Anda di halaman 1dari 347

MATERI OLIMPIADE MATEMATIKA NASIONAL

TEORI BILANGAN

A. TEORI BILANGAN
.

OPERASI DAN SIFAT BILANGAN BULAT


Secara umum himpunan bilangan real terbagi menjadi dua himpunan besar yaitu
Sifat-sifat operasi himpunan bilangan real. Beberapa sifat operasi pada bilangan
real antara lain adalah: himpunan bilangan rasional dan himpunan bilangan
irrasional.
Definisi bilangan rasional:
Sebuah bilangan r disebut bilangan rasional jika bilangan r tersebut
dapat dinyatakan sebagai pembagian dari dua buah bilangan bulat.
Dalam notasi matematika sebagai berikut
p
r , p,q bulat, q≠0
q
Kumpulan semua bilangan rasional disebut himpunan bilangan rasional yang
merupakan gabungan dari himpunan bilangan bulat dan himpunan bilangan
pecahan. Sebuah bilangan rasional dapat mudah kita kenal dari bilangan desimalnya
dimana pada bilangan desimalnya terdapat pengulangan digit yang secara teratur.
Contoh
Beberapa bilangan rasional yang dapat dilihat dari pola bilangan desimalnya adalah:
1
  0,5000...
2
1
  0,3333...
3
2
  0,285714285714285714...
7

Materi Olimpiade Matematika SMP - Persiapan OSN Page 1


4
   0,444444...
9
Definisi bilangan irrasional:
Sebuah bilangan disebut bilangan irrasional jika bilangan tersebut tidak
dapat dinyatakan sebagai pembagian dari dua buah bilangan bulat. Sebuah
bilangan irrasional dapat mudah kita kenal dari bilangan desimalnya dimana
pada bilangan desimalnya tidak terdapat pengulangan digit yang secara
teratur.
Contoh
Beberapa contoh bilangan yang merupakan bilangan irrasional:
 , e, 2 , 3, 5 ,...
Kumpulan semua bilangan irrasional disebut himpunan bilangan irrasional.

Definisi bilangan bulat positif (asli):


Bilangan asli atau bilangan natural adalah bilangan-bilangan yang
disimbolkan dengan angka 1, 2, 3, ….
Kumpulan semua bilangan asli disebut himpunan bilangan asli, yaitu N =
1, 2,3,4,..... Sedangkan gabungan antara bilangan nol dan himpunan

bilangan asli disebut himpunan bilangan cacah, yaitu C = N  {0} =  0,1,2,3,....

Materi Olimpiade Matematika SMP - Persiapan OSN Page 2


Definisi bilangan bulat negatif:
Sebuah bilangan x disebut bilangan bulat negatif bila bilangan x
merupakan kebalikan (invers) dari suatu bilangan bulat positif.
Jika a merupakan suatu bilangan bulat positif maka x di simbolkan
dengan x = -a.
Kumpulan semua bilangan bulat negatif disebut himpunan bilangan bulat negatif,
yaitu   1,2,3,...

Definisi faktor pembagi:


Jika a, b, dan c adalah bilangan-bilangan bulat, serta berlaku ab = c
maka a dan b disebut faktor pembagi dari c, sedangkan c disebut
kelipatan dari a dan b.

Definisi bilangan genap dan ganjil:


Sebuah bilangan bulat positif a disebut bilangan genap bila salah
satu faktor dari a adalah 2. Bilangan yang bukan genap disebut bilangan
ganjil.
Kumpulan semua bilangan genap disebut himpunan bilangan genap.
Sedangkan kumpulan semua bilangan ganjil disebut himpunan bilangan ganjil.

Definisi bilangan komposit:


Sebuah bilangan bulat positif k ≠ 1 disebut bilangan komposit bila
bilangan k tersebut dapat dinyatakan sebagai hasil kali dua atau lebih
bilangan bulat positif ≠ 1.

Kumpulan semua bilangan komposit disebut himpunan bilangan komposit.


Definisi bilangan prima:
Sebuah bilangan bulat positif p ≠ 1 disebut bilangan prima bila
bilangan p tersebut merupakan perkalian antara 1 dan p, atau
bilangan p hanya mempunyai 2 faktor yaitu 1 dan p sendiri.
Kumpulan semua bilangan prima disebut himpunan bilangan prima, yaitu
 2,3,5,7,...

Materi Olimpiade Matematika SMP - Persiapan OSN Page 3


Penerapan bilangan bulat dapat diperhatikan teorema berikut
Teorema Dasar Aritmatika.
Setiap bilangan bulat positif n > 1 dapat dituliskan secara tunggal sebagai
2
n = p11 p2 n
... pn

untuk suatu a 1 , a2 .... an > 0 dengan p1 ≤ p 2 ≤ ... ≤ p n merupakan bilangan


prima.

n  p1 1 p2 2 p3 3 ... pnmaka
a a a a n
Jika banyak pembagi bulat positif n
adalah a1  1a2  1. . . an  1
Apabila n suatu bilangan bulat positif, maka menyatakan jumlah semua

pembagi bulat positif dari n.

Contoh Soal:

2. Banyaknya bilangan genap yang kurang dari 1000 dan hasil kali angka-angka
penyusunnya 180 adalah …
Penyelesaian:
Faktor dari 180 adalah 1,2,3,4,5,6,9,10,12,15,18,20,30,45,60,90,180
Tiga diantara faktor-faktor di atas yang dapat disusun menjadi bilangan genap
dan jika angka penyusunnya dikalikan menghasilkan 180 adalah 594 dan 954
3. Untuk bilangan real dan didefinisikan suatu operasi * dengan aturan sebagai
berikut: dimana simbol dan + berturut-turut artinya
perkalian dan penjumlahan bilangan biasanya. Tentukan nilai yang memenuhi
!
Penyelesaian:

Materi Olimpiade Matematika SMP - Persiapan OSN Page 4


atau
4. Misalkan N adalah bilangan bulat terkecil yang bersifat bersisa 2 jika dibagi 5,
bersisa 3 jika dibagi 7 dan bersisa 4 jika dibagi 9. Hasil penjumlahan digit-digit
dari N adalah …
Penyelesaian:

Karena terkecil maka ambil sehingga


Hasil penjumlahan digit-digit dari N adalah
1 2 3 11
5. Misalkan N =  2  3  ...  11 . Dalam bentuk desimal nilai N adalah ....
10 10 10 10
Penyelesaian
1 2 3
= 0,1 = 0,02 = 0,003
10 100 1000
11
= 0,000.000.000.11
100.000.000.000
Jadi N = 0,123456789011.
Soal Latihan:
1. Jika a dan b adalah bilangan bulat genap, dengan a > b, maka banyaknya
bilangan bulat ganjil di antara a dan b adalah ...
2. Faktorisasi prima dari 5220 adalah ...
3. Dengan menggunakan angka-angka 1, 1, 2, 2, 3, 3, 4, 4 bilangan 8 angka
terbesar yang dapat dibentuk dengan syarat kedua angka 1 dipisahkan oleh
satu angka yang lain, kedua angka 2 dipisahkan oleh dua angka, kedua angka 3
dipisahkan oleh tiga angka, dan kedua angka 4 dipisahkan oleh empat angka
adalah .......
4. Untuk bilangan bulat a dan b, (a, b) artinya bilangan tak negatif yang
merupakan sisa a  b jika dibagi oleh 5. Bilangan yang ditunjukkan oleh (–3,4)
adalah ....

Materi Olimpiade Matematika SMP - Persiapan OSN Page 5


5. Banyaknya bilangan bulat n sehingga merupakan bilangan bulat
adalah....
6. Banyaknya pasangan solusi bilangan bulat positif yang memenuhi

adalah...
7. Banyaknya pasangan bilangan asli berbeda yang selisih kuadratnya 2012
adalah...
8. Hasil suatu bilangan genap dan suatu bilangan ganjil adalah 840. Bilangan
ganjil yang terbesar yang memenuhi syarat tersebut adalah ......
9. Hasil operasi terbesar yang dapat diperoleh dari penempatan angka-angka
4,6,7, dan 8 pada kotak kotak yang tersusun seperti di bawah ini adalah ...

10. Bilangan segitiga adalah bilangan yang berbentuk dengan n bilangan

asli. Banyaknya bilangan segitiga yang kurang dari 100 adalah....


11. Alfa mengalikan tiga bilangan prima berbeda sekaligus. Ada berapa faktor
berbeda dari bilangan yang dihasilkan?
12. Jumlah 101 bilangan bulat berurutan adalah 101. Berapakah bilangan bulat
yang terbesar di dalam barisan tersebut.

13. Semua bilangan n dan sehingga merupakan bilangan bulat adalah....

14. Misal Dalam bentuk desimal nilai N adalah...

11 13
15. Bilangan-bilangan manakah yang terletak di antara dan .
15 18
16. Untuk bilangan real dan didefinisikan suatu operasi * dengan aturan
sebagai berikut: dimana simbol dan + berturut-
turut artinya perkalian dan penjumlahan bilangan biasanya. Tentukan nilai
yang memenuhi
n 1
17. H adalah himpunan semua bilangan asli n demikian sehingga bentuk
n3
menghasilkan bilangan bulat kurang dari 1, maka banyaknya himpunan bagian
tak kosong dari H adalah ....
1 1 1 1 1
18. Bentuk sederhana dari     ....  adalah .....
2 6 12 20 2005 (2005  1)

19. Jumlah tiga bilangan prima pertama yang lebih besar dari 50 adalah

20. Bilangan asli n sehingga hasil kali

Materi Olimpiade Matematika SMP - Persiapan OSN Page 6


21. Banyak faktor dari 4200 yang merupakan bilangan ganjil positif adalah...

22. =...

23. Bilangan asli n terbesar sehingga jumlah 1 + 3 + 5 + ...( 2n -1 ) lebih kecil dari
2006 adalah..
24. Bilangan cacah lima digit dengan digit pertama tidak nol dan jumlah semua
digitnya sama dengan 2 ada sebanyak ….
25. Desi merayakan hari ulang tahun pada tanggal 27 Desember 2006. Jika pada
hari tersebut usia Desi sama dengan jumlah digit dari angka tahun
kelahirannya, maka Desi lahir pada tahun …
26. Suatu barisan hanya terdiri dari bilangan 1, 2, 3, 4, dan 5. Jika barisan tersebut
adalah 1,2, 2, 3, 3,3,4, 4, 4, 4, 5, 5, 5, 5, 5,1, 1, 1,1, 1, 1,1, 2, 2, 2, 2, 2, 2, 2, 3, 3, 3, 3,
3, 3, 3, 3, 4, …., maka suku ke 100 dari barisan tersebut adalah ….
27. Banyak bilangan asli yang kurang dari 10.000 dengan jumlah digit pertama dan
digit terakhirnya sama dengan 11 adalah ….
28. Jika bilangan 123.456.789 dikalikan dengan bilangan 999.999.999, maka
banyak angka 9 dari hasil perkalian kedua bilangan tersebut adalah …
29. Himpunan semua bilangan prima yang kurang dari seratus dan kuadrat
bilangan tersebut ditambah dua juga merupakan bilangan prima adalah ….
30. Jika P, Q, R angka-angka dari suatu bilangan dan (100P+10Q+R)(P+Q+R)=2008,
maka nilai P adalah...
31. Jika 173  a  1
,
maka 25a+5b+100c+500d sama dengan .
61 1
b
1
c
d
32. Misalkan n adalah bilangan asli yang tidak lebih dari 24, maka jumlah dari
semua nilai n yang memenuhi agar n dan 24 relatif prima adalah ...
33. Fachmy menghitung meningkat mulai dari 1000, kemudian bertambah 8
menjadi 1008, kemudian 1016, 1024,... demikian pula seterusnya, sedangkan
Zeldy pada saat yang sama menghitung menurus mulai dari 2008, berkurang 4
menjadi 2004, kemudian 2000, 1996, .... dan seterusnya. Bilangan tepat sama
saat mereka menghitung bersama-sama adalah….
34. Andi membuka sebuah buku setebal 650 halaman, hasil kali nomor halaman
yang nampak adalah 702. Jumlah nomor-nomor halaman buku yang terbuka
adalah ....

Materi Olimpiade Matematika SMP - Persiapan OSN Page 7


35. Banyaknya bilangan genap yang kurang dari 1000 dan hasil kali angka-angka
penyusun 180 adalah....
36. Jumlah 2009 bilangan bulat berurutan sama dengan 6027, maka selisih
bilangan terkecil dan terbesar sama dengan.....
37. Jika jumlah bilangan bulat positif berurutan adalah 2010 dengan , maka
terkecil yang mungkin adalah …
38. n adalah bilangan bulat positif terkecil sehingga 7 +30 n bukan merupakan
bilangan prima. Nilai dari adalah...
39. Bilangan tiga digit 2A3 jika ditambahkan dengan 326 akan menghasilkan
bilangan tiga digit 5B9. Jika 5B9 habis dibagi 9 maka A + B =....
40. Bilangan ratusan yang angka penyusunnya berupa bilangan prima dimana
perkalian ketiga angka penyusun tersebut adalah 10,ada sebanyak...buah
bilangan
41. Jika operasi # terhadap bilangan rasional positif didefinisikan sebagai a#b =

maka 3 # (3 # 3)

42. Menggunakan angka-a ngka 1, 2, 5, 6 dan 9 akan dibentuk bilangan


genap yang terdiri dari lima angka. Jika tidak ada angka yang
berulang, maka selisih bilanga n terbesar dan terkecil adalah ...
43. Diketahui 2012 bilangan bulat positif berurutan. Jika setiap bilangan tersebut
dibagi 5,kemudian sisa-sisa pembagiannya dijumlahkan,maka hasil
penjumlahan sisa-sisanya adalah...
44. Banyaknya solusi pasangan bilangan bulat positif persamaan 3x + 5y = 501
adalah....
45. Sejumlah bilangan disusun seperti dalam tabel berikut. Bilangan 2012 terletak
di bawah huruf....
P Q R S T U V
1 2 3
7 6 5 4
8 9 1
. .. . 0 1
. .. . 1
. .

Materi Olimpiade Matematika SMP - Persiapan OSN Page 8


46. Misalkan ab adalah bilangan terdiri dari dua angka. Jika bilangan itu ditambah
45, maka diperoleh bilangan ba. Pada bilangan ab, jika di antara a dan b

disisipkan angka 0, maka diperoleh bilangan yang nilainya kali bilangan ab.

Bilangan ab tersebut adalah....


47. Bilangan asli terkecil lebih dari 2012 yang bersisa 1 jika dibagi 2,3,4,5,6,7,8,9,10
adalah....
48. Jika bilangan m dibagi 5 memberikan sisa 3,dan bilangan n dibagi 5
memberikan sisa 2,maka bilangan mn jika dibagi 5 memberikan sisa....
49. Misalkan N adalah bilangan bulat terkecil yang bersifat bersisa 2 jika dibagi 5,
bersisa 3 jika dibagi 7 dan bersisa 4 jika dibagi 9. Hasil penjumlahan digit-digit
dari N adalah ....
50. Jika bilangan bulat x dan y dibagi 4, maka bersisa 3. Jika bilangan x–3y dibagi 4,
maka bersisa…..
51. Diketahui bilangan bulat positif n memiliki sifat-sifat berikut:2 membagi n, 3
membagi n+1, 4 membagi n+2, 5 membagi n+3, 6 membagi n+4, 7 membagi
n+5, 8 membagi n+6. Bilangan bulat positif pertama yang memiliki sifat-sifat ini
adalah 2. Tentukan bilangan bulat positif ke-5 yang memenuhi sifat- sifat di
atas.
52. Tentukan semua factor prima dari:
a. 27000001.
b. 1280000401
53. Ada berapa faktor positif dari:

a.22015375201372 yang merupakan kelipatan 10


b. 27355372 yang merupakan kelipatan 6

54. Nilai n terkecil sehingga bilangan 201220122012.......20122012 (ada n buah


2012) habis dibagi 999 adalah....
55. m dan n bilangan bulat positip dengan m.n=40000.Jika m dan n keduanya
tidak habis dibagi 10, tentukan m+n.
56. Fachmy menghitung, mulai dari 1000, kemudian bertambah 12 menjadi
1012,1024, 1036, 1048, ... Sedangkan Zeldy pada saat yang sama menghitung
mulai dari 2012, berkurang 8 menjadi 2004,1996, 1988, 1980,.... Bilangan
tepat sama saat mereka menghitung bersama-sama adalah...

Materi Olimpiade Matematika SMP - Persiapan OSN Page 9


57. Tentukan banyaknya bilangan positif 5-angka palindrom yang habis dibagi 3.
Palindron adalah bilangan/kata yang sama jika dibaca dari kiri ke kanan atau
sebaliknya. Sebagai contoh 35353 adalah bilangan palindron, sedangkan 14242
bukan
FPB dan KPK
1. FAKTOR PERSEKUTUAN TERBESAR (PEMBAGI SEKUTU TERBESAR) atau
FPB
Kita telah mengetahui bahwa semua faktor bulat positif dari 30 adalah 1, 2, 3, 5, 6,
10, 15, dan 30. Sedangkan semua faktor bulat positif dari 45 adalah 1, 3, 5, 9, 15 dan
45. Maka faktor-faktor persekutuan (pembagi-pembagi bersama) dari 30 dan 45
adalah 1, 3, 5 dan 15. Dan faktor persekutuan terbesar dari 30 dan 45 adalah 15.
Secara umum, pengertian tentang faktor persekutuan dari dua bilangan bulat
dituliskan sebagai definisi berikut ini.
Misalkan a, b bilangan bulat. d dikatakan pembagi sekutu dari a dan b
Jika d |a dan d | b.
Perhatikan bahwa 1 selalu merupakan pembagi dari bilangan bulat apapun.
Dengan demikian himpunan pembagi sekutu bukan merupakan himpunan yang
hampa.
Jika a=b=0maka setiap bilangan bulat merupakan pembagi sekutu dari a dan b.
Jika suatu bilangan tidak nol, maka ia hanya mempunyai sejumlah hingga
pembagi.
Dengan demikian jika paling sedikit salah satu dari a atau b tidak nol maka ada
berhingga banyaknya pembagi sekutu dari dua bilangan tersebut, dan diantara
bilangan tersebut ada yang terbesar, yang kemudian kita sebut sebagai pembagi
persekutuan terbesar (FPB)

Menentukan FPB
Untuk menentukan FPB dari dua atau lebih bilangan asli dapat dilakukan dengan
berbagai macam metode, yaitu:
1. Mendaftarkan semua faktor dari bilangan-bilangan tersebut
2. Metode faktor prima

Materi Olimpiade Matematika SMP - Persiapan OSN Page 10


Metode ini merupakan metode yang sangat umum digunakan, yaitu dengan
menguraikan setiap bilangan tersebut sebagai perkalian antar faktor primanya.
Kemudian untuk menentukan FPB dengan cara:
a. Tentukan semua faktor prima yang sama antar bilangan tersebut
b. Tentukan bilangan terkecil dari pemangkatan setiap faktor prima yang
sama di atas yang berada pada bilangan-bilangan tersebut
c. FPB dari bilangan-bilangan tersebut adalah perkalian antar semua
bilangan terkecil yang sudah diperoleh di atas
Namun metode ini dapat dilakukan jika bilangan-bilangan asli yang ingin dicari FPB
nya merupakan bilangan komposit. Secara matematis sebagai berikut:
Misalkan a, b bilangan asli yang mempunyai faktorisasi prima:
a = p1a1 p2a2 ... pnan dan b = p1b1 p2b2 ... pnbn

dimana pangkat adalah bilangan bulat tidak negatif, dan semua prima yang
muncul di faktorisasi a atau b muncul di faktorisasi kedua-duanya, bisa
dengan pangkat nol. FPB (a, b) = p1min(a1 ,b1 ) p2min(a2 ,b2 ) ... pnmin(an ,bn ) dimana min(x,

y)adalah nilai terkecil antara x dan y.

3. Algoritma pembagian / Algoritma Euclid


Teorema Algoritma pembagian.
Misalkan a bilangan bulat dan d bilangan bulat positif. Terdapat bilangan bulat q dan
r yang unik, dengan 0  r < d sehingga a = dq + r
Dalam persamaan yang diberikan pada algoritma pembagian, d disebut pembagi
(divisor), a disebut yang dibagi (divident), q disebut hasil bagi (quotient), dan r
disebut sisa (remainder). Secara sederhana metode yang dilakukan algoritma Euclid
adalah mencari faktor persekutuan terbesar dari bilangan-bilangan yang telah
direduksi terus menerus.
Cara mereduksi bilangan ini adalah dengan melihat sisa pembagian antara
satu bilangan dengan bilangan yang lain. Sisa tak nol terakhir adalah nilai FPB
yang dimaksud

Materi Olimpiade Matematika SMP - Persiapan OSN Page 11


Contoh Tentukan pembagi sekutu terbesar dari 12378 dan 3054.
Jawab
Dengan menggunakan Algoritma Euclid, kita peroleh
12378 = 4 . 3054 + 162
3045 =18 . 162 + 138
162 = 1. 138 + 24
138 = 5 . 24 +18
24 = 1.18+6
18 = 3 . 6 + 0
Dengan demikian (12378, 3054) = (18, 6) = 6.

Algoritma Euclides dapat juga digunakan untuk mencari persamaan


Diophantine
Suatu peramaan berbentuk ax + by = c dengan a, b, dan c bilangan bulat dan a, b
tidak nol disebut Persamaan Diopanthine, jika penyelesaiannya dicari pada
himpunan bilangan bulat.
Teorema
Persamaan diopanthine ax + by = c mempunyai penyelesaian jika dan hanya
jika FPB(a, b) membagi c.
Teorema
Jika d = FPB(a, b) dan x0, y0 adalah penyelesaian dari Persamaan
Diophantine ax + by = c, maka penyelesaian umum dari persamaan tersebut
adalah x  x0  db k dan y  y0  da k dengan k parameter bilangan bulat.

Contoh 1. Hitunglah (247,299) dan tentukan bilangan-bilangan bulat m dan n yang


memenuhi 247m + 299n = (247, 299)
Jawab
299 = 247.1 + 52
247 = 52.4 + 39
52 = 39.1 + 13
39 = 13.3
Jadi (247, 299) = 13
Selanjutnya,
13 = 52 - 39.1

Materi Olimpiade Matematika SMP - Persiapan OSN Page 12


= 52 - (247 - 52.4)
= 52.5- 247
= (299 - 247).5 - 247
13 = 299.5 + 247(-6)
Jadi m = -6 dan n = 5
Tetapi nilai m dan n yang memenuhi 247m + 299n = 13 tidak tunggal. Sebab 247(-6
+ 299t) + 299(5 - 247t), untuk setiap bilangan bulat t.
Jadi m = -6 + 299t dan n = 5 - 247t, untuk setiap bilangan bulat t.
Dari teorema di atas kita juga bisa katakan bahwa pembagi sekutu terbesar dari a
dan b adalah bilangan positif terkecil yang terdapat pada himpunan {ax + by | x,y
bilangan bulat }.
2. KELIPATAN PERSEKUTUAN TERKECIL (KPK)
Suatu bilangan bulat c dikatakan kelipatan persekutuan dari bilangan bulat taknol a
dan b jika a | c dan b | c. Perhatikan bahwa 0 merupakan kelipatan sekutu dari a dan
b. Untuk melihat apakah terdapat kelipatan sekutu yang tidak trivial, kita dapat
melihat bahwa ab dan -ab keduanya merupakan kelipatan sekutu dari a dan b, dan
salah satunya merupakan bilangan positif, yang kemudian menurut well ordering
principle, himpunan kelipatan sekutu yang positif dari a dan b, mempunyai unsur
terkecil, kita sebut sebagai kelipatan persekutuan terkecil (KPK)
Definisi
Kelipatan persekutuan terkecil dari dua bilangan bulat taknol a dan b, ditulis [a, b],
adalah bilangan bulat positif m yang memenuhi
(a). a | m dan b | m
(b). jika a | c dan b | c maka m ≤ c
Untuk menentukan KPK dari dua atau lebih bilangan asli dapat dilakukan dengan
berbagai macam metode, yaitu:
 Mendaftarkan semua kelipatan dari bilangan-bilangan tersebut
 Metode faktor prima
Namun metode ini hanya dapat dilakukan jika bilangan-bilangan asli yang
ingin dicari KPK nya merupakan bilangan komposit. Secara matematis
sebagai berikut:
Seperti FPB, KPK antara dua bilangan bulat juga dapat dicari dengan
faktorisasi prima dari masing-masing bilangan, dengan KPK(a, b) adalah

Materi Olimpiade Matematika SMP - Persiapan OSN Page 13


KPK (a, b) = p1maks( a1 ,b1 ) p2maks( a2 ,b2 ) ... pnmaks( an ,bn ) dimana maks (x, y)

menyatakan nilai terbesar antara x dan y.


Berikut adalah teorema yang mengkaitkan antara konsep pembagi sekutu terbesar
dan kelipatan sekutu terkecil
Teorema 1. Untuk bilangan bulat positif a dan b berlaku ab = (a, b) [a, b] ) dimana [a,
b] adalah KPK dari bilangan bulat positif a dan b
Contoh KPK dari 3054 dan 12378 adalah
Jawab
Dari contoh di atas didapatkan FPB (3054,12378) = 6.
Dengan demikian kelipatan sekutu terkecilnya adalah
3054  12378
[3054, 12378] =  6.300.402
6
Contoh Soal:
a. Dimas membeli majalah setiap 5 hari sekali, sedangkan Andre membeli
majalah setiap 8 hari sekali. Kemarin Dimas membeli majalah. Andre
membeli majalah hari ini. Keduanya paling cepat akan membeli majalah pada
hari yang sama, yaitu … hari lagi.

Penyelesaian:
Karena priode mereka membeli majalah berbeda sehari, maka Dimas dan
Andre akan membeli majalah pada hari yang sama adalah ,
dengan bilangan bulat positif. (KPK)

sehingga

Untuk maka (bukan bilangan bulat)

maka (bukan bilangan bulat)

maka (bilangan bulat)

Jadi paling cepat mereka membeli majalah pada hari yang sama adalah 25
hari lagi.
b. Diketahui FPB (a, 2008)=251. Jika , maka nilai terkecil yang
mungkin dari adalah …
Penyelesaian:
karena maka nilai terkecil adalah
2 1

Materi Olimpiade Matematika SMP - Persiapan OSN Page 14


Soal Latihan
1. Diketahui FPB (a, 2015)=403. Jika , maka nilai terkecil yang mungkin
dari adalah …
2. Ada berapa banyakkah pasangan terurut bilangan asli (a, b), dengan syarat a <
b dan FPB(a, b) = 4 serta KPK (a, b) = 140 ?
2015
3. Nilai dari  FPB (k ,7) adalah…
k 1

4. Tentukan pembagi sekutu terbesar dari 12378 dan 3054.


5. Tentukan (1479, 272)
6. Seratus karung beras dibagikan kepada 100 orang sebagai berikut: setiap
pemuda mendapat 3 karung, setiap gadis mendapat 2 karung, dan anak-anak
1
masing-masing mendapat karung. Berapa banyaknya masing-masing
2
pemuda, gadis dan anak-anak?
7. Tentukan [143, 227], [306, 657], [272, 1479].
8. Jika seekor ayam jago berharga 5 koin, seekor ayam betina berharga 3 koin,
dan tiga anak ayam berharga 1 koin. Berapa banyak ayam jago, ayam betina,
dan anak ayam yang jumlah totalnya 100 dapat dibeli oleh 100 buah koin?
9. Banyaknya anggota himpunan dari himpunan S={gcd(n3+1,n2+3n+9)|n∈ Z}
adalah…
10. Ada berapa banyak nilai k sehingga KPK (66, 88, k) = 1212.

3. BILANGAN BERPANGKAT
Definisi
Jika a adalah bilangan riel dan n bilangan bulat positip, maka didefinisikan :
an  a
.a
.a
..........
 ..a yaitu a ada n kali(n factor).
sebanyakn kali

Sifat
Jika a,b bilangan real dan m,n bilangan bulat positif, maka berlaku:
m
m n a am
a. a .a  a
m n
e.    m
b b
am
b.  a mn f. a 0  1
an

c. a m n
 a mn m
g. a 
1
am
d. (a.b) m  a m .b m
Semua sifat di atas juga berlaku untuk m,n bilangan rasional, untuk itu perlu
didefenisikan lagi satu aturan untuk bilangan rasional yaitu
n
a  a1/ n yang dikenal
dengan bentuk akar

Materi Olimpiade Matematika SMP - Persiapan OSN Page 15


Bentuk Akar
Menyederhanakan Bentuk Akar
Seperti yang dijelaskan di atas, salah satu bagian dari bilangan real adalah bilangan
irasional, dan salah satu bentuk dari bilangan irasional berupa bilangan dibawah
tanda akar. Jika bilangan di dibawah tanda akar berupa bilangan kuadrat sempurna
maka kita dapat menghilangkan tanda akar tersebut dengan aturan bentuk pangkat
di atas. Dalam perhitungan sering ditemukan bentuk akar yang memuat bilangan
yang berupa perkalian dari bilangan kuadrat sempurna dengan bilangan lain, di
bawah tanda akar tersebut. Untuk menyajikan bilangan tersebut disepakati bentuk
yang lebih sederhananya dengan memperkecil bilangan yang berada dibawah tanda
akar. Untuk lebih jelasnya perhatikan contoh berikut ini;
Operasi Bilangan Pada Bentuk Akar
1. Operasi Penjumlahan dan Pengurangan
Operasi penjumlahan dan pengurangan bentuk akar hanya dapat dilakukan, jika
bentuk akar-akarnya sejenis. =

- =
2. Operasi Perkalian Bentuk Akar
Perkalian pada bentuk akar bersesuaian dengan perkalian pada bentuk pangkat
berdasarkan hal tersebut diperoleh,

a x a = a x a = a 2 = a , untuk aR dan a > 0


dan a x b = a x b = ab , untuk a,bR dan a,b > 0
Untuk pecahan campuran diberlakukan aturan berikut;
axp

a b x p q = ap bq
bxq
Berdasarkan bentuk di atas dengan manipulasi aljabar sederhana diperoleh ;

( a b )2 = (a + b) 2 ab

( a b) = (a  b)  2 ab

Karena akar dari suatu bilangan selalu positif maka untuk pengurangan
diberikan syarat a > b.

Materi Olimpiade Matematika SMP - Persiapan OSN Page 16


Merasionalkan Penyebut Bentuk Akar
a
Bilangan Berbentuk
b

a b
Untuk merasionalkan penyebut , kalikan dengan
b b
c c
Bilangan Berbentuk atau
a  b a  b

Bentuk a + b dan a – b merupakan dua bentuk sekawan atau saling konjugat.


Untuk merasionalkan penyebut bentuk pecahan tersebut masing-masing penyebut
dan pembilang dari bilangan tersebut dikatakan saling sekawan atau konjugat.

Contoh Soal:

1.

= =

=
=
=

2.

Materi Olimpiade Matematika SMP - Persiapan OSN Page 17


3.

4.

5.

Soal Latihan

1. Urutan Bilangan-bilangan 25555 , 52222 , dan 33333 dari yang terkecil sampai
yang terbesar adalah ….

2. Urutan tiga bilangan 24444, 33333, dan 42222 dari yang terkecil sampai yang terbesar
adalah ….
3. ( 3)3  ( 3)2  ( 3)1  ( 3)0  ( 3)1  ( 3)2  ( 3)3  ...

4. Banyak bilangan bulat m sehingga merupakan bilangan bulat

adalah...

5. + =....

Materi Olimpiade Matematika SMP - Persiapan OSN Page 18


13  2 3    n 3
6. Bilangan bulat positif terkecil n dengan n  2015 sehingga
n
merupakan bilangan bulat adalah....
7. Selesaikanlah :

8. Nilai maksimum dan minimum dari bilangan berikut adalah ….

a. 20123333 , 20132222 , 20141111


b. p = 2013 (2012!) 2 , q = 2012 (2013!) 2 , dan r = (2014!)
1 1 1
2

c. ,

d.

9. Diketahui n = . Hitunglah nilai dari

10. Hitunglah

11. Tentukan hasil dari 123456789101112131415....20142015

12. Hitunglah nilai dari

13. Bentuk sederhana dari 4  15  4  15 adalah ...


14. Didefinisikan n! n.(n  1).(n  2)..3..2.1 . Bilangan bulat n yang memenuhi

n!  21035.52.7.11 adalah ...


15.

16. Nilai dari


adalah ….
17.

18.
19.

Materi Olimpiade Matematika SMP - Persiapan OSN Page 19


LATIHAN TEORI BILANGAN
OSN 2007

1. Untuk sebarang bilangan bulat a, b, dan c berlaku a  (b  c)  (a  b)  (a  c).

a. Cari contoh yang menunjukkan bahwa a  (b  c)  (a  b)  (a  c).

b. Kapan berlaku a  (b  c)  (a  b)  (a  c) ? Jelaskan jawaban Anda.

OSP 2003
5 2 1
2. Nilai    ....
12 7 8
1 1 1
3. Jika   maka x = ... .
6 12 x
4. Notasi x menyatakan bilangan bulat terbesar yang lebih kecil dari atau sama
denngan x. Sebagai contoh, 7
3
= 2,  12 = -1 Maka hubungan yang benar di

antara dua bilangan bulat s= 2- 3 dan t = 2 - 3 adalah ... .


n3
5. Tentukan nilai n sehingga n dan keduanya merupakan bilangan bulat.
n 1
OSP 2004
1. Setiap muka sebuah kubus diberi bilangan seperti pada gambar. Kemudian setiap
titik sudut diberi bilangan yang merupakan hasil penjumlahan bilangan pada
muka-muka yang berdekatan dengannya. Nilai tertinggi bilangan pada titik sudut
adalah ....
7

9 5 3 1

11

2. Pada suatu jam digital yang angka-angkanya tertera mulai dari 00:00 sampai
dengan 23:59, dimungkinkan terjadi penampakan bilangan Palindrome (bilangan
yang dibaca dari depan dan dari belakang sama nilainya, misalnya 12:21 dan
23:32). Dalam satu hari satu malam, banyaknya bilangan Palindrome tersebut
menampakkan diri adalah ....

Materi Olimpiade Matematika SMP - Persiapan OSN Page 20


3. Untuk bilangan bulat a dan b, (a, b) artinya bilangan tak negatif yang merupakan
sisa a  b jika dibagi oleh 5. Bilangan yang ditunjukkan oleh (–3, 4) adalah ....
4. Bilangan 10-angka terbesar menggunakan empat angka 1, tiga angka 2, dua
angka 3, dan satu angka 4, sehingga dua angka yang sama tidak terletak
bersebelahan adalah ....

5. Bentuk sederhana dari 4  15  4  15 adalah ....

6. Persegi Antimagic ukuran 4  4 adalah susunan persegi panjang dari bilangan-


bilangan 1 sampai dengan 16 sedemikian hingga jumlah dari setiap empat baris,
empat kolom, dan dua diagonal utamanya merupakan sepuluh bilangan bulat
yang berurutan. Diagram berikut ini menunjukkan sebagian dari persegi
Antimagic ukuran 4  4. Berapakah nilai dari *?

* 14

9 3 7

12 13 5

10 11 6 4

1 1 1 1 1
7.  2  3  2  ....  = ....
1 1 2  2 3  3 4  4
2
2004  2004
2

OSP 2005
1 2  3  n
1. Bilangan asli n terbesar yang memenuhi  2005 adalah
n
2. Bilangan A adalah bilangan asli terkecil yang merupakan hasil kali dari 3
bilangan prima pertama. Dua buah bilangan antara 200 dan 300 yang memiliki
faktor prima tepat sama dengan bilangan A tersebut adalah (Catatan: 10 dan 30
punya faktor prima yang tidak tepat sama, sedangkan 12 dan 18 memiliki faktor
prima yang tepat sama)
2 3
3. Semua pasangan bilangan asli m dan n yang memenuhi persamaan  1
m n
adalah
4. Dalam menentukan jawab perkalian bilangan 1493 dan 1507, seorang anak
mengurangkan langsung 49 dari 2.250.000. Dia sama sekali tidak mengalikan

Materi Olimpiade Matematika SMP - Persiapan OSN Page 21


kedua bilangan tersebut dengan panjang. Prinsip matematika yang digunakan
oleh anak tersebut adalah....
5. Ada berapa banyakkah pasangan terurut bilangan asli (a, b), dengan syarat a < b
dan FPB(a, b) = 4 serta KPK (a, b) = 140 ?
6. Suatu bilangan X terdiri dari 6 angka dan dimulai dari angka 1. Jika angka
pertama dipindahkan dari ujung paling kiri ke ujung paling kanan tanpa
mengubah susunan angka-angka yang lainnya, bilangan yang baru terbentuk
adalah tiga kali lipat bilangan semula. Berapakah bilangan X tersebut?
7. A adalah suatu himpunan bilangan. Himpunan A memiliki sifat tertutup terhadap
pengurangan, artinya hasil pengurangan dua bilangan di A akan menghasilkan
bilangan di A juga. Jika diketahui dua anggota dari A adalah 4 dan 9 tunjukkan
bahwa:
a. 0  A
b. –13  A
c. 74  A
Selanjutnya, daftarkan semua anggota himpunan A
8. (2, 0, 4, 1) adalah salah satu selesaian/jawab dari x1  x2  x3  x4  7 . Jika
semesta pembicaraan pada persamaan ini adalah himpunan semua bilangan bulat
tidak negatif, tentukan banyak penyelesaian yang mungkin dari
x1  x2  x3  x4  7 .

9. Tentukan semua pasangan bilangan bulat (x, y) yang memenuhi sistem


persamaan:
x( y  1)  y 2  1
y ( x  1)  x 2  1
10. Di antara bilangan 1/5 dan 1/4 terdapat tak hingga banyak bilangan pecah.
Tentukan 999 bilangan pecah di antara 1/5 dan 1/4 sehingga selisih antara
bilangan pecah berikutnya dengan bilangan pecah sebelumnya konstan.
(Maksudnya jika x1 , x2 ,, x999 adalah bilangan pecah yang dimaksudkan, maka

x2  x1  x3  x2  x4  x3    x999  x998 )

11. Untuk setiap pasangan bilangan asli a dan b, kita definisikan a*b = ab + a - b.
Bilangan asli x dikatakan penyusun penyusun bilangan asli n jika terdapat
bilangan asli y yang memenuhi x*y = n. Sebagai contoh, 2 adalah penyusun 6,

Materi Olimpiade Matematika SMP - Persiapan OSN Page 22


karena terdapat bilangan asli 4 sehingga 2*4 = 2.4 + 2 - 4 = 8 + 2 - 4 = 6.
Tentukan semua penyusun 2005.
12. Diketahui bentuk x 2  3 y 2  n , dengan x dan y adalah bilangan-bilangan bulat.
a. Jika n < 20, bilangan berapa sajakah n tersebut, dan diperoleh dari pasangan
(x, y) apa saja.
b. Tunjukkan bahwa tidak mungkin menghasilkan

OSP 2006
1. Nomor telepon di Kota Malang terdiri dari enam angka. Banyaknya nomor
telepon di kota itu yang habis dibagi 5 adalah ...
i. Nomor telepon di Kota Malang terdiri dari enam angka dan tidak ada angka
yang berulang. Banyaknya nomor telepon di kota itu yang habis dibagi 5
adalah ...
ii. Nomor telepon di Kota Malang terdiri dari enam angka yang tidak berulang,
dan selalu memuat angka 3. Banyaknya nomor telepon di kota itu yang habis
dibagi 5 adalah ...
2. Jika faktor persekutuan terbesar (fpb) dari bilangan bulat positif a dan b tidak
kurang dari 15, dan kelipatan persekutuan terkecil (kpk) nya tidak lebih dari 32,
maka banyaknya pasangan bilangan bulat a dan b yang mungkin adalah ...
3. Terdapat tiga penjaga taman hiburan Amin, Budi, dan Cepi. Amin berjaga setiap 3
hari, Budi setiap 4 hari, dan Cepi setiap 5 hari. Pada hari Minggu mereka berjaga
bersama-sama untuk yang pertama kalinya. Pada saat mereka akan bersama-
sama untuk yang kedua kali, Amin sakit, sehingga tidak masuk. Pada hari apa
mereka dapat berjaga bersama-sama untuk yang berikutnya?
4. Nilai dari 12  22  32  42    20052  20062  ...
1 4 3
5. Jika P dan Q keduanya adalah bilangan positif ganjil dan memenuhi   ,
P Q 5
maka selisih P dan Q adalah …
6. Pak Rahman memiliki satu kantong permen yang akan dibagikan kepada anak-
anak. Jika setiap anak diberi dua permen, maka di dalam kantong Pak Rahman
tersisa empat permen. Namun, Jika setiap anak diberi tiga permen, maka ada dua
anak yang tidak mendapat bagian dan satu anak yang mendapatkan dua permen.
Banyak permen Pak Rahman di dalam kantong sebelum dibagikan adalah …
7. Jika n adalah bilangan asli, maka bentuk paling sederhana dari perkalian

Materi Olimpiade Matematika SMP - Persiapan OSN Page 23


 1  1  1   1 
1  2 1  2 1  2   1  2 
 2  3  4   n 
8. Ucok bermain menyusun batang-batang korek api seperti tampak pada gambar
berikut. Apabila susunan batang korek api yang dibuat Ucok dilanjutkan,
tentukan banyak batang korek api yang diperlukan untuk membuat susunan ke-
20.
OSP 2007
1. Banyak bilangan prima antara 10 dan 99 yang tetap merupakan bilangan prima,
jika kedua digitnya dipertukarkan adalah ...
A. 9 B. 10 C. 11 D. 12 E 13
2. Diberikan dua bilangan bulat yang berjumlah 37. Jika bilangan yang lebih besar
dibagi dengan bilangan yang lebih kecil, maka hasil baginya adalah 3 dan sisanya
5. Selisih kedua bilangan tersebut adalah ...
A. 3 B. 5 C. 8 D. 21 E. 29
3. Jumlah dari setiap tiga bilangan asli yang terletak pada garis lurus pada Gambar 1
selalu sama. Nilai dari p+q+r+s yang mungkin adalah ...
p
A. 63 B. 69 C. 72 D. 84 E. 90

27 45

q 51 s

18 3
6

4. Diketahui a, b, c, dan d adalah bilangan asli. Jika c habis dibagi a, dan d habis
dibagi b, maka pernyataan berikut:
1. cd habis dibagi ab (ok)
2. c+d habis dibagi a+b (salah)
3. cd habis dibagi a (ok)
4. bc habis dibagi ab
5. d c habis dibagi b a
Yang selalu benar adalah ...
A hanya (i) B. hanya (1), (iii), dan (iv)

Materi Olimpiade Matematika SMP - Persiapan OSN Page 24


C. semuanya, kecuali (ii) D. semuanya kecuali (v)
E. semuanya
5. Didefinisikan n! n.(n  1).(n  2)..3..2.1 . Bilangan bulat n yang memenuhi

n!  21035.52.7.11 adalah ...


6. Semua pasangan bilangan bulat x dan y yang memenuhi persamaan
y 2 ( x  1)  1576  x 2 adalah …
3 6 9 12 96
7. Hasil dari penjumlahan     ...  adalah ...
20 60 126 224 24.192
8. Sebuah printer menomori semua halaman sebuah buku mulai dari nomor 1 dan
seluruhnya menggunakan 3201 digit. Banyak halaman buku tersebut adalah ...
Catatan: perhatikan banyak halaman yang terdiri dri 1 digit, 2 digit, 3 digit, dan
4 digit)
OSP 2008
1. Jika A= 1+11+111+1111+....+111.........111 (angka 1 ada 2008 angka) , maka 5
angka terakhir dari adalah ...

2.

3.

4.

5. Pak Asari akan mengikat semua buku yang dimilikinya. Ketika banyak buku
dalam setiap ikatan sama dengan 12, ada 2 buku yang tidak terikat. Dia
mengubah banyak buku dalam setiap ikatan. Sekarang dalam setiap ikatan
terdapat tepat 9 buku, ternyata juga masih bersisa 2 buku yang tidak terikat.
Seteiah dia mengikat 7 buku dalam setiap ikatan, tidak ada lagi buku yang
tersisa. Jika banyak buku yang dimiliki pak Asari berkisar antara 100 dan 200,
maka banyak buku yang dimiliki pak Asari adalah ....

Materi Olimpiade Matematika SMP - Persiapan OSN Page 25


6. Hasil kali 46 bilangan bulat sama dengan. Mungkinkah jumlah bilangan-bilangan
bulat yang memenuhi syarat tersebut sama dengan 0?

7. Perhatikan bahwa 1 + 2 + 3 + 45 + 6 + 78 + 9 = 144. Banyak cara yang mungkin


dilakukan untuk menghasilkan 144 dengan hanya menggunakan bilangan-
bilangan yang dibentuk dari angkaangka 1, 2, 3, 4, 5, 6, 7, 8, dan 9 secara
berurutan dari kiri ke kanan dan hanya menggunakan operasi penjumlahan
adalah ....
8. Untuk setiap pasangan bilangan asli dan, didefinisikan a b =a - b +ab. Bilangan
asli dikatakan mitra bilangan asli n jika terdapat bilangan asli y yang memenuhi
x y =n. Sebagai contoh, 7 adalah mitra dari 13 karena terdapat bilangan asli 1
sehingga 7 1=7 – 1+ 7.1 =7 - 1+7 = 13. Tentukan semua mitra dari 2008
9. Angka 1, 2 ,3, 4, 5, 6, 7, 8, dan 9 akan ditempatkan ke masing-masing kotak pada
gambar berikut sehingga jumlah mendatarnya sama dengan jumlah vertikalnya.
A adalah bilangan 5 angka yang dibentuk dengan cara membaca secara vertikal
dari atas ke bawah masing-masmg bilangan di dalam kotak tersebut. Berapa
banyak bilangan A yang mungkin terbentuk?

OSP 2009
1. Banyak bilangan bulat berbeda yang merupakan penjumlahan dari tiga bilangan
berbeda dalam { , 9, 1, 17, …., 41} adalah ….
2. Nilai dari adalah ….
3. Bilangan palindrom adalah bilangan yang dibaca dari kiri dan kanan selalu
sama, seperti 131. Banyak bilangan ganjil positip yang bersifat palindrom dan
terdiri dari sembilan angka serta dua kali bilangan tersebut juga merupakan
bilangan palindrom adalah …
4.

Materi Olimpiade Matematika SMP - Persiapan OSN Page 26


5. Suatu bilangan dikatakan “berprisque” jika bilangan tersebut merupakan
bilanan asli yang didapat oleh suatu bilangan prima dan suatu bilangan kuadrat
sempurna (contoh 3 adalah bilangan berprisque, tetapi 5 bukan bilangan
berprisque). Banyak bilangan berprisque yang kurang dari 100 adalah ….
6. Banyak cara untuk menyatakan bilangan 4725 sebagai perkalian dari dua
bilangan ganjil yang lebih besar dari 1 adalah ….
7. Banyak bilangan bulat positip n yang memenuhi bilangan kuadrat sempurna
adalah….
8.

9. 9.

OSP 2010
1.

2.

3.
4.

OSP 2011
1. Jika x adalah jumlah 99 bilangan ganjil terkecil yang lebih besar dari 2011 dan y
adalah jumlah 99 bilangan genap terkecil yang lebih besar dari 6, maka x + y = ...
2. Jika bilangan bulat x dan y dibagi 4, maka bersisa 3. Jika bilangan x–3y dibagi 4,
maka bersisa…..
3. Banyak bilangan 3 digit (angka) yang terdiri dari angka-angka 0,2,3,5,7,8 yang
lebih dari 243 dan kurang dari 780 adalah…..

Materi Olimpiade Matematika SMP - Persiapan OSN Page 27


4. Saat ini umur Agus dan umur Fauzan kurang dari 100 tahun. Jika umur Agus dan
umur Fauzan ditulis secara berurutan, maka diperoleh suatu bilangan empat
digit (angka) yang merupakan kuadrat sempurna. Dua puluh tiga tahun
kemudian, jika umur mereka ditulis dengan cara yang sama, maka diperoleh
bilangan empat digit lain yang juga merupakan kuadrat sempurna. Jika umur
mereka diasumsikan merupakan bilangan bulat positif, berapakah umur mereka
saat ini?
5. Diketahui bil.bulat positif n memiliki sifat-sifat berikut.
2 membagi n, 3 membagi n+1, 4 membagi n+2, 5 membagi n+3, 6 membagi n+4,
7 membagi n+5, 8 membagi n+6. Bilangan bulat positif pertama yang memiliki
sifat-sifat ini adalah 2. Tentukan bilangan bulat positif ke-5 yang memenuhi
sifat-sifat diatas.

OSP 2012
1.

2.

3.

4.

5.

6.

Materi Olimpiade Matematika SMP - Persiapan OSN Page 28


OSP 2013
1.

2.

3.

4.

OSP 2014
1.

2.

3.

4.

5.

Materi Olimpiade Matematika SMP - Persiapan OSN Page 29


B. ALJABAR

No. Kompetensi Materi Lingkup Materi


menganalisis pertidaksamaan
persamaan dan a. Persamaan linear satu dan
pertidaksamaan linear dua peubah
b. Pertidaksamaan linear
satu peubah
c. Persamaan kuadrat satu
peubah
d. Pertidaksamaan linear dan
kuadrat dua peubah
2. Sistem persamaan linear dua
peubah
d. Menerapkan dan 3. Barisan dan deret
menganalisis konsep a. Pola bilangan
barisan dan deret b. Barisan dan deret
aritmetika
c. Barisan dan deret geometri

RANGKUMAN MATERI
Pada materi ini,mempelajari konsep himpunan dimulai dari beberapa
pengertian seputar himpunan sampai akhirnya mengenai penerapan atau
pemakaian operasi-operasi pada himpunan. Di akhir materi ini akan diberikan
contoh soal mengenai himpunan
1. HIMPUNAN
Sebuah himpunan adalah setiap daftar, kumpulan atau kelas objek-objek
yang didefinisikan secara jelas. Objek-objek himpunan tersebut dapat berupa:
bilangan, orang, surat, sungai dan sebagainya. Setiap objek yang berada dalam suatu
himpunan disebut anggota atau elemen dari himpunan tersebut.
Notasi anggota himpunan
Bila suatu objek merupakan anggota suatu himpunan maka digunakan
lambang “”. Sebaliknya bila objek tersebut bukan merupakan anggota suatu
himpunan maka digunakan lambang “”.
Notasi banyaknya himpunan
Banyaknya anggota dari suatu himpunan dinotasikan dengan “n“. Untuk
menyatakan banyaknya himpunan C ditulis n(C).

Materi Olimpiade Matematika SMP - Persiapan OSN Page 30


Himpunan berhingga dan tak berhingga
Suatu himpunan dikatakan berhingga bila terdiri dari sejumlah tertentu
elemen-elemen yang berbeda. Bila tidak, maka himpunan tersebut dikatakan tak
berhingga.
Kesamaan himpunan
Himpunan A dikatakan sama dengan himpunan B jika kedua himpunan
tersebut memiliki anggota yang sama. Kesamaan himpunan A dan B dinotasikan
dengan A = B.
Himpunan kosong
Suatu himpunan disebut kosong bila himpunan tersebut tidak mengandung
atau tidak mempunyai anggota. Himpunan kosong dilambangkan dengan { } atau .
Himpunan bagian (subset) dan himpunan kuasa (power set)
Himpunan A dikatakan himpunan bagian dari himpunan B jika setiap anggota
dari himpunan A merupakan anggota dari himpunan B. Jika A subset dari B, maka
dinotasikan dengan A  B, jika A tidak subset B, maka dinotasikan dengan A  B.
Himpunan kuasa atau power set dari sebuah himpunan A adalah kumpulan
semua himpunan bagian dari himpunan A. Himpunan kuasa dari himpunan A
dinotasikan dengan 2A. Jika banyak anggota himpunan A adalah n, maka banyaknya
anggota power set dari himpunan A adalah 2n.
Power set dari himpunan A = {1, 3, 6} adalah himpunan {, {1}, {3}, {6}, {1, 3},
{1, 6},{3, 6}, A} yang mempunyai banyak anggota 23=8
Irisan dan gabungan dua himpunan
Irisan dua himpunan A dan B adalah himpunan semua anggota yang menjadi
anggota A dan sekaligus anggota B. Irisan antara himpunan A dan B biasa
dinotasikan dengan A  B.Dapat dinyatakan dengan A B = { x | x  A dan x  B }.
Gabungan dua himpunan A dan B adalah himpunan semua elemen yang menjadi
anggota himpunan A saja atau himpunan B saja atau anggota himpunan A dan
himpunan B. Gabungan antara himpunan A dan B dinotasikan dengan A  B. Dapat
dinyatakan dengan
A  B = { x | x  A atau x  B }
Jumlah Dua himpunan A + B = { x | x  A, | x  B,dan x  A  B }
Selisih Dua himpunan A – B = { x | x  A dan x  B }
Beda simetris antara himpunan A dan B disimbolkan dengan AB dengan definisi:

Materi Olimpiade Matematika SMP - Persiapan OSN Page 31


AB = (A  B) – (A  B) beda simetris ini juga biasa disimbolkan dengan A + B.
Komplemen suatu himpunan
Kompelemen dari suatu himpunan A adalah sebuah himpunan yang terdiri
dari anggota-anggota yang bukan merupakan anggota himpunan A namun masih
merupakan anggota dari himpunan semesta. Komplemen himpunan A dinotasikan
dengan AC.
Sifat-sifat operasi himpunan
Operasi dasar pada himpunan secara umum terdiri atas dua operasi, yaitu operasi
irisan dan operasi gabungan. Sifat-sifat yang ada pada operasi-operasi tersebut
adalah:
a. Sifat komutatif
Sifat komutatif berlaku pada operasi irisan dan gabungan dua himpunan.
Sifat komutatif pada operasi irisan dua himpunan adalah : A  B = B  A.
Sifat komutatif pada operasi gabungan dua himpunan adalah : A  B = B  A.
b. Sifat asosiatif
Sifat asosiatif berlaku pada operasi irisan dan gabungan tiga himpunan.
Sifat asositatif pada irisan tiga himpunan adalah : (A  B)  C = A  (B  C).
Sifat asositatif pada gabungan tiga himpunan adalah : (A  B)  C = A  (B  C).
c. Sifat distributif
Sifat distributif pada himpunan dibagi menjadi dua bagian, yaitu:
 Irisan terhadap gabungan himpunan
Sifat distributif irisan terhadap gabungan himpunan adalah:
A  (B  C) = (A  B)  (A  C).
 Gabungan terhadap irisan himpunan
Sifat distributif gabungan terhadap irisan himpunan adalah:
A  (B  C) = (A  B)  (A  C).

d. Hukum de Morgan
Sifat-sifat yang berhubungan dengan komplemen himpunan, gabungan dan
irisan dua himpunan dinamakan hokum de Morgan. Hukum de Morgan adalah
sifat-sifat berikut ini:
 (A  B)C = AC  BC
 (A  B)C = AC  BC
e. Sifat-sifat pada himpunan kosong dan semesta

Materi Olimpiade Matematika SMP - Persiapan OSN Page 32


Sifat-sifat yang berhubungan dengan himpunan kosong dan himpunan semesta
adalah:
 A=A
 SA=A
 A=
 SA=S
f. Sifat-sifat pada komplemen himpunan
Sifat-sifat yang berhubungan dengan komplemen himpunan adalah:
 (AC)C = A
 A  AC = S
 A  AC = 

Latihan Soal
1. Diketahui:
a. S = {1, 2, …, 10},
b. A = {1, 4, 7, 10},
c. B = {1, 2, 3, 4, 5}, dan
d. C = {2, 4, 6, 8}.
2. Tentukan: A  (B  C), Bc  (C – A), (A  B)c  C, (B Δ C)  (A  C)
3. Bila diketahui X = {1, 2, 3, 4} dan Y = { a | a bilangan bulat dan 0 < a < 5 } maka
akan berlaku hubungan sebagai berikut :
a. X  Y
b. Y  X
c. X = Y
4. Misalkan sebuah keluarga hendak berkemah di suatu bumi perkemahan.
Misalkan M adalah kejadian mereka mendapat kesulitan mekanis pada
kemahnya, T adalah kejadian mereka terkena denda pelanggaran lalu lintas, dan
V adalah kejadian bahwa mereka sampai di bumi perkemahan dan ternyata tidak
ada tempat yang kosong. Dengan melihat diagram Venn pada gambar di bawah
ini, manakah daerah-daerah yang yang menyatakan kejadian-kejadian berikut
ini:

Materi Olimpiade Matematika SMP - Persiapan OSN Page 33


a. Keluarga itu tidak mengalami kesulitan mekanis pada kemahnya dan tidak
melanggar lalu lintas, tetapi bumi perkemahan telah penuh.
b. Keluarga itu mengalami kesulitan mekanis pada kemahnya dan mendapatkan
bumi perkemahan telah penuh, tetapi tidak melanggar lalu lintas.
c. Keluarga itu mengalami kesulitan mekanis pada kemahnya atau
mendapatkan bumi perkemahan sudah penuh tetapi tidak melanggar lalu
lintas.

6. Dari 40 orang, 28 orang suka pisang dan 16 orang suka apel, sedangkan terdapat
10 orang yang suka kedua-duanya. Berapa orang yang tidak suka apel dan pisang
7. Di dalam sebuah grup terdapat 30 orang, 8 orang bisa berbahasa Inggris, 12
orang bisa berbahasa Spanyol dan 10 orang bisa berbahasa Perancis. Diketahui
bahwa 5 orang bisa berbahasa Inggris dan Spanyol, 5 orang bisa berbahasa
Spanyol dan Perancis dan 7 orang bisa berbahasa Inggris dan Perancis.
Sedangkan yang bias berbahasa ketiga-tiganya sebanyak 3 orang. Berapa banyak
yang hanya bisa berbahasa Indonesia ?
8. Apakah kamu percaya, sebuah toko mengadakan survey dengan hasil sebagai
berikut: dari 1000 orang, terdapat 816 suka coklat, 723 suka es krim, 645 suka
kue, 562 suka coklat dan es krim, 463 suka coklat dan kue, 470 suka es krim dan
kue erta terdapat 310 orang yang suka ketiga-tiganya ?
9. Sebuah perusahaan asuransi mempunyai 10.000 klien dengan klasifikasi sebagai
berikut:
a. Muda atau tua
b. Pria tau Wanita
c. Sudah menikah atau belum menikah
Dari 10.000 klien tersebut, terdapat 3000 yang muda, 4600 pria dan 7000 yang
sudah menikah. Kemudian terdapat pula 1320 pria muda, 3010 pria menikah
serta 1400 yang muda dan sudah menikah. Terakhir 600 pria muda dan sudah

Materi Olimpiade Matematika SMP - Persiapan OSN Page 34


menikah. Berapa banyak klien perusahaan tersebut yang muda, wanita dan
belum menikah ?
10. Di dalam sebuah kelas yang terdiri dari 40 siswa, 14 di antaranya sika
matematika, 16 suka biologi dan 11 suka fisika. Diketahui pula 7 siswa suka
matematika dan biologi, 8 suka biologi dan fisika, 5 suka matematika dan fisika
sedangkan yang suka ketiga-tiganya sebanyak 4 siswa. Berapa banyak siswa
yang tidak suka matematika atau tidak suka biologi atau tidak suka fisika ?
11. Di dalam sebuah grup terdapat tes pelajaran bahasa Perancis, bahasa Inggris dan
matematika. 92 orang ikut tes bahasa Perancis, 72 orang ikut tes bahsa Inggris
dan 63 orang ikut tes matematika. Diketahui bahwa tidak lebih dari 65 orang
ikut tes bahasa Perancis dan Inggris, tidak lebih dari 54 orang ikut tes bahasa
Perancis dan Matematika serta tidak lebih dari 48 ikut tes bahasa Inggris dan
Matematika.Tentukan jumlah orang terbanyak yang mungkin mengikuti seluruh
tes tersebut ?

2. RELASI DAN FUNGSI


Produk Kartesius Himpunan
Definisi
Misalkan himpunan A, B  , maka produk kartesius dari himpunan A dan B
(A  B) adalah himpunan pasangan terurut dari setiap elemen himpunan A
dan setiap elemen himpunan B, A  B  {(a, b); a  A, b  B}
Sifat :
 A  B  B  A, kecuali A = B
 Jika n(A) = banyaknya anggota pada himpunan A dan n(B) = banyaknya
anggota pada himpunan B, maka n(A  B) = n(A)n(B)
Catatan:
 Bila A = R dan B = R, maka
A  B = R  R = R2 = {(x, y) | x, y  R} dan
Relasi
Selanjutnya, dari produk kartesius kita dapat membuat relasi dari himpunan
A ke himpunan B. Relasi dari A ke B yang dimaksud adalah sebarang himpunan
bagian dari produk kartesius A  B, R  A  B  {(a, b); a  A, b  B}

Materi Olimpiade Matematika SMP - Persiapan OSN Page 35


Pengertian Fungsi
Dari relasi yang ada, terdapat relasi khusus yang memenuhi sifat bahwa
setiap elemen di domain mempunyai tepat satu pasangan di kodomain. Relasi
khusus itulah yang disebut sebagai fungsi.
Fungsi dari subhimpunan D dari A, selanjutnya disebut domain atau daerah
asal, ke himpunan B, yang selanjutnya disebut kodomain atau daerah kawan,
didefinisikan sebagai relasi dari D  A ke B yang memenuhi sifat bahwa setiap
elemen dari D memiliki tepat satu pasangan elemen dari B.
Secara matematis, fungsi dapat dituliskan sebagai pengaitan berikut ini:
f: D  A B
a  b = f(a); untuk setiap a di D  A dan b di B,
Maksudnya, untuk setiap a anggota D  A ada satu dan hanya satu b elemen B
sehingga b peta dari a, yang secara matematis ditulis sebagai berikut,
a  D  A ! b  B sehingga b  f (a) .
a disebut variabel bebas dan b disebut variabel tak bebas.
Daerah jangkauan atau range dari fungsi adalah himpunan yang beranggotakan
semua elemen kodomain yang dipasangkan oleh semua elemen dari domain,
R f  {b  B; b  f (a); a  D f }
Sifat :
 Jika n(A) = p dan n(B) = q , maka banyaknya fungsi yang dapat terbentuk dari
himpunan A ke himpunan B adalah qp
Operasi-operasi pada fungsi
Operasi-operasi dasar pada fungsi diantara lain adalah:
a. (f + g)(x) = f(x) + g(x)
b. (f g)(x) = f(x g(x)
c. (f g)(x) = f(x)g(x)
d. (f /g)(x) = f(x)/g(x); g(x)  0
e. (f n)(x) = (f(x))n
f.  f ( x) 
n n f ( x)

Untuk menggambarkan grafik fungsi, dapat dilakukan dengan memasangkan daerah


asal (domain) dari suatu fungsi atau dikenal sebagai sumbu-x kepada daerah kawan
(kodomain) atau dikenal sebagai sumbu-y pada bidang koordinat Cartecius
kemudian menghubungkan koordinat antar titik secara berurutan.

Materi Olimpiade Matematika SMP - Persiapan OSN Page 36


Persamaan Garis Lurus
Pada materi ini kita akan membahas materi aljabar yang khususnya mengenai
persamaan garis lurus.
a. Titik tengah
Titik tengah dari sebuah garis yang menghubungkan titik A(x1, y1) dan B(x2, y2)
 x  x 2 y1  y 2 
adalah titik C(x3, y3) =  1 , 
 2 2 
b. Jarak

Jarak antara titik A(x1, y1) dan B(x2, y2) adalah AB  ( x 2  x1 ) 2  ( y 2  y1 ) 2

y 2  y1
c. Gradien m AB 
x 2  x1

d. Kolinear
Tiga buah titik A(x1, y1), B(x2, y2) dan C(x3, y3) dikatakan terletak dalam satu
garis lurus yang sama (kolinear) jika dan hanya jika gradien AB = gradien BC =
y 2  y1 y 3  y 2 y 3  y1
gradien AC atau ditulis sebagai berikut:  
x 2  x1 x3  x 2 x3  x1

e. Persamaan garis lurus


a
Bentuk umum persamaan garis lurus adalah: ax  by  c  0, m
b
 Bentuk khusus 1
Persamaan garis lurus yang melalui (0, c) dan mempunyai gradient m adalah:
y  mx  c
 Bentuk khusus 2
Persamaan garis lurus yang memotong sumbu x di (a, 0) dan memotong
sumbu y di (0, b) adalah: bx  ay  ab

 Bentuk khusus 3
Persamaan garis lurus yang melalui dua titik (x1, y1) dan (x2, y2) ditentukan
y  y1 x  x1
oleh persamaan: 
y 2  y1 x 2  x1

Materi Olimpiade Matematika SMP - Persiapan OSN Page 37


 Bentuk khusus 4
Persamaan garis lurus yang melalui titik (x1, y1) dengan gradien m adalah:
y  y1  m( x  x1 )

f. Luas segitiga yang dibentuk oleh titik A(x 1, y1), B(x2, y2) dan C(x3, y3) ditentukan
oleh formula:

1 x1 x2 x3 x1
Luas =
2 y1 y2 y3 y1

1
 Luas = x1 y2  x2 y3  x3 y1  x2 y1  x3 y2  x1 y3 (harus selalu diambil bilangan
2
positif)
g. Luas segiempat yang dibentuk oleh titik-titik sudut A(x1, y1), B(x2, y2), C(x3,
y3)dan D(x4, y4) ditentukan oleh formula:

1 x1 x2 x3 x4 x1
Luas =
2 y1 y2 y3 y4 y1

1
= x1 y2  x2 y3  x3 y4  x4 y1  x2 y1  x3 y2  x4 y3  x1 y4
2
Fungsi Kuadrat
1. Bentuk umum fungsi kuadrat adalah sebagai berikut :
y = f(x) = + , dengan a,b,c
2. karakteristik grafik fungsi kuadrat
fungsi kuadrat memiliki bentuk umum dengan a,b,c dan 0. Dari
bentuk aljabar tersebut, grafik fungsi kuadrat dapat diilustrasikan sebagai
bentuk lintasan lengkung atau parabola dengan karakteristik sebagai berikut:
a. Jika , maka parabola terbuka ke atas.
b. Jika , maka parabola terbuka ke bawah.
c. Jika , maka parabola tidak memotong maupun menyinggung
sumbu-x.
d. Jika , maka parabola menyinggung sumbu-x.
e. Jika , maka parabola memotong sumbu-x di dua titik.

3. Langkah-langkah dalam membuat sketsa grafik fungsi kuadrat


+ adalah sebagai berikut:
a. Menentukan titik potong dengan sumbu-x, apabila .
b. Menentukan titik potong dengan sumbu-y, apabila .

Materi Olimpiade Matematika SMP - Persiapan OSN Page 38


c. Menentukan persamaan sumbu simetris. .

d. Menentukan titik puncak. P


Persamaan kuadrat memiliki beberapa sifat, yaitu:
1) Apabila untuk nilai x yang sama dengan p, mendapat tanda berlainan
dengan tanda a, maka persamaan mempunyai akar-akar yang real
serta berlainan yang dipisahkan oleh p. Apabila dan a mempunyai tanda
yang sama, sedang mempunyai akar-akar yang real, maka kedua
akar itu akan terletak dalam salah satu dari dua interval ( dan (+ ,

yaitu dalam interval yang mengandung .

X1 f(p) X2

a>0 , f(p) < 0

2) Jika f(p) dan f(q) mempunyai tanda yang berlainan, maka f(x)= 0 mempunyai
akar-akar yang real dan berlainan, sedangkan salah satu dari akar-akar itu
akan terletak diantara p dan q.

f(p)
q
p X1 f(q) X2

Materi Olimpiade Matematika SMP - Persiapan OSN Page 39


1. Grafik fungsi kuadrat yang melalui titik balik (xe, ye) dan sebuah titik tertentu
(x, y):
Y
(xe, ye)

(x, y)

0 X

y = a(x – xe)2 + ye

2. Grafik fungsi kuadrat yang memotong sumbu X di dua titik (x 1, 0), (x2, 0), dan
melalui sebuah titik tertentu (x, y):
Y

(x, y)

(x1, 0) (x2, 0)
X
0
y = a(x – x1) (x – x2)

Contoh Soal
1.

2. Misalkan suatu fungsi yang memenuhi untuk semua bilangan

real positif dan . Jika maka adalah ….


Penyelesaian:

Materi Olimpiade Matematika SMP - Persiapan OSN Page 40


3.

Materi Olimpiade Matematika SMP - Persiapan OSN Page 41


Latihan Soal
1. Jika A = {2, 3, 5, 6} dan B = {1, 2, 3, 4, 5, 6}. Himpunan pasangan terurut yang
menyatakan hubungan “faktor dari” dari himpunan A ke himpunan B adalah…
2. Apakah himpunan pasangan terurut pada soal no. 1 merupakan sebuah fungsi?
Jelaskan jawaban kalian!
3. Suatu himpunan pasangan terurut dinyatakan dengan {(1, 2), (1, 4), (3, 6), (5, 8),
(7, 10)}. Hubungan yang mungkin dari relasi tersebut adalah ….
4. Fungsi f(x) = ax + b. Jika f(8) = 12 dan f(4) = 4, maka nilai a + b = ….
5. Sebuah fungsi didefinisikan sebagai berikut
f :RR
x  5x
Tentukanlah:
 Bayangan (peta) dari –2
 Prapeta dari 1
625
f ( x)  1
6. Diberikan f dengan f(11) = 11 dan f(x + 3) = . Tentukan f(2008)
f ( x)  1
7. Tentukan semua fungsi yang memenuhi
x2 f(x) + f(1 – x) = 2x – x4 untuk setiap x  R
8. Untuk n bilangan bulat positif, fungsi f ditentukan oleh :
n  3, jika n bilangan ganjil

f ( n)   n

2
, jika n bilangan genap

Jika k adalah bilangan bulat ganjil dan f (f(f(k))) = 27. Berapakah jumlah digit k ?
9. Diketahui f(x) = ax7 + bx3 + cx – 5, dengan a, b dan c tetap bila f(-7) = 7.
Berapakah f(7) ?
10. Jika f adalah fungsi sehingga f(xy)= f(x–y) dan f(6) =1, maka f(–2) – f(4) =….
11. Jika untuk semua x anggota bilangan real maka nilai

dari

12. Misalkan f(n) adalah kuadrat dari jumlah angka-angka n. Misalkan juga

f2(n) didefinisikan sebagai f(f(n)), f3(n) sebagai f(f(f(n))) dan seterusnya.

Tentukan nilai dari f2014(12)


13. f(n) = banyaknya factor positif dari n N = f(1) + f(2) + f( ) + f( ) + … + f( )
Banyak 3N + 1 adalah....

Materi Olimpiade Matematika SMP - Persiapan OSN Page 42


14. Diketahui f(x) =

Jika f(1) = 3, f(2) = 6 dan f(3) = 9


Tentukan

3. PERBANDINGAN
Perbandingan adalah ukuran yang melihat besarnya kuantitas dari 2 hal yang
berbeda.
Perbandingan antara a dan b dengan b≠0 adalah a:b atau dan dibaca a

berbanding b

Skala
Skala adalah suatu perbandingan yang menyatakan hubungan antara ukuran objek
pada gambar atau peta dengan ukuran objek yang sebenarnya. Skala dirumuskan
dinyatakan dengan,

Hubungan antara benda sebenarnya dengan model/ gambarnya sebagai berikut :

Perbandingan Senilai
Untuk dan bilangan positif, perbandingan dan ( dikatakan senilai
dengan perbandingan c dan ( jikadan hanya jika . Jika maka

(perkalian silang)
Perbandingan berbalik Nilai
Untuk dan bilangan positif, perbandingan dan ( dikatakan berbalik

nilai dengan perbandingan c dan ( jikadan hanya jika atau

Materi Olimpiade Matematika SMP - Persiapan OSN Page 43


Latihan Soal
1. Suatu pekerjaan jika dikerjakan oleh Anto dan Dini dapat diselesaikan dalam
waktu 6 jam. Jika pekerjaan itu dikerjakan oleh Dini sendirian akan selesai lima
jam lebih lambat dibandingkan Anto. Pekerjaan itu dapat diselesaikan oleh
Anto sendirian dalam waktu …. jam
2. Pada akhir tahun 1994 Andi berusia setengah usia neneknya. Jumlah kedua
tahun kelahiran mereka adalah 3844. Berapakah usia Andi pada tahun 2012?
3. Wati mencatat, 3/20 dari total siswa di kelas IX adalah laki-laki, sedangkan
menurut catatan Budi, 1/7 dari total siswa dikelas IX selain dirinya adalah laki-
laki. Banyak siswa laki-laki kelas IX di sekolah mereka adalah…

4. OPERASI ALJABAR
Faktorisasi suku aljabar yang akan dijelaskan pada materi ini adalah yang
berbentuk:
a. (a  b)n

b. a 2  b2  (a  b)(a  b)

1. Bentuk (a  b)n
Misalkan a dan b bilangan real dan n bilangan asli, maka :
(a  b)n  (a  b)(a  b)(a  b)
  
n faktor

Untuk mencari koefisien dari suku-suku (a  b)n , dapat kita gunakan segitiga Pascal
berikut ini:
n=0 1
n=1 1 1
n=2 1 2 1
n=3 1 3 3 1
n=4 1 4 6 4 1
n=5 1 5 10 10 5 1
n=6 1 6 15 20 15 6 1
dst

Materi Olimpiade Matematika SMP - Persiapan OSN Page 44


Contoh
Tentukan koefisien suku x2y3 dari ( x  y)5

Tentukan koefisien suku x2y4 dari (2 x  y)6


Deret Binomial
Cara lain untuk menjabarkan bentuk (a  b)n dengan menggunakan deret binomial,
sebagai berikut:
n
(a  b)n   C (n, i)a n ibi  C (n, 0)a n  C (n, 1)a n 1b    C (n, n  1)abn 1  C (n, n)bn
i 0

dimana:
n!
 C (n, k ) 
k!(n  k )!

 n! 1.2.3.n
Contoh
Tentukan koefisien suku x4y3 dari ( y  3x)7

2. Bentuk a 2  b2  (a  b)(a  b)

Misalkan a dan b bilangan real, maka : a 2  b2  (a  b)(a  b)


Contoh
1. Tanpa menghitung yang rumit, sederhanakan bentuk 1252  1242
2. Tanpa menghitung yang rumit, sederhanakan bentuk 1234567892 –
123456790  123456788
3
 1 
3. Tentukanlah koefisien suku x1/2 dari   x
 x 
6
 3 
4. Tentukanlah koefisien suku x3/2 dan x3 dari  2 x  
 x
5. Jika a2 + a–2 = 4, carilah a6 + a –6

BENTUK PANGKAT DAN AKAR SERTA SIFAT-SIFATNYA


Definisi 1:
Bila a adalah bilangan riel dan n bilangan bulat positip, maka didefinisikan :

a n  a.a.a............a
yaitu a ada n kali(n factor).Jika a,b bilangan riel dan m,n bilangan bulat
positip,maka berlaku:

Materi Olimpiade Matematika SMP - Persiapan OSN Page 45


a. a m .a n  a mn e. (a.b) m  a m .b m
m
am a am
b.  a mn f.    m
a n
b b

c. a 
m n
 a mn g. a m 
1
am
d. a0  1

Keempat rumus di atas juga berlaku untuk bilangan rasional.Karena itu perlu

ditambah satu rumus lagi,yaitu


n
a  a1/ n

Karena juga berlaku untuk m,n bilangan rasional,maka sering disebut


bilangan berpangkat tak sebenarnya.Pada rumus diatas bilangan a disebut
bilangan pokok atau bilangan dasar.
1/ n
a
Coba bagaimana rumus dari:  
n
dan a.b .
b
Contoh Soal

1. Bentuk sederhana dari adalah ….

Penyelesaian

Materi Olimpiade Matematika SMP - Persiapan OSN Page 46


2. Misalkan , , , , dan . Berapakah nilai
?

a. 1 c. e.

b. 2 d. 3
Penyelesaian:

Latihan Soal
1. Jumlah semua bilangan real yang memenuhi
adalah …
2. Misalkan , , , , dan . Berapakah nilai
?
3. Jumlah dari solusi persamaan 4 x  15  3  2 x  1 adalah …
4. Tanpa menggunakan kalkulator, hitung
(1000000)(1000001)(1000002)(1000003)  1

5. Jika , maka nilai sama dengan …

a. 31/32 b.2/3 c. 1/3 d. 5/16


6. Misalkan a, b, c, d, dan e adalah bilangan-bilangan bulat sehingga
juga merupakan bilangan bulat.Jika diketahui bahwa nilai mutlak dari a, b, c,
d, dan e tidak lebih dari 2012 maka nilai terkecil yang mungkin dari a+ b+ c+
d+ e adalah....

Materi Olimpiade Matematika SMP - Persiapan OSN Page 47


5. PERSAMAAN dan PERTIDAKSAMAAN

Definisi persamaan linear


Persamaan linear adalah suatu kalimat matematika terbuka yang variabel
berderajat (berpangkat) satu.
Bentuk umum persamaan linear
Bentuk umum dari sebuah persamaan linear adalah:
ax = c (1 variabel)
ax + by = c (2 variabel)
ax + by + cz = d (3 variabel)
dimana a, b, c dan d konstanta.
Contoh
Contoh persamaan linear 1 , 2 dan 3 variabel adalah:
2x = 6
x + 5y = 10
2x + 6y = 4
x + 2y – z = 2
Penyelesaian (solusi) persamaan linear
Penyelesaian (solusi) persamaan linear ialah penentuan nilai dari setiap variabel
yang memenuhi persamaan tersebut dengan memperhatikan domain atau daerah
asalnya.
Contoh
Jika diberikan domain ialah himpunan bilangan bulat, maka
1. 2x = 6 hanya mempunyai satu solusi, yaitu: x = 3.
2. x + 5y = 10 mempunyai banyak solusi, yaitu : (x, y) = (0, 2) atau (10, 0)
atau (–10 , 4) atau (20, –2) dan lain-lain.
3. 2x + 6y = 4 mempunyai banyak solusi, yaitu : (x, y) = (2, 0) atau (–7,
3) atau (–10, 4) atau (8, –2) dan lain-lain.
Persamaan linier dua variable
Persamaan linier dua variable adalah persamaan yang memuat dua variable. Bentuk
umum persamaan linier dua variable adalah: ax+by +c=0 dengan a,b,c € R dan a ≠ 0
dimana, x dan y = variable, a dan b = koefisien, c = konstanta
Contoh: 2x – y + 4 = 0

Materi Olimpiade Matematika SMP - Persiapan OSN Page 48


Persamaan Kuadrat
Bentuk umum Persamaan Kuadrat
Sebuah persamaan kuadrat adalah sebuah persamaan polynomial derajat dua
yang mempunyai bentuk umum sebagai berikut: ax 2  bx  c  0, a  0 dan a,b dan
c bilangan-bilangan real..
a. Jenis-jenis akar persamaan kuadrat
Definisi diskriminan:
Jika diberikan persamaan kuadrat ax2  bx  c  0 , maka diskriminan dari
persamaan kuadrat
1. Akar-akar real berbeda (x1  x2)
Sebuah persamaan kuadrat memiliki akar-akar real berbeda x1  x2 jika nilai
diskriminan D positif (D > 0).
2. Akar real sama/kembar (x1 = x2)
Sebuah persamaan kuadrat memiliki akar-akar kembar x1=x2, jika nilai
diskriminan D nol (D=0).
3. Akar-akar kompleks
Sebuah persamaan kuadrat memiliki akar-akar kompleks x1,2 = p  qi (
i   1 ) jika nilai diskriminan D negatif (D < 0). Bilangan yang berbentuk
seperti ini disebut bilangan kompleks.
b. Solusi persamaan kuadrat
Untuk mencari solusi atau penyelesaian persamaan kuadrat dapat melalui berbagai
macam cara:
1. Faktorisasi
Faktorisasi adalah langkah pertama yang dapat kita lakukan untuk mencari akar-
akar persamaan kuadrat. Namun tidak semua persamaan kuadrat dengan mudah
kita faktorkan. Ada dua hal yang menyebabkan kita sulit memfaktorkan. Hal yang
pertama adalah berkenaan dengan keahlian kita memfaktorkan, sedangkan yang
kedua berkenaan dengan akar-akar yang tidak mudah difaktorkan (akar-akar
irrasional atau akar-akar kompleks).
Ada beberapa tipe persamaan kuadrat yang mudah kita faktorkan, yaitu:

Materi Olimpiade Matematika SMP - Persiapan OSN Page 49


o Bentuk ax2  c  0, b  0

Bentuk ax2  c  0 dapat kita faktorkan apabila a bernilai positif dan c bernilai
negatif atau sebaliknya.
o Bentuk ax2  bx  0, c  0
Bentuk ini sangat mudah juga difaktorkan, dengan menggunakan sifat
distributive sehingga diperoleh ax 2  bx  x(ax  b)  0 . Bentuk ini selalu
mempunyai nilai akar salah satunya yaitu x = 0
o Bentuk x2  bx  c  0, a  0

Bila bentuk x2  bx  c  0 mempunyai akar-akar rasional, maka bentuk ini dapat


difaktorkan menjadi bentuk ( x  q)( x  s)  0 dengan b  q  s dan c  qs .

o Bentuk ax2  bx  c  0, a  0
Bila bentuk ax2  bx  c  0 mempunyai akar-akar rasional, maka bentuk ini
(ax  p)(ax  q)
dapat difaktorkan menjadi bentuk  0 dengan b  p  q dan
a
ac  pq .
2. Kuadrat sempurna
Persamaan kuadrat ax2  bx  c  0 dapat diubah ke bentuk ( x  p)2  q ,
3. Rumus abc
Rumus abc sebenarnya adalah bentuk akhir dari cara kuadrat sempurna di atas
dan merupakan alternative terakhir jika kita sulit memfaktorkan persamaan

 b  b2  4ac
kuadrat tersebut. Rumus abc ditulis sebagai berikut: x1, 2  atau
2a
b D
x1, 2  D : nilai diskriminan = b2 –4ac.
2a
Hubungan akar-akar persamaan kuadrat dengan koefisien-koefisien
persamaan kuadrat
Hubungan akar-akar persamaan kuadrat dengan koefisien-koefisien dapat dilehat
sebagi berikut: Jika x1 dan x2 adalah akar-akar dari persamaan kuadrat maka
b c
x1  x 2   dan x1 x 2 
a a

Materi Olimpiade Matematika SMP - Persiapan OSN Page 50


PERTIDAKSAMAAN LINEAR SATU VARIABEL
Pertidaksamaan adalah sesuatu yang sering ditemukan dalam masalah-masalah
matematika. Konsep dasar dari pertidaksamaan adalah membandingkan nilai antar
bilangan yang ada (sifat keterurutan antar bilangan) dan aturan-aturan apa saja
yang dapat dijalankan untuk mencari solusi dari masalah pertidaksamaan yang
dihadapi.
Definisi
Misalkan a, b  R, maka:
a. a  b  0 jika a  b atau b  a
b. a  b  0 jika a  b atau b  a
a. Sifat-sifat dasar pertidaksamaan
Beberapa sifat yang penting dari pertidaksamaan antar lain adalah:
Misalkan a, b, c, d  R, maka:
a. Jika a  b maka a  c  b  c
b. Jika a  b dan c  d maka a  c  b  d
c. Jika a  b dan c  0 maka ac  bc
Jika a  b dan c  0 maka ac  bc
d. Jika a  0 maka 1 / a  0
Jika a  0 maka 1 / a  0
e. Jika a  b dan b  c maka a  c
f. Jika a  b dan b  a maka a  b
g. Untuk setiap a  R berlaku a 2  0
h. Jika ab  0 maka:
o a  0 dan b  0 atau
o a  0 dan b  0
i. Jika ab  0 maka:
o a  0 dan b  0 atau
o a  0 dan b  0

b. Menyelesaikan masalah pertidaksamaan


Untuk menyelesaikan masalah pertidaksamaan, kita gunakan sifat-sifat
pertidaksamaan sebelumnya sehingga diperoleh hasil yang benar-benar diinginkan.

Materi Olimpiade Matematika SMP - Persiapan OSN Page 51


Pertidaksamaan Linier
Pertidaksamaan adalah kalimat terbuka yang memuat atau menyatakan hubungan
tidak sama atau tanda ketaksamaan (<, >, ≥ dan ≤). Sifat-sifat pertidaksamaan, yaitu:
1. Di dalam pertidaksamaan dapat dilakukan penambahan atau pengurangan
dengan bilangan yang sama.
2. Di dalam pertidaksamaan juga dapat dilakukan perkalian dan pembagian
dengan suatu bilangan yang sama.
3. Setiap suku yang berpindah ruas, akan diikuti oleh perubahan tandanya,
misalnya dari positif ke negative atau sebaliknya.
Pertidaksamaan linier adalah kalimat terbuka yang mengandung variable
berpangkat satu pada salah satu atau kedua ruasnya dan menyatakan hubungan
dengan tanda <, >, ≥ dan ≤. Bentuk umumnya:

ax + b < 0 ax + b ≤

ax + b > 0 ax + b ≥
Dimana a ≠
Contoh:
2x – 7 < 4x – 2 ini merupakan pertidaksamaan linier dengan satu variable
yaitu x dan dinyatakan dengan kecil ( < ).
Contoh Soal
1. Jika dan , maka

Penyelesaian:

…………….(1)

……………..(2)

Materi Olimpiade Matematika SMP - Persiapan OSN Page 52


……………..(2)

2. Misalkan , sehingga . Persamaan kuadrat


memiliki dua akar real bila …
a. c. atau

b. d.

Penyelesaian:
Syarat persamaan kuadrat memiliki dua akar yang real adalah ,

maka

Jadi persamaan kuadrat tersebut memiliki dua akar real bila

Latihan Soal
1. Tentukan himpunan penyelesaian dari masing-masing pertidaksamaan berikut
ini, dimana xR :
b. –4 < 3x + 2 < 5
c. –3 < 4x – 9 < 11
d. 4 < 5 – 3x < 7
e. 2 + 3x < 5x + 1 < 16
f. 2x – 4  6 – 7x  3x + 6

Materi Olimpiade Matematika SMP - Persiapan OSN Page 53


2. Tentukan himpunan penyelesaian dari masing-masing pertidaksamaan berikut
ini :
a. 3x – 2  4x + 6, x bilangan bulat
b. 2(3x – 4) + 5 > 6(2x + 1) + 3, x bilangan bulat
c. 3x – 1  8 + 2x, x bilangan asli
d. 3y – (2 + 5y)  16, x bilangan asli
e. 5x + 1 > 8x + 7, x bilangan asli
f. 5(5 – 3x) – (–x + 6) > 8, x bilangan cacah
g. 4  3x < 7x + 1 < 22, x bilangan cacah
3. Tentukan himpunan penyelesaian dari sistem pertidaksamaan
a. 5x  25  25x  5 dan 8x  45  15x  10 , x bilangan bulat
b. 3x + 7 > 1 dan 2x + 1 < 3
c. 3x + 7 > 1 dan 2x + 1 > 4
d. 3x + 7 > 1 dan 2x + 1 < 4
4. Carilah interval dari nilai x yang memenuhi pertidaksamaan:
x  15 x  12 9  3x x  11
 dan 
6 3 2 5
5. Diketahui  2  x  5 , 3  y  12 dan 0,3  w  5 , berapakah

 Nilai terkecil yang mungkin dari (x+y)


 Nilai terbesar yang mungkin dari (y – x) dan y/w.

6. SISTEM PERSAMAAN LINEAR DUA PEUBAH


Materi kali ini salah satu materi yang penting untuk dipahami, karena banyak soal
matematika yang harus dimodelkan terlebih dahulu untuk mempermudah mencari
solusinya. Di antara model-model tersebut bisa merupakan suatu sistem persamaan
linear atau sistem persamaan non linear.
Definisi sistem persamaan linear (SPL)
Sistem persamaan linear ialah kumpulan dari persamaan-persamaan linear yang
saling berhubungan untuk mencapai tujuan tertentu.
Bentuk umum sistem persamaan linear
SPL dengan 2 variabel dari 2 persamaan, mempunyai bentuk umum sebagai berikut:

Materi Olimpiade Matematika SMP - Persiapan OSN Page 54


 a1 x  b1 y  c1

a 2 x  b2 y  c 2
SPL dengan 3 variabel dari 3 persamaan, mempunyai bentuk umum sebagai berikut:
 a1 x  b1 y  c1 z  d 1

a 2 x  b2 y  c 2 z  d 2
a x  b y  c z  d
 3 3 3 3

Penyelesaian (solusi) SPL


Penyelesaian SPL dapat dilakukan dengan berbagai macam cara, yaitu: substitusi,
eliminasi, metode cramer, eliminasi Gauss, eliminasi Gasuss-Jordan dan berbagai
macam cara lain dengan himpunan penyelesaiannya adalah { (x , y) }, dimana (x, y)
merupakan pasangan terurut dari variabel-variabel pada SPL dengan 2 variabel.
Namun metode penyelesaian yang kita pelajari sekarang adalah metode substitusi,
eliminasi dan cramer.
a. Metode eliminasi
Metode eliminasi adalah salah satu metode yang sederhana, yaitu dengan cara
menghilangkan suatu atau beberapa variabel dari semua persamaan yang lain,
sehingga diperoleh nilai dari variabel yang kita inginkan. Setelah itu
mensubstitusikan nilai variabel yang telah kita peroleh tersebut ke dalam
persamaan-persamaan lain sehingga diperoleh nilai variabel-variabel lainnya.
b. Metode substitusi
Metode sustitusi adalah salah satu metode lain yang sangat sederhana. Prinsip
yang dilakukan metode ini adalah dari salah satu persamaan linear kita buat nilai
eksplisit salah satu variabelnya terhadap variabel lainnya. Kemudian susbtitusi
nilai eksplisit variabel yang didapat ke dalam persamaan linear yang lainnya,
sehingga diperoleh nilai variabel yang diinginkan.
c. Metode cramer
Pada bagian ini akan diperkenalkan metode lain dalam mengerjakan SPL, yaitu
metode cramer.
Misalkan diberikan sebuah SPL 2 variabel dari 2 persamaan sebagai berikut:
 a1 x  b1 y  c1

a 2 x  b2 y  c 2
dimana a1b2  a2b1  0 ,

Materi Olimpiade Matematika SMP - Persiapan OSN Page 55


maka solusi x dan y dari SPL di atas adalah:
c1 b1
c b cb c b
 x 2 2  1 2 2 1
a1 b1 a1b2  a2b1
a2 b2
a1 c1
a c ac a c
 y 2 2  1 2 2 1
a1 b1 a1b2  a2b1
a2 b2
Latihan Soal
1. Tentukan nilai terbesar z yang memenuhi x + y + z = 5 dan xy + yz + xz = 3
2. Untuk nilai b yang manakah persamaan 2007x2 + bx + 7002 = 0 dan 7002x2 + bx
+ 2007 = 0 mempunyai akar persekutuan
3. Diberikan bahwa :
x1 + 4x2 + 9x3 + 16x4 + 25x5 + 36x6 + 49x7 = 1
4x1 + 9x2 + 16x3 + 25x4 + 36x5 + 49x6 + 64x7 = 12
9x1 + 16x2 + 25x3 + 36x4 + 49x5 + 64x6 + 81x7 = 123
Tentukan 16x1 + 25x2 + 36x3 + 49x4 + 64x5 + 81x6 + 100x7
4. Tentukan nilai bilangan real a, b, c yang memenuhi :
abc  ab  bc  ca  a  b  c  1
bcd  bc  cd  db  b  c  d  9


cda  cd  da  ac  c  d  a  9
dab  da  ab  bd  d  b  a  9

5. Selesaikan sistem persamaan :


 x 2  yz  3
 2
 y  xz  4
 z 2  xy  5

6. Selesaikan sistem persamaan :

 y  ( x  8)( x  2)
2 2

 2

 y  (8  4 x) y  (16  16 x  5 x )  0
2

7. Selesaikan sistem persamaan :

x+y= 4z  1
y+z= 4x  1
z+x= 4y  1

Materi Olimpiade Matematika SMP - Persiapan OSN Page 56


8. Jika a, b, c , d bilangan real sehingga a 2 + b2 = 1, c2 + d2 = 1 dan ac + bd = 0.
Tentukan nilai cb + cd = 0
9. Carilah solusi bilangan bulat persamaan simultan |x| + |y| = 3, |x – y| = 3
10. Misalkan a, b, c bilangan real berbeda yang memenuhi
a3 = 3(b2 + c2) – 25
b3 = 3(c2 + a2) – 25
c3 = 3(a2 + b2) – 25
Tentukan nilai abc

7. BARISAN DAN DERET


Pendahuluan
 Salah satu hal penting di dalam matematika adalah mempelajari pola.
 Jika diketahui beberapa bilangan, maka kita ingin mengetahui kelanjutan dari
bilangan tersebut.
 Langkah pertama adalah menyelidiki tentang perilaku bilangan tersebut dan
kemudian kita mencoba melanjutkan pola bilangan tersebut.
 Penyelidikan dilakukan dengan menggunakan operasi bilangan yang ada,
yaitu penjumlahan dan perkalian.

a. RUMUS – RUMUS DARI POLA BILANGAN


1. Jumlah dari n bilangan asli ganjil yang pertama = n2
2. Jumlah dari n bilangan asli genap yang pertama = n(n + 1)
3. Pada pola bilangan segitiga Pascal , jumlah bilangan pada baris ke n = 2n – 1
4. Bilangan persegi yang ke n = n2
5. Bilangan persegi panjang yang ke-n = n(n + 1)
1
6. Bilangan segitiga yang ke n = n (n + 1)
2
7. Pasangan bilangan yang hasil penjumlahannya sama dengan hasil
perkaliannya adalah :
n
n dan dengan n > 1
n 1
n n
n+ =n×
n 1 n 1

Materi Olimpiade Matematika SMP - Persiapan OSN Page 57


b. BARISAN DAN DERET ARITMATIKA
Bentuk umum : a, a + b, a + 2b, a + 3b, … , a + (n + 1)b
U1, U2, U3, U4,… ,Un
RUMUS-RUMUS PENTING
1. Un = a + (n – 1)b
2. Sn = ½ n (a + Un) → dipakai jika suku terakhir diketahui
Sn = ½ n(2a + (n – 1)b) → dipakai jika suku terakhir tidak diketahui
a  Un
3. U1 =
2
4. Un = Sn – Sn – 1 → dapat dipakai dalam deret aritmatika atau deret geometri
5. b = Un – Un – 1
dengan Un = Suku ke-n
Sn = Jumlah n suku pertama
a = Suku awal = U1
b = Beda
c. DERET GEOMETRI
Bentuk Umum : a, ar, ar2, ar3, …, arn-1
U1, U2, U3, U4,…, Un

RUMUS-RUMUS PENTING
Jika Un = Suku ke n; Ut = Suku tengah; Sn = Jumlah n
Un
Suku pertama; a = U1 = Suku awal; r = ratio r =
U n 1

Maka: Un = arn-1
a (r n  1) a (1  r n )
Sn = untuk r > 1 U1 = aU n dan Sn = untuk r < 1 Un = Sn – Sn-1
r 1 1 r

Contoh Soal
1. Nilai dari adalah …
Penyelesaian:
identik
dengan pola , sehingga

Materi Olimpiade Matematika SMP - Persiapan OSN Page 58


Pola merupakan jumlah suku deret
aritmatika dengan beda 4.

2. Agar bilangan sedekat mungkin ke 2.004,


haruslah
Penyelesaian:
Pola sesuai dengan deret geometri yaitu
sehingga dekat dengan 2004
dekat dengan ( lebih dekat dari pada dengan
2004)

Latihan Soal

1. Berapakah hasil perkalian dari ?

2. Nilai dari adalah …


3. Diketahui X0 = 1, X1 = 2, sedangkan untuk n  2 didefinisikan
X N 1  2 X N  2
Xn  , maka X2 + 2X3 = ... .
2 X N 1  X N  2

4. Diketahui N = 9 + 99 + 999 + …+ 9999…9.


2014angka
aa
Hitunglah jumlah dari semua angka pada N
5. Ditanyakan digit-digit apa yang muncul pada perkalian : 33
 3  66
...  6 .
...
666 3'an 666 6 'an

6. Jika x adalah jumlah 99 bilangan ganjil terkecil yang lebih besar dari 2011
dan y adalah jumlah 99 bilangan genap terkecil yang lebih besar dari 6, maka
x + y = ...

Materi Olimpiade Matematika SMP - Persiapan OSN Page 59


7. adalah barisan bilangan yang memenuhi

untuk n = 1,2,3, … jika = 100 tentukan nilai dari

8. Diketahui . tentukan nilai dari

9. Diketahui adalah suku-


suku suatu barisan bilangan, maka adalah….

LATIHAN ALJABAR
a. Himpunan
a.1. Himpunan bagian
a.2. Operasi himpunan
1. OSP 2005
H adalah himpunan yang didefinisikan oleh { x  B| x2  10, x - 1 < 2 } dengan
B adalah himpunan bilangan bulat. Banyaknya himpunan bagian tak kosong
dari H adalah
2. OSP 2008
Seorang peternak memiliki 114 hewan peliharaan yang terdiri dari kuda,
sapi, kambing, ayam dan bebek. Banyak hewan berkaki empat adalah 8 lebih
sedikit dibandingkan hewan berkaki dua. Sedangkan sapi miliknya adalah 3
lebih banyak dibanding kuda, tetapi 20 lebih sedikit dibanding kambing. Di
samping itu ayam miliknya adalah 13 lebih sedikit dibanding bebek. Banyak
sapi dan ayam milik peternak tersebut adalah...
3. OSP 2013

b. Relasi dan Fungsi


b.1. Pengertian relasi dan fungsi
b.2. Grafik fungsi dan transformasinya
b.3. Operasi fungsi
b.4 Fungsi kuadrat dan sifat- sifatnya

Materi Olimpiade Matematika SMP - Persiapan OSN Page 60


1. OSP 2006
Diberikan fungsi kuadrat f ( x)  ax 2  3x  c. Jika f (1)  4 dan
f (2)  7, maka f (1)  ...
2. OSP 2007
x
Diketahui fungsi bilangan real f ( x)  dengan x  1. Nilai dari
1 x
f (2007)  f (2006)  ...  f (3)  f (2)  f (1 / 2)  f (1 / 3)  ...  f (1 / 2007) Adalah ...
3. OSP 2008

4. OSP 2009

5. 2010

6. OSP 2011
Jika f adalah fungsi sehingga f(xy)= f(x–y) dan f(6) =1, maka f(–2) – f(4) =….
7. OSP 2013

8. OSN 2013

Materi Olimpiade Matematika SMP - Persiapan OSN Page 61


9. OSP 2014

10 OSP 2014

c. Perbandingan
c.1. Ukuran benda dengan skala
c.2. Perbandingan senilai
c.3. Perbandingan berbalik nilai

1. OSP 2003
Kapasitas tangki bahan bakar suatu mobil adalah 40 liter. Setiap menempuh
perjalanan sejauh 100 km, mobil tersebut menghabiskan 7,7 liter bahan
bakar. Suatu waktu, mobil tersebut digunakan untuk pergi dari Bandung ke
Yogyakarta yang jaraknya 428 km, Ketika memulai perjalanan, tangki mobil
tersebut terisi penuh bahan bakar. Dalam satuan liter terdekat, banyaknya
bahan bakar yang tersisa ketika tiba di Yogyakarta adalah……
2. OSP 2012
Jumlah tiga bilangan adalah 19. Jika bilangan pertama dan bilangan kedua
masing-masing dikurangi 1, maka diperoleh dua bilangan dengan rasio 1 : 3.
Jika bilangan kedua dan ketiga masing-masing ditambah 3, maka diperoleh
dua bilangan dengan rasio 5 : 6. Selisih bilangan terbesar dan terkecil adalah
….

Materi Olimpiade Matematika SMP - Persiapan OSN Page 62


3. OSP 2012
Dari hasil sensus diketahui bahwa penduduk suatu kota tak lebih dari
10.000 orang dan anak-anak 20% lebih banyak dari penduduk dewasa. Jika
anak laki-laki 10% lebih banyak dari anak perempuan, serta di antara
penduduk dewasa terdapat 15% lebih banyak perempuan, tentukan jumlah
terbesar yang mungkin dari penduduk kota tersebut.
4. OSP 2013
Suatu yayasan menyumbangkan 144 buku ke 4 sekolah. Banyak buku yang
diterima untuk setiap sekolah tidak sama. Selisih buku yang diterima
sekolah A dan B adalah 16. Selisih buku yang diterima sekolah B dan C
adalah 12. Selisih buku yang diterima sekolah C dan D adalah 8. Sekolah A
menerima buku paling sedikit dibandingkan dengan yang titerima sekolah
lain. Jika sekolah D menerima buku 2 kali lebih banyak dari pada buku yang
diterima sekolah A, tentukan banyak buku yang diterima masing-masing
sekolah.

d. Operasi aljabar
d.1. Operasi aljabar melibatkan bilangan berpangkat bulat dan bentuk
akar
d.2. Operasi aljabar melibatkan bilangan rasional

1. OSP 2003
Jika a + b = 1, dan a2 + b2 = 5, maka a3 + b3 = ... .
2. OSP 2003
Dalam suatu kelas, 3/5 bagian siswanya adalah wanita. Kedalam kelas
tersebut ditambahkan 5 siswa pria dan 5 siswa wanita. Sekarang, 3/7 bagian
siswanya adalah pria. Banyak siswa dalam kelas tersebut mula-mula adalah
... .
3. OSP 2004
Sebuah semangka yang beratnya 1 kg mengandung 93% air. Sesudah
beberapa lama dibiarkan di bawah sinar matahari, kandungan air semangka
itu turun menjadi 90%. Berapakah berat semangka sekarang?

Materi Olimpiade Matematika SMP - Persiapan OSN Page 63


4. OSP 2006
a 2 b2 a b
Misalkan a dan b bilangan real positif. Jika 2
 2  7, maka   ...
b a b a

5. OSN 2007
Jika a  3 p , b  3q , c  3r , dan d  3 s , dan jika p, q, r, dan s adalah bilangan
asli, berapakah nilai terkecil dari pqrs yang memenuhi

a 2  b3  c 5  d 7 ?

6. OSP 2008
Nilai x yang memenuhi persamaan adalah …

7. OSP 2008
Jika maka nilai adalah …

8. OSP 2009

9. OSP 2009

10. OSP 2010

Materi Olimpiade Matematika SMP - Persiapan OSN Page 64


11 OSP 2007
Jumlah dari setiap tiga bilangan asli yang terletak pada garis lurus pada
Gambar 1 selalu sama. Nilai dari p+q+r+s yang mungkin adalah ... A. 63 B. 69 C.
72 D. 84 E. 90

27 45

q 51 s

18 3
6

12. OSP 2010


Sejumlah siswa mengikuti ujian seleksi OSN tahun 2010 tingkat provinsi,
ternyata didapatkan data bahwa sebanyak 64 siswa yang lulus adalah wanita,
3/5 peserta yang lulus adalah laki-laki, Sedangkan jumlah peserta laki-laki
lulus adalah 4 kali lebih banyak dari pada jumlah peserta laki-laki yang tidak
lulus, serta jumlah peserta yang tidak lulus adalah 40 siswa. Berapa persenkah
jumlah peserta wanita yang mengikuti ujian seleksi tersebut?

13. OSP 2011


Diketahui Budi adalah siswa laki-laki dan Wati adalah seorang siswa
perempuan. Saat ini mereka duduk di kelas IX pada suatu sekolah. Mereka
mencatat banyak siswa kelas IX di sekolah mereka. Wati mencatat, 3/20 dari
total siswa di kelas IX adalah laki-laki, sedangkan menurut catatan Budi, 1/7
dari total siswa dikelas IX selain dirinya adalah laki-laki. Banyak siswa laki-
laki kelas IX di sekolah mereka adalah…

14. OSP 2012

Materi Olimpiade Matematika SMP - Persiapan OSN Page 65


15. OSP 2012
Diketahui sebuah bilangan rasional positip kurang dari 1 yang dinyatakan
dalam pecahan biasa dalam bentuk paling sederhana. Jika hasil kali pembilang
dan penyebut dari bilangan rasional tersebut adalah 20! = 1· 2· 3· … · 20,
tentukan semua bilangan yang dimaksud.

16. OSP 2013

e. Persamaan dan pertidaksamaan


e.1. Persamaan dan Pertidaksamaan linear satu peubah
e.2. Persamaan linear dua peubah
e.3. Persamaan kuadrat satu peubah
1. OSP 2003
Misalkan a dan b adalah dua bilangan tertentu. Jika a2 + (a + b) = a(b – a) + x,
maka x = ... .
2. OSP 2003
1 1 1
Jika   maka x = ... .
6 12 x
3. OSP 2003
x 1
Tentukan semua bilangan real x yang memenuhi pertaksamaan 2
x 1

4. OSN 2003
Buktikan bahwa jika a > 2 dan b > 3 maka ab + 6 > 3a + 2b.
5. OSN 2003
Perhatikan kumpulan pernyataan berurut berikut.
Diketahui bahwa
Karena maka
Sehingga
Akibatnya,

Materi Olimpiade Matematika SMP - Persiapan OSN Page 66


– –
(x – 1) (x + 1) = (x – 1) . 1
Dengan aturan pencoretan, diperoleh
x+1=1
1+1=1
2= 1
Pertanyaannya.
a. Kalau 2 = 1, maka setiap bilangan asli pasti sama dengan 1. Tunjukkan!
b. Hasil 2 = 1 adalah sesuatu yang tidak mungkin. Tentu ada yang salah di
dalam argumen di atas? Dimanakah letak kesalahannya? Mengapa itu kamu
anggap salah?
6. OSP 2004
Jika a + b = 1, b + c = 2, dan c + a = 3, maka a + b + c = ....
7. OSP 2004
Untuk bilangan real a dan b sembarang, buktikan bahwa a 2  b 2  2 (a  b)  2
8. OSP 2004
 x, jika x  0
Untuk bilangan real x didefinisikan x   , cari semua x yang
 x, jika x  0
memenuhi x 2  2 x  3  0 .

9 OSP 2011

10. OSP 2005


Bilangan 45 dapat dinyatakan sebagai selisih dari bilangan kuadrat, yakni
a 2  b 2 , dengan a dan b adalah bilangan asli. Semua pasangan bilangan asli a
dan b yang memenuhi a 2  b 2  45 adalah
11. OSP 2005
16 dapat dinyatakan sebagai 3x + 7y sebab jika x diganti dengan 3 dan y
diganti dengan 1, maka diperoleh 3.3+7.1 yang bernilai 16. 7 (tujuh)
bilangan antara 100 dan 122 yang dapat dinyatakan ke dalam bentuk 6x + 9y
adalah

Materi Olimpiade Matematika SMP - Persiapan OSN Page 67


12. OSP 2005
2 1
Bilangan-bilangan real x yang memnuhi x 2  2 x  1   2  0 adalah
x x
13. OSP 2006
Tentukan nilai m (real) agar persamaan
(2 x 2  2mx  (m  1))( x 2  mx  1)  0
mempunyai tepat dua solusi real.
14. OSP 2006
Himpunan penyelesaian dari
2( x  5) 1
1  4(3  x) 
7 7
dengan x   adalah …
15. OSP 2007
Semua pasangan bilangan bulat x dan y yang memenuhi persamaan
y 2 ( x  1)  1576  x 2 adalah …
16. OSP 2007
Diberikan dua bilangan bulat yang berjumlah 37. Jika bilangan yang lebih
besar dibagi dengan bilangan yang lebih kecil, maka hasil baginya adalah 3
dan sisanya 5. Selisih kedua bilangan tersebut adalah ...
A3 B. 5 C. 8 D. 21 E. 29
17. OSP 2008

18. OSP 2009

19. OSP 2009

Materi Olimpiade Matematika SMP - Persiapan OSN Page 68


20. OSN 2009
Dua persamaan kuadrat memiliki akar-akar bilangan asli. Persamaan
kuadrat yang pertama memiliki akar-akar a dan b, sedangkan persamaan
kuadrat yang kedua memiliki akar-akar b dan c , c ≠ a . Jika a, b, dan c
merupakan bilangan-bilangan prima kurang dari 15, ada berapa macam
pasangan persamaan kuadrat yang mungkin memenuhi persyaratan di atas?
21. OSP 2010
Akar-akar persamaan kuadrat – adalah real dan lebih
besar 1, Berapakah nilai p + q?
22. OSP 2011

23. OSN 2011

24. OSP 2012

25. OSP 2012

26. OSP 2013

27. OSP 2013


Diketahui parabola melalui titik (3; 4) dan (3; 16), serta
tidak memotong sumbu-X. Carilah semua nilai absis yang mungkin untuk
titik puncak parabola tersebut.

Materi Olimpiade Matematika SMP - Persiapan OSN Page 69


28 OSP 2014

29. OSP 2014

f. Sistem persamaan linear


1. OSN 2003
Diketahui a + b + c = 0. Tunjukkan bahwa

2 OSP 2008

2. OSP 2004
Jika selisih dua bilangan adalah 2 dan selisih kuadrat dua bilangan itu adalah
6, mak hasil tambah dua bilangan itu adalah ....
3. OSN 2004
Diberikan dua bilangan real positif x dan y sehingga xy  1.
1 1
Tentukan nilai minimum dari 4
 4!
x 4y
4. OSN 2005
Tentukan semua pasangan bilangan bulat (x, y) yang memenuhi sistem
persamaan:
x( y  1)  y 2  1
y ( x  1)  x 2  1

Materi Olimpiade Matematika SMP - Persiapan OSN Page 70


5. OSP 2006
Diketahui kurva y  x 2  7 x  2 dan garis y  2 x  8 saling berpotongan di
titik A dan titik B. Jarak antara titik A dan titik B adalah...
6. OSP 2006
Diketahui a 2  b 2  1 dan x 2  y 2  1. Lanjutkan proses aljabar berikut.

(a 2  b 2 )( x 2  y 2 )  (ax  by) 2  
a. Hubungan apakah yang bisa disimpulkan antara ax  by dengan 1?
b. Mengapa?
7. OSN 2006
Diketahui sistem persamaan empat variabel:
23x  47 y  3z  434
47 x  23 y  4w  183
19 z  17 w  91
dengan x, y, z, dan w adalah bilangan bulat positif. Tentukan nilai dari
(13x  14 y)3  (15z  16w)3 .
8. OSP 2007
Jika sistem persamaan x 7 y 5  r dan x 4 y 3  s dengan x, y, r dan s adalah
bilangan positif mempunyai penyelesaian x  r a s b dan y  r c s d , maka hasil
dari a+b+c+d adalah ...
A. 19 B. 2 C. 1 D. 7/12 E. ½
9. OSN 2007
1 Ax 2
Diketahui bahwa x   7. Tentukan nilai A agar 4 2 5 .
x x  x 1 6

10. OSP 2008

Materi Olimpiade Matematika SMP - Persiapan OSN Page 71


11. OSN 2008
Misalkan himpunan
A  {( x, y) | 3x  5 y  15, x  y 2  25, x  0, x, y bilangan bulat} . Tentukan
semua pasangan ( x, zx)  A dengan syarat z merupakan bilangan bulat.
12 OSP 2014

g. Barisan dan deret

1. OSP 2003
Diketahui X0 = 1, X1 = 2, sedangkan untuk n  2 didefinisikan
X  2 X N 2
X N  N 1 , maka X2 + 2X3 = ... .
2 X N 1  X N  2
2. OSP 2003
Diketahui suatu barisan
,
sehingga beberapa unsur pertamanya sebagai berikut U(1) = 6, U(2) = 18,
U(3) = 38, U(4) = 68, U(5) = 110. Tentukan nilai dari U(100).
3. OSN 2003
Diketahui bahwa . Untuk didefinisikan bahwa
. Tentukanlah

4. OSP 2004
1 1 1 1 1
 2  3  2  ....  = ....
1 1 2  2 3  3 4  4
2
2004  2004
2

5. OSN 2004
Suatu bola jika dijatuhkan tegak lurus ke tanah dari suatu ketinggian, maka
ia akan memantul kembali tegak lurus sepanjang sepertiga tinggi semula,
turun kembali tegak lurus dan memantul kembali sepertitiga tingginya, dan
seterusnya. Jika jarak yang ditempuh bola tersebut pada saat menyentuh
tanah yang keempat sama dengan 106 meter, dari ketinggian berapakah
bola tersebut dijatuhkan?

Materi Olimpiade Matematika SMP - Persiapan OSN Page 72


6. OSP 2006
Jika n adalah bilangan asli, maka bentuk paling sederhana dari perkalian
 1  1  1  1
1  2 1  2 1  2   1  2 
 2  3  4   n 
7. OSP 2006
Nilai dari 12  22  32  42    20052  20062  ...
8. OSN 2006
Diketahui N  9  99  999    9999
 9. Tentukan nilai N.

121angka

9. OSP 2007
3 6 9 12 96
Hasil dari penjumlahan     ...  adalah ...
20 60 126 224 24.192
10. OSP 2008
Jika , maka 5 angka terakhir dari

adalah …
11. OSP 2008
Diketahui

Bilangan terdekat dengan adalah …


12. OSN 2008
Suatu barisan bilangan real mempunyai suku-suku yang didefinisikan
 ar n 1 jika n  4m  3 atau n  4m  2
sebagai berikut. u n  
 ar
n 1
jika n  4m  1 atau n  4m

Dengan a  0, r  0, dan m bilangan bulat positif. Buktikan bahwa jumlah

a(1  r  r 2009  r 2010)


smeua suku ke-1 sampai dengan suku ke-2009 adalah
1 r2
13. OSP 2009
Nilai dari

Adalah …

Materi Olimpiade Matematika SMP - Persiapan OSN Page 73


14. OSP 2009

15. OSP 2010


Diberikan suatu barisan bilangan 1, 5, 6, 25, 26, 30, 31 , ... yang terdiri dari
barisan bilangan pemangkatan 5 atau jumlah bilangan-bilangan berbeda
hasil pemangkatan 5. Perhatikan bahwa
. Nilai suku ke-100 pada barisan tersebut adalah …
16. OSP 2011
Jika x adalah jumlah 99 bilangan ganjil terkecil yang lebih besar dari 2011
dan y adalah jumlah 99 bilangan genap terkecil yang lebih besar dari 6,
maka x + y = ...
17. OSN 2011

18. OSP 2012

19. OSP 2012

Materi Olimpiade Matematika SMP - Persiapan OSN Page 74


C. GEOMETRI

3. Menerapkan dan Geometri 1. Garis dan sudut


menganalisis konsep a. Kedudukan dua garis
garis dan sudut, bangun b. Sifat-sifat garis
datar, serta teorema c. Jarak dua titik dan jarak
phytagoras titik ke garis
d. Sifat-sifat sudut
Menerapkan konsep 2. Bangun datar
transformasi a. Sifat-sifat bangun datar
Menerapkan konsep dan b. Keliling dan luas
sifat bangun datar permukaan bangun datar
c. Kesebangunan dan
kekongruenan
3. Teorema Phythagoras
4. Transformasi (refleksi,
translasi, rotasi, dilatasi)
5. Bangun ruang
Luas permukaan, volume,
dan jaring-jaring dari kubus,
balok, tabung, prisma,
kerucut, limas, dan bola

RANGKUMAN MATERI
Dua buah garis dapat saling memiliki hubungan. Hubungan yang dimaksud dapat
berupa perpotongan dua buah garis atau tidak berpotongan sama sekali. Hubungan
antara dua buah garis yang akan kita lihat adalah hubungan saat kedua garis
tersebut sejajar (//). Saat dua buah garis berkedudukan sejajar, maka jarak antara
kedua garis tersebut selalu dipertahankan sama pada posisi dimana saja. Hubungan
kedua, yaitu tegak lurus ( ). Pada saat ini kedua buah garis berhubungan saling
membentuk sudut 900.

Garis
Garis adalah kumpulan titik-titik yang banyaknya tak terhingga yang jaraknya
sangat dekat dan memanjang ke dua arah.
contoh:

Garis lurus Garis Melengkung

Materi Olimpiade Matematika SMP - Persiapan OSN Page 75


1. Kedudukan dua garis
a. Sejajar a

Garis a sejajar dengan garis b ( a // b)


Dua buah garis dikatakan sejajar jika kedua garis tersebut tidak
berpotongan dan jarak kedua garis selalu tetap, serta terletak pada satu
bidang.
b. Berpotongan
a
O

b
Dua buah garis lurus hanya dapat berpotongan pada satu titik. Garis a dan
garis b berpotongan di titik O.
c. Berimpit

A C D B
Dua garis yang berimpit merupakan dua garis yang terletak pada satu garis
lurus, sehingga dua garis tersebut hanya tampak satu garis lurus. Garis AB
dan garis CD berimpit sehingga keduanya terletak pada satu garis
Sudut
1. Pengertian sudut
Sudut adalah daerah yang dibentuk oleh dua sinar garis yang bertemu pada satu
titik pangkal.
B O = titik sudut
OA dan OB = sinar garis/titik sudut
∠ AOB = ∠O = , nama sudut

O A

Materi Olimpiade Matematika SMP - Persiapan OSN Page 76


2. Jenis-Jenis Sudut
a. Sudut Lancip

Sudut yang besarnya lebih dari 00 dan kurang dari 900

00 < a0 < 900

a0
b. Sudut Tumpul

Sudut yang besarnya lebih dari 900 dan kurang dari 1800
0 0 0
90 < a <180

a
0
c. Sudut Siku-siku

Sudut yang besarnya 900

a0 = 900

a
0

d. Sudut Lurus

Sudut yang besarnya 1800


0
e. Sudut Refleks

Sudut yang besarnya antara 1800 sampai 3600 a0 = 1800

1800 < a0 < 3600


a
o

Materi Olimpiade Matematika SMP - Persiapan OSN Page 77


f. Sudut Putaran Penuh

Sudut yang besarnya 3600, disebut juga dengan sudut satu putaran penuh

3. Hubungan Antar Sudut


a. Sudut berpelurus (suplemen)

dua sudut yang jumlah sudutnya 1800


C

A O B

∠ AOC + ∠ BOC = 1800 ∠ AOC dan ∠ BOC saling berpelurus

b. Sudut berpenyiku (komplemen)


dua sudut yang jumlah sudutnya 900
B C

O A

∠ AOC + ∠ BOC = 900 ∠ AOC dan ∠ BOC saling berpenyiku

c. Sudut bertolak belakang


Sudut-sudut yang bertolak-belakang mempunyai sudut yang sama
∠A = ∠B ; ∠C = ∠D

A
C B
D

Materi Olimpiade Matematika SMP - Persiapan OSN Page 78


Selain itu kedua buah garis bisa saja memiliki hubungan saling berpotongan,
sehingga membentuk suatu sudut.
• Sudut sehadap sama besar
– A1 = B1 , A2 = B2
– A3 = B3 , A4 = B4
• Sudut dalam berseberangan sama besar
– A4 = B2 , A3 = B1
• Sudut luar berseberangan sama besar
– A2 = B4 , A1 = B3
• Sudut dalam sepihak
jumlah sudutnya 1800
– A4 = B1 , A3 = B2
• Sudut luar sepihak
jumlah sudutnya 1800
– A1 = B4 , A2 = B3

BANGUN DATAR
Segitiga
Sifat-sifat segitiga:
1. Jumlah besar sudutnya = 1800
2. Jumlah panjang dua sisi selalu lebih besar dari sisi ketiga
3. Panjang sisi-sisi segitiga berbanding lurus dengan besar sudutnya.
4. Pada segitiga sama sisi semua sisinya sama panjang dan semua sudutnya
sama besar yaitu 600.
5. Pada segitiga sama kaki dua sisinya sama panjang dan dua sudut kakinya
sama besar.
6. Besarnya sudut luar suatu sudut segitiga sama dengan jumlah besar dua
sudut yang lainnya.
7. Jika dua buah segitiga tingginya sama maka perbandingan luasnya sama
dengan perbandingan alasnya.
8. Jika dua buah segitiga alasnya sama maka perbandingan luasnya sama
dengan perbandingan tingginya

Materi Olimpiade Matematika SMP - Persiapan OSN Page 79


Segi Empat
1. Persegi panjang dan persegi
Luas persegi panjang = panjang x lebar
Keliling persegi panjang = 2(panjang + lebar)
Luas persegi = sisi x sisi
Keliling persegi = 4 x sisi
Sifatnya:
1) semua sudutnya siku-siku,

2) diagonalnya sama panjang: yaitu p2  l 2

3) pada persegi, diagonalnya saling tegak lurus dan panjangnya = sisi V2 ,


4) diagonalnya saling membagi dua sama panjang.
2. Jajar genjang dan belah ketupat
Luas jajar genjang = alas x tinggi
Luas belah ketupat = alas x tinggi, atau boleh juga
1
L (diagonal 1 x diagonal 2)
2
Keliling jajar genjang = 2(panjang + lebar),
Keliling belah ketupat = 4 x sisi.
Sifatnya:
1) sisi-sisi yang berhadapan sama panjang,
2) sudut yang berhadapan sama besar,
3) dua sudut yang berdekatan jumlahnya 180°,
4) dua diagonal pada belah ketupat saling tegak lurus.
3. Layang-layang
1
Luas layang-layang = L  (diagonal 1 x diagonal 2)
2
Keliling layang-layang = jumlah sisi-sisi layang-layang
Sifatnya:
1) dua diagonalnya saling tegak lurus,
2) diagonal terpanjangnya merupakan sumbu simetri,
sehingga:
a) diagonal terpendeknya terbagi dua sama panjang,
b) terdapat tiga pasang sudut yang mengapit sumbu simetri sama besar.

Materi Olimpiade Matematika SMP - Persiapan OSN Page 80


4. Trapesium
jumlah dua sisi sejajar
Luas trapesium = x tinggi
2
Keliling trapesium = jumlah semua sisi
Sifatnya:
1) mempunyai sepasang sisi berhadapan sejajar,
2) dua sudut pada dua sisi sejajar saling berpelurus.
Luas segiempat yang dibentuk oleh titik-titik sudut A(x1, y1), B(x2, y2), C(x3, y3)
dan D(x4, y4) ditentukan oleh formula:

1 x1 x2 x3 x4 x1
Luas =
2 y1 y2 y3 y4 y1
1
= x1 y2  x2 y3  x3 y4  x4 y1  x2 y1  x3 y2  x4 y3  x1 y4
2
Segi-n beraturan
Segi-n beraturan (dengan n > 3) dapat di buat dari n segitiga sama kaki yang
kongruen.
Luas segi-n beraturan = n x luas 1 unit segitiga sama kaki.
Sifat-sifat segi-n beraturan
1. Jumlah besar sudut segi-n = (n – 2)x1800
2. Besar sudut pusat pada tiap unit segitiga = 3600/n
3. Besar sudut pada tiap sisinya = 180 0 - 3600/n.
Luas Segitiga
Terdapat empat rumus untuk luas segitiga yaitu
1. Luas = ½ alas x tinggi
2. Luas = ½ ab sin C = ½ ac sin B = ½ bc sin A
3. Luas = s(s  a)(s  b)(s  c) , dengan s adalah ½ keliling = (a + b + c)/2

4. Jika segitiga dengan titik sudut (x1, y1), (x2, y2), dan (x3, y3) maka luasnya
x1 y1 1
1 1
= x2 y 2 1 = [( x1 y 2  x2 y3  x3 y1 )  ( x2 y1  x3 y 2  x1 y3 )]
2 2
x3 y3 1

5. Luas segitiga yang dibentuk oleh titik A(x1, y1), B(x2, y2) dan C(x3, y3)
ditentukan oleh fornula:

1 x1 x2 x3 x1
Luas =
2 y1 y2 y3 y1

Materi Olimpiade Matematika SMP - Persiapan OSN Page 81


1
 Luas = x1 y2  x2 y3  x3 y1  x2 y1  x3 y2  x1 y3 (harus selalu diambil bilangan
2
positif)
Kesebangunan dan Kesejajaran
Postulat 1. (Postulat kesamaan SdSd)
Jika dua sudut dari sebuah segitiga sama dengan dua sudut segitiga yang lain maka
kedua segitiga tersebut sebangun dan sisi yang bersesuaian mempunyai
perbandingan yang sama.
Postulat 2. (Postulat kesamaan S Sd S)
Dua segitiga sebangun, jika dua pasang sisinya berbanding seharga dan sudut
apitnya sama besar.
Postulat 3. (Postulat kesamaan S S S)
Dua segitiga sebangun, jika ketiga sisi segitiga yang pertama berbanding seharga
dengan sisi segitiga yang lain.
Postulat 4. (Postulat kesamaan S S Sd)
Dua segitiga sebangun, jika dua pasang sisinya berbanding seharga dan sudut di
depan salah satu sisi itu sama, sedangkan sudut di depan sisi yang satu lagi sejenis
(S S Sd).

D. Dalil Stewart
Misalkan CD = l adalah garis sebarang ke sisi AB
maka
l 2 c  a 2 c1  b 2 c2  c1c2 c

E. Dalil Menelaos
Jika sebuah garis transversal memotong sisi
AC, BC, dan AB di titik P, Q, dan R dari
BQ CP AR
segitiga ABC maka . . 1
QC PA RB

(Kebalikan Dalil Menelaos) Jika titik P, Q, dan R dari segitiga ABC memenuhi
BQ CP AR
. .  1 maka P, Q, dan R segaris (collinear)
QC PA RB

Materi Olimpiade Matematika SMP - Persiapan OSN Page 82


F. Dalil De Ceva
Jika titik D, E, dan F terletak pada sisi BC, AC,
dan AB pada segitiga ABC sedenikian sehingga
garis-garis AD, BE, dan CF berpotongan pada
satu titik (konkuren) melalui P maka
AF BD CE
. . 1
FB DC EA
(Kebalikan dalil De Ceva). Jika titik D, E, dan F terletak pada sisi BC, AC, dan AB
AF BD CE
pada segitiga ABC sedenikian sehingga . .  1 maka garis-garis AD, BE,
FB DC EA
dan CF berpotongan pada satu titik (konkuren)

DALIL PYTHAGORAS
Pada segitiga ABC yang siku-siku di A berlaku : a 2  b 2  c 2
Dalil Pythagoras berguna untuk
1. Menentukan jenis segitiga
a. Jika a 2  b 2  c 2 maka ΔABC siku-siku di A.
b. Jika a 2  b 2  c 2 maka ΔABC tumpul di A.
c. Jika a 2  b 2  c 2 maka ΔABC lancip.
2. Menghitung panjang sisi pada bidang datar dan bangun ruang
3. Menghitung jarak dua titik pada bidang koordinat.
TRANSFORMASI GEOMETRI
A. TRANLASI
Minggu lalu, Candra duduk di pojok kanan baris pertama di kelasnya. Minggu
ini, ia berpindah ke baris ketiga lajur keempat yang minggu lalu ditempati
Dimas. Dimas sendiri berpindah ke baris kedua lajur kedua yang minggu lalu
ditempati Sari. Perhatikan perpindahan tempat duduk Candra dan Dimas ini.

Materi Olimpiade Matematika SMP - Persiapan OSN Page 83


 Candra berpindah 2 lajur ke kiri dan 2 baris ke belakang. Saat berpindah
ini, Candra telah melakukan translasi 2 satuan ke kiri dan 2 satuan ke atas
  2
yang ditulis sebagai  
 2 
 Kemudian, Dimas berpindah 2 lajur ke kiri dan 1 baris ke depan. Saat
berpindah ini, Dimas telah melakukan translasi 2 satuan ke kiri dan 1
  2
satuan ke bawah yang ditulis sebagai  
 1 
 Misalkan, tempat duduk Candra minggu lalu di titik N(a, b) pada koordinat
  2
Cartesius. Dengan translasi   , diketahui tempat duduknya inggu ini
 2 
pada titik N ’(a-2,b+2).Kalian dapat menuliskan translasi ini sebagai berikut
 2 
 
N a, b  
 N ' a  2, b  2
 2

a
Dengan prinsip yang sama, jika titik P(x, y) ditranslasikan dengan T1   
b
maka diperoleh bayangannya P ' x  a, y  b . Secara matematis, ditulis
sebagai berikut.
a
T1  
Px, y   P ' x  a, y  b 
b

Sekarang, translasikan lagi bayangan yang telah kalian peroleh dengan


c
c T2  
T2    Didapat, P ' x  a, y  b    P '' x  a  c, y  b  d  Perhatikan
d

d 
bahwa P '' x  a  c, y  b  d   P '' x  a  c , y  b  d 

Materi Olimpiade Matematika SMP - Persiapan OSN Page 84


Ini berarti P '' x  a  c, y  b  d  diperoleh dengan mentranslasikan Px, y 
a c
dengan T    Translasi T ini merupakan translasi T1 dilanjutkan
b  d 
dengan T2, yang ditulis sebagai T1  T2
a c a c
Oleh karena T1    dan T2    maka T1  T2   
b d  b  d 
Akibatnya, titik Px, y  ditranslasikan dengan T1 dilanjutkan dengan
translasi T2 menghasilkan bayangan P '' sebagai berikut
 a c 
T1 T2  
Px, y   P '' x  a  c, y  b  d 
 bd 

Sifat:
a c
 Dua buah translasi berturut-turut   diteruskan dengan   dapat
b d 
a c
digantikan dengan translasi tunggal  
b  d 
 Pada suatu translasi setiap bangunnya tidak berubah.

Contoh:
 p
1. Translasi T1    memetakan titik A(1,2) ke titik A'(4,6)
q
a. Tentukan translasi tersebut !
b. Tentukanlah bayangan segitiga ABC dengan titik sudut A(1, 2), B(3, 4),
dan C( 5, 6) oleh translasi tersebut.
c. Jika segitiga yang kalian peroleh pada jawaban b ditranslasikan lagi
  1
dengan T2    Tentukan bayangannya!
  1
d. Translasikan segitiga ABC dengan translasi T2 ◦T1. Samakah
jawabannya dengan jawaban c?
Jawaban
 p
T1  
a. A1,2  A' 1  p, 2  q   A1 4,6
q

Diperoleh 1+p = 4 sehingga p = 3


2+q = 6 sehingga q = 4
 3
Jadi translasi tersebut adalah T1   
 4

Materi Olimpiade Matematika SMP - Persiapan OSN Page 85


 3
b. translasi T1    artinya artinya memindahkan suatu titik 3 satuan ke kanan
 4
dan 4 satuan ke atas. Dengan mentranslasikan titiktitik A', B', dan C' dari
segitiga ABC dengan translasi T1, kalian memperoleh segitiga A'B'C' sebagai
berikut
 3
T1  
A1,2  A' 1  3,2  4  A' 4,6
 4

 3
T1  
B3,4  B' 3  3,4  4  B' 6,8
 4

 3
T1  
C  5,6  C '  5  3,6  4  C '  2,10
 4

Jadi bayangan segitiga ABC adalah segitiga A'B'C' dengan titik A'(4,6),
B'(6,8), dan C'(-2,10)
 1 
T2  

c. A' 4,6  A' ' 4   1,6   1  A' ' 3,5
 1 

 1 
T2  
A' 6,8  A' ' 6   1,8   1  B' ' 5,7 
 1 

 1 
T2  
A' 4,6  A' '  2   1,10   1  A' '  3,9
 1 

Jadi bayangan segitiga A'B'C' adalah segitiga A''B''C'' dengan titik A''(3,5),
B''(5,7) dan C''(-3,9)
 3   1   2 
d. translasi titik T1  T2      
 4   1  3 
 2
 
A1,2  A' 1  2,2  3  A' 3,5
 3

 2
 
B3,4  B' 3  2,4  3  B' 5,7 
 3

 2
 
C  5,6  C '  5  2,6  3  C '  3,9
 3

Jadi bayangan segitiga ABC adalah segitiga A'B'C' dengan titik A'(3,5), B'(5,7)
dan C'(-3,9) Perhatikan bahwa segitiga yang kalian peroleh pada jawaban c
sama dengan segitiga yang kalian peroleh pada jawaban d.
  5
2. Tentukan bayangan lingkaran (x-3)2+(y+1)2=4 jika ditranslasikan T    !
 2 
Jawab
Ambil sembarang titik P(a,b) pada lingkaran (x-3)2 + (y+1)2 = 4 sehingga
diperoleh (a-3)2 + (b+1)2 = 4

Materi Olimpiade Matematika SMP - Persiapan OSN Page 86


Translasikan titik P dengan T    5  sehingga diperoleh
 2 
 
 5 
 
Pa, b  

2
 P' ' a  5, b  2

Jadi titik P'(a-5, b+2)


Perhatikan bahwa: a'= a - 5. Dari persamaan (*), didapat a = a' 5.
b'= b + 2. Dari persamaan (*), didapat b = b' - 2.
Dengan mensubstitusi nilai a dan b ini ke persamaan (*), akan
Diperoleh

Jadi bayangan dari jika ditranslasikan


  5
dengan T    adalah
 2 

B. REFLEKSI
Kalian pasti sering bercermin. Ketika bercermin, amatilah diri dan bayangan
kalian. Apakah memiliki bentuk dan ukuran yang sama? Amati pula jarak diri
kalian ke cermin. Samakah dengan jarak bayangan kalian ke cermin? Dengan
bercermin dan menjawab pertanyaan-pertanyaan tersebut, kalian akan
menemukan beberapa sifat pencerminan.

Dari gambar tersebut, kalian dapat mengatakan bahwa:


• Lingkaran Q kongruen dengan bayangannya, yaitu lingkaran Q’
• Jarak setiap titik pada lingkaran Q ke cermin sama dengan jarak setiap titik
bayangannya ke cermin, yaitu QA = Q’A dan PB = P’ B.
• Sudut yang dibentuk oleh cermin dengan garis yang menghubungkan setiap
titik ke bayangannya adalah sudut siku-siku.
Sifat-sifat tersebut merupakan sifat-sifat refleksi.
Matriks yang bersesuaian dengan tranformasi geometri

Materi Olimpiade Matematika SMP - Persiapan OSN Page 87


Refleksi Rumus Matriks
Refleksi terhadap 6  x'   1 0  x 
     
sumbu-x  y '   0  1 y 
Refleksi terhadap Ax, y  sb

.y
A'  x, y   x'    1 0  x 
     
sumbu-y  y '   0 1  y 
Refleksi terhadap Ax, y  
 A'  y, x 
yx
 x'   0 1  x 
     
garis y=x  y '   1 0  y 
Refleksi terhadap Ax, y    A'  y, x 
y  x
 x'   0  1 x 
     
garis y=-x  y '    1 0  y 
Refleksi terhadap Ax, y  x
k
 A' 2k  x, y 
garis x=k
Refleksi terhadap Ax, y  
y k
 A' x,2k  y 
garis y=k
Refleksi terhadap titik Ax, y  
 A' x' , y'
p ,q   x' p   cos 180  sin 180  x  p 
     
(p,q)  y 'q   sin 180 cos 180  y  q 
Sama dengan rotasi
pusat (p,q) sejauh 180˚
Refleksi terhadap titik Ax, y  
 A'  x, y 
0, 0 
 x'    1 0  x 
     
pusat (0,0)  y '   0  1 y 
Refleksi terhadap Ax, y    A' x' , y '
y  mx
 x'   cos 2 sin 2  x 
     
garis y=mx,m= tan α dengan x'  x cos 2  y sin 2  y '   sin 2  cos 2  y 
y'  x sin 2  y cos 2
Refleksi terhadap A x, y    A'  x' , y '
y  xk
 x'   0 1  x   0 
        
garis y=x+k dengan x'  y  k  y '   1 0  y  k   k 
y'  x  k

Refleksi terhadap Ax, y    A' x' , y '


y  x k
  x'   0  1 x   0 
        
garis y=-x+k dengan x'   y  k  y '    1 0  y  k   k 
y'   x  k

SIFAT-SIFAT
a. Dua refleksi berturut-turut terhadap sebuah garis merupakan suatu
identitas, artinya yang direfleksikan tidak berpindah.
b. Pengerjaan dua refleksi terhadap dua sumbu yang sejajar, menghasilkan
translasi (pergeseran) dengan sifat:

Materi Olimpiade Matematika SMP - Persiapan OSN Page 88


 Jarak bangun asli dengan bangun hasil sama dengan dua kali jarak
kedua sumbu pencerminan.
 Arah translasi tegak lurus pada kedua sumbu sejajar, dari sumbu
pertama ke sumbu kedua. Refleksi terhadap dua sumbu sejajar
bersifat tidak komutatip.
c. Pengerjaaan dua refleksi terhadap dua sumbu yang saling tegak lurus,
menghasilkaan rotasi (pemutaran) setengah lingkaran terhadap titik
potong dari kedua sumbu pencerminan. Refleksi terhadap dua sumbu
yang saling tegak lures bersifat komutatif.
d. Pengerjaan dua refleksi berurutan terhadap dua sumbu yang
berpotongan akan menghasilkan rotasi (perputaran) yang bersifat:
 Titik potong kedua sumbu pencerminan merupakan pusat perputaran.
 Besar sudut perputaran sama dengan dua kali sudut antara
kedua sumbu pencerminan.
 Arah perputaran sama dengan arah dari sumbu pertama ke sumbu
kedua.
C. ROTASI
Rotasi Rumus Matriks
Rotasi dengan Ax, y  R  A' x' , y '
0, 
 x'   cos   sin   x 
     
pusat (0,0) dengan x'  x cos   y sin   y '   sin  cos   y 
dan sudut y '  x sin   y cos 

putar α
Rotasi dengan Ax, y  R  A' x' , y '
 P ,   x'   cos   sin   x  a   a 
        
pusat P(a,b) dengan x'a  x  a  cos    y  b sin   y'   sin  cos   y  b   b 
dan sudut y 'b  x  a sin    y  b  cos 

putar α
Keterangan
α + : arah putaran berlawanan putaran jarum jam
α - : arah putaran searah putaran jarum jam
SIFAT-SIFAT
Dua rotasi bertumt-turut mempakan rotasi lagi dengan sudut putar dsama
dengan jumlah kedua sudut putar semula.Pada suatu rotasi, setiap bangun
tidak berubah bentuknya.

Materi Olimpiade Matematika SMP - Persiapan OSN Page 89


Catatan:
Pada transformasi pergeseran (translasi), pencerminan (refleksi) dan
perputaran (rotasi), tampak bahwa bentuk bayangan sama dan sebangun
(kongruen) dengan bentuk aslinya. Transformasi jenis ini disebut
transformasi isometri.

D. DILATASI
Aini dan teman-temannya berkunjung ke IPTN. Di sana, mereka mengamati
miniatur sebuah pesawat terbang. Miniatur pesawat terbang ini mempunyai
bentuk yang sama dengan pesawat terbang sesungguhnya, tetapi ukurannya
lebih kecil. Bentuk seperti miniatur pesawat terbang ini telah mengalami
dilatasi diperkecil dari pesawat terbang sesungguhnya. Selain dilatasi
diperkecil, terdapat pula dilatasi diperbesar, misalnya pencetakan foto yang
diperbesar dari klisenya. Faktor yang menyebabkan diperbesar atau
diperkecilnya suatu bangun ini disebut faktor dilatasi. Faktor dilatasi ini
dinotasikan dengan huruf kecil, misalnya k.
• Jika k > 1 atau k < -1, maka hasil dilatasinya diperbesar
• Jika -1 < k < 1, maka hasil dilatasinya diperkecil
• Jika k =  1, maka hasil dilatasinya tidak mengalami perubahan
Dilatasi Rumus Matriks
Dilatasi dengan pusat Ax, y  
 A' kx, ky
0, k 
 x'   k 0  x 
     
(0,0) dan faktor dilatasi k  y '   0 k  y 
Dilatasi dengan pusat Ax, y    A' x' , y '
P ,k 
 x'   k 0  x  a   a 
        
P(a,b) dan faktor dilatasi k dengan x'a  k x  a   y '   0 k  y  b   b 
y 'b  k  y  b 

E. KOMPOSISI TRANSFORMASI DENGAN MARIKS


Matriks yang bersesuaian dengan transformasi geometri
Transformasi Rumus Matriks
Identitas Ax, y  

1
A' x, y   x'   1 0  x 
     
 y '   0 1  y 

Materi Olimpiade Matematika SMP - Persiapan OSN Page 90


Translasi  p
   x'   x   p 
Ax, y   A' x  p, y  q 
q        
 y'   y   q 
Refleksi terhadap Ax, y  sb
 .x
 A' x, y   x'   1 0  x 
     
sumbu-x  y '   0  1 y 
Refleksi terhadap Ax, y  sb

.y
A'  x, y   x'    1 0  x 
     
sumbu-y  y '   0 1  y 
Refleksi terhadap Ax, y  
 A'  y, x 
yx
 x'   0 1  x 
     
garis y=x  y '   1 0  y 
Refleksi terhadap Ax, y    A'  y, x 
y  x
 x'   0  1 x 
     
garis y=-x  y '    1 0  y 
Refleksi terhadap Ax, y  x
k
 A' 2k  x, y 
garis x=k
Refleksi terhadap Ax, y  
 A' x,2k  y 
y k

garis y=k
Refleksi terhadap Ax, y  
 A' x' , y'
p ,q 
 x' p   cos 180  sin 180  x  p 
     
titik (p,q) Sama dengan rotasi pusat (p,q)  y'q   sin 180 cos 180  y  q 

sejauh 180˚
Refleksi terhadap Ax, y  
 A'  x, y 
0, 0 
 x'    1 0  x 
     
titik pusat (0,0)  y '   0  1 y 
Refleksi terhadap Ax, y    A' x' , y '
y  mx
 x'   cos 2 sin 2  x 
     
garis y=mx,m=tan dengan x'  x cos 2  y sin 2  y '   sin 2  cos 2  y 
α y '  x sin 2  y cos 2

Refleksi terhadap Ax, y    A' x' , y '


y  xk
 x'   0 1  x   0 
        
garis y=x+k dengan x'  y  k   
y ' 1 0  y  k  k 
y'  x  k
Refleksi terhadap Ax, y    A' x' , y '
y  x k
  x'   0  1 x   0 
        
garis y=-x+k dengan x'   y  k  y '    1 0  y  k   k 
y'   x  k
Rotasi dengan Ax, y  R  A' x' , y '
0, 
 x'   cos   sin   x 
     
pusat (0,0) dan dengan x'  x cos   y sin   y '   sin  cos   y 
sudut putar α y '  x sin   y cos 

Materi Olimpiade Matematika SMP - Persiapan OSN Page 91


Rotasi dengan Ax, y  R  A' x' , y '
 P , 
 x'   cos   sin   x  a   a 
        
pusat P(a,b) dan x'a  x  a  cos    y  b sin   y'   sin  cos   y  b   b 
sudut putar α y 'b  x  a sin    y  b  cos 

Dilatasi dengan Ax, y  


 A' kx, ky
0, k 
 x'   k 0  x 
     
pusat (0,0) dan  y '   0 k  y 
factor dilatasi k
Dilatasi dengan Ax, y    A' x' , y '
P ,k 
 x'   k 0  x  a   a 
        
pusat P(a,b) dan dengan x'a  k x  a   y '   0 k  y  b   b 
faktor dilatasi k y 'b  k  y  b 

Komposisi transformasi
1. komposisi dua translasi berurutan
a c
Diketahui dua translasi T1    dan T2    . Jika translasi T1 dilanjutkan
b d 
translasi T2 maka dinotasikan ” T1  T2 ” dan translasi tunggalnya adalah
T=T1+T2=T2+T1(sifat komutatif).
2. komposisi dua refleksi berurutan
a. refleksi berurutan terhadap dua sumbu sejajar
Jika titik A(x,y) direfleksikan terhadap garis x=a dilanjutkan terhadap
garis x=b. Maka bayangan akhir A adalah A' x' , y' yaitu:
x'=2(b-a)+x
y'=y
Jika titik A(x,y) direfleksikan terhadap garis y=a dilanjutkan terhadap
garis y=b. Maka bayangan akhir A adalah A' x' , y' yaitu:
x'=x
y'=2(b-a)+y
b. refleksi terhadap dua sumbu saling tegak lurus
Jika titik A(x,y) direfleksikan terhadap garis x=a dilanjutkan terhadap
garis y=b (dua sumbu yang saling tegak lurus) maka bayangan akhir A
adalah A' x' , y' sama dengan rotasi titik A(x,y) dengan pusat titik potong
dua sumbu (garis) dan sudut putar 180˚
c. refleksi terhadap dua sumbu yang saling berpotongan

Materi Olimpiade Matematika SMP - Persiapan OSN Page 92


Jika titik A(x,y) direleksikan terhadap garis g dilanjutkan terhadap garis h,
maka bayangan akhirnya adalah A' x' , y' dengan pusat perpotongan
garis g dan h dan sudut putar 2α(α sudut antara garis g dan h) serta arah
putaran dari garis g ke h.
m k  ml
tan  
1  m k  ml
Catatan ml  gradien garis l
mk  gradien garis k

d. sifat komposisi refleksi


Komposisi refleksi (refleksi berurutan) pada umumnya tidak komutatif
kecuali komposisi refleksi terhadap sumbu x dilanjutkan terhadap sumbu
y (dua sumbu yang saling tegak lurus).
3. rotasi berurutan yang sepusat
a. Diketahui rotasi R1(P(a,b),α) dan R2(P(a,b),β), maka transformasi
tunggal dari komposisi transformasi rotasi R1 dilanjutkan R2 adalah
rotasi R(P(a,b),α+β)
b. Rotasi R1 dilanjutkan R2 sama dengan rotasi R2 dilanjutkan R1
4. komposisi transformasi
a b   p q
Diketahui transformasi T1    dan T2    maka transformasi
c d   r s
tunggal dari transformasi:

a. T1 dilanjutkan T2 (T2 ◦ T1) adalah T=T2 . T1


b. T2 dilanjutkan T1 (T1 ◦ T2) adalah T=T1 . T2
Catatan T1 . T2 = T2 . T1

5. bayangan suatu kurva/bangun oleh dua transformasi atau lebih


Contoh: Tentukan bayangan garis -4x+y=5 oleh pencerminan terhadap garis
 3
y=x dilanjutkan translasi   !
 2
Jawab: misal titik P(x,y) pada garis -4x+y=5
P(x,y) dicerminkan terhadap garis y=x, bayangannya P'(y,x)

Materi Olimpiade Matematika SMP - Persiapan OSN Page 93


 3
P'(y,x) ditranslasi   . Bayangannya P''(y+3, x+2)=P''(x'',y'')
 2
Jadi x'' = y +3 → y = x''-3
y'' = x +2 → x = y'' -2
persamaan -4x+y=5 → -4(y'' -2) + (x'' - 3) = 5
-4y'' + 8 + x'' – 3 = 5
x'' - 4y''= 0
jadi bayangan akhirnya adalah x - 4y= 0
6. Luas bangun hasil tranformasi
Jika suatu bangun (segitiga, lingkaran, dan lain-lain) ditransformasikan maka:
a. Luas bangun bayangan tetap untuk transformasi : translasi, refleksi,
dan rotasi.
b. Luas bangun bayangan berubah untuk transformasi dilatasi, yaitu jika
luas bangun mula-mula L setelah didilatasi oleh [P(a,b),k], maka luas
bangun bayangannya adalah L'=k2 +L

SOAL TRANSFORMASI GEOMETRI

1. Tentukan bayangan titik A(-2,8) oleh


2 
a) Translasi  
  3
b) Refleksi terhadap garis
x = -6
c) Refleksi terhadap garis
y=x
d) Refleksi terhadap garis
y=4
e) Refleksi terhadap garis
y = -x

2. Diketahui garis k : 2x + 3y = 2
Tentukan persamaan bayangan garis k oleh :
  2
a) Translasi  
3 
b) Refleksi terhadap garis y = -4
c) Refleksi terhadap garis x + y = 0

Materi Olimpiade Matematika SMP - Persiapan OSN Page 94


BANGUN RUANG
Kubus
Kubus adalah suatu benda yang dibatasi oleh enam bidang datar yang
masing-masing berbentuk persegi yang sama dan sebagun (kongruen). Kubus yang
tampak disamping disebut kubus ABCD.EFGH. Kubus sering disebut bidang enam
beraturan atau helisaeder. Keenam bidang kubus disebut bidang batas, bidang sisi,
atau sisi kubus. Sisi ABCD disebut bidang dasar atau alas. Sisi EFGH disebut bidang
atas atau tutup, dan sisi-sisi lainnya disebut bidang-bidang sisi tegak. Pertemuan dua
sisi kubus berupa ruas garis disebut rusuk kubus. Kubus mempunyai 12 rusuk.
Rusuk-rusuk yang tidak terletak di bidang alas atau bidang atas disebut rusuk-rusuk
tegak.
Pertemuan 3 rusuk kubus disebut titik sudut kubus, yang merupakan pertemuan 3
bidang sisi. Kubus mempunyai 8 buah titik sudut. Garis yang menghubungkan dua
titik sudut yang berhadapan dalam satu sisi kubus disebut diagonal bidang atau
diagonal sisi.
Garis yang menghubungkan dua titik sudut yang berhadapan dalam kubus disebut
diagonal ruang. Bidang yang melalui dua rusuk yang berhadapan di dalam kubus
disebut bidang diagonal.
Ukuran kubus ditentukan oleh panjang rusuknya.
Perhatikan kubus ABCD.EFGH di samping!
Diagonal bidang atau diagonal sisi kubus pada gambar adalah
garis AC, BD, EG, HF, AF, BE, DG, CH, BG, CF, AH, dan DE.
Diagonal ruang pada gambar adalah garis AG, BH, CE, dan DF.
Bidang diagonal pada gambar adalah bidang ACGE, BDHF,
BGHA, DEFC, AFGD, dan BEHC.
Rumus Euler
Hubungan sisi, rusuk, dan titik sudut suatu bangun ruang dirumuskan oleh Euler
dalam bentuk : S+T=R+2
Dengan S : Banyak sisi T : Banyak titik sudut dan R : Banyak rusuk
Luas permukaan dan Volume kubus
Bila panjang rusuk kubus adalah a cm, maka:
a. Volume kubus = a3
b. Luas permukaan kubus = 6a2

Materi Olimpiade Matematika SMP - Persiapan OSN Page 95


c. Panjang diagonal sisi = a 2

d. Panjang diagonal ruang = a 3

e. Luas bidang diagonal = a2 2


Luas Permukaan dan Volume Balok
a. Jika balok dengan ukuran panjang = p, lebar = l, dan tinggi = t, maka Volume
balok = p . l . t
b. Luas permukaan balok = 2(pl + lt + pt)

c. Dr (panjang diagonal ruang ), ditentukan oleh : Dr = p2  l 2  t 2 .

Luas permukaan dan volume prisma


Luas permukaan prisma = luas bidang alas + luas bidang atas + luas bidang sisi tegak
(Selubung)
a. Luas selubung = keliling bidang alas x panjang rusuk tegak
b. Volume = luas alas x tinggi
Luas permukaan dan volume limas
Karena bentuk limas bergantung pada bidang alasnya, maka luas permukaan
dan volumenya bergantung pada bidang alasnya pula. Secara umum formula untuk
menentukan luas permukaan dan volume limas adalah sebagai berikut.
(i) Luas permukaan limas = luas alas + luas selimut
1
(ii) Volume limas = x luas alas x tinggi dan Volume bidang empat beraturan=
3
1 3
r 2
12

Silinder, Kerucut dan Bola


(i) Silinder atau tabung merupakan prisma tegak yang alasnya berupa lingkaran.
(ii) Kerucut merupakan limas yang alasnya berbentuk lingkaran.
(iii) Bola merupakan bangun ruang dengan tiap titik pada permukaannya
mempunyai jarak yang sama terhadap titik pusatnya.
Penentuan formula luas permukaan dan volume untuk silinder, kerucut, dan bola
dapat dilihat pada tabel berikut ini.

Materi Olimpiade Matematika SMP - Persiapan OSN Page 96


No. Gambar Nama Luas Permukaan Volume

1 Silinder 2 luas alas + luas selimut luas alas x tinggi


atau = 2 r (r  t ) =  r 2t
Tabung luas tabung tanpa tutup 1
=  d 2t
=  r (r  2t ) 4

2 Kerucut luas alas + luas selimut 1


luas alas x
=  r (r  s ) 3
1
dengan s = r2  t2 tinggi =  r 2t
3
3 Kerucut a(r1  r2 ) + r12  r22 1
3 t (r12  r22  r1r2 )
Terpancung

4 Bola 4 x luas lingkaran dengan jari-


jari r 4 r 2   d 2 dimana d = 2r 4
 r3
r 3
1
Luas bola padat = 3 r 2
2

CONTOH SOAL
1. Pada gambar berikut tabung berisi air, tinggi dan diameter tabung
tersebut adalah 18 cm dan 6 cm. kemudian ke dalam tabung dimasukkan
3 bola pejal yang identik (sama bentuk) sehingga bola tersebut
menyinggung sisi tabung dan air dalam tabung keluar, maka sisa air
dalam tabung adalah …
a. 51 c. 53 e. 55
b. 52 d. 54
Penyelesaian:
Jari-jari tabung jari-jari bola
V sisa air dalam tabung Volume air dalam tabung (volume tabung)
volume bola

Materi Olimpiade Matematika SMP - Persiapan OSN Page 97


2. Diketahui jajargenjang ABCD. Titik P dan Q terletak pada AC sehingga DP
dan BQ tegak lurus AC. Jika Panjang AD=13 cm, AC=25 cm dan luas
jajargenjang tersebut adalah 125 , maka panjang PQ adalah …
Penyelesaian:
karena maka
D C
Q
(sifat jajargenjang), maka

P
A B

siku-siku sehingga

maka

3. Sebuah segitiga ABC sama kaki dipotong menjadi dua buah segitiga sama
kaki (tidak harus kongruen) dengan membagi dua sama besar salah satu
sudut alasnya. Ukuran sudut yang terkecil dari segitiga ABC adalah …
Penyelesaian:
samakaki sehingga dan < <
< < dan <
sehingga ………………..(1)

garis adalah garis bagi di < , maka < <

samakaki sehingga dan < <


< <

Materi Olimpiade Matematika SMP - Persiapan OSN Page 98


C

…………………(2)
D
dari (1) dan (2)

A B

dan ( sudut terkecil)


4. Luas persegi panjang ABCD adalah 112 satuan luas. Titik E dan F berada
di diagonal AC seperti gambar di bawah ini sedemikian sehingga
. Luas segitiga DEF adalah …
Penyelesaian:
Luas
Luas Luas Luas
Luas Luas Luas
Luas 56 Luas
D C
Luas 56
F
Luas 56
E
Luas 56 A B
Luas
5. Diketahui jajargenjang ABCD, sudut A= sudut C= . Lingkaran dengan
pusat C melalui B dan D. AD diperpanjang memotong lingkaran di E dan
BE memotong CD di H. Perbandingan antara luas segitiga BCH dengan
segitiga EHD adalah …
Penyelesaian:
Karena segiempat ABCD adalah jajargenjang maka AB DC sehingga
< CDE=<A=
Perhatikan , (jari-jari lingkaran) maka <CDE = ,
<DCE = berarti siku-siku sama kaki

Materi Olimpiade Matematika SMP - Persiapan OSN Page 99


B C
sebangun dengan (sd-sd-sd)
sehingga H

A E
D

6. Diberikan persegi panjang ABCD. Titik O terletak di dalam ABCD


sedemikian rupa sehingga OA=3 cm, OB=12 cm dan OD=11 cm. panjang
OC adalah …
Penyelesaian:
Lukis garis AB sejajar PS, misalkan ,
sehingga
atau ………………..(1)
atau …………...(2)
Dari persamaan (1) dan (2) diperoleh A
S R

P Q
B

Materi Olimpiade Matematika SMP - Persiapan OSN Page 100


LATIHAN GEOMETRI
A. GARIS DAN SUDUT
A.1. KEDUDUKAN DUA GARIS
A.2. SIFAT-SIFAT GARIS
1. (OSP 2011)
Tiga garis lurus l1, l2, dan l3 mempunyai gradien berturut-turut 3, 4, dan 5.
Ketiga garis tersebut memotong sumbu -Y dititik yang sama. Jika jumlah absis
titik potong masing-masing garis dengan sumbu X= tentukan persamaan

garis l1.
2. (OSP 2006)
Diberikan segitiga PQR siku-siku di Q. Jika panjang PQ adalah x + 4, panjang QR
adalah
3 x + 2, dan panjang PR adalah 3 x + 4, maka panjang QR adalah …
A.3. SIFAT-SIFAT SUDUT
3. (OSP 2003)
Pada gambar berikut, garis PQ dan garis RS sejajar, demikian juga garis PS dan
QT sejajar. Tentukan nilai x nya. P Q
410
83 0

x0
S R

4. (OSN 2004) T
Diketahui titik A(1,2), B(0,2), C (3,0), dan D(3,1) seperti terlihat pada gambar
berikut. Tentukan besar sudut AOD!

Materi Olimpiade Matematika SMP - Persiapan OSN Page 101


5. (OSN 2004)
Diketahui rangkaian persegi yang disusun secara berkesinambungan dan
membentuk sudut-sudut sebagaimana pada gambar berikut. Tentukan nilai
sudut yang ditandai dengan huruf x!

6. (OSP 2007)
Besar sudut TUV pada Gambar di atas adalah ... derajat

5x
9x
V
5x 9x

9x

T
9x
8x
7x 9x

U
9x 9x

7. (OSP 2008)
Perhatikan gambar berikut!Hasil penjumlahan sudut berikut:

Materi Olimpiade Matematika SMP - Persiapan OSN Page 102


8. (OSN 2013)
Diketahui ABC adalah segitiga lancip dengan titik-titik sudutnya terletak pada
lingkaran yang berpusat di titik O. titik P terletak pada sisi BC sehingga AP
adalah garis tinggi segitiga ABC. Jika ABC + 3 ACB, buktikan
bahwa COP + CAB 9 .
A.4. TEOREMA PHYTHAGORAS DAN PENGGUNAANNYA
9. (OSN 2003)
Diberikan suatu persegi panjang dengan ukuran 16 cm × 25 cm, EBFG layang-
layang dan panjang AE = 5 cm. Tentukan panjang EF!

10. (OSP 2008)


Perhatikan gambar berikut. Diketahui ∆ABC adalah samasisi dengan panjang sisi
16 cm. Titik O terletak dalam ∆ABC. Dari titik O dibuat ruas garis OP, OQ, dan OR
yang tegak lurus terhadap sisi-sisi ∆ABC. Jumlah panjang ruas garis OP + OQ +
OR =...

11. (OSP 2010)


Perhatikan gambar jajar genjang di bawah ini, jika sudut BPC dan BQD siku-siku,
dan BP=4 cm, DP=4 cm, dan DC=7 cm, tentukan panjang BQ

Materi Olimpiade Matematika SMP - Persiapan OSN Page 103


12. (OSP 2010)
Sebuah persegi ABCD dengan luas x2, titik P terletak dalam persegi seperti
tampak pada gambar dengan jarak PA=PB=PM. Jika jarak tersebut dinyatakan
dengan y, nyatakan dalam x.

13. (OSP 2003)


Diketahui titik T adalah titik tengah sisi suatu segi-6 beraturan dengan panjang
sisi 1 satuan. Berapakah panjang TE?

14. (OSN 2004)


Balok ABCD.EFGH diperoleh dengan menempelkan dua kubus satuan
ABCD.PQRS dan PQRS.EFGH. Titik K adalah titik tengah rusuk AB, sedangkan titik
L dalah titik rengah rusuk SH. Berapak panjang ruas garis KL?

15. (OSN 2010)


Diketahui ABCD dan DEFG adalah dua jajargenjang. Titik E terletak pada AB dan
titik C terletak pada FG. Luas ABCD adalah 20 satuan. H adalah titik pada DG
sehingga EH tegak lurus DG. Jika panjang DG adalah 5 satuan, tentukan panjang
EH!

Materi Olimpiade Matematika SMP - Persiapan OSN Page 104


C. BANGUN DATAR
C.1. SIFAT-SIFAT BANGUN DATAR
C.2. KELILING DAN LUAS PERMUKAAN BANGUN DATAR

16. (OSP 2004)


Suatu garis memotong sumbu-x di titik A (a,0) dan memotong sumbu-y di titik
B (0,3) . Jika luas segitiga AOB sama dengan 6 satuan luas dengan titik O (0,0) ,
maka keliling segitiga AOB sama dengan ....
17. (OSP 2006)
Jika panjang sisi pada persegi yang terbesar adalah 1 satuan panjang dan persegi
berikutnya diperoleh dengan cara menghubungkan semua titik tengah pada ke
empat sisinya, maka jumlah luas daerah diarsir adalah ...

18. (OSP 2006)


Pada gambar di bawah, segitiga ABC adalah siku-siku di A dan AEDF adalah suatu
persegi. Jika panjang AB = 6 cm dan AC = 3 cm, maka luas daerah segitiga CDE
adalah ... cm2.
C

E D

B
A F

Materi Olimpiade Matematika SMP - Persiapan OSN Page 105


19. (OSP 2006)
Diberikan segitiga siku-siku sama kaki ABC dengan sudut siku-siku di C. Luas
segitiga ABC adalah 2 satuan luas. Busur l adalah busur lingkaran yang berpusat
di A dan membagi segitiga menjadi dua bagian yang sama luasnya. Busur m
adalah busur lingkaran yang berpusat di B dan menyinggung busur di l di titik
yang terletak di AB. Tentukan luas daerah yang diarsir. C

l m
l

A B
20. (OSP 2007)
Tiga segitiga samakaki (dengan panjang kakinya 2, 2 dan 2 2 ), satu persegi
(bujur sangkar dengan panjang sisi 2), dan satu jajargenjang (panjang alas dan
tingginya 2) dengan ukuran seperti pada Gambar 2 dapat disusun menjadi satu
persegi. Keliling persegi yang diperoleh adalah ….

2 2
2 2

2
2

21. (OSP 2007)


Empat bangun berbentuk layang-layang seperti gambar berikut (a>b, a dan b
bilangan asli kurang dari 10) ditata sedemikian rupa sehingga membentuk
persegi dengan lubang berbentuk persegipula ditengah-tengahnya. Lubang
berbentuk persegi ditengah-tengahnya tersebut memiliki keliling 16 satuan
panjang. Berapa keliling yang mungkin diperoleh dari persegi terluar yang

Materi Olimpiade Matematika SMP - Persiapan OSN Page 106


terbentuk, jika diketahui pula behwa a dan b adalah bilangan-bilangan yang
relative prima.

22. (OSP 2008)


Perhatikan ∆ABC pada gambar ! Diketahui , titik D terletak
ditengah AC dan titik E di tengah AB. Jika panjang AC= cm dan AB = 14 cm,
maka luas daerah BCDE adalah ...

23. (OSP 2009)


Misalkan titik A dan titik B adalah titik pada bidang datar yang jaraknya 2. Jika S
adalah himpunan dari titik-titik P, sehingga nilai dari (PA)2+(PB)2 paling besar
adalah 10, maka luas daerah dari S adalah …

24. (OSN 2009)


Diketahui segitiga ABC dengan A sebagi puncak dan BC sebagai alas. Titik P
terletak pada sisi CA. Dari titik A ditarik garis sejajar PB dan memotong
perpanjangan alas di D. Titik E terletak pada alas sehingga CE:ED=2:3 . Jika F
adalah tengah-tengah antara E dan Cdan luas segitiga ABC sama dengan 35 cm 2,
hitunglah luas segitiga PEF!

25. (OSP 2011)


Pada sebuah segiempat ABCD, sudut ABC dan sudut DAC adalah sudut siku-siku.
Jika keliling segi empat ABCD adalah 64 cm, keliling ABC adalah 24 cm, dan
keliling ACD adalah 60 cm, berapakah luas segiempat ABCD?

Materi Olimpiade Matematika SMP - Persiapan OSN Page 107


26. (OSP 2009)
Perhatikan gambar !

4 buah layang-layang kongruen yang termuat dalam persegi dan ternyata


masih tersisa bagian berbentuk persegi yang diarsir. Jika panjang
dan , maka luas yang diarsir adalah …
27. (OSP 2011)

Diketahui luas persegi ABCD adalah 25 m2. Jika E, F, dan G masing-masing


adalah titik tengah AB, AD, dan CD seperti pada gambar berikut, maka luas

trapesium BHFE adalah.... m2 .


D G C
H
F
A E B

28. (OSN 2011)


Bangun datar ABCD di bawah dalah trapezium dengan AB sejajar CD. Titik E dan
F terleteak pada CD sehingga AD sejajar BE dan AF sejajar BC. Titik H adalah
perpotongan AF dengan BE dan titik G adalah perpotongan AC dan BE. Jika
panjang AB adalah 4 cm dan panjang CD adalah 10 cm, hitunglah perbandingan
luas segitiga AGH dan luas trapesium ABCD.

Materi Olimpiade Matematika SMP - Persiapan OSN Page 108


29. (OSP 2012)
Pada gambar berikut kedua ruas garis putus-putus yang sejajar memabagi
persegi menjadi tiga daerahyang luasnya sama. Jika jarak kedua ruas garis putus-
putus tersebut 1 cm, maka luas persegi adalah …. cm2

30. (OSP 2003)


Titik sudut suatu segitiga memiliki koordinat (0,0), (4,3), dan (7,-1), maka lus
segitiga tersebut adalah ….
31. (OSP 2003)
Suatu persegi panjang dapat dipecah-pecah menjadi 5 persegi yang kongruen.
Jika luas persegi panjang tersebut adalah 720 cm 2, maka keliling satu perseginya
adalah ….
32. (OSP 2004)
Pada gambar di bawah, ABCD adalah persegi dengan panjang 4 cm. Titik-titik P
dan Q membagi diagonal AC menjadi 3 bagian sama panjang. Berapakah luas
PDQ? B C
Q

A D
33. (OSN 2010)
Diketahui segitiga ABC. Jika titik M terletak di tengah-tengah AC, titik N terletak
di tengah-tengah BC, dan titik P adalah sebarang titik pada AB. Tentukan luas
segiempat PMCN!

Materi Olimpiade Matematika SMP - Persiapan OSN Page 109


34 (OSN 2014)

C.3. KESEBANGUNAN DAN KEKONGRUENAN

34. (OSN 2003)


Perhatikan gambar tiga susunan persegi di bawah ini!

Buktikan bahwa
35. (OSN 2006)
Segitiga ABC pada gambar berikut ini adalah samakaki, dengan AB = AC = 90 cm
dan BC=108 cm. Titik P dan Q masing – masing terletak pada BC sedemikian
sehingga BP : PQ = 1 : 2 : 1. Titik S dan R berturut – turut terletak tepat di tengah
AB dan AC. Dari kedua titik ini masing – masing ditarik garis tegak lurus
terhadap PR sehingga memotong di PR di titik M dan N. Tentukan panjang MN ?

S N R

B P Q C

Materi Olimpiade Matematika SMP - Persiapan OSN Page 110


36. (OSP 2008)
Perhatikan gambar berikut!

Perbandingan luas segienam CHIJDG dan luas segienam ABCDEF adalah ...
37. (OSP 2009)
Perhatikan gambar berikut!

Jika segitiga TT1TX siku-siku sama kaki dan panjang TT1 = 8 cm, maka
TT1 + T1T2 + T2T3 + T3T4 + T4T5 + … adalah …
38. (OSN 2009)
Pada suatu segitiga ABC, titik D terletak pada sisi AB dan titik E terletak pada sisi
AC. Tunjukkan bahwa :

39. (OSP 2012)


Diketahui DABC dengan AB = 25 cm, BC = 20 cm, dan AC = 15 cm. Jika titik D
terletak pada sisi AB sedemikian sehingga perbandingan luas ADC dan ABC
adalah 14 : 25, tentukan panjang CD.

Materi Olimpiade Matematika SMP - Persiapan OSN Page 111


D. BANGUN RUANG
LUAS PERMUKAAN, VOLUME, DAN JARING-JARING KUBUS, BALOK,
TABUNG, PRISMA, KERUCUT, LIMAS, DAN BOLA
40. (OSP 2003)
Pada suatu kubus ABCD.EFGH, ruas garis AG adalah diagonal ruang dari kubus
tersebut. Ada berapa carakah perjalanan dari titik sudut G ke titik sudut A
dengan syarat perjalanan tersebut hanya melalui rusuk-rusuk kubus tanpa ada
yang dilalui lebih dari satu kali?
41. (OSN 2005)
Diketahui gambar berikut,ABCD adalah persegi dan E adalah titik sembarang di
luar persegi ABCD. E

D C

A B

Selidiki apakah berlaku hubungan ( AE ) 2  (CE ) 2  ( BE ) 2  ( DE) 2 ?


42. (OSP 2007)
Sebuah kubus padat terbuat dari bahan yang lunak akan dibelah mengikuti
segienam beraturan seperti tampak pada Gambar 5. Semua titik sudut segienam
tepat terletak di tengah rusuk-rusuk kubus. Jika rusuk kubus adalah a cm, maka
luas segienam tersebut adalah ...

Materi Olimpiade Matematika SMP - Persiapan OSN Page 112


43. (OSN 2008) Kubus ABCD.EFGH dipotong menjadi empat bagian oleh dua bidang.
Bidang pertama sejajar dengan sisi ABCD dan melalui titk tengah rusuk BF.
Bidang kedua melalui titik-titik tengah AB, AD, GH, dan FG. Tentukan rasio
volume dari bagian ruang yang terkecil dan bagian yang terbesar.
A D

B
C

44. (OSP 2009) F


G
Ada kubus sempurna dan ada kubus tidak sempurna dengan salah satu titik
sudut dipotong dengan potongan berbentuk limas. Jika panjang rusuk kubus 6a
cm, dan panjang rusuk tegak limas a cm, maka volume bangun baru adalah …

45. (OSN 2011)


Perhatikan kubus ABCD.EFGH dengan panjang rusuk 2 satuan. Titik A, B, C, dan D
terletak pada bidang sisi bagian bawah. Titik l merupakan titik perpotongan
garis diagonal pada bidang sisi bagian atas. Selanjutnya dibuat limas LABCD. Jika
limas LABCD dipotong oleh bidang diagonal yang menghubungkan titik-titik A, B,
G, dan H, tentukan volume limas terpancung bagian bawah.
46. (OSP 2012)
Sebuah silinder memiliki tinggi 5 cm dan volume 20 cm 3 . luas permukaan bola
terbesar yang mungkin diletakkan ke dalam silinder tersebut adalah…
47. (OSN 2013)
Diketahui TABC adalah limas segitiga beraturan dengan panjang rusuk 2 cm.
titik-titik P, Q, R dan S berturut-turut merupakan titik berat segitiga ABC, segitiga
TAB, segitiga TBC dan segitiga TCA. Tentukan volume limas segitiga beraturan
PQRS. (catatan: titik berat suatu segitiga adalah perpotongan ketiga garis berat)

Materi Olimpiade Matematika SMP - Persiapan OSN Page 113


48. (OSN 2014)

PENAKSIRAN ATAU PERHITUNGAN LUAS PERMUKAAN DAN VOLUME


BANGUN RUANG YANG TIDAK BERATURAN
49. (OSN 2006)
Apabila dalam delapan segitiga samasisi yang sisinya 12 cm disusun seperti
pada gambar berikut, diperoleh suatu jaring – jaring oktahedron. Tentukan
volume dari oktahedron tersebut.

50. (OSP 2009)


Perhatikan gambar!

Gambar di atas merupakan bangun ruang yang terdiri dari tabung berjari-jari R2
dan belahan bola yang berjari-jari R1. Tinggi tabung 8 cm, dan jari-jari bola 10
cm dan jari-jari tabung 5 cm. Hitunglah luas permukaan dari bangun ruang
tersebut !
51. (OSP 2010)
Perhatikan gambar berikut. Jika setiap persegi kecil memiliki luas 1 satuan/luas
daerah yang tertutup dibatasi oleh busur-busur lingkaran dibawah adalah

Materi Olimpiade Matematika SMP - Persiapan OSN Page 114


D. STATISTIKA DAN PELUANG

4. Menerapkan dan Statistika 1. Statistika


menganalisis ukuran dan a. Rata-rata, median, modus
pemusatan data dan Peluang data tunggal, dan
penyajian data penafsirannya
b. Penyajian data dalam
bentuk tabel, diagram,
grafik, dan penafsirannya
2. Peluang
Menerapkan dan a. Percobaan dan ruang
menganalisis aturan sampel
pencacahan dan peluang b. Aturan pencacahan
suatu kejadian (penjumlahan, perkalian,
permutasi, kombinasi)

RANGKUMAN MATERI
PENYAJIAN DATA
Statistika adalah ilmu yang berhubungan dengan bagaimana cara
mengumpulkan, menyajikan, menganalisis dan menginterpretasikan data yang
diperoleh dari hasil penelitian/ pengamatan/ perhitungan/ pengukuran.
Populasi adalah semua objek pengamatan, sedangkan sampel adalah sebagaian
objek yang menjadi pengamatan. Cakupan populasi lebih luas dibandingkan sampel.
Secara sederhana, dapat dikatakan bahwa sampel adalah bahagian dari populasi.
Datum adalah hasil yang diperoleh dari hasil pengukuran/ perhitungan/
pengamatan/ penelitian. Kelompok datum disebut data.
Contoh:
 Tinggi Nabil adalah 145 cm. Seratus empat puluh lima centimeter
merupakan datum.
Data yang diperoleh, dapat disajikan dengan menggunakan:

a. Tabel
Tabel merupakan daftar yang berisi ikhtisar sejumlah (besar) informasi,
biasanya berupa kata-kata dan bilangan yang tersusun secara rapi menurut
kolom dan baris.

Materi Olimpiade Matematika SMP - Persiapan OSN Page 115


Contoh:
Diketahui berat badan beberapa siswa kelas V SD Al Statistik yaitu Dina 34
kg, Dini 32 kg, Nana 30 kg, Nani 35 kg, Lucki 40 kg dan Randi 39 kg. Jika
disajikan menggunakan tabel, maka diperoleh tabel berikut.
Nama Berat(kg)
Dina 34
Dini 32
Nana 30
Nani 35
Lucki 40
Randi 39

b. Diagram batang
Diagram batang adalah diagram yang menyajikan data dalam gambar
berbentuk batang atau balok
Contoh:
Data jumlah pelanggan dan pemakai internet disebuah Provinsi disajikan
dalam tabel berikut Tabel Jumlah Pelanggan dan Pemakai Internet
Tahun Pelanggan Pemakai Jumlah
2005 866 8.081 8.947
2006 1.087 11.226 12.313
2007 1.500 16.400 17.900
2008 1.709 20.001 21.710
2009 2.010 25.195 27.205

Materi Olimpiade Matematika SMP - Persiapan OSN Page 116


DIAGRAM BATANG MAJEMUK

25100

20100
Pelanggan
JUMLAH 15100
Pemakai
25195
10100 20001
16400
5100 11226
8081
866 1087 1500 1709 2010
100 TAHUN
2005 2006 2007 2008 2009

DIAGRAM BATANG TEGAK


30000 27205

25000
21710
20000 17900

15000 12313 JUMLAH


10000 8947

5000

0
2005 2006 2007 2008 2009

Diagram lingkaran
Diagram lingkaran adalah diagram yang menyajikan data statistik yang
terbagi atas beberapa kelompok dalam bentuk daerah lingkaran. Daerah
lingkaran dibagi dalam potongan-potongan lingkaran. Besar setiap potongan
lingkaran sebanding dengan besar nilai data yang disajikan.
Diagram pastel adalah diagram lingkaran yang memiliki tebal (3 dimensi)

Materi Olimpiade Matematika SMP - Persiapan OSN Page 117


Contoh:
Mata pencaharian 300 penduduk Desa Makmur pada Tahun 2009
ditunjukkan oleh tabel berikut
Mata Pencaharian Frekuensi
Petani 90
Peternak 10
Pedagang 120
Guru 50
Karyawan 30
Jumlah 300

Mata Besar Sudut Pusat Persentasi


Pencaharian Juring
Petani

Peternak

Pedagang

Guru

Karyawan

DIAGRAM LINGKARAN
KARYAWAN
10%

PETANI
GURU
30%
16,67%

PEDAGANG PETERNAK
40% 3,33%

Materi Olimpiade Matematika SMP - Persiapan OSN Page 118


DIAGRAM PASTEL
KARYAWAN
10%
PETANI
GURU 30%
16,67%

PEDAGANG
40% PETERNAK
3,33%

c. Diagram Garis
Diagram Garis adalah garis yang diperoleh dengan menghubungkan nilai
setiap data menggunakan segmen garis. Biasanya, diagram garis digunakan
untuk menyajikan data yang diperoleh berdasarkan pengamatan dari waktu
ke waktu secara berurutan
Contoh :
Impor miyak sawit mentah di suatu kota/daerah dari Tahun 2001 sampai
2006 disajikan dalam tabel dan diagram garis berikut.
Tahun Impor (dalam Kuintal)
2004 9500
2005 4000
2006 4500
2007 10500
2008 1500
2009 2000

11000
IMPOR (Dalam

9000
Kuintal)

7000
5000
3000
1000
2004 2005 2006 2007 2008 2009
TAHUN

Materi Olimpiade Matematika SMP - Persiapan OSN Page 119


UKURAN PADA DATA
Setiap data dapat dinyatakan dalam ukuran pemusatan, ukuran lokasi ataupun
ukuran penyebaran. Ukuran pemusatan, menjelaskan bagaimana data yang ada
mengelompok terhadap pusat data itu. Ukuran lokasi menyatakan posisi dari data,
serta ukuran penyebaran meliputi cakupan yang dijangkau oleh data yang kita
miliki.
A. Ukuran Pemusatan
Ukuran pemusatan meliputi mean/ rata-rata, median dan modus.
 Mean adalah perbandingan jumlah seluruh data dengan banyaknya data
yang tersedia.

Sekumpulan data hanya akan memiliki satu buah nilai mean atau rata-rata,
atau dapat dikatakan bahwa nilai mean adalah tunggal.
 Median adalah nilai tengah data yang diperoleh setelah dikakukan
pengurutan terhadap data. Sama seperti mean, nilai median adalah tunggal.
Tidak mungkin suatu data memiliki median lebih dari satu. Jika banyak data
adalah maka median terletak pada data ke- . Jika ganjil, maka nilai

median adalah nilai dari data ke tersebut, tetapi jika genap, maka nilai

median adalah rata-rata dari data ke- dan data ke-

 Modus adalah data dengan frekuensi tertinggi atau data yang paling sering
muncul. Berbeda dengan mean dan median, nilai modus tidaklah harus
tunggal.
Contoh:
Diketahui matematika Devi dari 6 kali ujian adalah 5,8,7,8,7,9.
 Mean

 Median
Data diurutkan menjadi: 5,7,7,8,8,9
Median terletak pada data ke atau data ke 3,5. Artinya, milai

median adalah

Materi Olimpiade Matematika SMP - Persiapan OSN Page 120


 Modus
Angka 5 muncul 1 kali, angka 7 muncul 2 kali, angka 8 muncul 2 kali
dan angka 9 muncul 1 kali. Karena modus merupakan data dengan
frekuensi/ kemunculan paling tinggi, maka modus data di atas adalah
7 dan 8.

B. Ukuran Lokasi
Ukuran lokasi terdiri dari median, kuartil, desin dan persentil.
Secara umum, kesemua ukuran lokasi ini mirip dengan median. Nilainya akan
diperoleh setelah mengurutkan data yang ada. Prinsipnya, median membagi dua
data sama banyak, kuartil membagi data menjadi 4 bagian sama banyak, desil
membagi data menjadi 10 bagian sama banyak dan persentil membagi data
menjadi 100 bagian sama banyak.

Kuartil terdiri dari . Nilai dari terletak pada data ke-

Desil terdiri dari . Nilai dari terletak pada data

ke-

Persentil terdiri dari . Nilai dari terletak pada data

ke-

C. Ukuran Penyebaran
Ukuran penyebaran terdiri dari range/ jangkauan/ rentang, jangkauan antar
kuartil, simpangan kuartil, rata-rata simpang, simpangan baku, dan variansi.
 Range

 Jangkauan antar kuartil

Ukuran Pemusatan Data Tunggal


Rata-rata (Mean)
Di lain pihak, jika x1 , x2 , x3 ,, xn adalah nilai-nilai pengamatan dari suatu data
n

x i
maka: Rata-rata (mean) adalah x  i 1

Materi Olimpiade Matematika SMP - Persiapan OSN Page 121


Median
Misalkan x1 , x2 , x3 ,, xn adalah nilai-nilai pengamatan dari suatu data yang

telah diurutkan dari yang terkecil sampai terbesar atau sebaliknya, maka median
adalah nilai yang tepat berada di tengah-tengah jika banyaknya pengamatan ganjil
dan rata-rata dari dua nilai yang berada di tengah jika banyaknya pengamatan
genap.
Median dilambangkan dengan ~
x.
Modus
Modus dari suatu data adalah nilai yang paling sering muncul atau yang
mempunyai frekuensi yang paling tinggi.

PELUANG SUATU KEJADIAN


Definisi dari peluang adalah sebagai berikut:
Misalkan suatu ruang sampel S mempunyai N hasil yang mungkin yang banyaknya
berhingga dan masing-masing mempunyai kemungkinan yang sama untuk terjadi.
Misalkan A adalah suatu kejadian yang mempunyai k hasil, maka peluang kejadian A

adalah: P A 
k
N
Sifat-sifat Peluang :
 0  P A  1
 P   0  Kejadian yang tidak pernah terjadi
 PS   1  Kejadian yang pasti terjadi
Frekuensi Harapan Suatu Kejadian
Jika A adalah suatu kejadian pada ruang sampel S dengan peluang P(A), maka
frekuensi harapan munculnya kejadian A dalam M kali percobaan adalah P A M .
ATURAN PENCACAHAN
Aturan pencacahan dibagi menjadi dua, yaitu:
Pencacahan secara langsung
o Prinsip penjumlahan dan perkalian
o Permutasi
o Kombinasi

Materi Olimpiade Matematika SMP - Persiapan OSN Page 122


Prinsip Penjumlahan
Defnisi
Misalkan terdapat t himpunan X 1 , X 2, , X 3 ,... X t yang masing-masing

mempunyai n1 , n2, , n3 ,...nt anggota. Jika himpunan-himpunan tersebut saling lepas,

yaitu X i  X j   untuk i  j, maka banyaknya anggota yang bisa dipilih satu dari

masing-masing himpunan tersebut adalah n1  n2  n3  ....nt

Prinsip perkalian
Definisi
Jika terdapat aktiftas yang terdiri dari t langkah berurutan, dimana langkah 1
bisa dilakukan dalam n1 cara, setiap cara pada langkah 1 dapat dilakukan

langkah 2 bisa dilakukan dalam n2 cara, dan seterusnya sampai langkah ke-t

yang bisa dilakukan dalam nt cara; maka banyaknya aktiftas yang berbeda

yang dapat dilakukan adalah n1  n2  n3  ....nt cara. Kejadian dapat merupakan

jumlah dari cara pengaturan suatu pola atau pencacahan sejumlah objek.
Permutasi
Permutasi n obyek berbeda x1, x2, …, xn adalah penyusunan obyek tersebut dalam
suatu urutan tertentu dimana n! adalah permutasi dari n elemen
Permutasi-r
Permutasi r dari n elemen x1, x2, …, xn yang berbeda adalah penyusunan r elemen
dengan r < n.
Teorema
Untuk r < n, maka banyaknya permutasi r dari n objek yang berbeda adalah
n!
P(n, r ) 
(n  r )!

Atau P(n,r )  n(n 1)(n  2) (n  r  1)


Kombinasi
Misalkan X = { x1, x2, …, xn} adalah himpunan yang mengandung n elemen yang
berbeda. Kombinasi r dari X adalah penyusunan (tanpa memperhatikan urutan) r
elemen dari X, untuk r < n.
Banyaknya kombinasi r dari himpunan X adalah berupa koefisien binomial yaitu :
n!
C (n, r ) 
r! (n  r )!

Materi Olimpiade Matematika SMP - Persiapan OSN Page 123


Contoh Soal:
1. Nilai dari (OSK 2011)

a. 113/10! c. 73/10! e. 4/10!


b. 91/10! d. 71/10!
Penyelesaian:

2. Menggunakan angka-angka 1,2,5,6 dan 9 akan dibentuk bilangan genap yang


terdiri dari lima angka. Jika tidak ada angka yang berulang, maka selisih
bilangan terbesar dan terkecil adalah … (OSK 2011)
a. 70820 c. 80952 e. 83916
b. 79524 d. 81236
Penyelesaian:
Bilangan genap terbesar yang memenuhi adalah 96512
Bilangan genap terbesar yang memenuhi adalah 12596
sehingga selisihnya adalah –
3. Hasil penjumlahan adalah suatu bilangan yang angka
satuannya adalah … (OSK 2011)
a. 3 c. 5 e.
b. 4 d.
Penyelesaian:

sehingga agka satuan dari adalah satuan dari


, yaitu

Materi Olimpiade Matematika SMP - Persiapan OSN Page 124


4. Diketahui : , , , dengan
. Pengurutan yang benar dari ketiga bilangan di atas adalah ….
a. c. e.
b. d. :
Penyelesaian:

sehingga

sehingga

sehingga
sehingga
5. Rata-rata usia kelompok guru dan professor adalah 40 tahun. Jika rata-rata
kelompok guru adalah 35 tahun, sedangkan rataan kelompok professor adalah
50 tahun. Perbandingan banyaknya guru dan profesor adalah … (OSK
2011)
a. 2:1 c. 3:2 e. 3:4
b. 1:2 d. 2:3
Penyelesaian:
Misalkan: adalah banyaknya guru
adalah banyaknya profesor
Rata-rata usia guru maka jumlah usia guru
Rata-rata usia profesor maka jumlah usia guru
Rata-rata usia guru dan profesor
sehingga

6. Sebuah kotak berisi bola merah dan bola hijau. Jika empat bola merah
dikeluarkan dari kotak maka sepersepuluh sisanya adalah bola merah. Akan
tetapi, jika empat bola hijau dikeluarkan dari kotak maka seperlima sisanya
adalah bola merah. Banyaknya bola merah adalah … (OSK 2010)

Materi Olimpiade Matematika SMP - Persiapan OSN Page 125


Penyelesaian:
Misalkan: adalah banyaknya bola merah dalam kotak
adalah banyaknya bola hijau dalam kotak
……………(1)

……………(2)

dari (1) dan (2) diperoleh bahwa ……………(3)

dari (1) dan (3) diperoleh bahwa

7. Dalam sebuah kotak berisi 15 telur, 5 telur diantaranya rusak. Untuk


memisahkan telur baik dan telur yang rusak dilakukan pengetesan satu persatu
tanpa pengembalian. Peluang diperolehnya telur rusak ke 3 pada pengetesan ke
adalah … (OSK 2011)
a. b. c. d. e.

b. Penyelesaian:
Kemung Pengetesan ke- Peluang
kinan 1 2 3 4 5

I B B R R R

II B R B R R

III B R R B R

IV R R B B R

V R B R B R

VI R B B R R

Peluang diperolehnya telur


rusak ke 3 pada pengetesan
ke 5

Materi Olimpiade Matematika SMP - Persiapan OSN Page 126


8. Jika nilai ulangan siswa kelas VIII terdiri dari bilangan genap berurutan dengan
nilai terendah dan tertinggi , jangkauan interkuartil dari data tersebut
adalah …. (OSK 2009) 50
Penyelesaian:
Nilai ulangan siswa adalah 2,4,6,8,…,94,96,98
Banyaknya data adalah 49 buah
Jangkauan interkuartil dari data tersebut terlelak pada data ke

Nilai siswa yang terletak pada urutan ke-25 adalah 50.


9. Misalkan . Jika keempat anggota S diambil secara acak,
maka peluang terambilnya empat bilangan yang berjumlah genap adalah …
(OSK 2009)
a. 2/5 c. 11/21
b. ½ d. 2/3
Penyelesaian:
Kemungkinan kempat bilangan yang diambil dan berjumlah genap adalah
keempatnya genap atau keempatnya ganjil atau dua bilangan genap dan dua
bilangan ganjil, sehingga peluangnya dapat ditentukan dengan

10. Seorang pedagang menjajakan 10 jeruk manis dan 5 jeruk masam yang
kesemuanya terlihat sama dan diletakkan dalam keranjang yang sama pula. Jika
Ana ingin membeli dua buah jeruk dan mengambilnya sekaligus secara
sembarang, maka peluang Ana akan memperoleh dua jeruk dengan rasa yang
sama adalah … (OSK 2008)
a. 1/21 c. 2/15 e. 11/21
b. 1/105 d. 11/15

Materi Olimpiade Matematika SMP - Persiapan OSN Page 127


Penyelesaian:

11. Dua mata uang dilempar empat kali berturut-turut. Peluang muncul angka pada
pelemparan keempat adalah … (OSK 2007)
a. c. e.

b. d.

Penyelesaian:
Sisi mata uang yang muncul
Pelemparan
Mata uang Mata uang
ke-
pertama kedua
A A
A G
4
G A
G G
Peluang muncul angka pada pelemparan keempat (berarti muncul angka pada
kedua mata uang) adalah

12. Dalam suatu pertemuan terjadi 28 jabat tangan (salaman). Setiap dua orang
saling berjabat tangan paling banyak sekali. Banyaknya orang yang hadir dalam
pertemuan tersebut paling sedikit adalah … (OSK 2006)
a. 28 c. 14 e. 7
b. 27 d. 8
Penyelesaian:
Misalkan: jumlah orang yang hadir adalah n
banyaknya salaman adalah
sehingga

Materi Olimpiade Matematika SMP - Persiapan OSN Page 128


atau
Banyaknya orang yang hadir dalam pertemuan tersebut paling sedikit adalah 8
orang
13. Nomor polisi mobil-mobil di suatu Negara selalu terdiri atas empat angka. Jika
jumlah keempat angka pada setiap nomor juga habis dibagi 5, mobil yang
didaftar di negara tersebut paling banyak adalah …
a. 900 c. 2.000 e. 5.000
b. 1800 d. 4.500
Penyelesaian:

14. Agung mendapatkan bahwa nilai rata-rata dari tiga ulangan matematika yang
diikutinya adalah 81. Nilai ulangan pertama adalah 85. Nilai ulangan ketiga lebih
rendah 4 daripada nilai ulangan kedua. Berapakah nilai ulangan kedua Agung?
Penyelesaian:
Misalkan nilai ulangan ke-2 Agung adalah

maka

15. Dalam sebuah kotak terdapat 5 bola merah dan 10 bola putih. Jika diambil dua
bola secara bersamaan, peluang memperoleh dua bola yang berwarna sama
adalah …
a. c. e.

b. d.

Materi Olimpiade Matematika SMP - Persiapan OSN Page 129


Penyelesaian:

16. Upik melempar dadu, ia menghitung peluang terjadinya jumlah mata dadu
yang sama dengan 6. Untuk berapakah peluang tersebut paling besar?

Penyelesaian:
Misalkan maka
maka
maka

maka

maka
maka

maka
Jadi peluang terbesar adalah

Latihan Soal
1. Didefinisikan n! n.(n  1).(n  2)..3..2.1 . Bilangan bulat n yang memenuhi

n!  21035.52.7.11 adalah ... ( OSP 2007 )

2. Nilai dari (OSK 2011)

a. 113/10! c. 73/10! e. 4/10!


b. 91/10! d. 71/10!
3. Hasil penjumlahan adalah suatu bilangan yang angka
satuannya adalah …. (OSK 2011)
a.3 b. 4 c. 5 d. e.

Materi Olimpiade Matematika SMP - Persiapan OSN Page 130


4. Hitunglah atau sederhanakanlah bentuk dari : 1.1! + 2.2! + 3.3! + … +
2012.2012!

5. Diketahui : , , , dengan
. Pengurutan yang benar dari ketiga bilangan di atas adalah ….
a. c. e.
b. d.
6. Rata-rata usia kelompok guru dan professor adalah 40 tahun. Jika rata-rata
kelompok guru adalah 35 tahun, sedangkan rataan kelompok professor adalah
50 tahun. Perbandingan banyaknya guru dan profesor adalah
a. 2:1 b.1:2 c. 3:2 d. 2:3 e. 3:4
7. Sebuah kotak berisi bola merah dan bola hijau. Jika empat bola merah
dikeluarkan dari kotak maka sepersepuluh sisanya adalah bola merah. Akan
tetapi, jika empat bola hijau dikeluarkan dari kotak maka seperlima sisanya
adalah bola merah. Banyaknya bola merah adalah …
8. Dalam sebuah kotak berisi 15 telur, 5 telur diantaranya rusak. Untuk
memisahkan telur baik dan telur yang rusak dilakukan pengetesan satu persatu
tanpa pengembalian. Peluang diperolehnya telur rusak ke 3 pada pengetesan ke
adalah …
a. b. c. d. e.

9. Jika nilai ulangan siswa kelas VIII terdiri dari bilangan genap berurutan dengan
nilai terendah=2 dan tertinggi=98, jangkauan interkuartil dari data tersebut
adalah …. (OSK 2009)
10. Misalkan . Jika keempat anggota S diambil secara acak,
maka peluang terambilnya empat bilangan yang berjumlah genap adalah …
(OSK 2009)
a. 2/5 b.½ c. 11/21 d. 2/3
11. Seorang pedagang menjajakan 10 jeruk manis dan 5 jeruk masam yang
kesemuanya terlihat sama dan diletakkan dalam keranjang yang sama pula. Jika
Ana ingin membeli dua buah jeruk dan mengambilnya sekaligus secara
sembarang, maka peluang Ana akan memperoleh dua jeruk dengan rasa yang
sama adalah… (OSK 2008)
a.1/21 b.1/105 c. 2/15 d. 11/15 e. 11/21

Materi Olimpiade Matematika SMP - Persiapan OSN Page 131


12. Dua mata uang dilempar empat kali berturut-turut. Peluang muncul angka pada
pelemparan keempat adalah …
a. b. c. d. e.

13. Dalam suatu pertemuan terjadi 28 jabat tangan (salaman). Setiap dua orang
saling berjabat tangan paling banyak sekali. Banyaknya orang yang hadir dalam
pertemuan tersebut paling sedikit adalah …
14. Nomor polisi mobil-mobil di suatu Negara selalu terdiri atas empat angka. Jika
jumlah keempat angka pada setiap nomor juga habis dibagi 5, mobil yang
didaftar di negara tersebut paling banyak adalah …
15. Agung mendapatkan bahwa nilai rata-rata dari tiga ulangan matematika yang
diikutinya adalah 81. Nilai ulangan pertama adalah 85. Nilai ulangan ketiga lebih
rendah 4 daripada nilai ulangan kedua. Berapakah nilai ulangan kedua Agung?
16. Dalam sebuah kotak terdapat 5 bola merah dan 10 bola putih. Jika diambil dua
bola secara bersamaan, peluang memperoleh dua bola yang berwarna sama
adalah …
a. b. c. d. e.

17. Upik melempar dadu, ia menghitung peluang terjadinya jumlah mata dadu
yang sama dengan 6. Untuk berapakah peluang tersebut paling besar?
18. Nomor rumah disepanjang suatu jalan merupakan dua angka tanpa angka nol,
berapakah banyaknya rumah yang bernomor genap?
19. Berapa banyak bilangan genap tiga angka yang digit-digit dapat dibentuk dari
himpunan {0, 2, 4, 6, 7} yang kurang dari 700 dan digit-digitnya tak berulan.
20. Menggunakan angka-angka 1,2,5,6 dan 9 akan dibentuk bilangan genap yang
terdiri dari lima angka. Jika tidak ada angka yang berulang, maka selisih
bilangan terbesar dan terkecil adalah … (OSK 2011)
a.70820 b.79524 c. 80952 d. 81236 e. 83916
21. Wati menuliskan suatu bilangan yang terdiri dari 6 angka (6 digit) di papan
tulis, tetapi kemudian Iwan menghapus 2 buah angka 1 yang terdapat pada
bilangan tersebut sehingga bilangan yang terbaca menjadi 2002.Berapa banyak
bilangan dengan enam digit yang dapat Wati tuliskan agar hal seperti diatas
dapat terjadi?

Materi Olimpiade Matematika SMP - Persiapan OSN Page 132


22. Dari gambar dibawah, banyaknya cara yang dapat dilalui dari A ke C tanpa
melalui B dengan syarat hanya diperbolehkan berjalan ke kanan atau ke bawah
saja adalah…cara.

SOAL LATIHAN STATISTIKA


A. PENYAJIAN DAN PENAFSIRAN DATA
a. Pengumpulan, pengolahan, penginterpretasian, dan penyajian data hasil
pengamatan dalam bentuk tabel, diagram, dan grafik
{OSP 2008}
Sebuah boxplot, seperti gambar, biasanya digunakan untuk menampilkan data yang
menunjukkan nilai kuartil bawah, media, kuartil atas dan rentangnya.

Pada ruas garis yang panjangnya 10 cm, titik digunakan untuk menyatakan
data bernilai 0, titik untuk data bernilai 100. Jika diberikan data sebagai berikut:
65, 70, 67, 82, 71, 25, 83, 78, 58, 72, 94, 66, 86, 73, 71, 31, 71, 87, 65, 76, 86, 66, 98,
74, 84, 96, 100, 73 maka gambar boxplot data tersebut pada garis adalah …
b. Penerapkan pola dan generalisasi untuk membuat prediksi
c. Ukuran pemusatan
Contoh Soal OSP
{OSP 2013}
Rata-rata nilai dari 25 siswa adalah 40. Jika selisih rata-rata nilai 5 siswa terendah
dan 20 siswa sisanya adalah 25, maka nilai rata-rata siswa terendah adalah …

Materi Olimpiade Matematika SMP - Persiapan OSN Page 133


{OSP 2012}
Jika rata-rata 1000 bilangan ganjil positif berturutan adalah 2012, maka bilangan
terkecil dari bilangan-bilangan tersebut adalah …
{OSP 2011}
Tiga bilangan dan dipilih sehingga ketika setiap bilangan ditambahkan ke rata-
rata dua bilangan lainnya maka berturut-turut hasilnya adalah 80, 90 dan 100. Rata-
rata dari dan adalah …
{OSP 2009}
Tes matematika diberikan kepada tiga kelas dengan siswa yang berjumlah 100
orang. Nilai rata-rata kelas pertama, kedua dan ketiga masing-masing 7, 8 dan .

Jika banyak siswa pada kelas pertama 25 orang, dan banyak siswa pada kelas ketiga
5 orang lebih banyak dari jumlah siswa kelas kedua, maka nilai rata-rata seluruh
siswa tersebut adalah …
{OSP 2008}
Sebelas orang anggota tim sepakbola ditimbang secara berturutan. Setelah dua
orang selesai ditimbang, dihitung rata-ratanya. Begitu pula ketika orang ketiga
ditimbang, rata-rata baru dihitung kembali. Demikian seterusnya. Diketahui nilai
rata-rata ini selalu meningkat satu kilogram sampai semua pemain selesai
ditimbang. Selisih berat badan pemain yang paling berat dengan pemain yang
ditimbang pada urutan ketiga adalah … kilogram
{OSP 2006}
Rata-rata nilai matematika dari 24 siswa adalah 7,20. Setelah ditambah nilai dari 2
siswa, rata-ratanya menjadi 7,25. Jika nilai salah satu dari kedua siswa itu adalah
7,6 , maka nilai satu siswa yang lain adalah …
{OSP 2005}
Tiga bilangan bulat membentuk kumpulan data yang berata-rata 10. Banyaknya
kombinasi bilangan yang (sebutkan pula datanya), jika diketahui selisih data
terbesar dan terkecilnya tidak lebih dari 4 adalah …

Materi Olimpiade Matematika SMP - Persiapan OSN Page 134


Contoh Soal OSN
{OSN 2011}
Diberikan himpunan bilangan asli yang pertama. Jika salah satu bilangan dihapus,
maka rata-rata bilangan yang tersisa adalah . Tentukan bilangan yang dihapus

tersebut.
{OSN 2005}
Adi adalah karyawan pada salah satu perusahaan tekstil yang bertugas menyimpan data. Suatu ketika
Adi diminta pimpinan perusahaan untuk menyiapkan data tentang kenaikan produksi selama lima
periode. Setelah dicari Adi hanya menemukan empat data kenaikan, yaitu 4%, 9%, 7%, dan 5%. Satu
data lagi, yaitu data ke-5 tidak ditemukan. Selidiki data kenaikan produksi yang ke-5, bila Adi hanya
ingat bahwa rata-rata hitung dan medium dari kelima data tersebut adalah sama!

B. PELUANG KEJADIAN
Contoh Soal OSP
{OSP 2012}
Suatu tes matematika terdiri dari 5 soal pilihan ganda dengan lima pilihan dan
hanya satu pilihan yang benar. Jika Mulan menjawab soal secara menerka (secara
acak atau asal-asalan), maka peluang tepat dua soal dijawab dengan benar adalah …
a. b. c. d. e.

{OSP 2012}
Dalam sebuah karung terdapat 60 kaos bernomor 11,12,13,…,40. Ada 2 kaos untuk
setiap nomor (nomor 11 ada 2 kaos, nomor 12 ada 2 kaos dan seterusnya). Jika
diambil 2 kaos secara acak, maka peluang yang terambil adalah kaos yang bernomor
sama adalah …
a. b. c. d. e.

{OSP 2012}
Sehabis belanja, Ratina membawa pulang uang kembalian berupa 8 koin (uang
receh), yang terdiri dari ratusan, lima-ratusan, dan ribuan. Total nilai uang
kembalian adalah tiga ribu rupiah. Sayangnya, dalam perjalaan pulang salah satu
uang koin jatuh (hilang). Jika peluang kehilangan untuk satu ratusan, satu lima-
ratusan, dan satu ribuan adalah sama, maka peluang kehilangan satu lima ratusan
adalah …

a. b. c. d. e.

Materi Olimpiade Matematika SMP - Persiapan OSN Page 135


{OSP 2009}
Dalam sebuah kantong terdapat 5 bola warna putih, 2 bola warna hijau, dan 3 bola
warna merah. Akan diambil 3 bola secara satu persatu dengan pengembalian.
Artinya bila bola sudah diambil dikembalikan ke dalam kantong tersebut. Peluang
ketiga bola yang terambil berwarna hijau adalah …
{OSP 2007}
Dua dadu bersisi enam diberi nomor baru pada setiap sisinya. Dadu pertama diberi
nomor 1, 1, 2, 3, 3, 3 dan dadu kedua diberi nomor . Jika
kedua dadu dilempar bersamaan, maka peluang terjadinya jumlah bilangan pada
kedua sisi atas dadu bernilai positif adalah …
a. b. c. d. e.

{OSP 2006}
Indonesia akan mengirim delegasi Olimpiade Sains Internasional (OSI tingkat SMP
pada tahun 2006. Delegasi ini terdiri atas tiga siswa SMP yang harus dipilih secara
acak dari 10 kandidat yaitu enam siswa bidang Sains dan empat siswa bidang
Matematika. Berapa peluang terpilihnya delegasi OSI yang terdiri dari 2 siswa
bidang Sains dan 1 siswa dari bidang Matematika?
{OSP 2003}
Ada satu keluarga terdiri dari Ayah, Ibu dan tiga orang anak. Ibu lahir pada bulan
April. Berapakah peluang ada tepat satu orang anggota lain dalam keluarga tersebut
yang lahir juga di bulan April?
{OSP 2012}
Empat bola bernomor 1,2,3, dan 4 diletakkan dalam sebuah kotak. Sebuah bola
diambil secara acak dari kotak tersebut. Nomor yang muncul dicatat, kemudian bola
dikembalikan ke kotak semula. Jika proses pengambilan bola dilakukan sampai tiga
kali dengan cara serupa, maka peluang nomor bola yang terambil berjumlah 5
adalah …

a. b. c. d. e.

Materi Olimpiade Matematika SMP - Persiapan OSN Page 136


Contoh Soal OSN
{OSN 2013}
Terdapat tiga buah kotak A; B dan C masing - masing berisi tiga bola berwarna putih
dan dua bola berwarna merah. Selanjutnya dilakukan pengambilan tiga bola dengan
aturan sebagai berikut :
1. Tahap ke-1
Ambil satu bola dari kotak A.
2. Tahap ke-2
Jika bola yang terambil dari kotak A pada Tahap ke-1 berwarna putih, maka
bola tersebut dimasukkan ke kotak B. Selanjutnya dari kotak B diambil satu bola.
Jika yang terambil adalah bola berwarna putih maka bola tersebut dimasukkan ke
kotak C . Sedangkan, jika yang terambil adalah bola berwarna merah maka bola
tersebut dimasukkan ke kotak A.
Jika bola yang terambil dari kotak A pada Tahap ke-1 berwarna merah, maka
bola tersebut dimasukkan ke kotak C . Selanjutnya dari kotak C diambil satu bola.
Jika yang terambil adalah bola berwarna putih maka bola tersebut dimasukkan ke
kotak A. Sedangkan, jika yang terambil adalah bola berwarna merah maka bola
tersebut dimasukkan ke kotak B.
3. Tahap ke-3
Ambil satu bola dari masing - masing kotak A; B dan C .
Berapa peluang bahwa semua bola yang terambil pada Tahap ke-3 berwarna merah?
{OSN 2008}
Bilangan empat angka akan dibentuk dari angka-angka pada {0,1,2,3,4,5} dengan
syarat angka-angka pada bilangan tersebut tidak berulang, dan bilangan yang
terbentuk merupakan kelipatan 3. Berapakah peluang bilangan yang terbentuk
mempunyai nilai kurang dari 3000?
{OSN 2006}
Satu set soal terdiri dari 3 soal dengan pilihan jawaban Benar (B) atau Salah (S),
serta 3 soal pilihan ganda dengan jawaban A, B, C, atau D. Seseorang menjawab
semua soal secara acak. Berapa peluang ia hanya benar 2 soal?

Materi Olimpiade Matematika SMP - Persiapan OSN Page 137


(OSN 2014)

C. ATURAN PENCACAHAN
a. Aturan penjumlahan dan aturan perkalian
Contoh Soal OSP
{OSP 2013}
Tim sepakbola terdiri atas 25 orang, masing-masing diberi kaor bernomor 1 sampai
dengan 25. Banyak cara memilih tiga pemain secara acak dengan syarat jumlah
nomor kaos mereka habis dibagi tiga …
{OSP 2013}
Suatu byte didefinisikan sebagai susunan angka yang terdiri dari 8 angka (digit),
yaitu 0 atau 1. Contoh byte: 01110111. Banyak jenis byte yang memuat angka angka
1 tepat sebanyak adalah … a. 30 b. 45 c. 56 d. 62 e. 66
{OSP 2007}
Final lomba renang wanita 400 meter diikuti oleh 4 orang dinalis, yaitu Anita,
Bonita, Cantika, dan Dita. Diperoleh informasi bahwa Bonita selalu kalah bertarung
dengan Anita dan Cantika, namun selalu menang bertarung dengan Dita. Jika dalam
perlombaan tersebut akan ditentukan peraih medali emas, perak dan perunggu,
maka susunan dari penerima medali adalah … a. 6 b. 10 c. 12 d. 13 e. 14
{OSP 2006}
Nomor telepon di kota Malang terdiri dari enam angka. Banyaknya nomor telpon di
kota itu yang habis dibagi adalah …
{OSP 2004}
Bilangan 10-angka terbesar menggunakan empat angka 1, tiga angka 2, dua angka 3
dan satu angka 4, sehingga dua yang yang sama tidak terletak bersebelahan adalah…
{OSP 2004}
Sebuah jam digital angka-angka yang tertera mulai dari 00:00 sampai dengan 23:59.
Jam itu dapat menampilkan Palindrome (bilangan yang dibaca dari depan dan dari

Materi Olimpiade Matematika SMP - Persiapan OSN Page 138


belakang sama nilainya, misalnya 12:21 dan 23:32). Dalam satu hari satu malam,
banyaknya bilangan Palindrome yang muncul adalah …
{OSP 2006}
Lima orang pemuda pergi berekreasi menggunakan sebuah mobil. Mobil yang
digunakan memiliki dua tempat duduk di depan (termasuk untuk pengemudi) dan
tiga tempat duduk di belakang. Dari kelima pemuda tersebut hanya dua orang yang
bisa menjadi pengemudi. Banyak cara mereka duduk di mobil adalah …
(OSP 2014)

Contoh Soal OSN


{OSN 2013}
Pada suatu acara diundang 13 tamu istimewa yang terdiri dari 8 orang pria dan 5
orang wanita. Khusus untuk semua tamu istimewa tersebut disediakan 13 tempat
duduk pada satu baris khusus. Jika diharapkan tidak ada dua orang wanita yang
duduk bersebelahan, tentukan banyak posisi duduk yang mungkin untuk semua
tamu istimewa tersebut.
{OSN 2013}
Sebuah tabel yang berukuran n baris dan n kolom akan diisi dengan bilangan 1 atau
-1 sehingga hasil kali semua bilangan yang terletak dalam setiap baris dan hasil kali
semua bilangan yang terletak dalam setiap kolom adalah 1. Berapa banyak cara
berbeda untuk mengisi tabel tersebut?

Materi Olimpiade Matematika SMP - Persiapan OSN Page 139


{OSN 2009}
Perhatikan gambar berikut. Huruf-huruf a, b, c, d, dan e di dalam kotak akan diganti
dengan bilangan 1, 2, 3, 4, 5, 6, 7, 8 atau 9. Bila bilangan yang menggantikan a, b,c,d,
dan e harus berlainan dan ae = bd, ada berapa susunan yang mungkin terjadi?
A B
C
D E
{OSN 2009}
Suatu nomor telepon dengan 7 angka disebut Nomor Cantik bilamana angka-angka
yang muncul pada tiga angka pertama (ketiga angkanya harus berbeda) berulang
pada tiga angka berikutnya atau tiga angka terakhirnya. Contoh beberapa Nomor
Cantik: 7133719, 7131735, 7130713, 1739317, 5433354. Jika angka-angkanya
diambil dari 0, 1, 2, 3, 4, 5, 6, 7, 8, atau 9, tetapi angka pertama tidak boleh 0,
berapakah banyak Nomor Cantik yang bisa diperoleh.
{OSN 2009}
Pada turnamen catur, seorang pemain hanya bermain satu kali dengan pemain
lainnya. Seorang pemain memperoleh nilai 1 jika menang, 0 jika kalah, dan 1/2 jika
imbang. Setelah kompetisi berakhir, diketahui bahwa 1/2 total nilai yang diperoleh
oleh setiap pemain didapatkan dari bermain dengan 10 pemain yang mendapatkan
total poin terendah. Khusus untuk yang berada di peringkat sepuluh terbawah, 1/2
total poin yang diperolehnya didapatkan dari bermain dengan 9 pemain lainnya.
Berapakah banyak pemain dalam kompetisi tersebut?
{OSN 2008}
Diberikan tabel sebagai berikut.

Tabel 4 x 4 ini merupakan gabungan empat bagian tabel yang lebih kecil berukuran
2x2. Tebel ini akan diisi dengan empat bilangan bulat berbeda demikian sehingga:
 Jumlah mendatar bilangan-bilangan pada setiap barisnya sama dengan 10
 Jumlah vertikal bilangan-bilangan pada setiap kolomnya sama dengan 10

Materi Olimpiade Matematika SMP - Persiapan OSN Page 140


 Jumlah empat bilangan dalams etiap bagian 2x2 yang dibatasi oleh garis
tebal tersebut juga sama dengan 10
Tentukan berapa banyaknya susunan yang mungkin terbentuk
{OSN 2007}
Ada 13 kado berbeda yang akan dibagikan semuanya kepada Ami, Ima, Mai, dan
Mia. Jika Ami mendapat paling sedikit 4 kado, Ima dan Mai mesing-masing mendapat
paling sedikit 3 kado, dan Mia mendapat paling sedikit 2 kado, ada beberapa banyak
susunan kado yang mungkin diperoleh?
{OSN 2007}
Ucok bermaksud menyusun suatu kode kunci (password) yang terdiri atas 8 angka
dan memenuhi ketentuan berikut:
i. Angka yang dipakai adalah 1, 2, 3, 4, 5, 6, 7, 8, dan 9
ii. Angka pertama yang dipakai adalah minimal 1, angka kedua minimal 2,
angka ketiga minimal 3, dan seterusnya.
iii. Angka yang sama bisa digunakan beberapa kali.
a. Berapa banyak password berbeda yang mungkin disusun Ucok?
b. Berapa banyak password berbeda yang mungkin disusun Ucok jika ketentuan
(iii) diganti dengan tidak boleh ada angka yang digunakan lebih dari satu kali.

Permutasi dan kombinasi


Contoh Soal OSP
{OSP 2013}
Dalam sebuah kotak terdapat beberapa bola dengan empat macam warna yakni:
biru, merah, kuning dan putih. Paling sedikit terdapat 10 bola untuk masing-masing
warna. Bola diambil satu demi satu dari dalam kotak tersebut secara acak tanpa
pengembalian. Banyak pengambilan yang harus dilakukan untuk memastikan
mendapatkan 6 bola dengan warna sama adalah …
{OSP 2013}
Lima orang guru akan ditempatkan pada tiga sekolah yang berbeda, 2 orang di
sekolah pertama, 2 orang di sekolah kedua, dan 1 orang di sekolah ketiga. Banyak
cara menempatkan kelima orang guru tersebut…
a. 40 b. 30 c. 20 d. 10 e. 4

Materi Olimpiade Matematika SMP - Persiapan OSN Page 141


{OSP 2012}
Suatu antrian pembelian tiket masuk pertandingan sepak bola terdiri dari 2012
orang. Jika diantara 2 pria paling sedikit terdapat 3 wanita, maka banyak pria pada
antrian tersebut paling banyak adalah …
a. 501 b. 502 c. 503 d. 504 e. 505
{OSP 2004}
Enam belas tim sepakbola mengikuti turnamen. Pertama-tama mereka
dikelompokkan ke dalam 4 kelompok dengan masing-masing 4 tim di setiap
kelompoknya. Di setiap kelompok mereka saling bermain satu sama lain satu kali.
Dua tim yang memiliki peringkat teratas selanjutnya maju babak berikutnya yang
menggunakan sistem gugur (kalah langsung tereliminasi) sampai ditemukan
juaranya. Berapa banyak pertandingan yang berlangsung dalam turnamen tersebut?

Contoh Soal OSN


{OSN 2005}
Tiga orang hendak makan siang di suatu rumah makan. Untuk menentukan
siapakah yang membayar mereka membuat suatu permainan. Masing-masin
mengetos satu koin secara bersama-sama. Jika hasilnya muka atau belakang semua,
maka mereka megetos lagi. Jika tidak demikian, maka “orang ganjil” (yaitu orang
yang koinnya muncul berbeda dari dua orang lainnya) yang membayar. Tentukan
banyaknya semua hasil yang mungkin, jika permainan berakhir pada pengetosan:
a. Pertama.
b. Kedua.
c. Ketiga.
d. Kesepuluh.

{OSN 2004}
Sebuah dadu biasa 6-muka dilempar tiga kali. Berapakah besarnya kemungkinan
jumlah mata dadu pada ketiga lemparan adalah 12?

Materi Olimpiade Matematika SMP - Persiapan OSN Page 142


E. KAPITA SELEKTA

No. Kompetensi Materi Lingkup Materi


5. Menyelesaikan masalah Kapita Pemecahan masalah
matematika Selekta kontekstual yang berkaitan
dengan bilangan, aljabar,
geometri, statistika, dan
peluang.

Pemecahan Masalah Kontekstual Yang Berkaitan Dengan Aljabar


1. OSP 2003
Kapasitas tangki bahan bakar suatu mobil adalah 40 liter. Setiap menempuh
perjalanan sejauh 100 km, mobil tersebut menghabiskan 7,7 liter bahan
bakar. Suatu waktu, mobil tersebut digunakan untuk pergi dari Bandung ke
Yogyakarta yang jaraknya 428 km, Ketika memulai perjalanan, tangki mobil
tersebut terisi penuh bahan bakar. Dalam satuan liter terdekat, banyaknya
bahan bakar yang tersisa ketika tiba di Yogyakarta adalah……
2. OSP 2003
Suatu botol dengan kapasitas 875 mililiter digunakan untuk mengisikan
minyak ke dalam suatu jerigen berkapasitas 20 liter. Berapa kalikah botol
tersebut digunakan untuk membuat penuh sebuah jerigen kosong?
3. OSP 2005
Dalam menentukan jawab perkalian bilangan 1493 dan 1507, seorang anak
mengurangkan langsung 49 dari 2.250.000. Dia sama sekali tidak
mengalikan kedua bilangan tersebut dengan panjang. Prinsip matematika
yang digunakan oleh anak tersebut adalah ......

4. OSP 2005
Untuk setiap pasangan bilangan asli a dan b, kita definisikan a*b = ab + a - b.
Bilangan asli x dikatakan penyusun penyusun bilangan asli n jika terdapat
bilangan asli y yang memenuhi x*y = n. Sebagai contoh, 2 adalah penyusun 6,
karena terdapat bilangan asli 4 sehingga 2*4 = 2.4 + 2 - 4 = 8 + 2 - 4 = 6.
Tentukan semua penyusun 2005.
9. OSP 2008

Materi Olimpiade Matematika SMP - Persiapan OSN Page 143


10. OSP 2008

11. OSN 2008


Misalkan P adalah himpunan semua bilangan bulat positif antara 0 dan 2008
yang dapat dinyatakan sebagai jumlah dari dua bilangan bulat positif
berurutan atau lebih.(Contoh:11=5+6,90=29+30+31,100=18+19 +20+21+22
Jadi 11, 90, 100 adalah beberapa anggota P). Tentukan jumlah dari semua
anggota P!
12. OSP 2009

13. OSN 2009


Perhatikan gambar berikut. Huruf-huruf a, b, c, d, dan e di dalam kotak akan
diganti dengan bilangan 1, 2, 3, 4, 5, 6, 7, 8 atau 9. Bila bilangan yang
menggantikan a, b,c,d, dan e harus berlainan dan ae = bd, ada berapa
susunan yang mungkin terjadi?
A B
C
D E

14. OSP 2011


Saat ini umur Agus dan umur Fauzan kurang dari 100 tahun. Jika umur Agus
dan umur Fauzan ditulis secara berurutan, maka diperoleh suatu bilangan
empat digit (angka) yang merupakan kuadrat sempurna. Dua puluh tiga
tahun kemudian, jika umur mereka ditulis dengan cara yang sama, maka
diperoleh bilangan empat digit lain yang juga merupakan kuadrat sempurna.
Jika umur mereka diasumsikan merupakan bilangan bulat positif, berapakah
umur mereka saat ini?

Materi Olimpiade Matematika SMP - Persiapan OSN Page 144


15. OSP 2011
Data akhir suatu kompetisi yang diikuti oleh tiga tim sepakbola, masing-
masing tim saling berhadapan, dituliskan pada berikut.
Tim Menang Kalah Seri Gol (Memasukkan-Kemasukan)
Elang 1 0 1 5 2
Garuda 1 0 1 4 3
Merpati 0 2 0 3 7
Berapakah skor pertandingan antara Tim Garuda melawan Tim Merpati?

PEMECAHAN MASALAH KONTEKSTUAL YANG BERKAITAN DENGAN GEOMETRI


52. (OSP 2003)
Suatu botol dengan kapasitas 875 mililiter digunakan untuk mengisikan minyak
kedalam suatu jerigen berkapasitas 20 liter. Berapa kalikah botol tersebut
digunakan untuk membuat penuh sebuah jerigen kosong ?
53. (OSP 2004)
Setiap muka sebuah kubus diberi bilangan seperti pada gambar. Kemudian
setiap titik sudut diberi bilangan yang merupakan hasil penjumlahan bilangan
pada muka-muka yang berdekatan dengannya. Nilai tertinggi bilangan pada titik
sudut adalah .... 7

9 5 3 1

11

54. (OSN 2005) Pada gambar-gambar di bawah adalah: ”Gambar berikutnya


diperoleh dengan menambah gambar segitiga sama sisi berwarna hitam yang
ukuran sisinya setengah dari masing-masing segitiga warna putih yang tersisa
pada gambar selanjutnya”. Jika pola tersebut berkelanjutan (kontinu) sampai tak
hingga,

Gambar 1 Gambar 2 Gambar 3 Gambar 4


a. Jika diketahui luas segitiga sama sisi pada gambar 1 adalah 1 satuan luas,
tentukan luas keseluruhan daerah yang dibentuk oleh segitiga-segitiga hitam
pada gambar 5.

Materi Olimpiade Matematika SMP - Persiapan OSN Page 145


b. Andaikata anda diminta untuk menentukan luas keseluruhan luas daerah
yang dibentuk oleh segitiga-segitiga hitam pada gambar ke-20, rumus yang
bagaimanakah yang bisa anda gunakan
55. (OSP 2006)Sebuah ember terbuat dari seng seperti tampak pada gambar
(diameter alas 30 cm, diameter atas 50 cm, dan tinggi 40 cm). Luas seng yang
digunakan untuk membuat ember tersebut adalah ... cm2.
50

40

30
56. (OSP 2006)
Sebuah cerobong asap berbentuk kerucut terpancung, jari-jari alas cerobong
tersebut 5 meter, jari-jari atas 4 meter, dan tinggi 20 meter. Cerobong atas
tersebut akan dicat, biaya mengecat permeter persegi adalah Rp 5.000,-. Hitung
biaya pengecatan cerobong asap tersebut.
57. (OSN 2006) Dua orang petani, Si A dan Si B bermaksud mengubah batas tanah
mereka sehingga menjadi seperti garis lurus, tidak berbelok – belok seperti pada
gambar di bawah. Mereka tidak ingin luas daerah asalnya berkurang. Coba
tentukan garis batas yang seharusnya mereka sepakati, dan jelaskan mengapa
batas baru tersebut tidak mengurangi luas daerah asalnya masing – masing.

Si A

Si B

58. (OSP 2007)


Sebuah tugu akan dibangun dengan menumpuk kubus-kubus beton yang
rusuknya 50 cm, seperti tampak pada Gambar 6. Antar sisi-sisi kubus yang
berdempetan dan sisi kubus dengan lantai akan direkat dengan semen setebal 1
cm.
a. Jika tinggi tugu yang diinginkan adakah 51,51 m, berapa banyak kubus
beton yang diperlukan?

Materi Olimpiade Matematika SMP - Persiapan OSN Page 146


b. Jika semua permukaan kubus beton yang tampak dari tugu tersebut dicat
dengan harga pengecatan Rp 6.000,00 per meter persegi, tentukan besar
biaya yang diperlukan!

59. (OSP 2007)


Suatu kebun berbentuk persegi panjang seperti Gambar 7. Diketahui ukuran
AB = 15 m dan BC = 30 m. Seekor kucing berada pada titik A dan seekor tikus
berada pada titik E, yang merupakan titik tengah garis AD. Tikus berlari dengan
kecepatan 3 m/detik sepanjang garis lurus menuju titik C, sedangkan kucing
berlari dengan keceparan 5 m/detik sepanjang suatu garis lurus sedemikian
sehingga akan bertemu dengan tikus di titik F. Diketahui kucing dan tikus mulai
berlari pada saat yang bersamaan, dan di titik F kucing berhasil menerkam tikus.
Dalam berapa detik kucing berhasil menerkam tikus (bulatkan sampai 2 angka
desimal)? B C

D
A E

60. (OSP 2009)


Banyak Persegi pada gambar adalah …

61. (OSP 2008) Balok pejal ABCD.EFGH berukuran 15 cm × 10 cm × 6 cm. Titik P


terletak pada rusuk AB sedemikian sehingga AP = 3 cm. Seekor cicak yang ada di
sudut G akan menangkap nyamuk di P dengan merayap di permukaan balok. Jika

Materi Olimpiade Matematika SMP - Persiapan OSN Page 147


kecepatan cicak bergerak 2,5 cm/dtk, berapan waktu tercepat yang dibutuhkan
cicak agar dapat melahap nyamuk ?
62. (OSP 2009)
Perhatikan gambar!

Bangun ABCDEG adalah menunjukkan keadaan sebuah kamar . Keadaan


sebenarnya AD=DE, AB = 28 meter dan EF = 18 meter serta dengan luas kamar
adalah 624 m2. Jika sebuah penyekat dibuat dari E sampai C yang membagi luas
kamar menjadi dua bagian yang sama luas, maka jarak dari C ke G adalah…
63. (OSP 2013)
Sebuah drum berbentuk tabung yang berjari-jari 70 cm dan berisi air setinggi 40
cm (guanakan ). Seorang tukang pasang ubin memasukkan 110 buah ubin

keramik ke dalam drum sehingga setinggi permukaan air bertambah 8 cm. jika
permukaan setiap ubin keramik berukuran 40 cm x 40 cm, berapakah tebal ubin
keramik tersebut?
64. (OSN 2013)
Misalkan A, B dan P adalah paku-paku yang ditanam pada papan ABP. Panjang
AP = a satuan dan BP = b satuan. Papan ABP diletakkan pada lintasan X1X2 dan
Y1Y2 sehingga A hanya dapat bergerak bebas sepanjang lintasan X1X2 dan B hanya
bergerak bebas sepanjang lintasan Y 1Y2 seperti pada gambar berikut. Misalkan x
adalah jarak titik P terhadap lintasan Y1Y2 dan y jarak titik P terhadap lintasan

X1X2. Tunjukan bahwa persamaan lintasan titik P adalah

65. (OSP 2006)


Sebuah kebun berbentuk persegi panjang yang berukuran panjang 160 meter
dan lebar 50 meter. Di sepanjang tepi kebun dibangun parit dengan lebar yang
sama. Jika luas kebun tersebut sekarang menjadi ¾ luas kebun mula-mula, maka
lebar parit yang dibangun adalah ....

Materi Olimpiade Matematika SMP - Persiapan OSN Page 148


66. (OSP 2005)
Perhatikan gambar berikut. Andaikata Anda diminta untuk mencari luas daerah
di dalam kurva ABCDE. Jika jarak terdekat dua titik secara mendatar atau
vertikal adalah 5 cm, berapakah luas segilima ABCDE ?
B
       

A
       

       

D
       

 E      

       
C

67. (OSP 2005)


Perhatikan segi enam berikut ini. Banyaknya segitiga yang dapat ditemukan pada
gambar tersebut adalah:

68. (OSN 2012)


Di dalam Taman Khatulistiwa akan dibuat bangunan berbentuk limas dengan
alas segitiga sama sisi berbahan tembus pandang dengan panjang sisi alas 8 3 m
dan tinggi 8 cm. sebuah bola dunia akan ditempatkan di dalam limas tersebut.
Dengan mengabaikan ketebalan bahan pembuat limas, tentukan panjang
terbesar jari-jari bola dunia yang mungkin dapat dibuat.
69. (OSN 2010)
Lantai suatu aula berbentuk persegipanjang ABCD dengan AB = 30 meter dan
BC = 15 meter. Seekor kucing berada di posisi A. Melihat ada kucing, seekor tikus
yang berada di tengah-tengah AB lari dan mencoba menghindar dari kucing.
Tikus berlari dari tempatnya menuju titik C dengan kecepatan 3 m/dtk.
Lintasannya berbentuk garis lurus. Melihat tikus melarikan diri, dalam waktu

Materi Olimpiade Matematika SMP - Persiapan OSN Page 149


yang bersamaan dari titik A kucing pun mengejar dengan kecepatan 5
meter/detik. jika lintasan kucing juga berbentuk garis lurus dan tikus tertangkap
sebelum sampai di C, tentukan persamaan yang bisa digunakan untuk
menentukan posisi dan waktu tikus tertangkap kucing.

Pemecahan Masalah Kontekstual Yang Berkaitan Dengan Statistika Atau


Peluang
{OSP 2011}
Banyak bilangan 3 digit (angka) yang terdiri dari angka-angka 0,2,3,5,7,8 yang lebih
dari 243 dan kurang dari 780 adalah …
{OSP 2010}
Tentukan banyaknya cara membagikan 10 permen identik kepada tiga orang
sedemikian sehingga setiap orang sedikitnya mendapatkan satu permen
{OSP 2010}
Gambar berikut memberikan beberapa alternative jalan dari ke . sisi-sisi masing-
masing blok (persegi) menyatakan jalan dengan panjang satu satuan yang sama.
Tentukan banyaknya rute terpendek dari titik ke titik yang melalui titik 2,0,1,0
secara berturutan.

{OSP 2010}
Sebuah tabel permainan angka berukuran , setiap sel akan diisi dengan
bilangan 1 atau -1 sedemikian sehingga jumlah setiap baris dan setiap kolom adalah
0. Ada berapa banyak cara untuk menyusun tabel permainan yang dimaksud?
{OSP 2008}
Nia akan berulang tahun dalam waktu dekat. Karena keterbatasan biaya, dia hanya
mampu mengundang 10 dari 15 irang temannya. Diantara teman-temannya,
terdapat sahabat dekatnya, yaitu Ade,Dea, Ani dan Ina. Nia memutuskan bahwa Ade
dan Dea harus diundang. Tetapi, Ani dan Ina tidak mungkin diundang bersama-sama
karena mereka sedang berselisih faham. Banyak cara menentukan susunan nama-
nama yang akan diundang pada acara ulangtahun tersebut adalah …

Materi Olimpiade Matematika SMP - Persiapan OSN Page 150


{OSP 2008}
Dua puluh ubin yang kongruen akan disusun dalam 2 baris. Masing-masing baris
berisi 10 ubin. Diatara ubin-ubin tersebut terdapat 9 ubin bergambar bunga. Banyak
cara menyusun ubin tersebut agar sesame ubin bergambar bunga tidak saling
bersinggungan adalah …
{OSP 2008}
Pada suatu perusahaan, ada 3 lowongan pekerjaan yang disediakan hanya untuk
pekerja pria, 6 lowongan pekerjaan hanya untuk pekerja wanita, dan 4 lowongan
pekerjaan untuk pekerja pria atau wanita. Jika terdapat 20 pelamar dengan
komposisi 8 wanita dan 12 pria, maka …
{OSP 2007}
Suatu barisan berbentuk … (untuk dan
bilangan asli). Jika suku ke-7 barisan tersebut adalah 18, maka rata-rata dari 18
suku pertama barisan tersebut adalah …
{OSP 2007}
Kode sebuah ATM diketahui berupa bilangan lima digit dengan ciri-ciri sebagai
berikut.
 digit puluhan adalah dua kali lipat digit ribuan
 jika digit ratusan dan satuan dipertukarkan maka nilai bilangan tersebut
tidak berubah, dan
 digit puluh-ribuan adalah tidak nol
Jika pemilik kartu ATM tersebut lupa kodenya, tetapi ingat ciri-ciri tersebut maka
peluang ia dapat langsung menebaknya dengan benar adalah …
{OSP 2006}
Gambar di samping menunjukkan banyaknya siswa
dari kelas 7, kelas 8 dan kelas 9 yang mengikuti
kegiatan ekstrakurikuler sepakbola. Diketahui
bahwa banya siswa yang mengikuti kegiatan
tersebut semuanya adalah 28 orang. Dua orang
siswa dipilih secara acak untuk menjadi ketua dan
wakil ketua. Jika wakil ketua terlipih adalah siswa
kelas 7, maka peluang terpilihnya ketua yang
berasal dari kelas 9 adalah …

Materi Olimpiade Matematika SMP - Persiapan OSN Page 151


{OSP 2005}
Seseorang memiliki sejumlah koin 1000 rupiahan. Setelah diperhatikan dengan
seksama, ternyata koin yang dimilikinya terdiri dari 3 macam diantara 4 macam
koin sekarang yang masih berlaku (500-an, 200-an, 100-an). Selidiki dan tentukan
berapa banyak kombinasi koin yang mungkin dimiliki anak tersebut.
Contoh Soal OSN
{OSN 2011}
Dari pengukuran terhadap Sembilan pohon diperoleh data sebagai berikut.
a). Ada tiga hasil pengukuran yang berbeda
b). Semua data berupa bilangan positif
c). Mean = media = modus = 3
d). Jumlah kuadrat semua data adalah 87.
Tentukan semua kemungkinan ukuran Sembilan pohon tersebut.
{OSN 2008}
Alamat rumah di Jalan Bahagia hendak diberi nomor dengan aturan sebagai berikut.
Satu sisi jalan dinomori dengan nomor bilangan genap berurutan mulai dari nomor
2. Sisi seberang dinomori dengan nomor ganjil mulai dari nomor 3. Pada deretan
rumah bernomor genap, terdapat beberapa tanah kosong yang belum dibangun
rumah. Rumah pertama yang bernomor 2 memiliki tetangga di sebelahnya. Pada
waktu pengurus RT memesan nomor-nomor rumah tersebut, diketahui biaya
pembuatan setiap digitnya adalah Rp12.000,-. Untuk itu, total biaya yang harus
dikeluarkan adalah Rp 1.020.000,-. Diketahui pula bahwa biaya seluruh nomor
rumah sisi genap Rp132.000,- lebih murah dibanding sisi ganjil. Apabila tanah
kosong nanti sudah dibangun rumah, banyaknya rumah di sisi gena dan ganjil
adalah sama. Tentukan banyaknya rumah yang sekarangtelah ada di Jalan Bahagia
tersebut
{OSN 2007}
Hasil survey terhadap N orang dengan pertanyaan apakah mereka memelihara
anjing, burung, atau kucing di rumah adalah sebagai berikut: 50 orang memelihara
burung, 61 orang tidak memelihara anjing, 13 orang tidak memelihara kucing, dan
paling sedikit ada 74 orang yang memelihara paling sedikit dua jenis binatang di
rumah. Berapakah nilai maksimum dan minimum dari nilai N yang mungkin?

Materi Olimpiade Matematika SMP - Persiapan OSN Page 152


{OSN 2006}
Amir akan membuat lukisan dari lingkaran-lingkaran yang
setiap lingkarannya diisi dengan bilangan. Lukisan lingkaran
tersebut disusun mengikuti pola berikut. Dia membuat
aturan bahwa empat lngkaran terbawah akan diisi dengan
bilangan-bilangan positif kurang dari 10 yang dapat diambil
dari angka-angka pada tanggal kelahirannya, yakni 26-12-
1961 tanpa berulang. Sementara itu lingkaran-lingkaran di
atasnya akan diisikan dengan bilangan-bilangan yang
merupakan hasil kali dua bilangan pada lingkaran-lingkaran
di bawahnya.
a. Ada berapa carakah dia menempatkan bilangan-bilangan itu dari kiri ke
kanan pada lingkaran-lingkaran terbawah agar diperoleh nilai terbesar pada
lingkaran yang paling atas? Jelaskan!
b. Pada kesempatan yang lain, dia berencana memasukkan semua angka pada
tanggal kelahirannya tersebut sehingga jumlah lingkaran terbawah sekarang
harus sebanyak 8 lingkaran. Dia tidaklagi memperhatikan berulang tidaknya
bilangan-bilangan itu.
(1) Agar diperoleh nilai terkecil pada lingkaran yang paling atas,
bagaimanakah bilangan-bilangan itu disusun?
(2) Ada berapa susunan yang patut dipertimbangkan untuk menghasilkan
nilai terkecil?

{OSN 2003}
Untuk menarik minat pelanggan, suatu restoran penjual makanan cepat saji
memberikan kupon berhadiah kepada setiap orang yang membeli makanan di
restoran tersebut dengan nilai lebih dari Rp. 25.000. Di balik setiap kupon tersebut,
tertera salah satu dari bilanga-bilangan berikut. 9, 12, 42, 57, 69, 21, 15, 75, 24, dan
81. Pembeli yang berhasil mengumpulkan kupon dengan jumlah di balik kupon
tersebut sama dengan 100 akan diberi hadiah berupa TV 21”. Kalau pemilik restoran
tersebut menyediakan sebanyak 10 buat TV 21”, berapa banyak yang harus
diserahkan kepada para pelanggannya?

Materi Olimpiade Matematika SMP - Persiapan OSN Page 153


KUMPULAN SOAL - SOAL OSN
OLIMPIADE MATEMATIKA

TINGKAT KABUPATEN/ KOTA


TAHUN 2003 – 2019
Download di Folder OSN
https://folderosn.blogspot.co.id/
Download di Folder OSN
https://folderosn.blogspot.co.id/
Download di Folder OSN
https://folderosn.blogspot.co.id/
Download di Folder OSN
https://folderosn.blogspot.co.id/
OLIMPIADE SAINS NASIONAL SMP
SELEKSI TINGKAT KABUPATEN / KOTA TAHUN 2016

BAGIAN A : PILIHAN GANDA


( )
1. Nilai dari ( )
adalah …

A. 2012
B. 2013
C. 2014
D. 2015
2. Misalkan ⌈ ⌉ menyatakan bilangan bulat terkecil yang lebih besar daripada atau sama dengan x.

Jika , maka ⌈ ⌉ …

A. 35 B. 36 C. 37 D. 38
3. Jika ( – )( – ) 1
Maka ( – )( )
A. ( )
B. ( )
C. ( )
D.
4. Diketahui ABCD dan CEGH adalah dua persegipanjang kongruen dengan panjang 17 cm, dan
lebar 8 cm. Titik F adalah titik potong sisi AD dan EG. Luas segiempat EFDC adalah ...
A. 74,00
B. 72,25
C. 68,00
D. 63,75

5. Diketahui dua titik A(1,1) dan B(12, - 1). Garis l dengan gradien – ¾ melalui titik B. Jarak antara
titik A dan garis l adalah ... satuan panjang.
A. 4 C. 6
B. 5 D. 7
6. Perhatikan gambar di samping. Jika BE = 2 cm, EF = 6 cm, dan FC = 4 cm, maka panjang DE
adalah ... cm

A.

B.

C.

D.
7. Pada pagi hari yang cerah, suatu bola raksasa ditempatkan di tanah lapang yang datar. Panjang
bayangan bola tersebut apabila diukur dari titik singgung bola dengan tanah adalah 15 m. Di
samping bola tersebut terdapat tiang vertikal dengan tinggi 1m yang mempunyai bayangan
sepanjang 3 m. Radius bola tersebut adalah ... m.

A. C.
√ √

B. D.

8. Banyak bilangan real x yang memenuhi adalah …


A. 0 B. 1 C. 2 D. 3
9. Jika sistem persamaan
mx + 3y = 21
4x – 3y = 0
Memiliki penyelesaian bilangan bulat x dan y, maka nilai m+x+y yang mungkin adalah ...
A. 9 B. 10 C. 11 D. 12
10. Suatu survei dilakukan pada siswa kelas VII untuk mengetahui siswa yang berminat mengikuti
kegiatan Paskibra. Hasil survei adalah sebagai berikut:
 25% dari total siswa putra dan 50% dari total siswa putri ternyata berminat mengikuti
kegiatan tersebut;
 90% dari total peminat kegiatan Paskibra adalah siswa putri.
Rasio total siswa putri dan total siswa putra kelas VII di sekolah tersebut adalah ...
A. 9:1 B. 9:2 C. 9:3 D. 9:4

11. Suatu fungsi ditentukan dengan rumus ( ) {

Jika a adalah bilangan asli, maka nilai yang tidak mungkin untuk ( ) adalah …
A. 21
B. 39
C. 61
D. 77
12. Banyak bilangan bulat sehingga parabola y = + k tidak berpotongan dengan lingkaran
= 9 adalah ... .
A. 20
B. 19
C. 11
D. 10
13. Suatu perusahaan menjual dua jenis produk A dan B. Rasio hasil penjualan produk A dan B dari
tahun 2012 sampai dengan 2015 disajikan pada gambar berikut.

Diketahui banyak penjualan produk A selama 4 tahun adalah sebagai berikut.

Rata-rata banyak penjualan produk B dalam 4 tahun yang sama adalah ...
A. 1000
B. 1340
C. 1350
D. 1500
14. Di atas meja terdapat dua set kartu. Setiap set kartu terdiri atas 52 lembar dengan empat warna
berbeda (merah, kuning, hijau, dan biru). Masing-masing warna terdiri atas 13 kartu bernomor 1
sampai dengan 13. Satu kartu akan diambil secara acak dari dua set kartu tersebut. Peluang
terambil kartu berwarna merah atau bernomor 13 adalah ...

A.

B.

C.

D.
15. Terdapat lima bilangan bulat positif dengan rata-rata 40 dan jangkauan 10. Nilai maksimum yang
mungkin untuk bilangan terbesar dari lima bilangan tersebut adalah ...
A. 50
B. 49
C. 48
D. 45

BAGIAN B : ISIAN SINGKAT

1. Nilai dari ( ) adalah …

2. Bilangan bulat terbesar n agar 2 . 6. 10 .14 . 18 . ... . 198 dapat dibagi 6n adalah ... .
3. Ketika suatu segitiga siku-siku diputar pada salah satu sisi siku-sikunya, maka diperoleh kerucut
dengan volume 392 . Bila diputar pada sisi siku-siku lainnya, diperoleh kerucut dengan
volume 1344 . Panjang sisi miring segitiga siku-siku tersebut adalah ... cm.
4. Suatu balok tersusun atas kubus satuan seperti pada gambar di samping. Balok tersebut
dipancung sepanjang permukaan bangun datar yang dicetak tebal. Luas permukaan balok
terpancung adalah ... satuan luas.

5. Diketahui barisan fungsi ( ) ( ) ( ),.. sedemikian hingga ( ) dan ( )

( )
untuk bilangan bulat . Nilai dari ( )

6. Jika akar akar persamaan ( ) ( ) adalah m dan n dengan m>n,


dan akar akar persamaan adalah a dan b dengan a>b maka
7. Diketahui suatu barisan dengan suku ke-n adalah dengan

{
Jumlah seratus suku pertama barisan tersebut adalah ...
8. Misalkan x dan y merupakan bilangan asli berbeda yang memenuhi 4x+7y=2016. Banyak
pasangan (x,y) yang mungkin adalah ...
9. Delapan buku yang berbeda akan dibagikan kepada tiga orang siswa A, B, dan C sehingga
berturut-turut mereka menerima 4 buku, 2 buku, dan 2 buku. Banyak cara pembagian buku
tersebut adalah ...
10. Di kelas VIII terdapat 11 siswa. Pada saat ulangan Matematika, ada satu orang siswa yang sakit
sehingga harus mengikuti ulangan susulan. Nilai 10 siswa yang mengikuti ulangan pada
waktunya adalah 20, 10, 40, 80, 50, 60, 40, 70, 90, dan 30. Jika nilai siswa yang mengikuti
ulangan susulan diperhitungkan, maka rata-rata nilai yang diperoleh sama dengan median. Nilai
terbesar yang mungkin diperoleh siswa yang mengikuti ujian susulan adalah ...
OLIMPIADE SAINS NASIONAL SMP
SELEKSI TINGKAT KABUPATEN / KOTA TAHUN 2015

BAGIAN A: PILIHAN GANDA


1. Operasi * untuk himpunan bilangan { } didenifisikan sesuai table dibawah
ini

Jika untuk setiap bilangan bulat yang lebih besar daripada 1 definisikan ,
maka =…
A. 0
B. 1
C. 2
D. 3
2. Jika { },
{( )| },dan
T {( )| } maka anggota dari ada sebanyak….
A. 50
B. 1225
C. 1275
D. 2500
3. Nilai ujian lima orang siswa, yakni: Adi, Budi, Cici, Didi, dan Eki adalah bilangan bulat dan
mempunyai rata – rata yang sama dengan mediannya. Diketahui nilai tertinggi adalah 10 dan
terendah adalah 4. Jika yang memperoleh nilai tinggi adalah Adi dan yang terendah adalah
Eki, maka susunlah nilai yang mungkin ada sebanyak….
A. 3
B. 4
C. 13
D. 16
4. Diketahui lingkaran dengan pusat dan memiliki diameter . Sehingga siku – siku di
, pada diameter sehingga dan untuk jari – jari lingkaran adalah
1 cm, maka luas segitiga =…cm2
A.

B.

C.

D.

5. Toto dan Titi mulai berjalan dari titik bersamaan mengelilingi lapangan berbentuk persegi
yang panjang sisinya 180 meter. Diasumsikan Toto dan Titi berjalan dengan kecepatan
berturut – turut 72 meter/menit dan 60 meter/menit. Jika mereka bertemu untuk pertama
kalinya kembali dari titik setelah Toto berjalan putaran dan Titi berjalan putaran, maka
nilai adalah….
A. 6
B. 11
C. 20
D. 22
6. Diberikan tiga bilangan asli yakni dan . Jika sisa masing – masing bilangan
tersebut dibagi adalah sama yaitu dengan , maka hasil yang mungkin
adalah…
A. 165
B. 179
C. 344
D. 716
7. Dua dadu dan sekeping mata uang logam dilempar sekaligus, kemudian dicatat sisi yang
muncul. Jika diasumsikan munculnya setiap mata dadu seimbang dan munculnya setiap mata
uang seimbang, maka peluang akan didapatkan sisi angka pada mata uang dan kedua mata
dadu berjumlah 5 adalah…
A.

B.

C.

D.
8. Nilai yang memungkinkan agar 213 + 210 + 2n merupakan kuadrat sempurna adalah…
A. 5
B. 7
C. 12
D. 14
9. Didefinisikan fungsi ( ) untuk setiap bilangan asli . Nilai ( )
( ) ( ) adalah…
A. -31
B. -15
C. 15
D. 31

10. Nilai dari adalah…
√ √


A.

B.

C.

D.
11. Suatu taman kota dibatasi oleh suatu lintasan lari berbentuk lingkaran (seperti pada gambar)
dan tepat di titik pusat taman dibangun tugu ( ) yang dihiasi lampu. Di sepanjang tepi bagian
dalam taman, diletakkan 12 bangku permanen ( ) secara berurutan, sebut .
Jarak antara dua bangku berurutan dibuat sama (termasuk dari ). Jarak tugu ke
lintasan lari adalah 50 meter. Bakri, Bima dan Budi berlari pada lintasan lari mulai dari depan
bangku . Bakri dan Bima berlari searah dengan putaran jarum jam (dari ke arah ),
sedangkan Budi berlari mengambil arah yang berlawanan. Jika setelah 20 menit posisi Bakri

didepan bangku , Bima didepan bangku , dan Budi didepan bangku , maka jarak total
yang telah ditempuh tiga orang ini mendekati…meter ( = 3,14).
A. 549
B. 523
C. 471
D. 392
12. Diketahui adalah sebuah trapezium, sejajar dengan , dan . Jika
panjang , maka nilai adalah…
A. 46
B. 42
C. 38
D. 36
13. Anton dan kakaknya berulang tahun pada tanggal 1 januari. Pada tahun 2015, umur Anton
dan kakaknya sama dengan jumlah angka – angka tahun kelahirannya masing – masing. Jika
orang tua mereka menikah 25 tahun yang lalu, maka jumlah umur Anton dan kakaknya pada
tahun 2015 yang mungkin adalah…tahun
A. 22
B. 24
C. 26
D. 30
14. Penyedia jasa pengasuh bayi usia di bawah tiga tahun, memberlakukan tarif upah pengasuh
bayi sebagai berikut. Upah setiap jam sebesar untuk tiga jam pertama.
Selanjutnya diberlakukan aturan sebagai berikut. Untuk setiap satu jam berikutnya di siang
hari (mulai pukul 06.00 sampai dengan pukul 18.00), dikenakan upah sebesar lebih
banyak daripada upah satu jam sebelumnya. Adapun upah untuk malam hari di atas tiga jam
pertama dikenakan tetap sebesar setiap jam. Jika keluarga Adang menitipkan
bayinya pada pukul 16.00 sampai pukul 09.00 hari berikutnya, maka keluarga Adang harus
membayar biaya penitipan bayi tersebut sebesar …
A.
B.
C.
D.
15. Suatu kardus polos dari kertas berbentuk kubus. Volume kardus adalah 64.000 cm3. Fitri
memotong tepat pada rusuk kubus dan mengambil dua sisi bagian samping kardus tersebut.
Fitri melukis garis pada satu potongan sisi kardus dan diperoleh satu segitiga siku – siku yang
perbandingan dua sisi siku – siku adalah 1 : 2. Pada satu potongan sisi kardus yang lain
dilukis satu segitiga sama kaki (lihat gambar). Jika ternyata dua segitiga ini sama luasnya,
maka panjang sisi yang sama pada segitiga sama kaki adalah…cm.
A. 10
B. 102
C. 20
D. 202
BAGIAN B: ISIAN SINGKAT
1. Misalkan adalah suatu bilangan bulat dan adalah suatu bilangan prima, maka
nilai adalah…
2. Parabola melalui titik (-2,6) dan mempunyai sumbu simetri . Jika
dan merupakan bilangan genap positif berurutan, maka nilai adalah…
3. Perhatikan gambar berikut:

Titik dan masing – masing adalah titik singgung lingkaran pada sisi – sisi .

Diketahui , panjang panjang , dan panjang , Jika

sama kaki, maka luas adalah….

4. Dua botol yang berukuran sama diisi penuh dengan larutan gula. Rasio kandungan gula dan
air pada botol pertama adalah 2 : 11 dan pada botol kedua adalah 3 : 5. Jika isi kedua botol
tersebut dicampurkan, maka rasio kandungan gula dan air hasil campurannya adalah…
5. Misalkan ( ) . Jika terdapat dua bilangan bulat positif dan dengan

sehingga ( ) ( ) ( ), maka nilai

6. Jika jumlah 4 suku pertama suatu barisan aritmatika adalah 70 dan jumlah 12 suku berikutnya
adalah 690, maka suku ke-2015 barisan tersebut adalah…
7. Diketahui sebuah prisma yang dibentuk oleh bidang – bidang sisi berupa: dua trapezium yang
kongruen dan . Jika sejajar , panjang panjang , panjang
kali panjang , panjang panjang panjang panjang , dan
, maka perbandingan volume prisma . dan prisma adalah…
8. Mulai tahun ini materi OSN SMP bidang Fisika dan Biologi digabung menjadi satu, yaitu
IPA, sehingga wakil dari setiap sekolah tahun ini maksimal 3 orang. Diketahui bahwa di
Sekolah Teladan terdapat 6 calon siswa yang siap dikirim untuk mengikuti lomba OSN SMP
dengan kemampuan sebagai berikut:
a. Siswa A: Siap mewakili bidang lomba Matematika, IPA atau IPS
b. Siswa B dan C: Siap mewakili bidang lomba Matematika atau IPA
c. Siswa D: Siap mewakili bidang lomba Matematika atau IPS
d. Siswa E: Siap mewakili bidang lomba IPA atau IPS
e. Siswa F: Siap mewakili bidang lomba IPS
Siswa A dan B merupakan saudara kandung, sehingga sekolah mengambil kebijakan yakni
tidak mengijinkan dua orang yang bersaudara untuk mewakili sekolah (artinya jika A terpiliha
maka B tidak terpilih, begitu pula sebaliknya). Jika Sekolah Teladan memutuskan untuk

mengirimkan 3 siswa untuk mengikuti semua bidang lomba, maka cara yang mungkin untuk
memilih wakil sekolah tersebut ke OSN SMP tahun ini sebanyak…
9. Sebuah dicerminkan terhadap sumbu , kemudian dicerminkan lagi terhadap garis
sehingga hasil pencerminannya adalah . Jika koordinat titik – titik
( ) ( ) ( ), maka titik – titik koordinat dan berturut-turut adalah…
10. Tini ingin membuat gelang dari bahan manik – manik berwarna – warni yang terdiri dari
masing – masing 3 butir manik – manik berwarna merah, kuning, hijau, biru dan putih. Ia
ingin menyusun manik – manik tersebut sedemikian rupa sehingga di antara 2 manik – manik
berwarna putih selalu terdapat 4 manik – manik berwarna selain putih. Banyak susunan
gelang yang mungkin untuk dibuat adalah….
OLIMPIADE SAINS NASIONAL SMP
SELEKSI TINGKAT KABUPATEN / KOTA TAHUN 2014

BAGIAN A: PILIHAN GANDA


1. Sepuluh orang guru akan ditugaskan mengajar di tiga sekolah,yakni sekolah A, B, dan C, berturut
– turut sebanyak dua, tiga, dan lima orang. Banyak cara yang mungkin untuk menugaskan
kesepuluh guru tersebut adalah …
A. 2520
B. 5040
C. 7250
D. 10025
2. Berikut diberikan data siswa kelas VIII SMP Bina Prestasi. Tiga perlima bagian dari seluruh
siswa adalah perempuan. Setengah dari siswa laki – laki diketahui pergi ke sekolah naik bus
sekolah, sedangkan siswa perempuan hanya seperenamnya yang pergi ke sekolah naik bus
sekolah. Diketahui juga bahwa terdapat 147 siswa pergi ke sekolah tidak naik bus sekolah.
Banyak siswa kelas VIII sekolah tersebut adalah …
A. 330
B. 245
C. 210
D. 193
3. Diketahui FPB dan KPK dari bilangan 72 dan berturut – turut adalah 3 dan 1800. Pernyataan
berikut yang benar adalah …
A. kelipatan 5
B. kelipatan
C. x adalah genap
D. x adalah factor dari 3
4. Diberikan empat bilangan dan . Jika rata – rata dan adalah 50, rata – rata dan
adalah 75, serta rata – rata dan adalah 70, maka rata – rata dan adalah …
A. 35
B. 45
C. 50
D. 55
5. Rata – rata nilai dari 28 siswa adalah 80. Setelah ditambah nilai siswa A dan B, rata – ratanya
menjadi 78. Jika nilai A tiga kali nilai B, maka selisih antara nilai A dan B adalah …
A. 15
B. 25
C. 50
D. 75
6. Diketahui persamaan kurva dan . Jika kedua kurva
digambarkan pada bidang yang sama, maka banyak titik potong kedua kurva tersebut adalah …
A. 0
B. 1
C. 2
D. 3
7. Jika adalah faktor dari , maka bilangan bulat terbesar yang mungkin adalah …
A. 10
B. 15
C. 18
D. 20
8. Pada sebuah bidang terdapat sepuluh titik. Diantara sepuluh titik tersebut tidak ada tiga titik atau
lebih yang segaris. Banyak segitiga yang dapat dibentuk dengan menghubungkan sebarang tiga
titik pada bidang tersebut adalah …
A. 30
B. 60
C. 100
D. 120
9. Kubus ABCD.EFGH mempunyai panjang rusuk 2 satuan. Titik O adalah titik potong dua
diagonal pada bidang BCFG. Jarak titik O ke bidang BCEH adalah … satuan.

A.

B.

C.

D.
10. Perhatikan diagram batang berikut.

Gambar A
12

10

0
1 2 3 4 5 6 7 8 9 10

Gambar B
12
10
8
6
4
2
0
1 2 3 4 5 6 7 8 9 10

Pernyataan berikut yang salah adalah …


A. Modus pada gambar A Modus pada gambar B
B. Median pada gambar A Median pada gambar B
C. Quartil 1 pada gambar A Quartil 1 pada gambar B
D. Rata – rata pada gambar A Rata – rata pada gambar B
11. Banyak pasangan ( ) dengan dan bilangan asli yang memenuhi adalah …
A. 0
B. 1
C. 2
D. 3
12. Himpunan bilangan bulat dikatakan tertutup terhadap operasi penjumlahan jika hasil penjumlahan
dua bilangan bulat adalah bilangan bulat. Himpunan bilangan bulat dikatakan tidak tertutup
terhadap operasi pembagian karena ada hasil bagi dari sepasang bilangan bulat yang bukan
bilangan bulat. Jika { } adalah himpunan bulat positif genap, maka pernyataan
berikut yang benar adalah …
A. Himpunan A tertutup terhadap operasi perkalian saja
B. Himpunan A tertutup terhadap operasi penjumlahan saja
C. Himpunan A tertutup terhadap operasi penjumlahan dan perkalian
D. Himpunan A tertutup terhadap operasi penjumlahan dan pengurangan
13. Segitiga ABC adalah segitiga sama sisi dengan panjang sisi – sisinya 2 satuan. Selanjutnya,
dibentuk segitiga kedua dengan menghubungkan tiga titik tengah pada masing – masing sisi
segitiga ABC. Dengan cara serupa, dibentuk segitiga ketiga, keempat, kelima, keenam, dan
seterusnya. Luas seluruh segitiga – segitiga tersebut adalah …

A.

B.

C.

D.
14. Sepuluh titik pada suatu lingkaran diberi nomor 1, 2, …, 10. Seekor katak melompat searah jarum
jam satu satuan jika katak berada pada nomor yang merupakan bilangan prima, dan tiga satuan
jika bukan bilangan prima. Jika mula – mula katak berada pada posisi nomor 1, di manakah posisi
katak setelah melompat 2014 kali?
A. 1
B. 4
C. 7
D. 8
15. Diketahui garis sejajar garis dan garis sejajar garis .

𝐿 °
° 𝑦
𝐿 𝐿
°
𝑥 𝐿

Besar sudut adalah …


A. °
B. °
C. °
D. °
16. Suatu survey dilakukan terhadap 100 siswa peserta OSN tingkat kabupaten/kota berkaitan dengan
frekuensi pengiriman sms pada suatu hari. Hasil yang diperoleh sebagai berikut.
Jumlah sms Persentase
1 – 10 5%
11 – 20 10%
21 – 30 15%
31 – 40 20%
41 atau lebih 25%

Sisanya dilaporkan tidak mengirim sms. Jika dipilih seorang siswa secara acak, maka peluang
siswa tersebut mengirim sms tidak lebih dari 30 kali adalah …
A. 0,55
B. 0,30
C. 0,25
D. 0,15
17. Diketahui titik E, F, dan G pada trapezium ABCD. Sisi FE sejajar dengan sisi AB. Jika AB = 7,
DC = 14, DG = 8, FG = 4, GB = , dan GE = , maka nilai adalah …

14 C
D
8
4 𝑦
F G E

𝑥
A 7 B

A. 10
B. 11
C. 12
D. 13
18. Dari survey terhadap 75 orang diperoleh hasil sebagai berikut.
 50 orang berumur lebih dari 25 tahun, sisanya berumur tidak lebih dari 25 tahun
 27 orang menyukai masakan pedas, 7 diantaranya berumur tidak lebih dari 25 tahun
 28 orang menyukai masakan manis, 25 diantaranya berumur lebih dari 25 tahun
 5 orang menyukai masakan pedas dan juga masakan manis
 25 orang tidak menyukai masakan pedas maupun masakan manis, 7 diantaranya berumur
lebih dari 25 tahun
Banyak orang yang berumur tidak lebih dari 25 tahun yang menyukai masakan pedas dan juga
masakan manis adalah …
A. 2
B. 3
C. 4
D. 7
19. Jika luas satu persegi kecil adalah 4 m2, maka luas bangun datar pada gambar di bawah adalah ...

A. 36
B. 96
C. 144
D. 162
20. Seorang guru memiliki 3 kantong permen yang akan dibagikan kepada para siswanya. Masing –
masing kantong terdiri dari beberapa permen yang memiliki warna sama. Kantong pertama berisi
permen berwarna merah, kantong kedua berisi permen berwarna kuning, dan kantong ketiga berisi
permen berwarna hijau. Masing – masing siswa mendapatkan 7 permen dengan dua warna dan
kombinasi yang berbeda untuk setiap siswa. Sebagai contoh, bila siswa A mendapat 3 permen
berwarna merah dan 4 permen berwarna hijau, maka tidak ada siswa lain yang mendapat bagian
seperti siswa A. Maksimal banyak siswa yang ada di kelas tersebut adalah …
A. 15
B. 18
C. 21
D. 24
BAGIAN B: ISIAN SINGKAT

1. Bentuk paling sederhana dari adalah …

2. Banyak persegi pada gambar berikut adalah …

3. Berikut adalah gambar sebuah persegi panjang yang terdiri dari beberapa persegi yang dibuat dari
batang korek api. Sebagai contoh, bentuk 1 5 memerlukan 16 batang korek api, bentuk 2 5
memerlukan 27 batang korek api, seperti gambar berikut.
Bentuk 1 5

Bentuk 2 5
Banyak batang korek api yang diperlukan untuk membuat persegi panjang dengan bentuk 51 5
adalah …
4. Jika ⏟ , maka tiga angka terakhir dari M adalah …

( )( )
5. Semua nilai x yang memenuhi pertidaksamaan adalah …

6. Jika bilangan 2014 dinyatakan sebagai jumlah dari bilangan – bilangan asli berurutan, maka
bilangan asli terbesar yang mungkin adalah …
7. Delapan pensil dengan warna berbeda akan diletakkan dalam dua kotak mini untuk kepentingan
promosi. Banyak cara yang mungkin untuk meletakkan pensil – pensil tersebut sehingga tidak ada
kotak yang kosong adalah …
8. Jika hasil penjumlahan empat dari enam pecahan dan adalah maka hasil kali

dua pecahan lainnya adalah …


9. Perhatikan gambar di bawah ini. ABC adalah segitiga sama sisi. PQ tegak lurus AB, PS tegak
lurus AC, dan PR tegak lurus BC.

S
3
Q
1 P
2
B C
R
Jika PQ = 1, PR = 2, dan PS = 3, maka AB = …
10. Diberikan dua segitiga dan delapan persegi dengan sifat – sifat berikut.
(i) Dua segitiga siku – siku berukuran sama. Panjang sisi tegaknya 2 dan 4 satuan. Kedua
segitiga tersebut berwarna berbeda, satu berwarna biru, dan lainnya berwarna ungu.
(ii) Delapan persegi berukuran sama. Panjang sisi – sisinya 1 satuan. Tiga persegi berwarna
merah, tiga persegi berwarna kuning, dan lainnya berwarna hijau.
Dua segitiga dan delapan persegi tersebut akan disusun berimpitan sehingga membentuk persegi
berukuran 4 4 satuan yang akan dipakai sebagai hiasan dinding. Dengan memperhatikan
komposisi warna yang berbeda, banyak cara membentuk persegi berukuran 4 4 satuan di atas
adalah …
OLIMPIADE SAINS NASIONAL SMP
SELEKSI TINGKAT KABUPATEN / KOTA TAHUN 2013

BAGIAN A : PILIHAN GANDA


1. Bentuk x4 – 1 mempunyai faktor sebanyak … .
A. 3
B. 4
C. 5
D. 6
E. 7
2. Jika a, b, c, dan d adalah bilangan bulat positif dibagi 13 berturut-turut bersisa 12, 9, 11, dan 7,
maka 3a + 4b – 3c + 2d dibagi 13 akan bersisa
A. 0
B. 1
C. 7
D. 9
E. 11
3. Nilai rata-rata kelas A adalah 73, sedangkan nilai rata-rata kelas B adalah 88. Jika jumlah siswa
kedua kelas tersebut adalah 75 dan nilai rata-rata kedua kelas adalah 80, maka banyak siswa kelas
A adalah … orang
A. 35
B. 38
C. 40
D. 42
E. 45
4. Suatu hari perbandingan jumlah uang Netty dan Agit adalah 2 : 1. Sehari kemudian Netty
memberikan uangnya sejumlah Rp100.000,00 kepada Agit. Sekarang perbandingan uang Netty
dan Agit adalah 1 : 3. Jumlah uang Netty sekarang adalah
A. 240.000,00
B. 180.000,00
C. 120.000,00
D. 100.000,00
E. 60.000,00
5. Jika adalah fungsi linier, ( ) , dan ( ) ( ) maka nilai ( )
A. 762
B. 812
C. 832
D. 912
E. 1012
6. Diketahui :
{ | ( ) }.

Banyaknya himpunan bagian dari himpunan H adalah …


A. 4
B. 8
C. 16
D. 32
E. 64
7. Tiga orang A, B, dan C pinjam meminjam kelereng. Pada awalnya ketiga orang tersebut memiliki
sejumlah kelereng tertentu dan selama pinjam meminjam mereka tidak melakukan penambahan
kelereng selain melalui pinjam meminjam diantara ketiga orang tersebut. Pada suatu hari A
meminjami sejumlah kelereng kepada B dan C sehingga jumlah kelereng B dan C masing-masing
menjadi dua kali lipat jumlah kelereng sebelumnya. Hari berikutnya B meminjami sejumlah
kelereng kepada A dan C sehingga jumlah kelereng A dan C masing-masing menjadi dua kali lipat
jumlah kelereng sebelumnya. Hari terakhir C meminjami sejumlah kelereng kepada A dan B
sehingga jumlah kelereng A dan B masing-masing menjadi dua kali lipat jumlah kelereng
sebelumnya. Setelah dihitung akhirnya masing-masing memiliki 16 kelereng. Banyak kelereng A
mula-mula adalah ….
A. 8
B. 14
C. 26
D. 28
E. 32
8. Jika jumlah dua bilangan positip adalah 24, maka nilai terkecil dari jumlah kebalikan bilangan-
bilangan tersebut adalah … .
A. 1
B. ½
C. 1/3
D. ¼
E. 1/6
9. Jika ditulis dalam bentuk desimal, maka angka ke-2013 di belakang koma adalah …

A. 1
B. 2
C. 4
D. 5
E. 8
10. Diberikan angka disusun sebagai berikut: 987654321. Berapa banyak tanda operasi penjumlahan
harus disisipkan di antara angka-angka tersebut agar menghasilkan jumlah 99?
A. 3
B. 4
C. 5
D. 7
E. 8
11. Jika barisan berikut adalah barisan bilangan bulat positif berurutan yang dihilangkan semua
bilangan kelipatan tiga: 1, 2, 4, 5, 7, 8, 10, 11, 13, 14, …., maka suku ke - 67 barisan tersebut
adalah
A. 59
B. 62
C. 86
D. 92
E. 100
12. Jika rata-rata 51 bilangan bulat berurutan adalah 10, maka bilangan terkecil dari semua bilangan
tersebut adalah …
A. 5
B. 0
C. -5
D. -13
E. -15
13. Sebuah kantong berisi 15 bola merah, 12 bola biru, dan 3 bola hijau. Diambil sebuah bola secara
acak sebanyak 2 kali tanpa pengembalian. Peluang bola yang terambil merah pada pengambilan
pertama dan hijau pada pengambilan kedua adalah … .
A. 1/20
B. 3/58
C. 1/5
D. 3/29
E. 6/29
14. Lima orang anak akan naik mobil dengan kapasitas enam tempat duduk, yakni dua di depan
termasuk pengemudi (Sopir), dua di tengah, dan dua di belakang. Jika hanya ada dua orang yang
bisa mengemudi, banyak cara mengatur tempat duduk mereka adalah … .
A. 120
B. 200
C. 220
D. 240
E. 280
15. Jika diketahui panjang rusuk kubus ABCD.EFGH adalah 1 satuan, maka jarak titik E ke bidang
datar AFH adalah … satuan
A. ½

B.

C.

D.

E.
16. Diketahui sekelompok data memiliki sifat-sifat berikut:
a. Terdiri dari 5 data bilangan bulat positif dengan rataan = 7
b. Median = modus = 9
Jika jangkauan didefinisikan sebagai selisih data terbesar dengan data terkecil, maka jangkauan
terbesar yang mungkin adalah …
A. 11
B. 12
C. 13
D. 14
E. 15 l
17. Di dalam suatu keranjang terdapat 12 apel Malang, dua diantaranya diketahui busuk. Jika diambil
3 apel secara acak (random), maka peluang tepat satu di antaranya busuk adalah … .
A. 9/22
B. 5/11
C. 4/11
D. 9/44
E. 5/22
18. Sebuah silinder tegak diletakkan di dalam kubus ABCD.EFGH dengan panjang sisi kubus 2m.
Selanjutnya silinder dipancung oleh bidang miring yang melalui titik A, B, dan T dimana T adalah
titik perpotongan diagonal bidang CDHG. Volume terbesar silinder terpancung ini adalah … m3.

A.

B.

C.

D.

E.

19. Jika gambar di bawah adalah segi delapan beraturan, maka perbandingan luas antara daerah yang
diarsir dan luas segi delapan beraturan adalah …
A. 1 : 3
B. 1 : 4
C. 2 : 5
D. 3 : 8
E. 3 : 7

20. Beberapa bilangan empat angka memiliki angka-angka penyusun tak nol yang saling berbeda dan
berjumlah 10. Banyak bilangan yang dimaksud adalah … .
A. 24
B. 22
C. 20
D. 18
E. 16
BAGIAN B : ISIAN SINGKAT
1. Tino sedang memanjat tangga dan sekarang dia berada tepat di tengah tangga. Jika ia naik 3 anak
tangga ke atas, kemudian turun 5 anak tangga, serta naik kembali 10 anak tangga, maka Tino akan
sampai di puncak tangga. Banyak anak tangga yang dimiliki tangga tersebut adalah ….
2. Ani mempunyai uang Rp16.500,00. Sejumlah uang itu akan dihabiskan untuk membeli 6 buah
peralatan sekolah. Ia membeli beberapa pensil dengan harga Rp2.000,00 per pensil. Ia membeli
beberapa buku dengan harga Rp2.500,00 per buku, dan ia juga membeli beberapa kotak pensil
dengan harga Rp4.000,00 per kotak pensil. Banyak buku yang dibeli Ani adalah … .
3. Banyak bilangan positif n sehingga berupa bilangan bulat positif adalah…

4. Diberikan tabel bilangan berikut :

-7 x -8

2y -5 -4

x-2 -10 y

Jika diketahui bahwa jumlah masing-masing baris, kolom, dan diagonal adalah sama, maka nilai x
+ y adalah …
5. Jika himpunan A mempunyai anggota sebanyak x dan himpunan B mempunyai anggota sebanyak
y, x ≤ y, maka himpunan A ∪ B mempunyai anggota (maksimum) sebanyak … .
6. Semua bilangan asli n yang memenuhi sifat bahwa 6n2 + 5n – 4 adalah bilangan prima adalah… .
7. Jika adalah suku-suku suatu
barisan bilangan, Tentukan
8. Pada ABC terdapat titik D pada BC sehingga BD : DC = 1 : 3. Titik L pada AD sehingga AL :
LD =1 : 4. Perbandingan luas ACL dan BDL adalah …

9. Suatu string terdiri dari 10 angka 0, 1, atau 2. Bobot string didefinisikan sebagai jumlah angka
angka dalam string tersebut. Sebagai contoh, string 0002002001 mempunyai bobot 5. Banyak
string dengan bobot 4 adalah …
10. Tita memiliki tetangga baru yang memiliki 2 anak. Jika salah satu anak tetangga baru tersebut
adalah perempuan, maka besar peluang anak yang lain adalah laki-laki adalah … .
OLIMPIADE SAINS NASIONAL SMP
SELEKSI TINGKAT KABUPATEN / KOTA TAHUN 2012

BAGIAN A : PILIHAN GANDA


1. Pernyataan yang benar diantara pernyataan-pernyataan berikut adalah :
A. {Ø} Ø
B. {Ø} ⊆ Ø
C. Ø ⊆ Ø
D. {a,b} {a, b, {{a,b}}}
E. {a,Ø} ⊆ {a, {a,Ø}}
2. Diketahui persegi ABCD. Jika titik E terletak pada BC dan titik F terletak pada CD sehingga AE
dan AF membagi persegi panjang ABCD menjadi 3 daerah yang luasnya sama, maka
perbandingan luas segitiga AEF terhadap persegi ABCD adalah…
A. 4/18
B. 5/18
C. 6/18
D. 7/18
E. 8/18
3. Jika kedua akar persamaan p2x2 – 4px + 1 = 0 bernilai negatif, maka nilai p adalah…
A. p < 0
B. p < √ – ½
C. p < √ + ½
D. p < √
E. p < √
4. Jika f(x) = 3x + 1, g(x) = 1 – 2x, dan f(g(a)) = 28, maka nilai a adalah…
A. 7
B. 4
C. 4
D. 7
E. 13,5
5. Suatu byte didefinisikan sebagai susunan angka yang terdiri dari 8 angka (digit), yaitu 0 atau 1.
Contoh byte : 01110111. Banyak jenis byte yang memuat angka 1 tepat sebanyak 5 adalah…
A. 30
B. 45
C. 56
D. 62
E. 66
6. Perhatikan pola bilangan berikut. Bilangan 2012 akan terletak di bawah huruf…
A. Q
B. R
C. S
D. T
E. U
7. Jika m dan n adalah bilangan bulat positif sehingga m2 + 2m + 3n = 33, maka banyak bilangan n
yang memenuhi adalah…
A. 7
B. 6
C. 5
D. 4
E. 3
8. Enam pipa besar dapat mengeringkan kolam dalam 5 jam, sedangkan 8 pipa kecil dapat
mengeringkan kolam dalam 10 jam. Waktu yang diperlukan untuk mengeringkan kolam tersebut
apabila menggunakan 3 pipa besar dan 5 pipa kecil adalah…jam.
A. 60/13
B. 80/13
C. 90/13
D. 8
E. 8
9. Lima orang guru akan ditempatkan pada tiga sekolah yang berbeda, 2 orang disekolah pertama, 2
orang disekolah kedua, dan 1 orang disekolah ketiga. Banyak cara menempatkan kelima guru
tersebut adalah…
A. 40
B. 30
C. 20
D. 10
E. 4
10. Diketahui persegi panjang PQRS. Panjang PV = QT = PS = 6. Titik U adalah perpotongan antara
garis SV dan RT (seperti gambar dibawah ini). Jika PQ = 10, maka luas segiempat PTUS adalah…
A. 15
B. 17
C. 19
D. 21
E. 23
11. Empat bola bernomor 1, 2, 3, dan 4 diletakkan dalam sebuah kotak. Sebuah bola diambil secara
acak dari kotak tersebut. Nomor yang muncul dicatat, kemudian bola dikembalikan ke dalam kotak
semula. Jika proses pengambilan dilakukan selama 3 kali dengan cara yang serupa, maka peluang
nomor bola yang terambil berjumlah 5 adalah…
A. 5/256
B. 5/64
C. 1/16
D. 3/32
E. 3/16
12. Suatu antrian pembelian tiket masuk pertandingan sepak bola terdiri dari 2012 orang. Jika diantara
2 orang pria paling sedikit terdapat 3 wanita, maka banyak pria dalam antrian tersebut paling
banyak adalah…
A. 501
B. 502
C. 503
D. 504
E. 505
13. Diketahui bahwa abc dan def adalah bilangan 3 angka (digit) sehingga abc + def = 1000. Jika a, b,
c, d, e, dan f tidak satupun yang sama dengan 0, maka nilai a + b + c + d adalah…
A. 25
B. 26
C. 27
D. 28
E. 29
14. Suatu tes matematika terdiri dari 5 soal pilihan ganda dengan 5 pilihan dan hanya ada 1 pilihan
yang benar. Jika mulan menjawab soal secara menerka (secara acak atau asal-asalan), maka
peluang tepat 2 soal dijawab dengan benar adalah…
A. 32/725
B. 32/625
C. 64/725
D. 64/625
E. 128/625
15. Untuk setiap bilangan bulat x didefinisikan fungsi f dengan f(x) adalah banyaknya angka (digit)
dari bilangan x. Contoh : f(125) = 3 dan f(2012) = 4. Nilai dari f(22012) + f(52012) adalah…
A. 2013
B. 2014
C. 2015
D. 2016
E. 2025
16. Dalam sebuah karung terdapat 60 kaos bernomor 11, 12, 13, 14, ... , 40. Ada 2 kaos untuk setiap
nomor (ada 2 kaos bernomor 11, ada 2 kaos bernomor 12, dan seterusnya). Jika diambil 2 kaos
secara acak, maka peluang yang terambil adalah kaos bernomor sama adalah…
A. 1/59
B. 2/35
C. 2/33
D. 2/31
E. 2/29
17. Sehabis belanja, Retina membawa pulang uang kembalian berupa 8 koin (uang receh), yang terdiri
dari ratusan, lima-ratusan, dan ribuan. Total nilai uang kembalian adalah tiga ribu rupiah.
Sayangnya, dalam perjalanan pulang salah satu uang koin jatuh (hilang). Jika peluang kehilangan
untuk satu ratusan, satu lima-ratusan, dan satu ribuan adalah sama, maka peluang kehilangan satu
koin lima-ratusan adalah…
A. 1/8
B. 2/8
C. 3/8
D. 4/8
E. 5/8
18. Jika 2, 3, 5, 6, 7, 10, 11, ... adalah barisan bilangan yang terdiri dari semua bilangan asli yang
bukan bilangan kuadrat dan bukan bilangan pangkat tiga, maka bilangan 270 adalah suku ke…
A. 247
B. 248
C. 249
D. 250
E. 251
19. Suatu balok dengan volume 240 satuan mempunyai panjang a, lebar b, dan tinggi c (a, b, dan c
adalah bilangan asli). Jika a + b + c = 19 dan a > b > c > 3, maka luas permukaan balok yang
sisinya mempunyai rusuk b dan c adalah…
A. 64
B. 60
C. 48
D. 40
E. 30
20. Perhatikan gambar di bawah ini. Jika lingkaran besar berjari-jari 4 dan lingkaran kecil berjari-jari
2, serta luas daerah yang diarsir adalah 5/12 dari luas lingkaran besar, maka besar ∠RPQ adalah…
A. 60o
B. 90o
C. 120o
D. 135o
E. 150o

BAGIAN B : ISIAN SINGKAT


1. Diketahui 2012 bilangan bulat positif berurutan. Jika setiap bilangan tersebut dibagi 5, kemudian
sisa-sisa pembagiannya dijumlahkan, maka hasil penjumlahan sisa-sisanya adalah…
2. Jika , dan ( ) ( ) ( ) , maka nilai
adalah…
3. Jika segitiga ABC siku-siku di B, AB = 6, AC = 10, dan AD adalah garis bagi sudut BAC, maka
panjang AD adalah…
4. Semua nilai yang memenuhi persamaan √( ) √ adalah…
5. Jika rata-rata dari 1000 bilangan ganjil positif berurutan adalah 2012, maka bilangan terkecil dari
bilangan-bilangan tersebut adalah…
6. Jalan Majapahit sejajar dengan jalur kereta api yang membentang lurus. Anton menumpang bus
OSN di jalan Majapahit dengan kecepatan konstan (tetap) 40 km/jam. Dari arah yang berlawanan
dengan bus yang ditumpangi Anton, berpapasan dengan kereta api barang yang bergerak dengan
kecepatan konstan 20 km/jam. Anton mencatat bahwa bus dan kereta api berpapasan selama
seperempat menit terhitung mulai dari lokomotif (bagian depan) sampai bagian paling belakang.
Panjang kereta api tersebut adalah ... meter.
7. Banyak himpunan bagian dari himpunan {a,b,c,d,e,f} yang memuat sedikitnya satu huruf vokal
adalah…
8. Empat titik ditempatkan pada lingkaran berjari-jari ½ satuan. Jika keempat titik tersebut
dihubungkan sehingga membentuk persegi panjang, maka luas terbesar (maksimum) yang
mungkin bagi persegi panjang tersebut adalah…
9. Kubus ABCD.EFGH mempunyai panjang rusuk 2 cm. Jika titik T adalah titik potong diagonal
bidang BCGF, titik P adalah titik tengah rusuk AB, dan titik Q adalah titik tengah rusuk DC, maka
jarak antara titik T dengan bidang PQHE adalah…cm.
10. Misalkan ab adalah bilangan terdiri dari dua angka. Jika bilangan itu ditambah 45, maka diperoleh
bilangan ba. Pada bilangan ab, jika diantara a dan b disisipkan angka 0, maka diperoleh bilangan
yang nilainya 7⅔ kali bilangan ab. Bilangan ab tersebut adalah…
OLIMPIADE SAINS NASIONAL SMP
SELEKSI TINGKAT KABUPATEN / KOTA TAHUN 2011

BAGIAN A : PILIHAN GANDA


1. Nilai

A. 113/10!
B. 91/10!
C. 73/10!
D. 71/10!
E. 4/10!
2. Menggunakan angka-angka 1, 2, 5, 6 dan 9 akan dibentuk bilangan genap yang terdiri dari lima
angka. Jika tidak ada angka yang berulang, maka selisih bilangan terbesar dan terkecil adalah
A. 70820
B. 79524
C. 80952
D. 81236
E. 83916
3. Pada gambar berikut tabung berisi air, tinggi dan diameter tabung tersebut adalah 18 cm dan 6 cm.
Kemudian ke dalam tabung dimasukkan 3 bola pejal yang identik (sama bentuk) sehingga bola
tersbut menyinggung sisi tabung dan air dalam tabung keluar, maka sisa air di dalam tabung adalah
... cm .
A. 51π
B. 52π
C. 53π
D. 54π
E. 55π
4. Seorang ilmuwan melakukan percobaan terhadap 50 ekor kelinci dan melaporkan hasilnya sebagai
berikut:
 25 ekor diataranya kelinci jantan.
 25 ckor di l at i h menghindari jebakan, 10 ekor diantaranya jantan.
 20 ekor( dari total 50 ekor) berhasil menghindari jebakan, 4 ekor diantaranya jantan.
 15 ekor yang pernah dilatih berhasil menghindari jebakan, 3 ekor diantaranya jantan.
Berapa ekor kelinci betina yang tidak pcrnah dilatih, tidak dapat mcnghindari jebakan?
A. 5
B. 6
C. 7
D. 8
E. 9
5. Banyaknya bilangan bulat x sehingga merupakan bilangan bulat adalah ...
√ √

A. 2
B. 3
C. 5
D. 6
E. 7
6. Urutan tiga bilangan 24444 , 33333 , dan 42222 dari yang terkecil sampai yang terbesar adalah….
A. 24444 , 42222 , 33333
B. 24444, 33333 , 42222
C. 33333, 42222 , 24444
D. 42222 ,33333, 24444
E. 33333 ,24444, 42222
7. Lima pasang suami istri akan duduk di 10 kursi secara memanjang. Banyaknya cara mengatur
tempat duduk mereka sehingga setiap pasang suami istri duduk berdampingan adalah....
A. 3800
B. 3820
C. 3840
D. 3900
E. 3940
8. Dalam sebuah kotak berisi 15 telur, 5 telur diantaranya rusak. Untuk memisahkan telur baik dan
telur yang rusak dilakukan pengetesan satu persatu tanpa pengembalian. Peluang diperoleh telur
rusak ke 3 pada pengetesan ke 5 adalah ...

A.

B.

C.

D.

E.

9. Diketahui limas T.ABCD. panjang Rusuk AB 2 cm dan TA 4 cm. Jarak titik B dan rusuk TD
adalah ...
A. √
B. √
C. √
D. √
E. √
10. Sembilan lingkaran kongruen terletak di dalam persegi seperti terlihat pada gambar. Jika keliling
sebuah lingkaran 62,8 cm dengan π = 3,14, maka luas daerah yang diarsir adalah …cm2
A. 344
B. 364
C. 484
D. 688
E. 728
11. Suatu jam dinding selalu menghasilkan keterlambatan lima menit untuk setiap jamnya. Jika saat
sekarang jam tersebut menunjukkan waktu yang tepat, maka jam tersebut akan menunjukkan
waktu yang tepat setelah ...jam.
A. 105
B. 110
C. 114
D. 124
E. 144
12. Di dalam kotak terdapat 18 bola identik (berbentuk sama), 5 berwarna hitam, 6 berwarna putih dan
7 berwarna hijau. Jika diambil dua bola secara acak, maka peluang yang terambil bola berwarna
sama adalah ...

A.

B.

C.

D.

E.

13. Perhatikan gambar di atas, persegi ABCD dengan panjang sisi 14 cm menyinggung Lingkaran.
Masing-masing sisi persegi dibuat setengah lingkaran dengan diameter sisi persegi tersebut. Jika π
= 3,14, maka luas daerah yang diarsir adalah ... cm2
A. 49
B. 56
C. 112
D. 178
E. 196
14. Diketahui 22x + 2-2x = 2 . Nilai 2x + 2- x = .....
A. 1
B. 2
C. √
D. 3
E. √
15. Rataan usia kelompok guru dan profesor adalah 40 tahun. Jika rataan kelompok guru adalah 35
tahun sedangkan rataan kelompok profesor adalah 50 tahun, perbandingan banyaknya guru dengan
profesor adalah . ..
A. 2 : 1
B. 1 : 2
C. 3 : 2
D. 2 : 3
E. 3 : 4
16. Diketahui jajargenjang ABCD. Titik P dan Q terletak pada AC sehingga DP dan BQ tegak lurus
AC. Jika panjang AD = 13 cm, AC = 25 cm dan luas jajargenjang tersebut adalah 125 cm2, maka
panjang PQ adalah ... cm
A.

B. 1
C. √
D. √
E.

17. √ √ √ √ √ √
A. 10
B. 11
C. 12
D. √
E. √
18. Hasil penjumlahan 1! + 2! + 3! + ... + 20 11 ! adalah suatu bilangan yang angka satuannya adalah
...
A. 3
B. 4
C. 5
D. √
E. √
19. Lima orang akan pergi ke pantai menggunakan sebuah mobil berkapasitas 6 tempat duduk. Jika
hanya ada dua orang yang bisa menjadi sopir. maka banyaknya cara mengatur tempat duduk di
dalam mobil adalah ...
A. 60
B. 120
C. 180
D. 240
E. 280
20. Sebuah bingkai foto yang berbentuk persegi diputar 45o dengan sumbu putar titik perpotongan
diagonal-diagonalnya. Jika panjang sisi persegi adalah 1 cm. Luas irisan antara bingkai foto
sebelum dan sesudah diputar adalah ... cm2.
A. 1+ 2 √
B. 2 + 2 √
C. 1
D. 2 - 2 √
E. 2 √ - 2
BAGIAN B : ISIAN SINGKAT
1. Lima permen identik (berbentuk sama). satu rasa apel. dua rasa jeruk dan dua rasa jahe akan
dibagikan kepada lima sekawan Anto, Bono, Carli, Dodo dan Edo, sehingga masing-masing
mendapat satu permen. Peluang Anto mendapat permen rasa jahe adalah ...
2. Jumlah angka-angka dari hasil kali bilangan 999999999 dengan 12345679 adalah ...
3. Perhatikan gambar berikut. ABCD persegi dengan panjang sisi sisinya adalah 2 cm. E adalah titik
tengah CD dan F adalah titik tengah AD. Luas daerah EDFGH adalah ...

4. Nilai jumlahan bilangan berikut adalah …


12 – 22 + 32 –42 + 52 –….–20102 + 20112
5. Jika barisan x1, x2, x3, …. mememihi . x1+ x2+ x3 +…..+xn = n3 untuk semua n bilangan asli, maka
x100 = ...
6. Semua pasangan bilangan bulat (a,b) yang mememihi 2a = b2 – 1 adalah ...
7. Tersedia beberapa angka 2. 0, dan 1. Angka dua ada sebanyak lima buah masing-masing berwarna
merah, hijau, kuning, biru dan nila. Angka nol dan satu masing-masing ada sebanyak empat buah
dengan warna masing-masing merah, hijau, kuning dan biru. Selanjutnya menggunakan angka -
angka tersebut akan dibentuk bilangan 2011 sehingga angka-angka yang bersebelahan tidak boleh
sewarna. Contoh pewarnaan yang dimaksud: 2 (merah) 0 (hijau) 1(merah) 1 (biru). contoh bukan
pewarnaan yang dimaksud: 2 (merah) 0 (hijau) 1 (hijau 1 (biru). Banyaknya bilangan 2011 dengan
komposisi pewarnaan tersebut adalah ...
8. Sebuah kotak berisi 500 kelereng berukuran sama yang terdiri dari 5 warna dimana masingmasing
kelereng sewarna berjumlah 100. Minimum banyaknya kelereng yang harus diambil secara acak
sedemikian sehingga kelereng yang terambil dijamin memuat sedikitnya 5 kelereng yang berwarna
sama adalah ...
9. Jika (3 + 4 ) (32 + 42 ) (34 + 44 ) (38 + 48 ) (316 + 416 ) (332 + 432 ) = (4x - 4y ). Maka x – y = …
10. Suatu himpunan disebut berjenis H jika memenuhi sifat:
a) Himpunan tersebut beranggotakan tiga bilangan bulat tak negatif.
b) Rata-rata ketiga bilangan anggota himpunan tersebut adalah 15.
Banyaknya semua Himpunan berjenis H ini adalah ...
OLIMPIADE SAINS NASIONAL SMP
SELEKSI TINGKAT KABUPATEN / KOTA TAHUN 2010

BAGIAN A : PILIHAN GANDA


1. Garis l melalui titik (–4, –3) dan (3, 4). Jika garis l juga melalui titik (a, b), maka nilai

A. 23
B. 1.
C. –1
D. –28
E. –31
2. Jika bilangan ganjil dikelompokkan seperti berikut: {1}, {3,5}, {7,9,11}, {13,15,17,19}, maka
suku tengah dari kelompok ke-11 adalah...
A. 21
B. 31
C. 61
D. 111
E. 121
3. n adalah bilangan bulat positif terkecil sehingga 7 + 30n bukan bilangan prima. Nilai dari
adalah...
A. 1
B. 4
C. 9
D. 16
E. 25
4. Dijual 100 lembar kupon, 2 diantaranya berhadiah. Ali membeli 2 lembar undian. Peluang Ali
mendapat 2 hadiah adalah...
A.

B.

C.

D.

E.

5. Bilangan tiga digit 2A3 jika ditambah dengan 326 akan menghasilkan bilangan tiga digit 5B9. Jika
5B9 habis dibagi 9, maka A + B = ....
A. 5
B. 6
C. 7
D. 8
E. 9
6. Sebuah mata uang dan sebuah dadu dilantunkan bersama-sama. Bila diketahui mata uang muncul
angka, maka peluang munculnya mata dadu lebih dari 2 adalah ...
A.

B.

C.

D.

E.
7. Diberikan dua buah bilangan bulat berbeda yang berjumlah 37. Apabila bilangan yang lebih besar
dibagi dengan bilangan yang lebih kecil, maka hasil baginya adalah 3 dan sisanya 5. Selisih kedua
bilangan tersebut adalah ...
A. 21
B. 22
C. 23
D. 24
E. 25

8. Jika x : y =3 : 4, maka nilai adalah ...

A.

B.

C.

D.

E.

9. Roda A dengan jari-jari 40 cm dan roda B dengan jari-jari 10 cm dihubungkan dengan sebuah tali
yang melingkari keduanya. Jika jarak pusat kedua roda adalah 60 cm, maka panjang tali yang
dibutuhkan adalah ... cm
A. (√ )
B. (√ )
C. (√ )
D. (√ )
E. (√ )
10. Pada segitiga ABC (siku-siku di C), titik Q pada AC, titik P pada AB, dan PQ sejajar BC. Panjang
sisi AQ = 3 ; AP = 5 ; BC = 8, maka luas DABC adalah …
A. 48
B. 36
C. 24
D. 22
E. 12
11. Jika diberikan ( ) dengan n bilangan asli, maka nilai
adalah...
A. -5
B. 0
C. 17
D. 28
E. 30
12. Tersedia tujuh gambar yang berbeda akan dipilih empat gambar yang akan dipasang membentuk
barisan memanjang. Banyaknya cara yang dapat dilakukan jika sebuah gambar yang terpilih harus
selalu dipasang di ujung adalah ...
A. 420
B. 504
C. 520
D. 720
E. 710

13. Diketahui 3x, , dan adalah bilangan bulat. Manakah dari ketiga bentuk di bawah ini yang juga

merupakan bilangan bulat untuk nilai-nilai x yang memenuhi ketiga bentuk di atas?

I. II. 2x III. 6x

A. I
B. II
C. Ill
D. I dan III
E. II dan III
14. Bilangan ratusan yang berupa bilangan prima dimana perkalian ketiga angka penyusun bilangan
tersebut adalah 10 ada sebanyak ... buah bilangan.
A. 6
B. 5
C. 4
D. 3
E. 2
15. Sebuah prisma segiempat berukuran 15 cm x 15 cm x 10 cm, terbuat dari baja. Prisma tersebut
setiap rusuknya diberi kerangka terbuat dari kawat dan setiap sisi dicat. Harga baja tiap 1 cm2
adalah Rp800,00; setiap 4 cm kawat harganya Rpl.300,00; dan setiap 10 cm2 membutuhkan cat
seharga Rpl.600,00. Biaya untuk membuat prisma segiempat tersebut adalah ...
A. Rp2.020.000,00
B. Rpl.160.000,00
C. Rpl.060.000,00
D. Rpl.050.000,00
E. Rpl.030.000,00
16. JikaP(x) = Q(x)(x - a), dimana P(x) dan Q(x) polinom, maka:
A. P(a) 0
B. x - a bukan faktor dari P(x)
C. kurva y = P(x) memotong sumbu x di titik (a, 0)
D. kurva y = P(x) memotong sumbu x di titik (–a, 0)
E. titik potong terhadap sumbu x tidak dapat ditentukan
17. Empat kubus identik dengan panjang rusuk 5 cm disusun menjadi suatu bangun ruang dengan cara
menempelkan sisi-sisinya. Banyak bangun ruang berbeda yang terbentuk adalah...
A. 10
B. 8
C. 6
D. 5
E. 3
18. Fungsi f(x) = x2 -ax mempunyai grafik berikut.

Grafik fungsi g(x) = x2 + ax + 5 adalah …


E
A. B.

C D

19. Terdapat 3 orang Indonesia, 4 orang Belanda, dan 2 orang Jerman akan duduk dalam bangku yang
memanjang. Banyaknya susunan yang terjadi jika duduknya berkelompok menurut
kewarganegaraannya adalah...
A. 24
B. 48
C. 288
D. 536
E. 1728
20. Anto mempunyai 20 lembar seribuan, 4 lembar lima ribuan dan 2 lembar sepuluh ribuan. Jika x, y,
dan z adalah banyaknya seribuan, lima ribuan, dan sepuluh ribuan, maka banyak cara berbeda
sehingga jumlahnya dua puluh ribu adalah ...
A. 6
B. 7
C. 8
D. 9
E. 10
BAGIAN B : ISIAN SINGKAT
1. Sebuah DABC sama kaki dipotong menjadi dua buah segitiga sama kaki (tidak harus kongruen)
dengan membagi dua sama besar salah satu sudut alasnya. Ukuran sudut yang terkecil dari segitiga
DABC adalah …
2. Sebuah kotak berisi bola merah dan hijau. Jika empat bola merah dikeluarkan dari kotak maka
sepersepuluh sisanya adalah bola merah. Akan tetapi jika empat bola hijau dikeluarkan dari kotak
maka seperlima sisanya adalah bola merah. Banyak bola merah yang semula berada di dalam kotak
tersebut adalah ...
3. Sebuah perahu motor meninggalkan kapal induk ke arah utara menuju suatu target dengan
kecepatan tetap 80 km/jam. Kapal induk bergerak ke arah timur dengan kecepatan tetap 40
km/jam. Apabila perahu motor tersebut hanya mempunyai bahan bakar yang cukup untuk berjalan
4 jam saja, maka jarak maksimum target yang dapat ditujunya agar ia dapat kembali ke kapal induk
dengan tanpa masalah adalah … km.
4. Suatu pekerjaan jika dikerjakan oleh Anto dan Dini dapat diselesaikan dalam waktu 6 jam. Jika
pekerjaan itu dikerjakan oleh Dini sendirian akan selesai lima jam lebih lambat dibandingkan
Anto. Pekerjaan itu dapat diselesaikan oleh Anto sendirian dalam waktu … jam.
5. Diketahui jajargenjang ABCD; ∠A = ∠C = 45°. Lingkaran K dengan pusat C melalui B dan D. AD
diperpanjang memotong lingkaran di E dan BE memotong CD di H. Perbandingan luas antara
ABCH dengan AEHD adalah ...
6. Jika jumlah k bilangan bulat positif berurutan adalah 2010, dengan k > 1, maka k terkecil yang
mungkin adalah ...
7. Diketahui ABCD adalah persegi. Titik E merupakan perpotongan AC dan BD pada persegi ABCD
yang membentuk persegi baru EFGH. EF berpotongan dengan CD di I dan EH berpotongan
dengan AD di J. Panjang sisi ABCD adalah 4 cm dan panjang sisi EFGH adalah 8 cm. Jika ∠EID =
60°, maka luas segiempat EIDJ adalah ...cm2.
8. Kereta penumpang berpapasan dengan kereta barang. Laju kereta penumpang 40 km/jam
sedangkan kereta barang 20 km/jam. Seorang penumpang di kereta penumpang mencatat bahwa
kereta barang berpapasan selama 15 detik. Panjang rangkaian KA barang adalah... m

9. Jika operasi * terhadap bilangan rasional positif didefinisikan sebagai , maka (

)=…
10. Sebuah kubus akan diberi warna sedemikian sehingga setiap dua sisi yang berdekatan (yakni dua
sisi yang dipisahkan oleh tepat satu rusuk) diberi warna yang berbeda. Jika diberikan 5 warna yang
berbeda, maka banyak cara yang berbeda untuk mewarnai kubus adalah ...
OLIMPIADE SAINS NASIONAL SMP
SELEKSI TINGKAT KABUPATEN / KOTA TAHUN 2009

BAGIAN A : PILIHAN GANDA


1. Jika a, b, 15, c, dan d membentuk barisan aritmetika, mka a + b + c + d = …
A. 45
B. 60
C. 75
D. 90
2. Misalkan S = {21, 22, 23, …, 30}. Jika empat anggota S diambil secara acak, maka peluang
terambilnya empat bilangan yang berjumlah genap adalah …
A.

B.

C.

D.

3. Diketahui koordinat segiempat ABCD adalah A(0, 0), B(30, 0), C(0, 40), dan D(30, 40). Titik E
dan F masing-masing membagi sisi CD dan AC menjadi dua bagian sama panjang. Jika pada
segitiga CEF dibuat lingkaran dalam maka koordinat titik pusat lingkaran adalah ….
A. (5, 35)
B. (35, 5)
C. ( , 10)

D. (10, )

4. Berat seekor gajah pada awal tahun adalah 655,36 kg. Selama bulan Januari, berat gajah naik 25%.
Karena debu dan efek meteorit yang menghalangi sinar matahari sepanjang Bulan Februari, berat
gajah turun 25%. Kemudian sepanjang bulan Maret, sinar matahari kembali normal dan berat gajah
kembali naik 25%. Pada bulan April, karena keracunan makanan, gajah terserang sakit perut yang
menyebabkan beratnya kembali turun 25%. Keadaan seperti ini berlanjut hingga bulan-bulan
berikutnya. Berat gajah pada akhir Juli adalah …. Kg.
A. 675, 00
B. 625, 00
C. 600,00
D. 540,00
5. Gambar di bawah ini menunjukkkan suatu persegi yang dibagi menjadi 6 bagian yang sama. Setiap
bagian berupa persegipanjang yang mempunyai keliling 70 cm. Luas persegi tersebut adalah ….
A. 625 cm2
B. 784 cm2
C. 900 cm2
D. 961 cm2

6. Pada bulan Januari harga tas di Toko Asia adalah Rp 150.000,00. Pada bulan Februari harga tas
naik 10%, tetapi bila yang membeli pelajar memperoleh potongan 10%. Pada bulan Maret harga
tas tersebut menjadi Rp 135.000,00 tetapi pembeli dibebani pajak pembelian sebasar 10% dan
diskon bagi pelajar tidak berlaku lagi. Dua orang pelajar, Andi dan Anton membeli tas tersebut.
Andi membeli pada bulan Februari, sedangkan Anton membeli pada bulan Maret. Pernyataan
berikut yang benar adalah ....
A. Jumlah uang yang dibayarkan Andi sama dengan jumlah uang yang dibayarkan Anton.
B. Anton membayar sebesar Rp 150.000,00 untuk membayar tas yang dibelinya.
C. Di antara tiga bulan yang disebut di atas, bulan Januari adalah bulan yang paling
menguntungkan bagi pelajar untuk membeli tas.
D. Jumlah uang yang dibayarkan Andi lebih besar dari jumlah uang yang dibayarkan Anton.
7. Pada hari Minggu, jumlah uang Tora dan Ani berbanding 3 : 1. Pada hari Senin, tora memberi
uang sejumlah Rp. 50.000,00 kepada Ani. Sekarang perbandingan jumlah uang Tora dan Ani
menjadi 1 : 2. Jumlah uang Tora dan uang ani pada hari Minggu adalah ….
A. Rp. 720.000,00
B. Rp.600.000,00
C. Rp. 450.000,00
D. Rp.400.000,00
8. Misalkan a dan b bilangan bulat sehingga a( a + b) = 34. Nilai terkecil a–b adalah ….
A. –17
B. –32
C. –34
D. –67

9. Jika√ √ √ √ , maka nilai x sama dengan....

A.

B.

C.

D.
10. Andi membuka sebuah buku setebal 650 halaman, hasil kali nomor halaman yang nampak adalah
702. Jumlah nomor-nomor halaman buku yang terbuka adalah ….
A. Lebih dari 53
B. Kurang dari 50
C. Lebih dari 52
D. Kurang dari 54
11. Titik-titik (1, –1), (3, 4), (m, n), dan (11, –1) adalah titik-titik sudut suatu jajargenjang, m dan n
bilangan bulat. Panjang diagonal terpendeknya adalah ….
A. 10
B. √
C. √
D. 5
12. Tujuh orang tukang kayu dalam waktu 5 jam menghasilkan 6 papan tulis. Dalam waktu 1 jam
papan tulis yang dihasilkan oleh seorang tukang kayu adalah ….
A.

B.

C.

D.

13. Edy berangkat ke sekolah pukul 6.00 setiap pagi. Bila bermobil dengan kecepatan 40 km/jam, dia
tiba di sekolah terlambat 20 menit. Bila kecepatan 60 km/jam, dia tiba 15 menit lebih awal. Di
sekolah Edy, pelajaran jam pertama dimulai pukul ….
A. 7. 30
B. 7. 25
C. 7. 15
D. 7. 00
14. Misalkan , a R sehingga √ . Persamaan kuadrat √
memiliki dua akar real bila ….
A. √
B. √

C. √ atau √

D. √ √
15. Suatu percobaan dilakukan dengan ketentuan sebagai berikut :
i. Pertama kali dilakukan pelemparan sekeping mata uang.
ii. Jika dalam pelemparan mata uang muncul sisi gambar, percobaan dilanjutkan dengan
pelemparan mata uang. Sedangkan jika muncul sisi angka, percobaan dilanjutkan dengan
sebuah dadu bersisi enam.
iii. Jika sampai dengan pelemparan mat uang ketiga kalinya selalu muncul gambar, percobaan
dihentikan.
iv. Jika dalam pelemparan dadu muncul angka genap, pelemparan dihentikan.
v. Jika dalam pelemparan dadu muncul angka ganjil, pelemparan diulang sekali dn selanjutnya
pelemparan dihentikan apapun angka yang muncul.
Peluang bahwa dalam percobaan tersebut tidak pernah terjadi pelemparan dadu adalah ….
A. 1
B.

C.

D.

16. Suatu sekolah mengikutsertakan 3 siswa laki-laki dan 2 siswa perempuan dalam seleksi OSN
tingkat kabupaten/ kota. Diberikan 3 soal pilihan benar-salah. Peluang bahwa tidak ada satupun
siswa laki-laki yang menjawab semua soal dengan benar, sedangkan ada satu siswa perempuan
yang dapat menjawab semua soal dengan benar adalah ….

A.

B.

C.

D.

17. Untuk sembarang p bilangan prima, misalkan h = 14p–4. Pernyataan berikut yang benar adalah….
A. h tidak dapat dinyatkan dalam bentuk kuadrat dari bilangan asli.
B. h dapat dinyatakan dalam bentuk kuadrat dari bilangan asli.
C. Ada bilangan asli n sehingga berlaku 14p-4 = n3
D. Terdapat n bilangan ganjil sehingga 14p–4 = n2

18. Nilai x yang memenuhi persamaan (√ ) ( ) √ adalah ….

A.

B.

C.

D.
19. Rata-rata dari empat bilangan beruurtan adalah 2m–1, maka nilai dari empat kali bilangan terkecil
adalah ….
A. 8m+8
B. 8m+3
C. 8m–7
D. 8m–10
20. Pada pemilihan calon ketua kelas yang diikuti oleh 5 kontestan, diketahui bahwa pemenangnya
mendapat 10 suara. Jika diketahui juga bahwa tidak ada dua kontestan yang memperoleh jumlah
suara yang sama, maka perolehan terbesar yang mungkin untuk kontestan dengan suara paling
sedikit adalah ….
A. 3
B. 4
C. 5
D. 6

BAGIAN B : ISIAN SINGKAT


1. Banyaknya bilangan genap yang kurang dari 1000 dan hasil kali angka-angka penyusun 180
adalah ….
2. Luas persegipanjang ABCD adalah 112 satuan luas. Titik E dan F berada di diagonal AC
seperti gambar di bawah ini sedemikian hingga 3(AE + FC) = 4EF. Luas segitiga DEF adalah
… satuan luas.

3. Jika f(n) menyatakan banyak faktor bilangan asli n, maka f(f(f(2009))) = ….


4. Rata-rata 15 bilangan adalah 0. Bila bilangan-bilangan v, w, x, y, dan z ditambahkan, maka
rataratanya bertambah 5. rata-rata bilangan-bilangan yang ditambahkan adalah ….
5. Lantai suatu ruangan berbentuk persegi. Lantai tersebut akan dipasang keramik berbentuk
persegi juga. Bila keramik yang terletak pada diagonalnya sebnayk 33, maka banyaknya
keramik yang menutupi lantai adalah ….
6. Faisal memperoleh nomor antrean ke-2009 untuk menaiki bus antarkota dalam propinsi, dari
kota malang ke Surabaya. Bus berangkat setiap 5 menit dan setiap pemberangkatan, bus
memuat 55 orang. Jika pemberangkatan pertama berangkat pukul 5.01 pagi, maka Faisal
berangkat pada pukul
7. Jumlah 2009 bilangan bulat berurutan samadengan 6027, maka selisih bilangan terkecil dan
terbesar sama dengan ….
8. Jika nilai ulangan siswa kelas VIII terdiri dari bilangan genap beruurtan dengan nilai terendah
2 dan tertinggi 98, jangkauan interkuartil dari data tersebut adalah ….
9. Dua belas segi delapan beraturan dengan panjang sisi 2 cm. disusun dalam sebuah persegi
seperti gambar berikut.

Luas persegi di atas sama dengan….


10. Jumlah semua bilangan riril x yang memenuhi persamaan berikut adalah ….
( ) (( ) ) ( ( ) )
OLIMPIADE SAINS NASIONAL SMP
SELEKSI TINGKAT KABUPATEN / KOTA TAHUN 2008

BAGIAN A : PILIHAN GANDA


1. Jika P, Q, R angka – angka dari suatu bilangan dan (100P + 10 Q + R)(P + Q + R) = 2008, maka
nilai Q adalah ….
A. 3 C. 5 E. 7
B. 4 D. 6

2.  3    3    3    3    3    3    3   ....
3 2 1 0 1 2 3

A. 1 C. 10
1
4
4
3
E. 81 3
9 9

B. 14
5
3
D. 4
1
4
4
3
9 3 9

3. Misalkan n adalah bilangan asli yang tidak lebih dari 24, maka jumlah dari semua nilai n yang
memenuhi agar n dan 24 relatif prima adalah ….
A. 120 C. 95 E. 81
B. 96 D. 82
4. Perhatikan gambar berikut :
P

Q R
S
Segitiga PQR merupakan segitiga sama sisi. Jika SPQ 20 o dan TQR  35o , maka SUT ....
A. 135o C. 125o E. 95o
B. 130o D. 105o
5. Jika rata – rata dari 15 bilangan asli berbeda adalah 12, maka bilangan asli terbesar yang mungkin
adalah….
A. 45 C. 89 E. 166
B. 75 D. 105
3n 1
6. Jika dibagi 9, maka sisanya sama dengan ….
2
A. 2 C. 4 E. 8
B. 3 D. 6
7. Fahmy menghitung, mulai dari 1000, kemudian bertambah 8 menjadi 1008, 1016, 1024, 1032, ….
Sedangkan Zeldy pada saat yang sama menghitung mulai dari 2008, berkurang 4 menjadi 2004,
2000, 1996, …. Bilangan tepat sama saat mereka menghitung bersama – sama adalah ….
A. 1672 C. 1656 E. 1640
B. 1664 D. 1648
f (b)  f (a)
8. Jika f(z) = az + b, maka nilai dari adalah ….
ba
A. b C. a E. ab
2 2
B. b D. a
9. Suatu bilangan terdiri dari 5 angka. Jika jumlah dari angka – angka tersebut adalah A dan jumlah
dari angka – angka pada bilangan A adalah B, maka nilai terbesar dari B yang mungkin adalah ….
A. 9 C. 11 E. 13
B. 10 D. 12
10. Perhatikan gambar berikut :
P

Q
T
Jika QT garis singgung lingkaran yang berpusat di O dan besar TQR  112o , maka besar
PQT  ....

A. 56o C. 34o E. 24o


B. 44o D. 26o
173 1
11. Jika  a , maka 25a + 5b + 100c +500d = ….
61 1
b
1
c
d
A. 6325 C. 5555 E. 3475
B. 5635 D. 4545
12. Bapak dan ibu Zainal sedang merencanakan nama bagi anak mereka yang akan segera lahir
dengan nama yang terdiri dari 3 kata dengan nama belakang zainal. Mereka menginginkan inisial/
singkatan nama anak tersebut adalah terurut menurut abjad dengan tak ada huruf yang berulang,
sebagai contoh GTZ, tetapi mereka tidak mau TGZ. Banyak pilihan inisial nama yang dapat
dipergunakan adalah ….
A. 25 C. 150 E. 600
B. 125 D. 300
13. Pada bulan Januari harga tas di Toko Rima adalah Rp. 150.000,00. Pada bulan Februari harga tas
naik 10%, tetapi bila yang membeli pelajar memperoleh potongan 10%. Pada bulan Maret
potongan bagi pelajar tidak berlaku lagi, tetapi harga tas tersebut turun menjadi Rp. 135.000,00
dan pembeli dibebani pajak pembelian sebesar 10%. Dua orang pelajar, Andi dan Anton, membeli
tas tersebut. Andi membeli pada bulan Pebruari, sedangkan Anton membeli pada bulan Maret.
Pernyataan berikut yang benar adalah ….
A. Anton membayar sebesar Rp 150.000,00 untuk tas yang dibelinya.
B. Andi membayar sebesar Rp 150.000,00 untuk tas yaqng dibelinya.
C. Jumlah uang yang dibayarkan Andi sama dengan jumlah uang yang dibayarkan Anton.
D. Jumlah uang yang dibayarkan Andi lebih besar dari jumlah uang yang dibayarkan Anton
E. Diantara tiga bulan tersebut di atas, bulan Januari adalah bulan yang paling menguntungkan
bagi pelajar untuk membeli tas.
14. Pada gambar berikut diketahui panjang tali busur AB = 24 cm dan MO = ON, maka luas daerah
yang diarsir adalah ….
A. 24 cm2
B. 72 cm2
C. 104 cm2
D. 144 cm2
E. 152 cm2
15. Huruf ke– 2008 dari pola : O,L,I,M,P,I,A,D,E,S,A,I,N,O,L,I,M,P,I,A,D,E,S,A,I,N, …. adalah ….
A. A C. E E. M
B. D D. I
16. Seorang pedagang menjajakan 10 jeruk manis dan 5 jeruk masam yang kesemuanya terlihat sama
dan diletakkan dalam satu keranjang yang sama. Jika Ana ingin membeli 2 buah jeruk dan
mengambilnya sekaligus secara sembarang, maka peluang Ana akan memperoleh 2 jeruk dengan
rasa yang sama adalah ….
A. B. C. D. E.

17. Perhatikan gambar berikut :

ABCD merupakan persegi panjang dan EFGH adalah jajaran genjang, maka panjang sisi x adalah
A. 6,8 C. 7,6 E. 8,1
B. 7,2 D. 8,0
18. Suatu deret aritmatika mempunyai suku pertama a dan beda 10. Jumlah n suku pertama adalah
10.000. Jika suku ke – n kurang dari 500. Maka nilai n terbesar yang mungkin adalah ….

A. 73 C. 71 E. 69
B. 72 D. 70

Bilangan – bilangan 3, 4, dan 7 disubstitusikan sebarang dan boleh berulang untuk menggantikan
konstanta – konstanta a,b, dan c pada persamaan kuadrat ax2 + bx + c = 0. Peluang persamaan
kuadrat itu mempunyai akar – akar real adalah ….
A. 1 C. 1 E. 1
3 9 27
B. 1 D. 1
18 18
19. Perhatikan gambar berikut :

Pada segiempat ABCD dibuat setengah lingkaran pada sisi AD dengan pusat E dan segitiga sama
sisi BEC. Jika BC = 20cm, maka luas daerah yang diarsir adalah ….
A.
100 
3  50 cm 2

B.  50 
100 3    cm 2
 3 

C.  50 
100 3    cm 2
 6 

D.  100 
100 3    cm 2
 3 

E.
100  100 3 cm 2
BAGIAN B : ISIAN SINGKAT
1. Pada saat makan siang, Taufan menghabiskan 1/3 dari uang yang ia miliki. Setelah makan siang,
ia menerima uang dari temannya sebesar Rp. 25. 000,00. Sore harinya, ia membeli tiket bioskop
sebesar Rp. 40.000,00 dan membeli makanan seharga Rp. 12.500,00. sekarang uangnya tersisa
Rp. 52.500,0, berapa uang taufan sebelum makan siang ?
2. Lima orang dalam satu keluarga dicatat nama dan umurnya, sebagaimana tampak pada table
berikut.
Anak Anak Anak
Nama Ayah Ibu
I II III
Umur
40 36 8 6 2
(tahun)
Rata – rata umur keluarga tersebut lima tahun yang lalu sama dengan….
3. Garis g melalui titik (-2,3), memotong sumbu – X dititik A, dan memotong sumbu – Y dititik B.

Jika jarak titik O dengan titik A sama dengan jarak titik O dengan titik B, maka persamaan garis g
adalah ….
4. Intan berjalan kaki dengan kecepatan tetap 4, 5 km/jam pada suayu jalur lurus kea rah utara.
Dikejauhan pada jarak 2, 7 km dari arah utara pada jalur yang sama, Mufti mengendarai sepeda
dengan kecepatan lima kali lipat kecepatan intan. Lama waktu yang diperlukan sehingga mereka
akan kembali berjarak 2,7 km satu sama lain adalah ….
5. Misalkan banyak anggota hmpunan A dan B berturut – turut adalah m dan n, dengan m > n.
Banyak anggota himpunan A  B paling sedikit adalah ….
6. Diberikan sebuah persegi dengan sisi a satuan, sebagaimana tampak pada gambar berikut. Empat
buah segitiga siku – siku dipotong dari persegi tersebut seperti digambarkan sebagai daerah
berarsir abu – abu. Diketahui semua sisi siku – siku yang lebih pendek memiliki panjang 3/8 a
satuan. Luas daerah tak berarsir pada persegi tersebut adalah ….

7. Anto memiliki sejumlah kubus kecil berwarna putih yang ia susun menjadi sebuah kubus yang
lebih besar. Sedikitnya satu sisi kubus besar dicat dengan warna hijau, tetapi masih ada setidaknya
satu sisi tetap berwarna putih. Kubus besar tersebut kemudian dibongkar kembali dan ditemukan
bahwa ada 1000 buah kubus kecil yang tetap berwarna putih disemua sisinya. Banyak sisi kubus
besar yang telah diberi warna hijau adalah ….
8. Diketahui z adalah bilangan asli yang memenuhi semua syarat berikut :
a. z terdiri dari 5 angka
b. Angka penyusun z tidak ada yang berulang
c. Penjumlahan semua angka penyusun z adalah 10
d. Jika z ditambah dengan bilangan cerminnya maka akan diperoleh sebuah bilangan lima
angka yang semua angkanya sama
Bilangan z terbesar yang mungkin adalah …
Keterangan : Bilangan cermin adalah bilangan dengan angka penyusun yang sama tetapi
memiliki urutan angka terbalik. Disamping itu, bilangan cermin dapat memiliki angka 0 pada
posisi pertama, sedangkan bilangan semula tidak.

9. Suatu kerucut tegak tertutup yang berisi air dengan diameter d cm dan tinggi x cm. Tinggi air
pada kerucut adalah 1/2 x cm. Jika posisi kerucutnya dibalik, maka tinggi air pada kerucut
tersebut adalah ….
10. Perhatikan gambar berikut .

Dengan mulai dari angka 2 pada lingkaran ditengah, bilangan 2008 dapat dibentuk dari
pergerakan satu lingkaran ke satu lingkaran lainnya jika lingkarannya saling bersinggungan.
Banyak cara untuk membentuk bilangan 2008 adalah ….
OLIMPIADE SAINS NASIONAL SMP
SELEKSI TINGKAT KABUPATEN / KOTA TAHUN 2007

BAGIAN A : PILIHAN GANDA


1. Urutan bilangan–bilangan 25555, 52222, dan 33333 dari terkecil sampai yang terbesar adalah ….
A. 25555,52222,33333 D. 52222,25555,33333
B. 52222,33333,25555 E. 33333,52222,25555
C. 33333,25555,52222
2. Misalkan a, b, dan c bilangan bulat. Pernyataan berikut yang salah adalah ….
A. Jika a membagi b dan b membagi c, maka a membagi c
B. Jika a membagi b dan c, maka a membagi b + c
C. Jika a membagi b dan c, maka a membagi bc
D. Jika a membagi c dan b membagi c, maka ab membagi c
E. Jika a membagi b, maka a membagi bc
ab
3. Misalkan untuk bilangan bulat a dan b didefinisikan a*b = . Untuk semua bilangan bulat a,
2
b, dan c.
I. a*b = b*a
II. a*a = a
III. a*(b*c) = (a*b)*c
Pernyataan yang benar adalah ….
A. I saja C. III saja E. I, II, dan III
B. II saja D. I dan II saja
4. Bilangan cacah lima digit dengan digit pertama tidak nol dan jumlah semua digitnya sama dengan
2 ada sebanyak ….
A. 1 C. 3 E. 5
B. 2 D. 4
5. Perhatikan gambar!

Nilai dari a + b + c + d + e + f + g + h + I adalah ….


A. 360 C. 720 E. 1.260
B. 540 D. 900
6. Suatu bilangan kuadrat jika dibagi 3, maka kemungkinan sisanya adalah ….
A. 0 C. 2 E. 0, 1, atau 2
B. 1 D. 0 atau 1
7. Seorang pedagang membeli 25 kg beras jenis A seharga Rp6.000,00. Setiap kg dan 15 kg beras
jenis B seharga Rp4.000,00 setiap kg. Kedua jenis beras tersebut kemudian dicampur. Agar
mendapat untung 4% setiap kg beras tersebut dijual seharga ….
A. Rp5.200,00 D. Rp5.580,00
B. Rp5.460,00 E. Rp6.240,00
C. Rp5.520,00
8. Jika f fungsi dari himpunan bilangan asli ke himpunan bilangan asli yang memenuhi f(x) + f(x +
1) = 2x2 dan f(31) = 99, maka f(99)= ….
A. 8.673 C. 8.871 E. 9.604
B. 8.772 D. 9.505
9. Diketahui suatu segitiga sama sisi dan setengah lingkaran seperti pada gambar. Jika panjang sisi
segitiga tersebut adalah 14 cm, maka luas daerah yang diarsir adalah

3
A. 49 3 – 14 D. 98 3 – 36 
4

1 1
B. 49 3 – 24  E. 98 3 – 24 
2 2

C. 49 3 – 183/8
10. Suatu lapangan rumput berbentuk persegi ABCD seperti pada gambar dengan lapangan AB = 7 m.
Seekor kambing diikat di E dengan tali sepanjang 4 m. Jika jarak AE = 2 m. Luas daerah rumput
yang dapat dimakan kambing tersebut adalah …. m2.
A B

E 

D C

A. 2 3 + 16 / 3 D. 4 + 4

B. 2 3 + 4 E. 8 + 4
C. 2 2 + 16 / 3
11. Banyak jalan terpendek dari P ke Q adalah ….

A. 4 C. 22 E. 80
B. 16 D. 60
12. Pada pukul 10.15 penerjun payung melompat dari pesawat sambil membuka parasutnya. Setelah 8
detik, ketinggiannya 2 m dari permukaan tanah. Lima detik kemudian ketinggiannya 1900 m dari
permukaan tanah. Misalkan mulai detik ke 8 sampai dengan 1 menit kecepatannya tetap.
Ketinggiannya pada pukul 10.16 adalah …. meter.
A. 860 C. 940 E. 980
B. 890 D. 960
13. Desi merayakan hari ulang tahun pada tanggal 27 Desember 2006. Jika pada tersebut usia Desi
sama dengan jumlah digit dari angka tahun kelahirannya, maka Desi lahir pada tahun ….
A. 1994 C. 1989 E. 1979
B. 1992 D. 1984
14. Suatu barisan hanya terdiri dari bilangan 1, 2, 3, 4, dan 5. Jika barisan tersebut adalah 1, 2, 2, 3, 3,
3, 4, 4, 4, 4, 5, 5, 5, 5, 5, , 1, 1, 1, 1, 1, 1, 2, 2, 2, 2, 2, 2, 2, 3, 3, 3, 3, 3, 3, 3, 3, 4, 4,… maka
urutan ke 100 dari barisan tersebut adalah ….
A. 1 C. 3 E. 5
B. 2 D. 4
8
 2
15. Konstanta dari  3x 3   adalah ….
 x

A. 14.328 C. 16.112 E. 17.128


B. 15.552 D. 16.128
16. Banyaknya bilangan asli yang kurang dari 10.000 dengan jumlah digit pertama dan digit
terakhirnya sama dengan 11 adalah ….
A. 999 C. 800 E. 400
B. 888 D. 444
17. Perhatikan gambar. Jika bilangan pada derah persegi tidak diarsir diperoleh dengan
menjumlahkan dua bilangan pada persegi tidak diarsir di bawah dan terhubung dengannya, maka
nilai x adalah

A. 1 C. 9 E. 54
B. 6 D. 27
18. Perhatikan gambar, diketahui PQRS adalah jajargenjang dan misalkan garis SU memotong
diagonal PR di titik T, memotong ruas garis QR di titik U, dan memotong garis PQ di titik T.
S R

V
P Q
Maka panjang UV adalah ….
A. 12 C. 20 E. 24
B. 18 D. 22
19. Dua mata uang dilempar empat kali berturut–turut. Peluang muncul angka pertama kali pada
pelemparan keempat adalah ….
1 3 1
A. C. E.
44 44 4

2 1
B. D.
44 42

20. Untuk meningkatkan penjualan, suatu perusahaan memberikan hadiah yang dimuat dalam setiap
kotak susu yang dijual satu dari empat seri buku secara acak. Jika Gina membeli empat kotak
susu, maka peluang Gina mendapatkan semua seri buku hadiah adalah ….
A. 1 C. 3 E. 1
256 32
B. 3 D. 1
256 4

BAGIAN B : ISIAN SINGKAT


1. Jika bilangan 123.456.789 dikalikan dengan bilangan 999.999.999, maka banyak angka 9 dari
hasil perkalian kedua bilangan tersebut adalah ….
2. Kota A terletak 50 km di sebelah utara kota B, dan kota C terletak 120 km di sebelah timur kota
B, dan kota D terletak di tengah antara kota B dan kota C. Jarak kotak D dari kota A adalah ….
3. Perhatikan dua lingkaran konsentrik (memiliki pusat yang sama) seperti pada gambar.

Jika keliling besar lebih panjang 4 meter dari keliling lingkaran kecil, maka jarak d adalah….
4. Perhatikan gambar. Jika pada setiap persegi ditempatkan suatu bilangan bulat positif sedemikian
rupa sehingga perkalian bilangan – bilangan dari sembarang lima persegi yang berurutan
menghasilkan 360, maka jumlah bilangan pada semua persegi tersebut adalah…

5. Luas daerah yang diarsir adalah….

6. Jika H adalah himpunan semua pembagi positif dari 2007, maka banyak himpunan bagian dari H
yang tidak kosong adalah….
7. Suatu pabrik pembuat tas memilki pekerja laki – laki sama banyak dengan pekerja wanita.
Kecepatan kerja pekerja laki – laki dan wanita sama. Dalam waktu 6 hari, 6 pekerja laki – laki dan
8 pekerja wanita dapat menghasilkan 4.200 tas. Jika dalam waktu 7 hari, seluruh pekerja pabrik
dapat menghasilkan 5.600 tas, maka pekerja laki – laki pada pabrik tersebut ada sebanyak ….
Orang.
8. Himpunan semua bilangan prima yang kurang dari 100 dan kuadrat bilangan tersebut ditambah 2
juga merupakan bilangan prima adalah….
9. Perhatikan gambar . Banyak daerah persegi yang terletak pada daerah persegi ABCD berukuran 9
x 9 dan paling sedikit satu sisinya terletak pada persegi ABCD adalah ….
Petunjuk : salah satu daerah persegi yang dimaksud adalah daerah yang diarsir.

10. Di laboratorium matematika terdapat 6 batang kayu sejenis yang panjangnya berturut – turut 2
dm, 4 dm, 4 dm, 10 dm, 22 dm, dan 37 dm. Jika keenam batang kayu tersebut harus digunakan
untuk membuat trapesium sama kaki, maka banyaknya trapesium sama kaki yang dapat dibentuk
adalah ….
OLIMPIADE SAINS NASIONAL SMP
SELEKSI TINGKAT KABUPATEN / KOTA TAHUN 2006

BAGIAN A : PILIHAN GANDA


1. Jumlah dua bilangan bulat yang berbeda adalah 14. Jika hasil bagi kedua bilangan tersebut adalah
juga bilangan bulat, maka salah satu bilangan yang mungkin adalah ….
A. 2 C. 6 E. 9
B. 4 D. 7
1 1 1
2. Jika   , maka x = ….
6 12
A. 4 D. 2 dan –2
B. 4 dan –4 E. Tidak ada jawaban yang benar
C. 2
3. Pada suatu peta tertulis perbandingan 1 : 200.000. Jika jarak antara dua kota adalah 50 km, maka
jarak kedua kota tersebut dalam peta adalah ….
A. 0,25 cm C. 25 cm E. 10 cm
B. 2,5 cm D. 1 cm
4. Ali, Ani, dan Budi pergi ke suatu toko untuk membeli pensil dan buku yang sama. Ali membeli
dua buku dan dua pensil, Ani membeli 3 pensil dan 4 buku, sedangkan Budi membeli satu pensil
dan dua buku. Jika Ali dan Ani berturut–turut membayar Rp2.500,00 dan Rp4.500,00 maka Budi
harus membayar ….
A. Rp1.000,00 D. Rp2.500,00
B. Rp1.500,00 E. Rp3000,00
C. Rp2.000,00
5. Diberikan kerucut dengan volume 77 cm3. Jika tinggi kerucut itu 6 cm, maka jari–jari alasnya
adalah ….
A. 2 cm C. 7 cm E. 12,25 cm
B. 3,5 cm D. 10,3 cm
6. Jika panjang diagonal suatu persegi adalah 4 cm, maka luas persegi itu (dalam cm2) adalah ….
A. 2 C. 6 E. 16
B. 4 D. 8

7. Bilangan asli n sedemikian sehingga hasil kali ( )( )( ) ( )

merupakan bilangan bulat adalah ….


A. n ganjil D. n sebarang
B. n genap E. tidak ada n yang memenuhi
C. n kelipatan 3
1 1
8. Selisih terbesar dari 2 bilangan rasional x yang memenuhi pertidaksamaan  2x  adalah ….
5 2
1 1
A. D.
20 80
1 E. Jawaban A, B, C, dan D salah
B.
10
1
C.
8
9. Misalkan A = {1, 2, 3} dan B = {a, b, c}. Banyaknya korespondensi satu–satu yang dapat dibuat
dari A ke B adalah
A. 1 C. 6 E. 27
B. 3 D. 9
10. Seorang Ayah berumur 39 tahun mempunyai dua orang anak bernama Budi dan Wati. Tahun
depan, selisih umur Ayah dan Budi dibandingkan selisih umur Ayah dan Wati adalah 14 : 19. Jika
umur Ayah sekarang adalah tiga kali umur Budi ditambah enam kali umur Wati, maka jumlah
umur Wati dan umur Budi ditambah enam kali umur Wati, maka jumlah umur Wati dan umur
Budi tiga tahun yang akan datang adalah ….
A. 17 C. 19 E. 21
B. 18 D. 20
11. Suatu garis lurus memotong sumbu X di titik (a, 0) dan memotong sumbu Y di titik (0, b) dengan
a dan b adalah bilangan bulat. Jika luas segitiga AOB adalah 12 satuan luas, maka banyaknya
bilangan bulat a dan b yang mungkin adalah ….
A. 4 pasang C. 16 pasang E. 64 pasang
B. 8 pasang D. 32 pasang
12. Misalkan a, b, dan c adalah panjang sisi–sisi suatu segitiga, dengan a, b, dan c berupa bilangan
asli yang berurutan yang rata–rata hitungnya 6. Jika ditarik garis tinggi terhadap sisi yang
panjangnya b, maka panjang garis tinggi tersebut adalah ….
A. 6 6 C. 2 6 E. 2 2

B. 4 6 D. 4 2

13. Pada segitiga PQR, S adalah titik tengah QP dan T titik tengah QR. Perbandingan antara TS dan
QR adalah ….
A. 1 : 2 C. 2:3 E. 3 : 5
14. Luas daerah yang diarsir setengah dari luas yang tidak diarsir. Panjang AB dibagi panjang AC
adalah…. C
1 1 1
A. 2 C. 5 E. 5
2 5 7 B
1 1
B. 3 D. 7
3 7
A
1 1 1 1 1
15. Misalkan m dan n adalah bilangan bulat dan 0 < m < n. Jika   , maka  = ….
m n 3 m n

2 1 5
A. C. – E.
3 6 6

1 2
B. D. –
6 3

16. Banyaknya bilangan bulat dari – 1006 sampai dengan 2006 yang merupakan kelipatan 3 tetapi
bukan kelipatan 6 adalah ….
A. 500 bilangan D 503 bilangan
B. 501 bilangan E. 504bilangan
C. 502 bilangan
17. Bentuk sederhana dari (y + x)(x – y) [ x(x – y) + y(y + x)]} adalah ….
A. x4 + y4 D. –(x4 + y4)
B. x4 – y4 E. tidak ada jawaban yang benar
C. y –x4 4

18. Jika 5 ≤ x ≤ 10 dan 2 ≤ y ≤ 6, maka nilai minimum untuk (x – y)(x + y) adalah ….


A. –21 C. –11 E. 12
B. –12 D. 11
19. Perhatikan gambar di bawah ini. Jika CE = EB, AD = DB, besar ABC = 30o, dan panjang CA =
4 cm, maka panjang CF adalah

E
F

300
A B
D
4 2 1
A. 28 C. 7 E. 7
3 7 3

1 4
B. 28 D. 7
3 3

20. Perhatikan gambar di bawah ini. Jika luas BCDE = luas ABE, dan panjang CD = 8 ,
maka panjang BE = ….
A

B E

C D

1
A. 4 D. 2
2
E. Jawaban A, B, C, dan D
B. 2
tidak ada yang benar
C. 2
BAGIAN B : ISIAN SINGKAT
1. Jika jumlah dua bilangan adalah 3 dan selisih kuadrat bilangan itu adalah 6, maka hasil kali kedua
bilangan ….
2. Panjang jalan tol Bogor Jakarta 60 km. Pada pukul 12.00 mobil A berangkat dari pintu tol Bogor
menuju Jakarta dengan kecepatan rata–rata 80 km/jam. Pada saat yang sama mobil B berangkat
dari pintu tol Jakarta menuju Bogor dengan kecepatan rata–rata 70 km/jam. Kedua mobil tersebut
akan berpapasan pada pukul ….
3. Jika segi n beraturan besar sudut–sudutnya 135o, maka n =
1 1
4. Semua bilangan bulat x sehingga  merupakan bilangan bulat adalah ….
2 2
5. Bilangan asli n terbesar sehingga jumlah 1 + 3 + 5 + … + (2n – 1) lebih kecil 2006 adalah ….
6. Semua pasangan bilangan real (x,y) yang memenuhi x2 + y2 = 2x – 4y – 5 adalah ….
7. Perhatikan gambar di bawah ini. Jika panjang AB = 2 cm, panjang CD = 3 cm, dan panjang AC =
9 cm, maka panjang BC adalah ….
C

B x  x
D

8. Banyaknya faktor dari 4200 yang merupakan bilangan ganjil positif adalah ….

9. =

10. Perhatikan gambar di bawah ini. Jika panjang AB = 3 cm, panjang AD = 8 cm, panjang CD = 5
cm, dan titik E terletak pada ruas garis BC, maka panjang minimal dari AE + AD adalah

B C

D
OLIMPIADE SAINS NASIONAL SMP
SELEKSI TINGKAT KABUPATEN / KOTA TAHUN 2005

BAGIAN A : PILIHAN GANDA


0,036
1. = ….
0,9

A. 0,002 C. 0,2 E. 20
B. 0,02 D. 2
11 13
2. Di antara bilanga–bilangan berikut, manakah yang terletak di antara dan ?
15 18
11 15 24
A. C. E.
18 18 33
13 11
B. D.
15 13
3. Perhatikan tiga barisan enam bilangan berikut.
1) 8, 16, 32, 64, 128, dan 256
2) 7, 11, 16, 22, 29, 37
3) 2, 9, 2, 16, 2, 25
manakah dari 3 barisan tersebut yang mungkin menjadi 6 suku berikutnya dari suatu barisan
bilangan yang tiga suku pertamanya adalah 1, 2, dan 4?
A. (1) C. (3) E. Semua
B. (2) D. (1) dan (2)
4. Perhatikan gambar berikut!
Jika jarak terdekat titik–titik tersebut secara vertial maupun horizontal adalah 2 satuan, maka luas
daerah persegi pada gambar di atas adalah …. satuan luas.

A. 10 C. 20 E. 50
B. 40 D. 30
5. Uang sebesar Rp2.000,00 dapat dinyatakan dengan beberapa koin 50 rupiahan, 100 rupiahan, 200
rupiahan dan/atau 500 rupiahan. Diketahui ternyata bahwa uang tersebut terdiri dari tepat dua
koin 500 rupiahan, dan dua koin jenis lainnya. Dengan mengikuti aturan tersebut banyak cara
yang mungkin untuk menyatakan uang sebesar Rp2.000,00 dengan koin–koin tersebut adalah ….
A. 17 C. 100 E. 8
B. 20 D. 10
6. Sekumpulan data yang terdiri dari 5 bilangan asli yang memiliki rata–rata hitung 8 dan rentang
(selisih terbesar dan terkecil) 12. Bilangan asli terkecil yang tidak mungkin menjadi anggota dari
kumpulan tersebut adalah ….
A. 1 C. 18 E. 15
B. 20 D. 6
7. Bilangan 43 dapat dinyatakan ke dalam bentuk 5a + 11b, karena untuk a = 13 dan b = –2, nilai
dari 5a + 11b adalah 43. Manakah dari 3 bilangan 37, 254, dan 1986 yang tidak dapat dinyatakan
dalam bentuk 5a + 11b?
A. 1983 C. 254 dan 1986 E. tidak ada
B. 254 D. semua
8. Tiga ekor ayam (Besar, Sedang, dan Kecil) ditimbang. Jika yang Besar dan yang Kecil ditimbang,
beratnya adalah 2,6 kg. Jika yang Besar dan Sedang ditimbang beratnya adalah 3 kg, dan jika
yang Sedang dan yang Kecil ditimbang, beratnya adalah 2 kg. Berat ketiga ayam itu seluruhnya
adalah ….
A. 4 kg C. 3,8 kg E. 5 kg
B. 4,2 kg D. 4,6 kg
9. Salah satu faktor dari 173 – 53 adalah ….
A. 5 C. 13 E. 399
B. 17 D. 273
10. Bilangan yang ditunjukkan oleh
1
adalah ….
(1 2 )(2  3 )(1 2 )(2  3 )

A. Bilangan irrasional positif


B. Bilangan rasional tidak bulat
C. Bilangan bulat negatif
D. Bilangan irrasioanl negatif
E. Bilangan bulat positif

BAGIAN B : ISIAN SINGKAT


1. 10 pasang suami istri mengikuti pesta. Mereka kemudian saling berjabatan tangan satu sama lain.
Namun demikian, setiap suami istri tidak berjabatan tangan. Maka banyaknya jabatan tangan yang
terjadi adalah
2. Misalkan a dan b menyatakan luas daerah yang diarsir pada gambar di bawah. Kelima lingkaran
kecil berjari–jari r. Titik–titik pusat empat lingkaran kecil yang menyinggung lingkaran besar
merupakan titik–titik sudut persegi. Jika a sama dengan 10 cm2, maka b =….
3. Diberikan persegi ABCD dengan panjang sisi satu satuan panjang. Misalkan P suatu titik di
dalam, sehingga ukuran sudut APB adalah 120. Jumlah luas daerah segitiga APB dan segitiga
PCD adalah ….
4. Untuk bilangan real a dan b didefinisikan suatu operasi * dengan aturan sebagai berikut : a*b = (a
 b) + (a + b) dimana simbol  dan + berturut–turut artinya adalah perkalian dan penjumlahan
bilangan biasanya. Tentukan nilai a yang memenuhi ketentuan a*a = 3.
n 1
5. H adalah himpunan bilangan asli n demikian sehingga bentuk menghasilkan bilangan bulat
n
kurang dari 1. Banyaknya himpunan bagian tak kosong dari H adalah ….
6. Dalam satu tahun harga sebuah mobil berkurang 10% dari harga tahun sebelumnya. Paling sedikit
berapa tahun sehingga harga mobil itu kurang dari setengahnya harga semula?
7. Setiap kotak piramid di samping akan diisi dengan bilangan. Mula–mula yang harus diisi adalah
kotak–kotak pada alas piramid. Kotak di atasnya diperoleh dari menjumlahkan bilangan–bilangan
yang ada di dalam dua kotak yang ada di bawahnya. Andaikan dasar piramid hendak diisi dengan
bilangan–bilangan 7, 12, 5, 4, dan 9, berapakah nilai terbesar yang mungkin dari bilangan pada
kotak teratas?

8. Bentuk sederhana dari


adalah ….
( )( )

9. Sebuah balok memiliki sisi–sisi yang luasnya 24 cm2, 32 cm2, dan 48 cm2. Berapakah jumlah
panjang semua rusuk balok tersebut?
10. Pompa air merek Tangguh sanggup memompa air 25 liter setiap menit. Pompa merek Perkasa
sanggup memompa air 400 cc setiap detik, sedangkan merek Tahan Banting sanggup memompa
1,6 m3 setiap jam. Pompa manakah yang paling cepat mengisi sebuah tangki air berkapasitas 500
liter?
OLIMPIADE SAINS NASIONAL SMP
SELEKSI TINGKAT KABUPATEN / KOTA TAHUN 2004

BAGIAN A : PILIHAN GANDA

5050  4950 = ….

A. 10 C. 1.000 E. 100.000
B. 100 D. 10.000
2. Persegi panjang besar berukuran 9 cm  5 cm. Daerah yang diarsir adalah satu–satunya bangun di
dalam persegi panjang yang bukan persegi. Berapa luas persegi tersebut?
A. 1,5 cm2 D. 3,5 cm2
B. 2 cm2 E. 4 cm2
C. 3 cm2

b
3. Jika a = , maka b dinyatakan dalam a adalah..
1 b

a2
A. b = 1 + a2 D. b =
1 a2

1 a2 a2
B. b= E. b=
a2 1 a2

1 a2
C. b=
a2
4. Bilangan segitiga adalah bilangan yang berbentuk n(n+1)/2, dengan n adalah bilangan asli.
Banyaknya bilangan segitiga yang kurang dari 100 adalah ….
A. 8 B. 9 C. 10 D. 13 E. 15
5. Joko mengalikan 3 bilangan prima berbeda sekaligus. Ada berapa faktor berbeda yang dihasilkan?
A. 3 C. 5 E. 8
B. 4 D. 6
6. Persegi pada gambar di samping ini memiliki luas satu satuan luas. Pecahan yang menyatakan
luas daerah yang tidak diarsir adalah ….
A. 1/3 D. 3/7
B. 2/5 E. 3/8

C. 3/5
s
7. Pecahan adalah pecahan sejati, jika s < t dan faktor persekutuan terbesarnya adalah 1. Jika t
t
memiliki nilai mulai 2 sampai dengan 9, dan s bilangan positif, maka banyaknya bilangan sejati
berbeda yang dapat dibuat adalah ….
A. 26 C. 28 E. 30
B. 27 D. 29
8. 3% dari 81 sama dengan 9% dari ….
A. 27 C. 72 E. 243
B. 54 D. 90
9. Jumlah 101 bilangan bulat berurutan adalah 101. Berapakah bilangan bulat terbesar di dalam
barisan bilangan itu?
A. 51 C. 100 E. 150
B. 56 D. 101
10. Dengan menggunakan uang koin Rp 50,00; Rp 100,00; Rp 200,00; ada berapa carakah kita
menyatakan uang sebesar Rp 2.000,00?
A. 20 C. 95 E. 121
B. 65 D. 106

BAGIAN B : ISIAN SINGKAT


1. Pada gambar di samping, garis PQ dan garis RS sejajar, demikian juga garis PS dan QT sejajar.
Nilai x sama dengan… P Q
41
83

x
S R

2. Alex selalu berbohong pada hari–hari Kamis, Jumat, dan Sabtu. Pada hari–hari lai Alex selalu
jujur. Di lain pihak, Frans selalu bohong pada hari–hari Minggu, Senin, dan Selasa, dan selalu
jujur pada hari lainnya. Pada suatu hari, keduanya berkata : “Kemarin saya berbohong”. Hari
mereka mengucapkan perkataan tersebut adalah hari…
n3
3. Semua n sehingga n dan keduanya merupakan bilangan bulat adalah …
n 1
1 2 3 11
4. Misalkan N =  2  3    11 . Dalam bentuk desimal, nilai dari N adalah…
10 10 10 10
5. Diberikan tempat air berbentuk kerucut (lihat gambar). Untuk mengisi air sampai pada ketinggian
t diperlukan air sebanyak 38,5 liter. Dalam liter, volume air yang dibutuhkan untuk memenuhi
tempat tersebut adalah..

½t
6. 213 jika dibagi dengan 13 akan memberikan sisa sama dengan
7. Tujuh ekor kambing menghabiskan rumput seluas 7 kali ukuran lapangan sepak bola dalam waktu
7 hari. Waktu yang diperlukan oleh 3 ekor kambing untuk menghabiskan rumput seluas 3 kali
lapangan ukuran lapangan sepakbola adalah …. Hari.
8. Rata – rata sembilan bilangan adalah 6. Satu diantara kesembilan bilangan dibuang. Rata – rata
1
delapan bilangan yang tinggal adalah 6 . Bilangan yang dibuang adalah
2

9. Jumlah semua angka pada bilangan 22005 . 52003 adalah ….


10. perhatikan gambar berikut. Panjang CP adalah ….
D C

3 P

A B
OLIMPIADE SAINS NASIONAL SMP
SELEKSI TINGKAT KABUPATEN / KOTA TAHUN 2003

BAGIAN A : PILIHAN GANDA


1. 44 + 44 + 44 + 44 = ….
A. 27 C. 210 E. 1034
B. 54 D. 512
2. Kelipatan persekutuan terkecil dari 210, 42, dan 70 adalah ….
A. 14 C. 210 E. 420
B. 7 D. 1260
3. Joko tidur dari 9.20 tadi malam dan bangun pagi harinya pukul 4.35. Ia tidur selama ….
A. 4 jam 45 menit D. 7 jam 15 menit
B. 5 jam 15 menit E. 19 jam 15 menit
C. 5 jam 45 menit
4. Gabah hasil panen sawah mempunyai kadar 25%. Setelah dijemur kadar airnya menyusut
sebanyak 80%. Kadar air gabah tersebut saat ini adalah ….
A. 2,5% C. 5% E. 10%
B. 15% D. 2%
5. Jika a dan b adalah bilangan bulat genap a > b, maka banyaknya bilangan bulat ganjil di antara a
dan b adalah ….
( a  b) (a  b  2) E. Tidak dapat
A. C.
2 2 ditentukan
B. a – b D. a – b + 1
6. Di dalam suatu lingkaran yang berjari–jari 4 cm dibuat persegi ABCD sehingga titik–titik sudut
persegi tersebut berada pada lingkaran. Luas persegi ABCD tersebut adalah…
A 64 cm2 C. 16 cm2 E. 4 cm2
B. 32 cm2 D. 8 cm2
7. Kendaraan A berjalan dengan laju 60 km/jam. Dua jam berikutnya kendaraan B dengan laju 80
km/jam berangkat dari tempat dan menuju arah yang sama. Setelah berapa jam kendaraan B
menyusul kendaraan A?
A. 2 jam C. 4 jam E. 6 jam
B. 3 jam D. 5 jam
8. Pada gambar di samping, ABCD adalah persegi dan ABE adalah segitiga sama sisi. Besar sudut
DAE adalah ….

D C

A B

A. 15 C. 45 E. 75


B. 30 D. 50
9. Faktorisasi prima dari 5220 adalah ….
A. 22. 32.145 C. 22.32.5.29 E. 22.35.5
10. Harga sepotong kue turun dari Rp 250,00 menjadi Rp 200,00. Dengan uang Rp 4.000,00 berapa
potong kue lebih banyak yang dapat dibeli sekarang?
A. 4 C. 20 E. 6
B. 8 D. 2

BAGIAN B : ISIAN SINGKAT


11. Dengan menggunakan angka–angka 1, 1, 2, 2, 3, 3, 4, 4 bilangan 8–angka terbesar yang dapat
dibentuk dengan syarat kedua angka 1 dipisahkan oleh satu angka yang lain, kedua angka 2
dipisahkan oleh kedua angka yang lain, kedua angka 3 dipisahkan oleh tiga angka, dan kedua
angka 4 dipisahkan oleh empat angka yang lain adalah ….
12. Hasil kali suatu bilangan genap dan suatu bilangan ganjil adalah 840. Bilangan ganjil yang
terbesar yang memenuhi syarat tersebut adalah ….
13. Jumlah dua bilangan sama dengan 12. Hasil kali dua bilangan tersebut nilainya akan paling besar
jika salah satu bilangannya adalah ….
14. Perhatikan gambar berikut! Banyaknya bilangan bulat hitam pada gambar ke sepuluh adalah ….
             
             
           
        
   dibawah
15. Banyaknya segitiga pada gambar   adalah ….
16. Gambar bangun berikut dibangun oleh 5 persegi yang kongruen. Jika keliling bangun 72 cm,
maka luas bangun tersebut adalah ….

17. Pada gambar berikut, ABCD adalah persegi dengan sisi 6 satuan. Titik E dan F membagi diagonal
AC menjadi tiga bagian sama panjang. Luas segitiga BEF = ….
D C

A B

18. Diketahui sebuah bak berbentuk balok yang terisi penuh dengan air. Bak tersebut akan
dikosongkon tanpa sisa dengan menggunakan pompa yang mampu menyedot air 0,7 liter per
detik. Dalam waktu 30 menit bak dapat dikosongkan tanpa sisa. Jika luas alas bak adalah 10.500
cm2, maka tinggi bak tersebut adalah ….
19. Hasil operasi terbesar yang dapat diperoleh dari penempatan angka–angka 4, 6, 7, dan 8 pada
kotak–kotak yang tersusun di bawah ini adalah ….
 + ( : )  
20. Pada suatu peta dengan skala 1 : 100.000, luas tanah sebuah sekolah adalah 50 cm2. Luas tanah
sekolah tersebut pada peta dengan skala 1 : 200.000 adalah …
KUMPULAN SOAL - SOAL OSN
OLIMPIADE MATEMATIKA
OLIMPIADE SAINS NASIONAL (OSN) SMP 2003 - 2018
TINGKAT KABUPATEN/ KOTA,PROPINSI & NASIONAL

OLIMPIADE SAINS NASIONAL (OSN) SMP


TINGKAT PROVINSI
TAHUN 2003 - 2018
OLIMPIADE SAINS NASIONAL SMP
SELEKSI TINGKAT PROVINSI TAHUN 2018
BIDANG STUDI MATEMATIKA
WAKTU : 150 MENIT
21 April 2018

SOAL ISIAN SINGKAT

5k  1
1. Diketahui bilangan bulat positif k sehingga juga bilangan bulat positif. Dua nilai k yang
3k  18
memenuhi adalah ....

2. Suatu partikel bergerak pada bidang Cartesius dimulai dari titik (0,0). Setiap langkah
1
pergerakan adalah satu satuan. Peluang partikel bergerak pada arah sumbu-X positif adalah ,
2
2
sedangkan peluang bergerak pada arah sumbu-Y positif adalah . Setelah bergerak 10 langkah,
5
peluang partikel tersebut sampai pada titik (6,4) dengan melalui titik (3,4) adalah ....

3. Diberikan himpunan A = {1,2,3, ..., 25}. Banyak himpunan bagian berunsur dua yang hasil kali
unsur-unsurnya kuadrat sempurna adalah ....

4. Diketahui bilangan x dan y, masing-masing tidak lebih dari 2018 dan x2 + y2 habis dibagi 121.
Jika pasangan (x,y) dan (y,x) tidak dibedakan, maka banyak pasangan (x,y) yang memenuhi
adalah ....

5. Suatu tabung berada di dalam prisma tegak segitiga. Tabung tersebut tepat menyinggung
prisma pada alas, tutup, dan semua sisi prisma. Alas prisma berbentuk segitiga sama sisi dengan
panjang sisi 8 cm dan tinggi prisma 6 cm . Volume tabung tersebut adalah ....
6. Diketahui ABC mempunyai panjang sisi AB = AC = 3 cm dan BC = 2 cm. Titik D dan E
terletak pada AC sehingga BD adalah garis tinggi dan BE adalah garis berat ABC. Luas BDE
adalah ... cm2.

7. Sebuah kode terdiri dari 6 digit angka akan disusun dengan ketentuan sebagai berikut:
a) Angka pertama adalah tak nol
b) Nilai angka pertama adalah dua kali angka terakhir
c) Jika angka ke-2 dan ke-3 dipertukarkan, tidak akan mengubah nilai bilangan.
Banyaknya susunan angka kode yang mungkin adalah ....

8. Misalkan k adalah garis yang menyinggung kurva y = x2 – 1 di titik (x1,y1), dengan x1 > 1. Jika k
melalui titik (1,–1), maka k memotong sumbu-y di titik ....

9. Misalkan suku-suku suatu barisan diberikan dengan x1 = 1, xn+1 = xn + n, untuk n > 1. Nilai n
terbesar sehingga x1 + x2 + x3 + ... + xn ≤ 2018 adalah ....

10. Nilai x dan y yang memenuhi sistem


2 2 4
 3 x  3 y   63

 y  1 x  13
 2 42
adalah ....

11. Bilangan bulat dari 1, 2, 3, ..., 1000 ditulis berurutan pada keliling lingkaran. Seseorang
menendai bilangan 1, bilangan 13, bilangan 25 dan setiap bilangan ke-12 setelahnya (berarti
bilangan yang ditandai adalah 1, 13, 25, 37, ...). Proses ini berlangsung terus sampai dengan
bertemu bilangan yang pernah ditandai. Bilangan bulat pada keliling lingkaran tersebut yang
tidak ditandai ada sebanyak ....

12. Diberikan suatu segitiga samakaki ABC dengan AB = AC = 10 cm. Titik D terletak pada sisi AB
sejauh 6 cm dari A, serta titik E pada sisi AC sejauh 4 cm dari A. Selanjutnya dari A ditarik garis
a
tinggi dan memotong BC di F. Jika bilangan rasional menyatakan perbandingan luas
b
segiempat ADFE terhadap luas segitiga ABC dalam bentuk yang paling sederhana, maka nilai
a  b adalah ....
13. Diketahui ABC siku-siku di C. D titik tengah AC dan AC = BD = 2 10 . P pada BD sehingga
CP  BD . Luas CDP adalah ....

14. Persegi panjang ABCD mempunyai panjang sisi AB = 4 cm dan BC = 8 cm . Titik F pada AD, G
pada BC, sehingga garis FG sejajar sisi CD, dan panjang AF = 2 cm. Titik E merupakan titik
tengah CD. Selanjutnya dilukis diagonal BD dan garis AE. Banyak segiempat pada persegi
panjang ABCD adalah ....

15. Didefinisikan ⟦x⟧ = bilangan bulat yang lebih kecil atau sama dengan x, contoh ⟦2⟧ = 2; ⟦0,1⟧ =

0; dan ⟦1,8⟧ = 1. Jika J = ⟦ 1918 ⟧ + ⟦ 1919 ⟧ + ⟦ 1920 ⟧ + ... + ⟦ 2018 ⟧ maka nilai J
adalah ....

SOAL URAIAN

16. Tentukan semua penyelesaian dari sistem persamaan



 x  6 y  xy  x  3 y  0;
2 2

 2

 x  5 x  3 y  y  10  0.
2

17. Sebuah permainan dengan nama “Halang Rintang” mempunyai aturan permainan bahwa jika
seseorang berada pada rintangan ke-n, orang tersebut harus melemparkan dadu sebanyak n kali.
Jika jumlah mata dadu dari n pelemparan ini lebih besar dari 2n, maka orang tersebut berhasil
melewati rintangan. Tentukan peluang bahwa seseorang berhasil melewati tiga rintangan
pertama. Diasumsikan bahwa dadu yang digunakan adalah dadu yang setimbang.

18. Seseorang mengamati Pelat Nomor Kendaraan Bermotor (PNKB) yang terdiri atas empat
angka. Dengan angka pertama tak nol. Orang tersebut mendefinisikan PNKB istimewa jika
memenuhi dua syarat, yaitu:
 PNKB tersebut memuat tiga atau empat suku barisan aritmetika
 beda atau selisih barisan tersebut merupakan bilangan bulat positif.
Tentukan banyak PNKB istimewa dimaksud.
SOAL OLIMPIADE SAINS NASIONAL TINGKAT PROVINSI (OSP)
TAHUN 2017
BIDANG MATEMATIKA SMP

BAGIAN A : SOAL ISIAN SINGKAT

1. Diketahui x dan y adalah dua bilangan bulat positif. Banyak (x,y) sehingga kelipatan
persekutuan terkecil dari x dan y sama dengan 233557 adalah ... .
2. Jika A = {a, b, c} dengan a, b, dan c merupakan bilangan asli lebih besar daripada 1, serta
a  b  c  180 , maka banyak himpunan A yang mungkin adalah ... .
1
3  16 3 4 3 1 
3. Bentuk sederhana dari ekspresi 5  3    adalah ... .
 25 25 25 
1
4. Diketahui n1 = 1 dan n k 1  untuk k  {1,2,3,...,2016 } .
1
1
nk
Nilai dari n1n2  n2n3  n3n 4  ...  n2016n2017 adalah ... .
5. Diberikan persegi dengan setengah lingkaran L1, yang berpusat pada titik tengah alasnya.
Lingkaran L2, dengan radius r menyinggung sisi atas dan sisi tegak persegi, serta L1.
Sedangkan lingkaran L3 dengan radius s menyinggung L1, L2, dan sisi tegak persegi. Rasio
dari r : s adalah ... .

6. Dua lingkaran L1 dan L2 mempunyai radius berturut-turut 12 cm dan 5 cm. Titik P1 pada
L1 dan titik P2 pada L2. Mula-mula L1 dan L2 bersinggungan luar di P1 dan P2. Kemudian
L2 digelindingkan sepanjang L1, sehingga tetap bersinggungan luar. Titik P2 pertama kali
bertemu kembali dengan P1 ketika L2 telah digelindingkan sebanyak ...kali.
7. Bilangan 3 angka yang habis dibagi 3 dengan semua angka penyusunnya merupakan
anggota dari S = {2,3,5,6,7,9} ada sebanyak ... .
8. Sekolah A memiliki 3 kelas yang akan mengikuti ujian komputer pada sekolah B. Sekolah
B menyediakan 2 pilihan waktu setiap harinya selama 5 hari berturut-turut. Setiap waktu
yang disediakan dibuka dua kelas paralel. Jika setiap kelas sekolah A hanya mengikuti satu
kali ujian, dan waktu ujian ditentukan secara acak, maka peluang bahwa tiga kelas
tersebut mengikuti ujian pada hari yang berbeda adalah ... .

BAGIAN B : SOAL URAIAN

1. Diketahui m adalah suatu bilangan bulat lima angka. Angka di tengah dari penyusun m
m
dihapus sehingga diperoleh n yang merupakan bilangan empat angka. Jika adalah suatu
n
bilangan bulat, tentukan semua nilai m yang memenuhi.
2. Diketahui fungsi kuadrat f ( x )  ax 2  bx  c dengan a  0 dan f (0)  4 . Tentukan
semua kemungkinan nilai a, b, dan c agar 0  f ( x )  4 untuk 0  x  3 .
3. Pada segitiga ABC, berturut-turut melalui titik A, B, dan C dibuat garis lurus yang
memotong sisi di hadapannya. Ketiga garis tersebut berpotongan di titik X sehingga
diperoleh enam segitiga seperti pada gambar. Jika masing-masing luas segitiga yang
diarsir adalah satu, buktikan bahwa masing-masing luas segitiga yang tidak diarsir juga
satu.

4. Misalkan n menyatakan banyak perubahan posisi berurutan dari laki-laki ke perempuan


atau sebaliknya dalam suatu antrian. Urutan sesama laki-laki atau sesama perempuan
tidak dibedakan. Contohnya, dalam antrian yang terdiri dari 4 laki-laki (L) dan 6
perempuan (P) dengan susunan antrian LPPLLPPLPP, diperoleh n = 5 karena ada lima
posisi laki-laki dan perempuan. Tentukan rata-rata nilai n dari semua kemungkinan
urutan antrian yang terdiri dari 3 laki-laki dan 5 perempuan.
OLIMPIADE SAINS NASIONAL SMP
SELEKSI TINGKAT PROPINSI
TAHUN 2016

BAGIAN A: ISIAN SINGKAT


1. Misalkan adalah bilangan asli ganjil berurutan yang jumlahnya
merupakan bilangan kuadrat. Nilai x2016 terkecil yang mungkin adalah ...
2. Jika ab + ab + ab = cbb dan setiap huruf yang berbeda menyatakan angka yang berbeda juga,
maka nilai a, b, dan c adalah …
3. Pada gambar berikut diketahui serta .

Jika diketahui panjang , maka panjang adalah ... cm.


4. Pada gambar berikut terdapat lima persegi sepusat (semua diagonal persegi berpotongan di satu
titik): , , , dan . Titik-titik sudut terletak pada sisi-sisi dan membaginya dengan
perbandingan 1 : 4. Dengan cara yang serupa titik-titik sudut terletak pada sisi-sisi untuk
{ }. Perbandingan luas dan adalah …

5. Banyak cara mendapatkan empat bilangan asli ganjil (dengan urutan tidak diperhatikan) yang
berjumlah 22 adalah …
6. Garis dengan memotong parabola di titik A dan B. Jika
C adalah titik puncak parabola tersebut sehingga luas segitiga ABC sama dengan 6 satuan luas,
maka nilai adalah …
7. Diberikan persamaan ( ) ( )( ) . Jika dan adalah
bilangan asli, maka jumlah dari semua nilai yang mungkin adalah …
8. Pada gambar berikut, segitiga sama sisi terletak di dalam sebuah persegi. Perbandingan luas
segitiga dan persegi adalah ...

9. Dito mencatat bahwa semester ini dia telah mengikuti delapan ulangan harian pelajaran
Matematika. Nilai ulangan diberikan pada skala 100. Catatan Dito menunjukkan bahwa rata-rata
nilai setelah ulangan ke-7 naik 2 poin dibandingkan rata-rata nilai sampai ulangan ke-6.
Sedangkan rata-rata nilai sampai ulangan ke-8 juga naik 2 poin dibanding rata-rata nilai sampai
ulangan ke-7. Selisih nilai ulangan ke-8 dan ke-7 adalah ... poin
10. Diketahui banyak suku suatu barisan aritmatika adalah genap. Jumlah suku-suku dengan nomor
ganjil adalah 32 dan jumlah suku-suku dengan nomor genap adalah 50. Jika selisih suku terakhir
dan suku pertamanya adalah 34, maka banyak suku pada barisan tersebut adalah …

BAGIAN B: URAIAN
1. Diberikan Kubus ABCD.EFGH dengan panjang rusuk 2 cm. Titik P terletak pada perpanjangan
HE sehingga PE = 1 cm. Tentukan jarak titik P ke bidang yang memuat segitiga AHF.

2. Empat orang siswa makan siang di suatu kantin. Di kantin tersebut masih tersedia 3 porsi nasi
goreng, 20 porsi nasi pecel, dan 25 porsi nasi rawon, 19 gelas jus alpukat, 17 gelas jeruk panas,
dan 15 gelas jus sirsak. Mereka ingin memesan 4 porsi makanan dan 3 gelas minuman. Tentukan
banyak pilihan komposisi makanan dan minuman yang mungkin mereka pesan.
3. Fungsi f didefinisikan pada bilangan bulat yang memenuhi ( ) dan ( ) ( )
( ) ( ) ( ) untuk semua n >1. Hitunglah nilai ( )
OLIMPIADE SAINS NASIONAL SMP
SELEKSI TINGKAT PROPINSI
TAHUN 2015

BAGIAN A: ISIAN SINGKAT


1. Banyak faktor persekutuan dari 145152 dan 544320 yang merupakan bilangan genap positip
adalah …
2. Pak Tani memiliki 500 ekor ayam yang terdiri dari ayam pedaging dan ayam petelur.
Sebagian ayam berwarna merah dan sebagian lagi berwarna putih. Banyak ayam petelur dan
berwarna merah adalah 100 ekor. Jika diambil satu ekor ayam secara acak, maka peluang
untuk mendapatkan ayam pedaging adalah sama dengan peluang untuk mendapatkan ayam
berwarna putih, yaitu sebesar 3/5. Banyak ayam pedaging yang berwarna merah adalah …
3. Diketahui adalah segiempat talibusur pada lingkaran yang memiliki jari-jari luar .
Diketahui diameter lingkaran, panjang , dan panjang . Keliling
adalah
4. Rani dan Susi masing-masing memilih empat angka berbeda yang merupakan anggota dari
{ } untuk menyusun dua buah bilangan dua angka. Jika mereka masing - masing
menjumlahkan kedua bilangan yang disusun, maka hasilnya adalah bilangan tiga angka.
Notasikan jumlah bilangan yang diperoleh Rani dan Susi berturut-turut adalah dan .
Diketahui bahwa bersisa 2 jika dibagi 47. Jika s memiliki nilai terbesar yang mungkin,
maka r + s = …
5. Diketahui dan adalah dua bilangan bulat. Banyak anggota himpunan penyelesaian dari
persamaan √ √ √ adalah …
6. Diketahui barisan himpunan beranggotakan beberapa bilangan asli berurutan sedemikian rupa
sehingga banyak anggota himpunan - himpunan tersebut membentuk barisan aritmatika.
Empat suku pertama barisan himpunan tersebut adalah
{ }{ }{ }{ } Bilangan 2015 berada pada suku/
himpunan ke …
7. Diketahui siku-siku di , serta lingkaran yang berpusat di menyinggung sisi dan
berturut-turut di dan . Selanjutnya dan adalah diameter lingkaran. Jika adalah
jari-jari lingkaran, maka luas daerah yang diarsir adalah … satuan luas.
8. Delegasi perwakilan pelajar Kota Bahagia ke suatu pertemuan pelajar nasional terdiri dari 5
orang. Ada 10 siswa laki-laki dan 10 siswa perempuan yang mencalonkan diri untuk menjadi
anggota delegasi. Jika disyaratkan bahwa paling sedikit seorang delegasi harus laki-laki, maka
banyak cara untuk memilih delegasi tersebut adalah …
9. Jika salah satu akar dari persamaan kuadrat ( ) adalah bilangan
prima, maka nilai terbesar yang mungkin adalah …
10. Jika kurva parabola dicerminkan terhadap garis , kemudian digeser
kearah sumbu-X positif sejauh 2 satuan, maka diperoleh kurva dengan persamaan …

BAGIAN B: URAIAN
1. Diberikan himpunan { } Berapakah banyak himpunan bagian dari
yang memiliki anggota sehingga jumlah semua anggota tersebut habis dibagi ?
2. Pada gambar berikut, bangun adalah persegi, bangun persegi panjang dan luas
dua bangun ini sama yaitu . Garis dan garis berpotongan di titik dan
perbandingan panjang Diketahui perbandingan panjang .
Jika titik potong diagonal persegi dan titik potong diagonal persegi panjang
, berapakah panjang ?

3. Pada sebuah permainan disediakan sejumlah kartu bernomor semua bilangan prima berbeda
yang bernilai kurang dari 100 dalam suatu wadah tertutup. Permainan dilakukan dengan
mengambil 2 kartu secara acak dan memeriksa bilangan yang tertera pada kartu, apakah
jumlahnya merupakan bilangan prima atau bukan. Jika jumlahnya bukan bilangan prima, ia
diberi kesempatan mencoba kembali sampai total 3 kali pengambilan. Seorang pemain akan
memenangkan permainan, jika ia berhasil mendapatkan jumlah prima pada maksimal
pengambilan ke tiga. Berapa peluang seorang pemain memenagkan permainan tersebut?
OLIMPIADE SAINS NASIONAL SMP
SELEKSI TINGKAT PROPINSI
TAHUN 2014

BAGIAN A: ISIAN SINGKAT


1. Diketahui dan adalah bilangan bulat positif. Salah satu solusi dari
adalah ( ) ( ). Salah satu solusi yang lain adalah …
2. Jika dan merupakan bilangan real yang memenuhi , maka nilai terbesar dari
perkalian dan adalah
3. Sebuah lingkaran berada dalam seperempat lingkaran besar, seperti pada gambar di bawah. Jika
jari-jari lingkaran besar = 8 satuan, maka luas daerah yang diarsir adalah …

4. Jumlah bilangan bulat positif berbeda adalah . Dimana Tidak ada satupun dari
bilangan-bilangan tersebut yang lebih besar dari . Minimal banyaknya bilangan ganjil
pada deret bilangan tersebut adalah …
5. Terdapat bilangan ribuan dengan jumlah angka-angkanya 8. Contoh bilangan ini adalah .
Bilangan yang memenuhi sifat ini ada sebanyak …
6. Misalkan adalah suatu daerah trapezium sedemikian sehingga perpanjangan sisi dan
perpanjangan sisi berpotongan di titik . Diketahui panjang , dan tinggi
trapesium tersebut adalah . Jika dan masing-masing adalah titik tengah dan , maka
luas segitiga adalah …
7. Diberikan dua persamaan berikut.

dan

Nilai yang memenuhi kedua persamaan tersebut adalah …

8. Jika dan bilangan bulat ganjil serta maka banyak bilangan bulat diantara dan
adalah …
9. Fungsi dari himpunan ke himpunan Y dikatakan satu-satu, jika untuk setiap
dengan ( ) ( ) berlaku . Jika { } dan { }, maka
fungsi berbeda dari ke yang merupakan fungsi satu-satu dan setiap bilangan anggota
tidak dikaitkan dengan faktornya di ada sebanyak …
10. Indah dan Nian bermain lempar dadu secara bergantian dimulai dengan lemparan pertama
giliran Indah. Seseorang akan memenangkan permainan jika ia mendapatkan mata dadu tetapi
lawannya tidak mendapatkan mata dadu atau pada lemparan sebelumya. Peluang Indah
pada giliran yang ketiga melempar (lemparan kelima) akan menang adalah …

BAGIAN B: URAIAN
1. Temukan semua bilangan real yang memenuhi persamaan √
2. Dikethui jumlah buah bilangan bulat positif ganjil berurutan adalah . Tentukan
terkecil yang mungkin.
3. Diberikan kerangka limas dengan alasnya adalah daerah segitiga siku-siku .
Diketahui sisi siku-sikunya adalah dan dengan panjang √ dan panjang
, rusuk tegak lurus dengan bidang , dan panjang . Jika pada rusuk
terdapat titik sehingga sebuah bola dengan sebagai diameternya menyinggung bidang
alas , hitung jari-jari bola tersebut.
4. Sebuah kode rahasia terdiri dari dua huruf dan satu bilangan antara dan . Aturan yang
harus dipenuhi adalah sebagai berikut.
(i) Semua angka dan huruf harus saling berbeda,
(ii) Jika tiga angka membentuk bilangan genap maka kedua huruf yang dipilih adalah huruf
vocal
(iii) Jika tiga angka membentuk bilangan ganjil maka kedua huruf yang dipilih adalah huruf
konsonan
Tentukan banyak kode rahasia yang mungkin dibuat.
5. Untuk bilangan real, dirumuskan suatu fungsi

( )

Maka hitunglah hasil penjumlahan berikut.

( ) ( ) ( )
OLIMPIADE SAINS NASIONAL SMP
SELEKSI TINGKAT PROPINSI
TAHUN 2013

BAGIAN A : ISIAN SINGKAT


1. Diketahui segitiga sama sisi dengan panjang sisi 10 cm. Jika dibuat lingkaran yang berpusat di
titik tengah salah satu sisi segitiga dengan jari-jari 5 cm, maka luas daerah di dalam lingkaran
dan di luar segitiga adalah … cm2
2. Rata-rata nilai dari 25 siswa adalah 40. Jika selisih rata-rata nilai 5 siswa terendah dan 20 siswa
sisanya adalah 25, maka nilai rata-rata 5 siswa terendah adalah …
3. Dalam sebuah kotak terdapat beberapa bola dengan empat macam warna yakni: biru, merah,
kuning, dan putih. Paling sedikit terdapat 10 bola untuk masing-masing warna. Bola diambil satu
demi satu dari dalam kotak tersebut secara acak tanpa pengembalian. Banyak pengambilan yang
harus dilakukan untuk memastikan mendapatkan 6 bola dengan warna sama adalah …

4. Jika , maka nilai

5. Himpunan penyelesaian pertidaksamaan di bawah ini adalah …

6. Jika nilai
, maka nilai adalah …
7. Sebuah drum berbentuk tabung yang berjari-jari 70 cm dan berisi air setinggi 40 cm (gunakan
). Seorang tukang pasang ubin memasukkan 110 buah ubin keramik ke dalam drum
sehingga tinggi permukaan air bertambah 8 cm. Jika permukaan setiap ubin keramik berukuran
40 cm 40 cm, berapakah tebal ubin keramik tersebut?
8. Diketahui bilangan bulat positif. Jika ditambah angka-angka pembentuknya menghasilkan
313, maka semua nilai yang mungkin adalah …
9. Diketahui dua buah himpunan dan dengan
{( )| } dan
{( )| }. Banyaknya anggota himpunan
adalah …
10. Tim sepakbola terdiri atas 25 orang, masing-masing diberi kaos bernomor 1 sampai dengan 25.
Banyak cara memilih tiga pemain secara acak dengan syarat jumlah nomor kaos mereka habis
dibagi tiga adalah …
BAGIAN B : URAIAN
1. Suatu yayasan menyumbangkan 144 buku ke 4 sekolah. Banyak buku yang diterima untuk setiap
sekolah tidak sama. Selisih buku yang diterima sekolah A dan B adalah 16. Selisih buku yang
diterima sekolah B dan C adalah 12. Selisih buku yang diterima sekolah C dan D adalah 8.
Sekolah A menerima buku paling sedikit dibandingkan dengan yang diterima sekolah lain. Jika
sekolah D menerima buku 2 kali lebih banyak daripada buku yang diterima sekolah A, tentukan
banyak buku yang diterima masing-masing sekolah.
2. Satu set kartu remi/bridge terdiri dari 52 lembar. Diambil 5 lembar kartu secara acak. Tentukan
peluang terambil 2 kartu warna merah dan 3 kartu warna hitam, yang diantaranya terdapat tepat 1
kartu King.
3. Misalkan 10 lingkaran yang berjari-jari 1 cm dimasukkan dalam lingkaran berjari-jari cm
seperti pada gambar berikut. Tentukan .

4. Gunakan delapan bilangan prima yang berbeda dan kurang dari 25 untuk melengkapi persegi
ajaib di bawah, sehingga setiap kotak di dalam persegi terisi oleh satu bilangan prima serta
jumlah bilangan pada setiap baris dan setiap kolom selalu sama.

5. Didefinisikan ⟦ ⟧ adalah bilangan bulat terbesar yang kurang dari atau sama dengan . Sebagai

contoh: ⟦ ⟧ karena . Jika dan adalah bilangan real dengan ⟦√ ⟧ dan

⟦ √ ⟧ , tentukan nilai dari ⟦√⟦√ ⟧⟧.


OLIMPIADE SAINS NASIONAL SMP
SELEKSI TINGKAT PROPINSI
TAHUN 2012

BAGIAN A: ISIAN SINGKAT


1. Sebuah silinder memiliki tinggi 5 cm dan volume 20 cm3. Luas permukaan bola terbesar yang
mungkin diletakkan ke dalam silinder tersebut adalah …
2. Jumlah tiga buah bilangan adalah 19. Jika bilangan pertama dan bilangan kedua masing-masing
dikurangi 1, maka diperoleh dua bilangan dengan rasio 1 : 3. Jika bilangan kedua dan ketiga
masing-masing ditambah 3, maka diperoleh dua bilangan dengan rasio 5 : 6. Selisih bilangan
terbesar dan terkecil adalah…
3. Jika , maka …

4. Lima belas bilangan prima pertama dituliskan berturut-turut pada lima belas kartu. Jika semua
kartu tersebut diletakkan dalam sebuah kotak dan kemudian diambil secara acak dua buah kartu
berturut-turut tanpa pengembalian, maka peluang terambil dua kartu dengan jumlah dua bilangan
yang tertulis merupakan bilangan prima adalah …
5. Perhatikan gambar bangun datar setengah lingkaran dengan diameter AD dan pusat lingkaran M
berikut. Misalkan B dan C adalah titik-titik pada lingkaran sedemikian sehingga AC BM dan
BD memotong AC di P. Jika besar ∠CAD = so, maka besar sudut ∠CPD =…o.

6. Lima angka, yakni 1, 2, 3, 4, dan 5, dapat disusun semuanya tanpa pengulangan menjadi 120
bilangan berbeda. Jika bilangan-bilangan tersebut diurutkan dari yang terkecil ke yang terbesar,
maka bilangan yang menempati urutan ke-75 adalah …
7. Diketahui 1 + k habis dibagi 3, 1+ 2k habis dibagi 5, dan 1 + 8k habis dibagi 7. Jika k adalah
bilangan bulat positif, maka nilai terkecil untuk k adalah …
8. Jika = 20102 + 20112 dan = 20122 + 20132, maka nilai sederhana dari √ ( )
adalah …
9. Jika dan adalah penyelesaian dari persamaan kuadrat , maka nilai dari

adalah …
10. Pada gambar berikut, kedua ruas garis putus-putus yang sejajar membagi persegi menjadi tiga
daerah yang luasnya sama. Jika jarak kedua ruas garis putus-putus tersebut adalah 1 cm, maka
luas persegi adalah … cm2.

BAGIAN B : URAIAN
1. Tentukan semua bilangan real x yang memenuhi persamaan berikut :

2. Pada gambar berikut, sembilan lingkaran kecil dalam lambang olimpiade akan diisi masing-
masing dengan bilangan 1, 2, 3, 4, 5, 6, , 8, atau 9. Tentukan pengisian tersebut sehingga jumlah
bilangan di dalam setiap lingkaran besar adalah 14.

3. Diketahui ∆ABC dengan AB = 25 cm, BC = 20 cm, dan AC = 15 cm. Jika titik D terletak pada
sisi AB sedemikian sehingga perbandingan luas ∆ADC dan ∆ABC adalah 14 : 25, tentukan
panjang CD.
4. Dari hasil sensus diketahui bahwa penduduk suatu kota tak lebih dari 10.000 orang dan anak-
anak 20% lebih banyak daripada penduduk dewasa. Jika anak laki-laki 10% lebih banyak
daripada anak perempuan, serta di antara penduduk dewasa terdapat 15% lebih banyak
perempuan, tentukan jumlah terbesar yang mungkin dari penduduk kota tersebut.
5. Diketahui sebuah bilangan rasional positif kurang dari 1 yang dinyatakan dalam pecahan biasa
dalam bentuk paling sederhana. Jika hasil kali pembilang dan penyebut dari bilangan rasional
tersebut adalah 20! = 1 x 2 x 3 x ⋅⋅⋅ x 20, tentukan semua bilangan yang dimaksud.
OLIMPIADE SAINS NASIONAL SMP
SELEKSI TINGKAT PROPINSI
TAHUN 2011

BAGIAN A : ISIAN SINGKAT


1. Jika adalah jumlah 99 bilangan ganjil terkecil yang lebih besar dari 2011 dan adalah jumlah
99 bilangan genap terkecil yang lebih besar dari 6, maka =…
2. Jika adalah fungsi sehingga ( ) ( – ) dan ( ) , maka ( )– ( )
3. Jika bilangan bulat dan dibagi 4, maka bersisa 3. Jika bilangan – dibagi 4, maka bersisa

4. Perhatikan gambar berikut. Suatu lingkaran berjari-jari 2 satuan berpusat di A. suatu persegi
memiliki titik sudut di A dan satu titik sudut yang lain di lingkaran. Di dalam persegi tersebut
terdapat lingkaran yang menyinggung keempat sisi persegi. Di dalam lingkaran terdapat persegi
yang keempat titik sudutnya berada di lingkaran tersebut. Di dalam persegi ini terdapat lingkaran
yang menyinggung keempat sisi persegi. Luas daerah yang diarsir sama dengan …

5. Banyaknya 3 digit (angka) yang terdiri dari angka-angka 0, 2, 3, 5, 7, 8 yang lebih dari 243 dan
kurang dari 780 adalah …
6. Diketahui Budi adalah seorang siswa laki-laki dan Wati adalah seorang siswa perempuan. Saat ini
mereka duduk di kelas IX pada suatu sekolah. Mereka mencatat banyaknya siswa kelas IX di
sekolah mereka. Wati mencatat 3/20 dari total siswa di kelas IX adalah laki-laki. Sedangkan
menurut Budi, 1/7 dari total siswa kelas IX selain dirinya adalah laki-laki. Banyak siswa laki-laki
dikelas IX disekolah mereka adalah …
7. Diketahui luas persegi ABCD adalah 25 m2. Jika E, F, dan G masing-masing adalah titik tengah
AB, AD, dan CD seperti pada gambar berikut, maka luas trapesium BHFE adalah … m2.
8. Tiga bilangan a, b, dan c dipilih sehingga ketika setiap bilangan ditambahkan ke rata-rata dua
bilangan lainnya, maka berturut-turut hasilnya adalah 80, 90, dan 100. Rata-rata dari a, b, dan c
adalah …
9. Sebuah bilangan bulat x diambil secara acak dari { }.
Peluang bahwa x adalah penyelesaian pertidaksamaan √ adalah …

10. Misalkan n adalah suatu bilangan asli dan x adalah bilangan riil positif. Jika ,

maka nilai sama dengan …

BAGIAN B : URAIAN
1. Saat ini umur agus dan umur fauzan kurang dari 100 tahun. Jika umur agus dan umur fauzan
ditulis secara berurutan, maka diperoleh suatu bilangan empat digit (angka) yang merupakan
kuadrat sempurna. Dua puluh tiga tahun kemudian, jika umur mereka ditulis dengan cara yang
sama, maka diperoleh bilangan empat digit lain yang juga merupakan kuadrat sempurna. Jika
umur mereka diasumsikan merupakan bilangan bulat positif, berapakah umur mereka saat ini!
2. Pada sebuah segiempat ABCD, sudut ABC dan sudut DAC adalah sudut siku-siku. Jika keliling
segiempat ABCD adalah 64 cm, keliling ABC adalah 24 cm dan keliling ACD adalah 60 cm.
berapakah luas segiempat ABCD?
3. Diketahui bilangan bulat positif n memiliki sifat-sifat berikut. 2 membagi n, 3 membagi n+1, 4
membagi n+2, 5 membagi n+3, 6 membagi n+4, 7 membagi n+5, dan 8 membagi n+6. Bilangan
bulat positif pertama yang memiliki sifat-sifat ini adalah 2. Tentukan bilangan bulat positif ke-5
yang memenuhi sifat-sifat diatas!
4. Tiga garis lurus l1, l2, dan l3 mempunyai gradien berturut-turut 3, 4, dan 5. Ketiga garis tersebut
memotong sumbu-Y dititik yang sama. Jika jumlah absis titik potong msing-masing garis dengan
sumbu-X adalah , tentukan persamaan garis l1.

5. Data akhir suatu kompetisi yang diikuti oleh tiga sepak bola, masing-masing tim saling
berhadapan, dituliskan pada berikut.
Tim Menang Kalah Seri Gol (Memasukkan-Kemasukan)
Elang 1 0 1 5 2
Garuda 1 0 1 4 3
Merpati 0 2 0 3 7

Berapakah skor pertandingan antara Tim Garuda melawan Tim Merpati?


OLIMPIADE SAINS NASIONAL SMP
SELEKSI TINGKAT PROPINSI
TAHUN 2010

BAGIAN A : ISIAN SINGKAT


1. Jika f (x) = 3x2 + 18x + 28 dan 12 + 22 + 32 + 42 + … + (2009)2 + (2010)2 = A, maka f (0) + f (1) + f
(2) + f (3) + … + f (2010) = …
2. Jika dan , maka nilai dari p2 + pq + q2 adalah …
√ √ √ √

3. Diberikan suatu barisan bilangan 1, 5, 6, 25, 26, 30, 31 , ... yang terdiri dari barisan bilangan
pemangkatan 5 atau jumlah bilangan-bilangan berbeda hasil pemangkatan 5. Perhatikan bahwa 1 =
5°, 6 = 1 + 5, 31 = 1 + 5 + 52, ... Nilai suku ke-100 pada barisan tersebut adalah …
4. Bilangan asli terkecil yang tidak sama dengan satu yang selalu dapat membagi habis bilangan yang
terdiri dari 6 angka abcabc adalah ...
5. Perhatikan gambar berikut Jika setiap persegi kecil memiliki luas 1 satuan/ luas daerah tertutup
yang dibatasi oleh busur-busur lingkaran di bawah adalah ...

6. Perhatikan gambar jajaran genjang di bawah ini, Jika sudut BPC dan BQD siku-siku, dan BP = 4
cm, DP = 4 cm dan DC = 7 cm, tentukan panjang BQ

7. Tentukan banyaknya cara membagikan 10 permen identik kepada tiga orang sedemikian sehingga
setiap orang sedikitnya mendapatkan satu permen.

8. Gambar berikut memberikan beberapa altematif jalan dari A ke B, Sisi-sisi masing-masing blok
(persegi) menyatakan jalan dengan panjang satu satuan yang sama. Tentukan banyaknya rute
terpendek dari titik A ke titik B yang melalui titik-titik 2, 0, 1, 0 secara berurutan.
9. Dipunyai persegi ABCD dengan luas x2 , titik P terletak dalam pcrsegi seperti tampak pada
gambar dengan jarak PA = PB = PM. Jika jarak tersebut dinyatakan dengan y, nyatakan y dalam
x.

10. Jika 3996 = psqt ru ,dengan p, q, r adalah bilangan prima, maka nilai p + q + r + s + t + u adalah
...

BAGIAN B : URAIAN
1. Tentukan bilangan asli terbesar yang jika membagi bilangan-bilangan 1723, 2010, dan 5741
selalu memberikan sisa 1 adalah ...

2. Akar-akar persamaan kuadrat – adalah real dan lebih besar 1, Berapakah nilai
?

3. Dipunyai panjang jari-jari lingkaran A = 8 cm dan jari-jari lingkaran B = 2 cm. Tentukan panjang
jari-jari lingkaran C.

4. Sebuah tabel permainan angka berukuran 4 4, setiap sel akan diisi dengan bilangan 1 atau –1
sedemikian sehingga jumlah setiap baris dan kolom adalah 0. Ada berapa banyak cara untuk
menyusun tabel permainan yang dimaksud ?
5. Sejumlah siswa mengikuti ujian seleksi OSN tahun 2010 tingkat provinsi, ternyata didapatkan
data bahwa sebanyak 64 siswa yang lulus adalah wanita, peserta yang lulus adalah laki-laki,

Sedangkan jumlah peserta laki-laki lulus adalah 4 kali lebih banyak dari pada jumlah peserta laki-
laki yang tidak lulus, serta jumlah peserta yang tidak lulus adalah 40 siswa. Berapa persenkah
jumlah peserta wanita yang mengikuti ujian seleksi tersebut ?
OLIMPIADE SAINS NASIONAL SMP
SELEKSI TINGKAT PROPINSI
TAHUN 2009

BAGIAN A : ISIAN SINGKAT


1. Banyak bilangan bulat berbeda yang merupakan penjumlahan dari tiga bilangan berbeda dalam
{5, 9, 1, 17, …., 41} adalah …
2. Nilai dari adalah ...
3. Dalam sebuah kantong terdapat 5 bola warna putih, 2 bola warna hijau, dan 3 bola warna merah.
Akan diambil 3 bola secara satu persatu dengan pengembalian artinya bila bola sudah diambil
dikembalikan ke dalam kantong tersebut. Peluang ketiga bola yang terambil berwarna hijau
adalah …
4. Bentuk sederhana dari :

√( )

5. Bilangan palindrom adalah bilangan yang dibaca dari kiri dan kanan selalu sama, seperti 131.
Banyak bilangan ganjil positip yang bersifat palindrom dan terdiri dari sembilan angka serta dua
kali bilangan tersebut juga merupakan bilangan palindrom adalah …
6. Tes matematika diberikan kepada tiga kelas dengan siswa yang berjumlah 100 orang. Nilai rata -
rata kelas pertama, kedua, dan ketiga masing-msing adalah 7, 8, dan . Jika banyak siswa pada

kelas pertama 25 orang, dan banyak siswa pada kelas ketiga 5 orang lebih banyak dari jumlah
siswa kelas kedua, maka nilai rata-rata seluruh siswa tersebut adalah …
( ) ( )
7. Jika , maka nilai y sama dengan ...

8. Perhatikan Gambar 1.

Jika segitiga TT1Tx siku-siku sama kaki dan panjang TT1 = 8 cm, maka TT1 + T1T2 + T2T3 + T3T4
+ T4T5 + … adalah …
9. Diketahui dua persamaan :

Nilai yang memenuhi kedua persamaan tersebut adalah …


10. Misalkan A dan B adalah titik pada bidang datar yang jaraknya adalah 2. Jika S merupakan
himpunan dari titik-titik P sehingga nilai (PA)2 + (PB)2 paling besar adalah 10, maka luas daerah
dari S adalah …
11. Suatu himpunan A beranggotakan sebelas bilangan bulat positip yang berbeda. Jika rata-rata dari
kesebelas bilangan tersebut adalah 12, maka bilangan bulat positip terbesar dalam A yang
mungkin adalah ….
12. Semua bilangan real x yang memenuhi persamaan √ √ adalah ....
13. Perhatikan Gambar 2, yaitu 4 buah layang-layang kongruen yang memuat pada persegi dan
ternyata masih tersisa daerah persegi yang diarsir. Jika panjang p = 3 √ cm, dan q = 5 √ cm,
maka luas daerah yang diarsir adalah ….

14. Suatu bilangan dikatakan “berprisque” jika bilangan tersebut merupakan bilangan asli yang
didapat oleh suatu bilangan prima dan suatu bilangan kuadrat sempurna (contoh 3 adalah bilangan
berprisque, tetapi 5 bukan bilangan berprisque). Banyak bilangan berprisque yang kurang dari 100
adalah …
15. Banyak persegi pada Gambar 3 adalah …
16. Perhatikan Gambar 4(a) sebagai kubus sempurna dan Gambar 4(b) merupakan kubus yang sama
dengan Gambar 4(a) dengan salah satu titik sudut dipotong dengan potongan berbentuk limas.
Jika panjang rusuk kubus 6a cm dan panjang rusuk tegak limas 2 a cm. maka volume bangun
baru adalah …

17. Banyak cara untuk menyatakan bilangan 4725 sebagai perkalian dari dua bilangan ganjil yang
lebih besar dari 1 adalah …
18. Banyak bilangan bulat positip n yang memenuhi bilangan kuadrat sempurna n4 + n3 +1 adalah …
19. Perhatikan gambar 5.

Bangun ABCDEG adalah menunjukkan keadaan sebuah kamar. Keadaan yang sebenarnya AD =
DE, AB = 28 meter, dan EF = 18 meter serta luas kamar 624 m2 . Jika sebuah penyekat dibuat
dari E sampai C yang membagi luas kamar menjadi dua bagian yang sama luas, maka jarak dari C
ke G adalah ….
20. Diketahui bilangan bulat x1 = 34, x2 = 334, x3 = 3334,…. , dan xn= ⏟ . Banyaknya angka

3 pada bilangan 9(x670)3 adalah ….


BAGIAN B : URAIAN
1. Perhatikan Gambar 6!

Gambar 6 merupakan bangun ruang yang terdiri dari tabung berjari-jari R2 dan belahan bola padat
berjari-jari R1. Tinggi tabung 8 cm, jari-jari bola R1 = 10 cm dan jari-jari tabung R2 = 5 cm.
Hitunglah luas permukaan dari bangun ruang tersebut!
2. Carilah semua bilangan bulat positip n sehingga ̅̅̅̅̅ (merupakan

bilangan desimal berulang tak berhenti) dengan a, b, dan c adalah angka-angka yang berbeda dari
0 sampai dengan 9!
3. Diketahui suatu fungsi ( ) . Jika ( ) ( ) ( )
dan ( ) , maka hitunglah jumlah angka angka dari ( )
OLIMPIADE SAINS NASIONAL SMP
SELEKSI TINGKAT PROPINSI
TAHUN 2008

BAGIAN A : ISIAN SINGKAT


1. Jika ⏟ , maka 5 angka terakhir dari A adalah …

2. Seorang peternak memiliki 114 hewan peliharaan yang terdiri dari kuda, sapi, kambing, dan
bebek. Banyak hewan berkaki empat adalah 8 lebih sedikit dibandingkan dengan banyak hewan
berkaki dua. Sedangkan sapi miliknya adalah 3 lebih banyak disbanding kuda, tetapi 20 lebih
sedikit dibanding kambing. Disamping itu ayam miliknya adalah 13 lebih sedikit dibanding
bebek. Banyak sapi dan ayam milik peternak tersebut adalah …
3. Perhatikan gambar 1. Hasil penjumlahan sudut :
1  2  3  4  5  6  7  8  9  ...o

4. Sebelas orang anggota suatu tim sepak bola ditimbang secara berurutan. Setelah dua orang selesai
ditimbang, dihitung rata – ratanya. Begitu pula ketika orang ketiga ditimbang, rata – rata baru
dihitung kembali. Demikian pula seterusnya. Diketahui nilai rata – rata ini selalu meningkat satu
kilogram sampai semua pemain selesai ditimbang. Selisih berat badan antara pemain yang paling
berat dengan pemain yang ditimbang pada urutan ketiga adalah ... Kilogram.
5. Nilai x yang memenuhi persamaan 4x(32006 ) = 32009 – 32007 + 24 adalah …
3a  2b  4c 6c
6. Jika  1 , maka nilai adalah …
4a  b  c ab
o
7. Perhatikan ABC pada gambar 2. Diketahui  BAC = 135 , titik D terletak ditengah AC, dan

titik E ditengah AB. Jika panjang AC = 10 2 cm dan AB = 14 cm, maka luas daerah BCDE
adalah … cm2
8. Perhatikan Gambar 3. Diketahui ABC adalah sama sisi dengan panjang sisi 16 cm. Titik O
terletak di dalam ABC. Dari titik O dibuat ruas garis OP, OQ, dan OR yang tegak lurus
terhadap sisi – sisi ABC. Jumlah panjang ruas garis OP + OQ + OR adalah ... cm

9. Angka satuan dari 12008 + 32008 + 52008 + 72008 + 92008 + 112008 + 132008 adalah …
10. Nia berulang tahun dalam waktu dekat. Karena keterbatasan biaya, dia hanya mampu
mengundang 10 dari 15 orang temannya. Diantara teman – temannya, terdapat sahabat dekatnya,
yaitu Ade, Dea, Ani, dan Ina. Nia memutuskan bahwa Ade dan Dea harus diundang. Tetapi, Ani
dan Ina tidak mungkin diundang bersama – sama karena mereka sedang berselisih faham. Banyak
cara menentukan susunan nama – nama yang akan diundang pada acara ulang tahun tersebut
adalah …
11. Dua puluh ubin persegi yang kongruen akan disusun dalam 2 baris. Masing – masing baris berisi
10 ubin. Diantara ubin – ubin tersebut terdapat 9 ubin bergambar bunga. Banyak cara menyusun
ubin tersebut agar sesama ubin bergambar bunga tidak saling bersinggungan adalah …
(Catatan : dua ubin dikatakan bersinggungan jika ada salah satu sisi yang saling berimpit)
1 1 1 1
12. Diketahui : A =    ...  .
1 2 2 3 32 9999  100
Bilangan kuadrat terdekat dengan A adalah …
13. Perhatikan gambar 4. Perbandingan luas daerah segienam beraturan CHIJDG dan luas daerah
segienam beraturan ABCDEF adalah …

A B

F C
G H

E D I
J
14. Pak Asari akan mengikat semua buku yang dimilikinya. Ketika banyak buku dalam setiap ikatan
sama dengan 12, ada 2 buku yang tidak terikat. Dia mengubah banyak buku dalam setiap ikatan.
Sekarang dalam setiap ikatan terdapat tepat 9 buku, ternyata juga masih bersisa 2 buku yang tidak
terikat. Setelah dia mengikat 7 buku dalam setiap ikatan, tidak ada lagi buku yang tersisa. Jika
banyak buku yang dimiliki pak Asari berkisar antara 100 dan 200, maka banyak buku yang
dimiliki pak Asari adalah …
15. Perhatikan bahwa 1 + 2 + 3 + 45 + 6 + 78 + 9 = 144. Banyak cara yang mungkin dilakukan untuk
menghasilkan 144 dengan hanya menggunakan bilangan – bilangan yang dibentuk dari angka –
angka 1, 2, 3, 4, 5, 6, 7, 8, dan 9 secara berurutan dari kiri kekanan dan hanya menggunakan
operasi penjumlahan adalah …
16. Diketahui dan adalah dua bilangan bulat positif yang memenuhi persamaan
1 1 1 1
   2 . Nilai terkecil dari adalah …
a 2a 3a x  2 x

2x  4
17. Jika ( ) = , ≠ 0, dan bilangan real,maka ( ) …
x
Catatan : Notasi f 2(x) = f (f(x)), notasi f 3 (x) = f (f(f(x))) dan seterusnya
18. Sebuah boxplot, seperti pada gambar 5, biasanya digunkan untuk menampilkan data yang
menunjukkan nilai kuartil bawah, median, kuartil atas, dan rentangnya

A B

Data Quartil Median Quartil Data


Terkecil Bawah Atas Terbesar

Pada ruas garis AB yang panjangnya 10 cm, titik A digunakan untuk menyatakan data bernilai 0,
titik B untuk data bernilai 100. jika diberikan data sebagai berikut : 65, 70, 67, 82, 71, 25, 83, 78,
58, 72, 94, 66, 86, 73, 71, 31, 71, 87, 65, 76, 86, 66, 98, 74, 84, 96, 100, 73 maka gambar boxplot
data tersebut pada ruas garis AB adalah …
19. Pada suatu perusahaan, ada 3 lowongan pekerjaan yang disediakan hanya untuk pekerja pria, 5
lowongan pekerjaan hanya disediakan untuk pekerja wanita, dan 4 lowongan pekerjaan untuk
pekerja pria dan wanita. Jika terdapat 20 pelamar dengan komposisi 8 wanita dan 12 pria, maka
banyak cara mengisi lowongan pekerjaan tersebut adalah …
20. Diketahui empat pesamaan garis berikut.
ax + by = c, dx + ey = f, px + qy = r, dan sx + ty = u
Agar terbentuk persegi panjang, hubungan yang mungkin antara a, b, c, d, e, f, p, q, r, s, t dan u
adalah …
BAGIAN B : URAIAN
1. Pasangan bilangan bulat ( ) memenuhi system persamaan
2x  y  xy  2

 y  3z  yz  7
2x  3z  xz  30

Berapakah nilai dari x3  10 y 3  z 3

2. Balok pejal ABCD.EFGH berukuran 15 cm x 10 cm x 6 cm. Titik P terletak pada rusuk AB


sedemikian hingga AP = 3 cm. Seekor cecak yang ada disudut G akan menangkap nyamuk yang
ada di P dengan merayap pada permukaan balok. Jika kecepatan cicak bergerak 2,5 cm/detik,
berapa waktu tercepat yang dibutuhkan cicak agar dapat melahap nyamuk ?
3. Hasil kali 46 bilangan bulat sama dengan 1. Mungkinkah jumlah bilangan – bilangan bulat yang
memenuhi syarat tersebut sama dengan 0 ?
4. Angka 1, 2, 3, 4, 5, 6, 7, 8, dan 9 akan ditempatkan ke masing – masing kotak pada gambar
berikut sehingga jumlah mendatarnya sama dengan jumlah vertikalnya. A adalah bilangan 5
angka yang dibentuk dengan cara membaca secara vertikal dari atas kebawah masing – masing
bilangan didalam kotak tersebut. Berapa banyak bilangan A yang mungkin terbentuk ?

5. Untuk setiap pasangan bilangan asli a dan b, didefinisikan a  – . Bilangan asli


dikatakan mitra bilangan asli jika terdapat bilangan asli yang memenuhi  = . Sebagai
contoh, 7 adalah mitra 13 karena terdapat bilangan asli 1 sehingga 7  1 = 7 – 1 + 7 . 1 = 7 – 1 + 7
= 13. Tentukan semua mitra dari 2008
OLIMPIADE SAINS NASIONAL SMP
SELEKSI TINGKAT PROPINSI
TAHUN 2007

BAGIAN A : PILIHAN GANDA


1. Banyak bilangan prima antara 10 dan 99 yang tetap merupakan bilangan prima jika kedua digitnya
dipertukarkan adalah …
A. 9 C. 11 E. 13
B. 10 D. 12
2. Diberikan dua bilangan bulat yang berjumlah 37. Jika bilangan yang lebih besar dibagi dengan
yang lebih kecil, maka hasil baginya adalah 3 dan sisanya 5. Selisih kedua bilangan tersebut adalah

A. 3 C. 8 E. 29
B. 5 D. 21
3. Dua buah dadu berisi enam diberi nomor baru pada setiap sisinya. Dadu pertama diberi nomor
1,1,2,3,3,3 dan dadu kedua diberi nomor –1,–1,–1,–2,–2,–3. Jika kedua dadu dilempar bersamaan,
maka peluang terjadinya jumlah bilangan pada kedua sisi atas dadu bernilai positif adalah …
1 2 4
A. C. E.
4 3 5
1 3
B. D.
2 4
4. Jika sistem persamaan x7y5 = r dan x4y3 = s dengan x, y, r, dan s adalah bilangan positif
mempunyai penyelesaian x = rasb dan y = rcsd, maka hasil dari a + b + c + d adalah …
1
A. 19 C. 1 E.
2
7
B. 2 D.
12
5. Jumlah dari setiap tiga bilangan asli yang terletak pada garis lurus pada gambar selalu sama. Nilai
p + q + r + s adalah …

A. 63 C. 71 E. 90
B. 69 D. 84
6. Diketahui a, b, c, dan d adalah bilangan asli. Jika c habis dibagi a, dan d habis dibagi b, maka
pernyataan berikut :
i) cd habis dibagi ab
ii) (c + d) habis dibagi (a + b)
iii) cd habis dibagi a
iv) bc habis dibagi ab
v) dc habis dibagi ba
yang selalu benar adalah…
A. hanya i)
B. hanya i), iii), dan iv)
C. semuanya, kecuali ii)
D. semuanya, kecuali v)
E. semuanya
7. Tiga segitiga sama kaki, satu persegi (bujur sangkar), dan jajargenjang dengan ukuran seperti
gambar dapat disusun menjadi satu persegi. Keliling persegi yang diperoleh adalah …

2 2 2
2

A. 8 C. 16 E. 16 2

B. 12 D. 8 2
8. Final lomba renang 400 meter diikuti oleh 4 orang finalis yaitu Anita, Bonita, Cantika, dan Dita.
Diperoleh informasi bahwa Bonita selalu kalah bertarung dengan Anita dan Cantika, namun
selalu menang bertarung dengan Dita. Jika dalam perlombaan tersebut akan ditentukan peraih
medali emas, perak, dan perunggu, maka kemungkinan susunan penerima medali adalah
A. 6 C. 12 E. 14
B. 10 D. 13
9. Jumlah koefisien dari hasil penguraian (19x – 20y)2007 adalah …
A. 192007– 202007 D. 1
B. –1 E. 192007+194014
C. 0
10. Tiga lingkaran kongruen saling bersinggungan seperti tampak pada gambar 3. Garis AB melalui
ketiga titik pusat lingkaran dan garis AC merupakan garis singgung lingkaran yang berpusat di B.
Jika diketahui jari–jari lingkaran adalah 3 cm, maka panjang DE adalah …
C
D
E

A B

2 2 5
A. 2 cm C. 3 cm E. 4 cm
3 3 6
1 4
B. 3 cm D. 4
3 5
BAGIAN B : ISIAN SINGKAT
1. Besar sudut TUV pada gambar 4 adalah ... derajat.

5x

V
T
5x

Q S
7x

8x
9x U

2. Definisikan n! = n(n – 1)(n – 2). … .2.1.


Bilangan bulat n yang memenuhi n! = 210 . 35 . 52 . 7 . 11 adalah
3. Semua bilangan–bilangan bulat x dan y yang memenuhi persamaan y2(x + 1) = 1576 + x2 adalah
4. Seorang nelayan menaiki perahu motor yang melaju dengan kecepatan konstan 50 km/jam
melawan arus sungai yang kecepatan airnya konstan. Di tengah perjalanan tanpa disadari
pelampungnya terjatuh ke air dan terbawa arus sungai. Dua puluh menit kemudian ia baru
mengetahuinya, dan segera berbalik arah untuk mencarinya. Jika ia berhasil menemukannya pada
posisi 30 km ke arah hilir dari tempatnya berbalik tadi, maka laju arus sungai tersebut adalah …
km/jam.
5. Diketahui fungsi bilangan real ( ) = x untuk x  1.
1 x
1 1 1
Nilai dari f(2007) + f(2006) + … + f(3) + f(2) + f( )+( )+…+( ) adalah ….
2 3 2007
6. Hasil dari penjumlahan

adalah ….
7. Suatu barisan berbentuk a, b, a + b, a + 2b, 2a + 3b, 3a + 5b, … Jika suku ke–7 dari barisan
tersebut adalah 18, maka rata–rata dari 18 suku pertama barisan tersebut adalah ….
8. Sebuah printer menomori semua halaman sebuah buku mulai dari nomor 1 dan seluruhnya
menggunakan 3201 digit. Banyak halaman buku tersebut adalah ….
9. Sebuah kubus padat terbuat dari bahan yang lunak akan dibelah mengikuti segienam beraturan
seperti tampak pada gambar 5. Semua titik sudut segienam tepat terletak di tengah rusuk–rusuk
kubus. Jika rusuk kubus adalah a cm, maka luas segienam tersebut adalah …. cm2.

10. Kode sebuah kartu ATM diketahui berupa bilangan lima digit dengan ciri–ciri berikut:
- Digit puluhan adalah dua kali lipat digit ribuan,
- Jika digit ratusan dan satuan dipertukarkan maka nilai bilangan tersebut tidak berubah, dan
- Digit puluh ribuan adalah tidak nol.
Jika pemilik kartu ATM tersebut lupa kode, tetapi ingat dengan ciri–ciri tersebut, maka peluang ia
dapat langsung menebaknya dengan benar adalah …

BAGIAN C : URAIAN
1. Sebuah tugu akan dibangun dengan menumpuk kubus–kubus beton yang rusuknya 50 cm, seperti
tampak pada gambar 6. Antar sisi kubus yang berdempetan dan sisi kubus dengan lantai akan
direkat dengan semen setebal 1 cm.

Gambar 6
a. Jika tinggi tugu yang diinginkan adalah 51,51 m, berapa banyak kubus beton yang
diperlukan?
b. Jika semua permukaan kubus beton yang tampak dari tugu tersebut di cat dengan harga
pengecatan Rp 6.000,00 per meter persegi, tentukan besar biaya yang diperlukan.
2. Suatu kebun berbentuk persegi panjang seperti gambar 7. Diketahui ukuran AB = 15 m dan BC =
30 cm. Seekor kucing berada pada titik A dan seekor tikus berada pada titik E, yang merupakan
titik tengah garis AD. Tikus berlari dengan kecepatan 3 m/detik sepanjang garis lurus menuju titik
C, sedangkan kucing berlari dengan kecepatan 5 m/detik sepanjang garis lurus sedemikian
sehingga akan bertemu dengan tikus di titik F. Diketahui kucing dan tikus mulai berlari pada saat
bersamaan, dan di titik F kucing berhasil menerkam tikus (bulatkan sampai 2 angka desimal)?
B C

A D
E
OLIMPIADE SAINS NASIONAL SMP
SELEKSI TINGKAT PROPINSI
TAHUN 2006

BAGIAN A : ISIAN SINGKAT


1. Diberikan segitiga PQR siku–siku di Q. Jika panjang PQ adalah x + 4, panjang QR adalah 3x + 2,
dan panjang PR adalah 3x + 4, maka panjang QR adalah …
2. Diberikan fungsi kuadrat f(x) = ax2 – 3x + c. Jika f(1) = 4 dan f(2) = 7, maka f(–1) = …
3. Nomor telepon di kota Malang terdiri dari enam angka. Banyaknya nomor telepon di kota itu
yang habis dibagi 5 adalah
4. Perhatikan gambar berikut ini.
Jika panjang sisi pada persegi yang terbesar adalah 1 satuan panjang dan persegi berikutnya
diperoleh dengan cara menghubungkan semua titik tengah pada keempat sisinya, maka jumlah
luas yang diarsir adalah …

5. Rata–rata nilai matematika dari 24 siswa adalah 7,20. Setelah ditambah nilai dari 2 siswa, rata–
ratanya menjadi 7,25. Jika nilai salah satu dari kedua siswa itu adalah 7,65, maka nilai satu siswa
yang lain adalah …
6. Jika faktor persekutuan terbesar dari bilangan bulat positif a dan b tidak kurang dari 15, dan
kelipatan persekutuan terkecilnya tidak lebih dari 32, maka banyaknya pasangan bilangan bulat a
dan b yang mungkin adalah …
7. Sebuah kebun berbentuk persegi panjang yang berukuran panjang 160 m dan lebar 50 m. Di
sepanjang tepi kebun dibangun parit dengan lebar yang sama. Jika luas kebun tersebut sekarang
menjadi ¾ luas kebun mula–mula, maka lebar parit yang dibangun adalah ….
8. Terdapat tiga penjaga taman hiburan A, B, dan C. A berjaga setiap 3 hari, B setiap 4 hari, dan C
setiap 5 hari. Pada hari Minggu mereka berjaga bersama–sama untuk pertama kalinya. Pada saat
mereka akan berjaga bersama–sama untuk kedua kalinya, A sakit, sehingga tidak masuk. Pada
hari apa mereka dapat berjaga bersama–sama untuk berikutnya?
a2 b2 a b
9. Misalkan a dan b bilangan real positif. Jika 2
 2
 7 , maka  =…
b a b a
10. Nilai dari 1 – 2 + 3 – 4 + 5 – 6 + … + 2005 – 20062 = …
2 2 2 2 2 2 2
11. Himpunan penyelesaian dari :
2( x  5) 1
1  4(3  x)  , dengan x   adalah …
7 7
1 4 3
12. Jika P dan Q keduanya adalah bilangan positif ganjil dan memenuhi   , maka selisih dari
P Q 5

P dan Q adalah …
13. Diketahui kurva y = x2 + 7x + 2 dan garis y = 2x + 8 saling berpotongan di titik A dan titik B.
Jarak antara titik A dan B adalah …
14. Sebuah ember terbuat dari seng seperti terlihat pada gambar. Luas seng yang digunakan untuk
ember tersebut adalah …

50 cm

40 cm

30 cm

15. Dari gambar di samping ini diketahui bahwa jari–jari lingkaran yang kecil adalah 3 cm dan jari–
jari lingkaran yang besar adalah 5 cm. Panjang CD adalah … cm.

A C  A
B

16. Pak Rahman memiliki sekantong permen yang akan dibagikan kepada anak–anak. Jika tiap anak
diberi dua permen, maka di dalam kantong Pak Rahman masih tersedia empat permen. Namun,
jika tiap anak diberi 3 permen, akan ada dua anak yang tidak mendapat bagian dan satu anak
mendapat dua buah permen. Banyak permen Pak Rahman di dalam kantong sebelum dibagikan
adalah …
17. Lima orang pemuda pergi berekreasi menggunakan sebuah mobil. Mobil yang digunakan
memiliki dua tempat duduk di depan (termasuk untuk pengemudi) dan tiga tempat duduk
dibelakang. Dari kelima pemuda tersebut hanya dua orang bisa menjadi pengemudi. Banyak cara
mereka duduk di mobil tersebut adalah …
18. Jika n adalah bilangan asli, maka bentuk paling sederhana dari perkalian

( )( )( ) ( ) adalah …
19. Gambar di bawah menunjukkan banyaknya siswa dari kelas 7, kelas 8, dan kelas 9 yang
mengikuti ekstrakurikuler sepak bola. Diketahui banyak siswa yang mengikuti kegiatan tersebut
adalah 28 orang. Dua orang siswa dipilih secara acak untuk menjadi ketua dan wakil ketua. Jika
wakil ketua terpilih adalah siswa kelas 7, maka peluang terpilihnya ketua yang berasal dari kelas 9
adalah …

Banyak siswa

Kelas 7 Kelas 8 Kelas 9

20. Pada gambar berikut ini segitiga ABC adalah siku–siku di A dan AEDF adalah suatu persegi. Jika
panjang AB = 6 cm dan AC = 3 cm, maka luas daerah segitiga CDE adalah …

E D

A F B
BAGIAN B : URAIAN
1. Sebuah cerobong asap berbentuk kerucut terpancung, jari–jari alas cerobong tersebut 5 m, jari–
jari atas 4 m, dan tinggi 20 m. Cerobong asap tersebut akan dicat, biaya mengecat per meter
persegi adalah Rp 5.000,00. Hitunglah biaya pengecatan cerobong asap itu.
2. Tentukan m agar persamaan (2x2 + 2mx – (m + 1))(x2 + mx + 1) = 0 mempunyai tepat satu solusi
real!
3. Diberikan segitiga siku–siku sama kaki ABC dengan sudut siku–siku di C. Luas segitiga ABC
adalah 2 satuan luas. Busur l adalah busur lingkaran yang berpusat di A dan membagi segitiga
menjadi dua bagian yang sama luasnya. Busur m adalah busur lingkaran yang berpusat di B dan
menyinggung busur l di titik yang terletak di AB. Tentukan luas daerah yang diarsir.

4. Ucok bermain menyusun batang–batang korek api seperti tampak pada gambar berikut. Apabila
susunan batang korek api yang dibuat Ucok dilanjutkan, tentukan banyak batang korek api yang
diperlukan untuk membuat susunan ke–20.

5. Indonesia akan mengirimkan delegasi Olimpiade Sains Internaisonal (OSI) tingkat SMP pada
tahun 2006. Delegasi ini terdiri atas tiga siswa SMP, yang harus dipilih secara acak dari 10
kandidat yaitu enam siswa bidang Sains dan empat siswa bidang Matematika. Berapa peluang
terpilihnya delegasi OSI yang terdiri dari 2 siswa dari bidang Sains dan 1 siswa dari bidang
Matematika?
OLIMPIADE SAINS NASIONAL SMP
SELEKSI TINGKAT PROPINSI
TAHUN 2005

BAGIAN A : ISIAN SINGKAT


1. Perhatikan segi enam berikut. Banyaknya segitiga yang dapat ditemukan pada gambar tersebut
adalah….

2. Bilangan asli n terbesar yang memenuhi

3. Bilangan A adalah bilangan asli yang terkecil yang merupakan hasil kali dari 3 bilangan prima
pertama. Dua buah bilangan antara 200 dan 300 yang memiliki faktor prima tepat sama dengan
bilangan A tersebut adalah …
(Catatan : 10 dan 30 punya faktor prima yang tidak tepat sama, sedangkan 12 dan 18 memiliki
faktor prima yang tepat sama)

4. Semua pasangan bilangan asli m dan n yang memenuhi pasangan adalah …

5. Bilangan 45 dapat dinyatakan sebagai selisih dari bilangan kuadrat, yakni a2 – b2, dengan a dan b
adalah bilangan asli. Semua pasangan bilangan asli a dan b yang memenuhi a2 – b2= 45 adalah …
6. Bilangan 16 dapat dinyatakan sebagai 3x+7y sebab jika x diganti dengan 3 dan y diganti dengan
1, maka diperoleh 3.3 + 7.1 yang bernilai 16. Tujuh bilangan antara 100 dan 122 yang dapat
23
dinyatakan ke dalam bentuk 6x+9y adalah … m n 1

7. Tiga bilangan bulat membentuk kumpulan data yang berata–rata 10. Banyaknya kombinasi
bilangan yang mungkin (sebutkan juga datanya), jika diketahui data terbesar dan terkecilnya tidak
lebih dari 4 adalah …
8. H himpunan yang didefinisikan oleh {x  B  x2 ≤ 10, x – 1 < 2}, dengan B adalah himpunan
bilangan bulat. Banyaknya himpunan bagian tak kosong dari H adalah …
9. Bilangan–bilangan real x yang memenuhi :
2 1
x2 – 2x – 1 –  = 0 adalah …
x x2
10. Dalam menentukan jawab perkalian bilangan 1493 dan 1507, seorang anak mengurangkan
langsung 49 dari 2.250.000. Dia sama sekali tidak mengalikan kedua bilangan tersebut dengan
cara panjang. Prinsip matematika yang digunakan oleh anak tersebut adalah …
BAGIAN B : URAIAN
1. Perhatikan gambar di samping. Anda diminta untuk mencari luas daerah di dalam kurva ABCDE.
Jika jarak terdekat dua titik secara horizontal atau vertikal adalah 5 cm, berapakah luas segilima
B
ABCDE?        
  A     
       
       
D
 E
      
       C
2. Seseorang memiliki sejumlah koin senilai 1000 rupiah. Setelah diperhatikan dengan seksama,
ternyata koin yang dimilikinya terdiri dari 3 macam koin di antara 4 macam koin yang sekarang
masih berlaku (500–an, 200–an, 100–an, dan 50–an). Selidiki dan tentukan berapa banyak
kombinasi koin yang mungkin dimiliki oleh orang tersebut!
3. Suatu bilangan X terdiri dari 6 angka dan dimulai dari angka 1. Jika angka pertama dipindahkan
dari ujung paling kanan tanpa mengubah susunan angka–angka yang lainnya, bilangan yang baru
terbentuk adalah tiga kali lipat bilangan semula. Berapakah bilangan X tersebut?
4. Pada gambar di samping, titk O adalah titk pusat lingkaran yang berjari–jari r. Jika panjang ruas
1
garis ED juga sama dengan r, buktikan lah bahwa DEC = AOB!
3
B

D r
r

E A
C O r

5. Ada berapa banyakkah pasangan terurut bilangan asli (a,b) dengan syarat a < b, dan FPB(a,b) = 4
serta KPK(a,b) = 140?
OLIMPIADE SAINS NASIONAL SMP
SELEKSI TINGKAT PROPINSI
TAHUN 2004
BAGIAN A : ISIAN SINGKAT
1. Setiap muka sebuah kubus diberi bilangan seperti pada gambar. Kemudian setiap titik sudut diberi
bilangan yang merupakan hasil penjumlahan bilangan pada muka–muka yang berdekatan
dengannya. Nilai bilangan tertinggi pada titik sudut adalah …

9 5 3 1

11

2. Jika a + b = 1, b + c = 2, dan c + a = 3, maka a + b + c = …


3. Pada suatu jam digital yang angka–angkanya tertera mulai dari 00:00 sampai 23:59,
dimungkinkan terjadi penampakan bilangan palindrome (bilangan yang dibaca dari depan dan
dari belakang sama nilainya, misalnya 12:21 dan 23:32). Dalam satu hari satu malam, banyaknya
bilangan Palindrome tersebut menampakkan diri adalah …
4. Untuk bilangan bulat a dan b, (a, b) artinya bilangan tak negative yang merupakan sisa a  b
dibagi oleh 5. Bilangan yang ditunjukkan oleh (–3, 4) adalah …
5. Bilangan 10-angka terbesar yang menggunakan empat angka 1, tiga angka 2, dua angka 3, dan
satu angka 4, sehingga dua bilangan yang sama tidak terletak bersebelahan adalah …
6. Jika selisih dua bilangan adalah 2 dan selisih kuadrat dua bilangan itu 6, maka hasil tambahan dua
bilangan itu adalah …

7. Bentuk sederhana dari 4  15  4  15 adalah ...

8. Suatu garis memotong sumbu–x di titik A(a,0) dan memotong sumbu y di titik B(0,3). Jika luas
segitiga AOB sama dengan 6 satuan luas, dengan O(0,0), maka keliling segitiga AOB sama
dengan …
9. Persegi Antimagic ukuran 4  4 adalah persegi panjang dari bilangan–bilangan 1 sampai
dengan 16 sedemikian sehingga jumlah dari setiap empat baris, empat kolom dan dua diagonal
utamanya merupakan sepuluh bilangan yang berurutan. Diagram berikut menunjukkan sebagian
dari Persegi Antimagic ukuran 4  4. Berapakah nilai dari *?

* 14
9 3 7
12 13 5
10 11 6 4

10. = ...
BAGIAN B : URAIAN
1. Enam belas tim sepakbola mengikuti turnamen. Pertama–tama mereka dikelompokkan ke dalam
empat kelompok dengan masing–masing 4 tim di setiap kelompoknya. Di setiap kelompok
mereka saling bermain satu sama lain satu kali. Dua tim yang memiliki peringkat teratas
selanjutnya maju ke babak berikutnya yang menggunakan sistim gugur (kalah langsung
tereliminasi) sampai ditemukan juaranya. Berapa banyak pertandingan yang berlangsung dalam
turnamen tersebut?
2. Pada gambar di bawah ini, ABCD adalah persegi dengan panjang 4 cm. Titik P dan Q membagi
diagonal AC menjadi tiga bagian sama panjang. Berapakah luas PDQ?

B C

D
A

3. Untuk bilangan real x didefinisikan


 x, jika x  0
x 
 x, jika x  0

Jika x2 + 2 x –3 = 0, nilai x yang memenuhi adalah ...

4. Sebuah semangka yang beratnya 1 kg mengandung 93% air. Sesudah beberapa lama dibiarkan di
bawah sinar matahari, kandungan air semangka itu turun menjadi 90%. Berapakah berat
semangka sekarang?
5. Untuk bilangan real a dan b sebarang, buktikan bahwa
a2 + b2  2(a + b) – 2
OLIMPIADE SAINS NASIONAL SMP
SELEKSI TINGKAT PROPINSI
TAHUN 2003

BAGIAN A : ISIAN SINGKAT


5 2 1
1.   =…
12 7 8
2. Suatu botol dengan kapasitas 875 mililiter digunakan untuk mengisikan minyak kedalam suatu
jerigen berkapasitas 20 liter. Berapa kalikah botol tersebut digunakan untuk membuat penuh
sebuah jerigen kosong?
3. Titik–titik sudut segitiga ABC memiliki koordinat A(0,0), B(4,3), dan C(7,–1). Maka luas segitiga
ABC tersebut adalah …
4. Suatu persegi panjang dapat dipecah-pecah menjadi 5 persegi yang kongruen. Luas persegi
panjang tersebut adalah 720 cm2. Maka keliling satu perseginya adalah …
5. Notasi x menyatakan bilangan bulat terbesar yang lebih kecil dari atau sama dengan x. Sebagai
7  1
contoh,    2,    1 . Maka hubungan yang benar di antara dua bilangan bulat
3  2

s  2  3  dan t   2    3  adalah …
6. Kapasitas tangki bahan bakar suatu mobil adalah 40 liter. Setiap menempuh perjalanan 100 km,
mobil tersebut menghabiskan 7,7 liter bahan bakar. Mobil tersebut digunakan untuk pergi dari
Bandung ke Yogyakarta sehingga menempuh perjalanan sejauh 428 km. Ketika mulai perjalanan,
tangki mobil tersebut terisi penuh bahan bakar. Dalam satuan liter terdekat, banyaknya bahan
bakar yang tersisa ketika tiba di Yogyakarta adalah …
7. Ada suatu keluarga terdiri dari ayah, ibu dan 3 orang anak. Ibu lahir pada bulan April. Berapakah
peluang ada tepat satu orang anggota lain dalam keluarga tersebut yang lahir juga dibulan April?
8. Pada suatu kubus ABCD.EFGH ruas garis AG adalah diagonal ruang dari kubus tersebut. Ada
berapa carakah perjalanan terpendek dari titik sudut G ke titik sudut A dengan syarat perjalanan
tersebut hanya melalui rusuk–rusuk kubus dan tidak ada rusuk yang dilalui lebih dari satu kali?
9. Misalkan a dan b adalah dua bilangan tertentu. Jika a2+(a+b)b=a(b–a)+x, maka x = …
10. Gaji Yuni dan Yuli pada tahun 2001 sama besarnya. Pada tanggal 1 Januari 2002, gaji Yuni naik
15% sedangkan gaji Yuli naik 10%. Tepat 1 tahun kemudian, gaji Yuli naik 15% dan gaji Yuni
naik 10%. Siapakah yang gajinya sekarang lebih besar?
BAGIAN B : URAIAN
x 1
1. Tentukan semua bilangan real x yang memenuhi pertaksamaan 2.
x 1
2. Diketahui T adalah titik tengah suatu sisi dari segi–6 beraturan ABCDEF. Jika panjang sisi segi–6
tersebut 1 satuan, berapakah panjang garis TE? D

C E

B F

T
A
3. Diketahui suatu barisan U(n) = 2.3 + 3.4 + 4.5 + 5.6 + 6.7 + … + (n+1)(n+2) sehingga beberapa
suku awal dari barisan tersebut adalah sebagai berikut U(1) = 6, U(2) = 18, U(3) = 38, U(4) = 68, U(5)
= 110. Tentukan nilai dari U(20)?
x n 1  2 x n  2
4. Diketahui x0 = 1, x1=2. Sedangkan untuk n 2 didefinisikan xn = . Maka x2 + 2x3 =…
2 xn 1  xn  2

3
5. Dalam suatu kelas, bagian siswanya adalah wanita. Ke dalam kelas tersebut ditambahkan 5
5
3
siswa pria dan 5 siswa wanita. Sekarang, bagian siswanya adalah pria. Berapa banyakkah siswa
7
dalam kelas mula–mula?
KUMPULAN SOAL - SOAL OSN
OLIMPIADE MATEMATIKA
OLIMPIADE SAINS NASIONAL (OSN) SMP 2003 – 2018
TINGKAT KABUPATEN/ KOTA,PROPINSI & NASIONAL

OLIMPIADE SAINS NASIONAL (OSN) SMP


TINGKAT NASIONAL
TAHUN 2003 - 2018
OLIMPIADE SAINS NASIONAL SMP
SELEKSI TINGKAT NASIONAL
TAHUN 2017

SOAL OSN SMP TAHUN 2017

1. Carilah bilngan real 𝑥 yang memenuhi pertidaksamaan


𝑥2 − 3 𝑥2 + 5 𝑥2 − 5 𝑥2 + 3
+ ≥ +
𝑥2 − 1 𝑥2 + 3 𝑥2 − 3 𝑥2 + 1

2. Diketahui 𝑚 adalah bilangan asli empat angka dengan angka satuan dan ribuan sama. Jika 𝑚
merupakan bilangan kuadrat, tentukan semua nilai 𝑚 yang mungkin.

3. Pada gambar berikut, Δ𝐴𝐵𝑃 adalah segitiga samakaki. Dengan 𝐴𝐵 = 𝐵𝑃 dan titik 𝐶 pada 𝐵𝑃.
Hitunglah volume dari benda yang diperoleh dari hasil pemutaran Δ𝐴𝐵𝐶 mengelilingi garis 𝐴𝑃.

C P
B
12 12
18

4. Acara perpisahan suatu kelas terdiri dari 10 anak laki-laki dan 12 anak perempuan. Wali kelas
dari kelas tersebut menyediakan enam hadiah untuk siswanya dipilih secara acak. Hadiah yang
disediakan adalah satu tas sekolah, dua buah novel dan tiga buah kalkulator. Jika total siswa laki-
laki mendapatkan hadiah sama banyak dengan total siswa perempuan yang mendapatkan
hadiah, ada berapa banyak susunan yang mungkin dari siswa yang mendapatkan hadiah?

5. Diketahui 𝑆 = 1945,1946,1947, ⋯ ,2016,2017 . Jika 𝐴 = 𝑎, 𝑏, 𝑐, 𝑑, 𝑒 merupakan himpunan


bagian dari 𝑆 dengan 𝑎 + 𝑏 + 𝑐 + 𝑑 + 𝑒 habis dibagi 5. Tentukan banyak 𝐴 yang mungkin.

6. Parabola 𝑦 = 𝑎𝑥 2 + 𝑏𝑥, 𝑎 < 0 memiliki puncak di titik 𝐶 dan memotong sumbu 𝑥 di titik 𝐴 dan
𝐵 yang berbeda. Garis 𝑦 = 𝑎𝑥 memotong parabola tersebut di titik berbeda 𝐴 dan 𝐷. Jika luas
segitiga 𝐴𝐵𝐶 sama dengan |𝑎𝑏| kali luas segitiga 𝐴𝐵𝐷, tentukan nilai 𝑏 sebagai fungsi dari 𝑎
tanpa menggunakan nilai mutlak.
Catatan: |𝑥| disebut nilai mutlak 𝑥 dengan
𝑥, 𝑗𝑖𝑘𝑎 𝑥 ≥ 0
𝑥 =
−𝑥, 𝑗𝑖𝑘𝑎 𝑥 < 0

7. Diketahui 𝑎 bilangan prima dan 𝑘 adalah bilangan bulat positif. Jika 𝑘 2 − 𝑎𝑘 adalah bilangan
bulat positif, tentukan nilai 𝑘 sebagai fungsi 𝑎.
SOAL OSN SMP TAHUN 2017

8. Terdapat 5 titik yang berbeda, 𝑇1 , 𝑇2 , 𝑇3 , 𝑇4 dan 𝑇 pada sebuah lingkaran 𝐿. Misalkan 𝑡𝑖𝑗 adalah
jarak titik 𝑇 ke garis 𝑇𝑖 𝑇𝑗 atau perpanjangannya. Buktikan bahwa
𝑡𝑖𝑗 𝑇𝑇𝑖 𝑡12 𝑡13
= dan =
𝑡𝑗𝑘 𝑇𝑇𝑘 𝑡24 𝑡34

𝑇4

𝑇1

𝑇2 𝑇3

9. Diketahui barisan bilangan bulat positif 7-angka 𝑎1 , 𝑎2 , ⋯ , 𝑎2017 dengan 𝑎1 < 𝑎2 , ⋯ < 𝑎2017 .
Setiap suku barisan bilangan tersebut memiliki angka-angka penyusun tak naik. Diketahui
𝑎1 = 1000000, dan 𝑎𝑛+1 adalah bilangan terkecil yang mungkin lebih besar dari 𝑎𝑛 . Sebagai
contoh diperoleh 𝑎2 = 1100000 dan 𝑎3 = 1110000. Tentukan 𝑎2017

10. Pada kilang minyak di daerah Duri, tersedia pompa-1 dan pompa-2. Kedua pompa tersebut
digunakan untuk mengisi tangki penampungan dengan volume 𝑉. Tangki tersebut dapat diisi
penuh menggunakan pompa-1 saja dalam waktu 4 jam, atau menggunakan pompa-2 saja dalam
waktu 6 jam. Mula-mula kedua pompa digunakan secara bersama-sama dalam waktu 𝑎 jam,
kemudian dilanjutkan hanya menggunakan pompa-1 selama b jam dan dilanjutkan dengan
pompa-2 selama c jam. Jika biaya operasional pompa-1 adalah 15(𝑎 + 𝑏) ribu per jam dan biaya
operasional pompa-2 adalah 4(𝑎 + 𝑐) ribu per jam, tentukan 𝑏 dan 𝑐 agar biaya operasional
seluruh pompa adalah minimum (nyatakan 𝑏 dan 𝑐 sebagai fungsi 𝑎). Tentukan juga nilai 𝑎 yang
mungkin.
𝑗 𝑇𝑘
OLIMPIADE SAINS NASIONAL SMP
SELEKSI TINGKAT NASIONAL
TAHUN 2016

HARI PERTAMA
1. Tentukan semua bilangan real yang memenuhi persamaan
(1 + x2 + x4 + .... + x2014)(x2016 + 1) = 2016x2015
2. Misalkan A adalah suatu bilangan bulat dan
A = 2 + 20 + 201 + 2016 + 20162 + .... + ⏟

Tentukan tujuh angka terakhir dari A berurutan mulai dari angka jutaan sampai dengan
satuan.
3. Pada segitiga ABC, Titik P dan Q berada pada sisi BC sehingga panjang BP sama dengan CQ, ∠BAP
= ∠CAQ dan ∠APB lancip. Apakah segitiga ABC sama kaki? Tulis alasan Anda.
4. Ayu akan membuka koper tetapi dia lupa kuncinya. Kode koper tersebut terdiri dari Sembilan angka
yakni empat angka 0 (nol) dan lima angka 1. Ayu ingat bahwa tidak ada empat angka sama yang
berurutan. Berapa banyak kode yang mungkin harus dicoba sehingga dipastikan koper tersebut
terbuka?
5. Fulan memelihara 100 kalkun dengan bobot kalkun ke- i adalah xi untuk i {1, 2, 3, .... , 100}. Bobot
kalkun ke-i dalam gram diasumsikan mengikuti fungsi xi(t) = Sit + 200 – i dengan t menyatakan waktu
dalam satuan hari dan Si merupakan suku ke-i suatu barisan aritmatika dengan suku pertama adalah

bilangan positif a dengan beda b = . Diketahui rata-rata data bobot seratus kalkun tersebut pada saat

t = a adalah 150,5 gram. Hitunglah median data bobot kalkun tersebut pada saat t = 20 hari.

HARI KEDUA
1. Diketahui ( ) , untuk . Didefinisikan untuk semua bilangan rasional positif

dan . Perhatikan barisan dengan , ( ) untuk


Tentukan ( ) dan
2. Diketahui a dan b adalah bilangan bulat positif dengan a > b > 2.

Apakah merupakan bilangan bulat? Tulis alasan Anda.


3. Diberikan kubus ABCD.EFGH dengan panjang rusuk 1 dm. Terdapat persegi PQRS pada bidang
diagonal ABGH dengan titik P pada HG dan Q pada AH seperti ditunjukkan pada gambar di bawah.
Titik T adalah titik pusat persegi PQRS. Garis HT diperpanjang sehingga memotong garis diagonal
BG di N. Titik M adalah proyeksi N terhadapat BC. Tentukan volume prisma terpancung DCM.HGN.

4. Sembilan pasang suami istri ingin berfoto dalam posisi tiga baris dengan latar belakang Jembatan
Ampera Palembang. Terdapat 4 orang di baris depan, 6 orang di baris tengah, dan 8 orang di baris
belakan. Mereka sepakat bahwa setiap pasang suami istri harus dalam baris yang sama, serta setiap
dua orang yang bersebelahan haruslah pasangan suami istri atau berjenis kelamin sama. Tentukan
banyak susunan posisi berbeda yang mungkin dilakukan
5. Suatu hotel menyediakan empat jenis kamar dengan kapasitas, tarif, dan banyak kamar seperti
disajikan pada tabel berikut.

Satu rombongan yang terdiri dari empat keluarga ingin menginap semalam di hotel tersebut. Masing-
masing keluarga terdiri dari suami-sitri dan anak-anak mereka yang belum menikah. Banyak anggota
keluarga menurut jenis kelamin disajikan pada tabel berikut
Ketua rombongan memberlakukan ketentuan sebagai berikut.
I. Setiap pasang suami-istri harus sekamar dan tidak boleh sekamar dengan pasangan suami-istri
lainnya.
II. Laki-laki dan perempuan tidak boleh sekamar kecuali mereka berasal dari satu keluarga.
III. Paling sedikit ada satu kamar yang ditempati oleh semua perwakilan keluarga (“kamar
perwakilan”)
IV. Setiap keluarga menempati paling banyak 3 jenis kamar.
V. Tidak ada kamar yang ditempati oleh lebih dari satu keluarga kecuali kamar perwakilan.
Anda diminta mengatur kamar untuk rombongan tersebut agar total biaya penginapan semurah
mungkin. Berikan dua alternatif kemungkinan pengaturan kamar untuk setiap keluarga dan tentukan
total biayanya.
OLIMPIADE SAINS NASIONAL SMP
SELEKSI TINGKAT NASIONAL
TAHUN 2015

HARI PERTAMA
1. Tentukan bilangan bulat yang memiliki sifat sifat berikut :
a) Setiap dua angka (digit) bersebelahan pada bilangan tersebut adalah prima
b) Semua bilangan prima yang dimaksudkan pada butir (a) diatas adalah berbeda

2. Tentukan semua bilangan bulat sehingga √ √ √ √


3. Gambar brikut menunjukkan jalur untuk membentuk rangkaian huruf angka “OSN2015”.
Tentukan banyak jalur berbeda yang mungkin untuk membentuk rangkaian huruf angka tersebut
dengan mengikuti anak panah.

4. Diberikan segita lancip ABC dengan L sebagai lingkaran luarnya. Dari titik A dibuat garis tinggi
pada ruas garis BC sehingga memotong lingkaran L dititik X. dengan cara serupa dibuat garis
tinggi dari titik B dan titik C sehingga memotong lingkaran L, berturut turut dititik Y dan titik Z.
apakah panjang busur AY=panjang busur AZ?
5. Siswa kelas VII.3 dibagi menjadi lima kelompok: A, B, C, D dan E. setiap kelompok melakukan
lima percobaan IPA selama lima minggu. Setiap minggu masing masing kelompok melakukan
satu percobaan yang berbeda dengan percobaan yang dilakukan oleh kelompok lain. Tentukan
paling sedikit dua jadwal percobaan yang mungkin pada minggu kelima, berdasarkan informasi
berikut:
 Pada minggu pertama, kelompok D mengerjakan percobaan 4
 Pada minggu kedua, kelompok C mengerjakan percobaan 5
 Pada minggu ketiga, kelompok E mengerjakan percobaan 5
 Pada minggu keempat, kelompok A mengerjakan percobaan 4 dan kelompok D
mengerjakan percobaan 2
HARI KEDUA
1. Diketahui m dan n adlah dua bilangan positif yang berturut turut tersiri dari empat angka dan tiga
angka. Kedua bilangan tersebut memuat angka 4 dan 5. Bilangan 59 adalah faktor prima dari m.
sisa pembagian n oleh 38 adalah 1. Jika selisih m dan n tidak lebih dari 2015, tentukan semua
pasangan bilangan ( ) yang mungkin.
2. Diketahui persamaan dengan mempunyai dua akar real yang berbeda
dan persamaan ( ) tidak mempunyai akar real.
Apakah ?
3. Suatu kompetisi bola basket diikuti oleh 6 tim. Setiap tim membawa satu bendera tim yang
dipasang pada tiang yang terdapat di pinggir lapangan pertandingan. Terdapat empat lokasi dan
setiap lokasi memiliki lima tiang berjajar. Pasangan bendera di setiap lokasi dimulai dari tiang
paling kanan secara berurutan. Jika tidak semua tiang di setiap lokasi harus dipasang bendera
tentukan banyak susunan bendera yang mungkin.
4. Diketahui dua lingkaran dan berturut turut berpusat di M dan N. jari jari lingkaran dan
berturut turut adalah 5 satuan panjang dan 6 satuan panjang. Lingkaran melalui titik N dan
berpotongan dengan lingkaran di titik P dan dititik Q. titik U terletak pada lingkaran
sehingga ruas garis PU adalah suatu diameter lingkaran . Titik T terletak pada perpanjangan

ruas garis PQ sehingga luas segiempat QTUN adalah satuan luas. Tentukan panjang QT.

5. Sebuah bula es memiliki volume awal . Setelah n detik volume bola es menjadi dan luas
permukaannya . Bola es mencair dengan perubahan volume per detik sebanding dengan luas
permukaannya, yaitu untuk setiap n, dengan a adalah suatu konstanta positif.
Selain itu, diketahui bahwa perbandingan perubahan volume dan perubahan jari jari per detik

sebanding dengan luas permukaannya, yaitu , dengan k adalah suatu konstanta

positif. Jika dan bola es mencair keseluruhannya tepat pada saat h detik, tentukan
nilai h.
OLIMPIADE SAINS NASIONAL SMP
SELEKSI TINGKAT NASIONAL
TAHUN 2014

HARI PERTAMA
1. Bahri bertempat tinggal cukup dekat dengan jam gadang di kota Bukit Tinggi Sumatera Barat.
Bahri memiliki jam antik. Pada hari senin tanggal 4 maret 2013 pukul 10.00 pagi, jam antik
terlambat Bahri terlambat dua menit dibandingkan jam Gadang. Sehari kemudian, jam antiknya
terlambat empat menit dibandingkan Jam Gadang. Tanggal 6 Maret 2013 jam tersebut terlambat
enam menit dibanding Jam Gadang. Hari-hari berikutnya Bahri mengamati bahwa jam antiknya
menunjukkan pola keterlambatan yang sama. Pada hari apa dan tanggal berapakah di tahun 2014
jam antik Bahri (jarum pendek dan jarum panjang) menunjuk angka yang sama dengan Jam
Gadang ?
2. Pada satu musim kompetisi Liga Sepak Bola Indonesia diikuti oleh 20 tim sepaknola. Setiap tim
bertanding dengan setiap tim lain sebanyak dua kali. Nilai hasil setiap pertandingan adalah 3 jika
menang, 1 jika imbang (seri), dan 0 jika kalah. Setiap minggu ada 10 pertandingan yang
melibatkan semua tim. Pada akhir minggu ke berapakah paling cepat yang mungkin, juara
kompetisi pada musim tersebut dapat dipastikan?
3. Perhatikan gambar berikut.

Segiempat adalah segiempat talibusur (segiempat yang keempat titik sudutnya terletak
pada lingkaran). Diketahui tegak lurus dan tegak lurus . Apakah pertanyaan
berikut benar? Tuliskan alasan Anda !

4. Misalkan . Jika jumlah semua angka (digit) penyusun bilangan sama dengan
dan jumlah semua angka penyusun bilangan sama dengan , maka tentukan jumlah semua
angka penyusun .

5. Tentukan semua bilangan bulat positif sehingga ( ) adalah kuadrat dari


suatu bilangan bulat.
HARI KEDUA
1. Halaman rumah Siti Nurbaya yang berbentuk persegi panjang akan ditutupi dengan sejumlah
paving blok yang berbentuk segienam beraturan atau potongannya seperti gambar dibawah ini.
Panjang sisi segienam tersebut adalah 12 cm. Pemasangan paving blok yang lain atau
potongannya sehingga seluruh permukaan halaman tertutup penuh. Untuk menutupi seluruh
permukaan halaman rumah tersebut diperlukan 603 paving blok. Berapa paving blok tersebut
harus dipotong menjadi model A, B, C dan D untuk keperluan penutupan. Jika diperlukan 17
potongan paving blok model A, berapakah ukuran panjang dan lebar halaman rumah Nurbaya?
Hitung juga berapa banyak masing masing potongan paving model B, C dan D yang digunakan.

2. Diketahui persegi PQRS. Jika salah satu sisinya terletak pada garis dan dua titik
sudutnya terletak pada parabola , tentukan luas maksimum persegi PQRS yang mungkin.
3. Pada limas segitiga T.ABC titik E, F, G dan H berturut turut terletak pada AB, AC, TC dan TB
sehingga . Tentukan perbandingan volume kedua
bagian limas segitiga yang terbagi oleh bidang EFGH
4. Diketahui x bilangan bulat tak negatif dan y bilangan bulat. Tentukan semua pasangan ( )
yang memenuhi
5. Pelatih tim nasional bola basket Indonesia akan memilih pemain untuk menjadi anggota tim inti.
Pelatih akan menilai lima pemain A, B, C, D dan E dalm satu pertandingan simulasi ( atau uji
coba) dengan total waktu pertandingan 80 menit. Setiap saat hanya ada satu diantara lima pemain
tersebut yang bermain. Tidak ada pembatas banyaknya pergantian pemain selama pertandingan.
Total waktu bermain untuk masing masing pemain A, B dan C adalah kelipatan 5 menit,
sedangkan total waktu bermain masing masing pemain D dan E adalah kelipatan 7 menit.
Berapakah banyak cara setiap pemain berada dilapangan berdasarkan total waktu bermain?
OLIMPIADE SAINS NASIONAL SMP
SELEKSI TINGKAT NASIONAL
TAHUN 2013

HARI PERTAMA

1. Diketahui f adalah suatu fungsi sehingga f(x) + 2 ( ) = 3x untuk setiap x ≠ 0. Carilah nilai x

yang memenuhi f(x) = f(-x).


2. Diketahui ABC adalah segitiga lancip dengan titik-titik sudutnya terletak pada lingkaran yang
berpusat di titik O. Titik P terletak pada sisi BC sehingga AP adalah garis tinggi segitiga ABC.
Jika ∠ABC + 30O ≤ ∠ACB, buktikan bahwa ∠COP + ∠CAB < 90 O
3. Tentukan semua bilangan asli a, b, dan c yang lebih besar dari 1 dan berbeda, serta memenuhi
sifat bahwa abc membagi habis bc + ac + ab + 2.
4. Misalkan A, B, dan P adalah paku-paku yang ditanam pada papan ABP. Panjaang AP = a satuan
dan BP = b satuan. Papan ABP diletakkan pada lintasan X1X2 dan Y1Y2 sehingga A hanya
bergerak bebas sepanjang lintasan X1X2 dan hanya bergerak bebas sepanjang lintasan Y1Y2
seperti pada gambar berikut. Misalkan x adalah jarak titik P terhadap lintasan Y1Y2 dan y adalah

terhadap lintasan X1X2. Tunjukkan bahwa persamaan lintasan titik P adalah

5. Terdapat tiga buah kotak A, B, dan C masing-masing berisi 3 bola berwarna putih dan 2 bola
berwarna merah. Selanjutnya dilakukan pengambilan tiga bola dengan aturan sebagai berikut:
1. Tahap ke – 1
Ambil satu bola dari kotak A
2. Tahap ke – 2
 Jika bola yang terambil dari kotak A pada tahap ke-1 berwarna putih, maka bola tersebut
dimasukkan ke kotak B. selanjutnya dari kotak B diambil satu bola. Jika yang terambil
adalah bola berwarna putih, maka bola tersebut dimasukkan ke kotak C, sedangkan jika
yang terambil bola merah, maka bola tersebut dimasukkan ke kotak A.
 Jika bola yang terambil dari kotak A pada tahap ke-1 berwarna merah, maka bola tersebut
dimasukkan ke kotak C. selanjutnya dari kotak C diambil satu bola. Jika yang terambil
adalah bola berwarna putih, maka bola tersebut dimasukkan ke kotak A, sedangkan jika
yang terambil bola merah, maka bola tersebut dimasukkan ke kotak B.
3. Tahap ke – 3
Ambil masing-masing satu bola dari kotak A, B, dan C
Berapa peluang bahwa semua bola yang terambil pada tahap ke – 3 berwarna merah?

HARI KEDUA
1. Apakah ada bilangan asli n sehingga n2 + 5n + 1 habis dibagi oleh 49? Jelaskan!
2. Diketahui parabola y = ax2 + bx + c melalui titik (-3, 4) dan 3, 16), serta tidak memotong sumbu-
x. Carilah semua nilai absis yang mungkin untuk titik puncak parabola tersebut.
3. Diketahui T.ABC adalah limas segitiga beraturan dengan panjang rusuk 2 cm. Titik-titik P, Q, R,
dan S berturut-turut merupakan titik berat segitiga ABC, segitiga TAB, segitiga TBC, dan segitiga
TCA. Tentukan volume limas segitiga beraturan P.QRS. (catatan : titik berat suatu segitiga adalah
perpotongan ketiga garis berat)
4. Pada suatu acara diundang 13 orang tamu istimewa yang terdiri dari 8 orang pria dan 5 orang
wanita. Khusus untuk semua tamu istimewa tersebut disediakan 13 tempat duduk pada satu baris
khusus. Jika diharapkan tidak ada dua orang wanita yang duduk bersebelahan, tentukan banyak
posisi duduk yang mungkin untuk semua tamu istimewa tersebut
5. Sebuah tabel yang berukuran n baris dan n kolom akan diisi dengan bilangan 1 atau -1 sehingga
hasil kali semua bilangan yang terletak dalam setiap baris dan hasil kali semua bilangan yang
terletak dalam setiap kolom adalah – 1. Berapa banyak cara berbeda untuk mengisi tabel tersebut?
OLIMPIADE SAINS NASIONAL SMP
SELEKSI TINGKAT NASIONAL
TAHUN 2012
HARI PERTAMA
1. Jika diketahui himpunan
{( )|( ) }
Tentukan banyak himpunan bagian dari H.
2. Seorang pesulap menyatakan dirinya ahli menebak pikiran dengan pertunjukan berikut. Salah
seorang penonton awalnya diminta secara tersembunyi menuliskan sebuah bilangan lima angka,
lalu menguranginya dengan jumlah angka-angka penyusun bilangan tersebut, kemudian
menyebutkan 4 dari 5 angka penyusun bilangan hasil (dengan urutan sembarang).Selanjutnya
pesulap tersebut dapat menebak angka yang masih disembunyikan. Sebagai contoh, jika penonton
menyebutkan empat angka bilangan hasil : 0,1,2,3, maka pesulap akan tahu bahwa angka yang
disembunyikan adalah 3.
a. Berilah suatu contoh Anda sendiri dari proses di atas.
b. Jelaskan secara matematis bentuk umum dari proses tersebut.
3. Pada suatu keranjang buah terdapat 20 apel, 18 jeruk, 16 mangga, 10 nanas, dan 6 pepaya. Jika
seseorang ingin mengambil 10 buah dari keranjang tersebut, ada berapa banyak komposisi buah
terambil yang mungkin ?
4. Di dalam taman Khatulistiwa akan dibuat bangunan berbentuk limas dengan alas segitiga sama
sisi berbahan tembus pandang dengan panjang sisi alas √ dan tinggi . Sebuah bola dunia
akan ditempatkan di dalam limas tersebut. Dengan mengabaikan ketebalan bahan pembuat limas,
tentukan panjang terbesar jari-jari bola dunia yang mungkin dapat dibuat.
5. Berapakah sisa dari dibagi

HARI KEDUA
1. Pada suatu hari, seorang peneliti menempatkan dua kelompok spesies yang berbeda, yakni
amoeba dan bakteri pada suatu media yang sama, masing-masing dalam jumlah tertentu (dengan
satuan sel). Peneliti tersebut mengamati bahwa pada hari berikutnya, yakni hari kedua, ternyata
setiap sel masing-masing spesies membelah diri menjadi dua sel. Pada hari yang sama setiap sel
amoeba memangsa tepat satu sel bakteri. Pengamatan selanjutnya yang dilakukan setiap hari
menunjukkan pola yang sama, yakni setiap sel masing-masing spesies membelah diri menjadi dua
sel dan kemudian setiap sel amoeba memangsa tepat satu sel bakteri. Pengamatan pada hari ke-
100 menunjukkan bahwa setelah masing-masing spesies membelah diri dan kemudian setiap sel
amoeba memangsa tepat satu sel bakteri, ternyata membuat bakteri punah. Tentukan
perbandingan jumlah amoeba dengan bakteri pada hari pertama.
2. Diketahui adalah bilangan bulat positif. Jika

( )
√ √
Tentukan ( ) ( ) ( ) ( ).
3. Budi menyusun empat belas buah bola masing-masing berjari-jari . Sembilan buah bola
pertama diletakkan di atas meja sedemikian sehingga membentuk persegi dan saling
bersinggungan. Empat buah bola berikutnya diletakkan di atas Sembilan bola pertama sehingga
saling bersinggungan. Bola keempat belas ditaruh di atas empat bola tadi, sehingga menyinggung
empat bola tersebut. Jika Bambang mempunyai lima puluh lima buah bola yang masing-masing
juga berjari-jari cm, dan semua bola tersebut disusun mengikuti pola susunan bola yang
dilakukan Budi, hitunglah ketinggian pusat bola yang paling atas diukur dari permukaan meja
pada susunan bola yang dilakukan Bambang.
4. Diketahui sebuah segitiga dengan panjang sisi-sisinya adalah
√ .Tentukan luas maksimum persegi panjang yang mungkin dapat dibuat
di dalam segitiga tersebut.
5. Ada 12 orang yang antri untuk membeli tiket masuk suatu pertunjukan dengan harga satu tiket
adalah Rp 5.000. Diketahui 5 orang diantara mereka hanya mempunyai uang kertas Rp 10.000
dan sisanya hanya mempunyai uang kertas Rp 5.000. Jika penjual tiket awalnya hanya
mempunyai uang Rp 5.000, berapakah peluang penjual tiket tersebut mempunyai cukup
kembalian untuk melayani semua orang sesuai dengan urutan mereka dalam antrian ?
OLIMPIADE SAINS NASIONAL SMP
SELEKSI TINGKAT NASIONAL
TAHUN 2011

HARI PERTAMA
1. Dari pengukuran terhadap tinggi sembilan pohon diperoleh data sebagai berikut.
a) Ada tiga hasil pengukuran yang berbeda (dalam satuan meter)
b) Semua data berupa bilangan positip
c) Mean = median = modus = 3
d) Jumlah kuadrat semua data adalah 87
Tentukan semua kemungkinan ukuran tinggi Sembilan pohon tersebut.
2. Didefinisikan nilai mutlak |x| sebagai berikut.
|x| = x, jika x ³ 0 dan |x| = -x, jika x < 0
Jika x dan y adalah bilangan bulat, tentukan banyak pasangan (x,y) yang memenuhi |x| + |y| ≤ 50.
3. Bangun datar ABCD di bawah ini adalah trapesium dengan AB sejajar CD. Titik E dan F terletak
pada CD sehingga AD sejajar BE dan AF sejajar BC. Titik H adalah perpotongan AF dengan BE
dan titik G adalah perpotongan AC dengan BE. Jika panjang AB adalah 4 cm dan panjang CD
adalah 10 cm hitunglah perbandingan luas segitiga AGH dan luas trapesium ABCD.

4. Seorang calon dokter diharuskan magang di rumah sakit selama lima hari pada bulan juli 2011.
Pimpinan rumah sakit memberikan aturan sebagai berikut.
a) Magang tidak boleh dilakukan dua hari berturut-turut.
b) Magang hari kelima baru boleh dilakukan setelah empat hari terhitung sejak magang hari
keempat. Misalkan magang hari keempat adalah tanggal 20, maka magang hari kelima baru
boleh dilaksanakan setidaknya tanggal 24.
Tentukan banyak pilihan jadwal yang mungkin bagi calon dokter tersebut.
5. Perhatikan barisan bilangan asli berikut :
5, 55, 555, 5555, 55555, ...,⏟ ...

Barisan di atas mempunyai aturan : suku ke-n terdiri dari n angka (digit) 5. Tunjukkan bahwa ada
diantara suku-suku barisan tersebut yang habis dibagi oleh 2011.
HARI KEDUA
1. Diberikan himpunan n bilangan asli yang pertama. Jika salah satu bilangan dihapus, maka rata-
rata bilangan yang tersisa adalah . Tentukan bilangan yang dihapus tersebut.

2. Ipin dan Upin melakukan permainan Tic Tac Toe dengan sebuah papan berukuran 3 x 3. Ipin
mendapat giliran pertama dengan memainkan X. Upin memainkan O. Mereka harus mengisi
tanda X atau O pada papan catur secara bergantian. Pemenang pada permainan ini adalah orang
pertama yang berhasil menyusun tanda secara horizontal, vertical, atau diagonal. Tentukan
banyak posisi akhir yang mungkin, jika Ipin menang pada langkah ke-4. Sebagai contoh, salah
satu posisi akhir adalah seperti gambar di bawah ini.

3. Bilangan 1 sampai 10 disusun pada segilima sehinga jumlah tiga bilangan pada setiap sisi adalah
sama. Sebagai contoh, pada gambar di samping jumlah tiga bilangan tersebut adalah 16. Untuk
semua susunan yang mungkin, tentukan nilai terbesar dan terkecil dari jumlah tiga bilangan
tersebut.

4. Didefinisikan
( ) ∑ ( ) ( ) ( ) ( ) . Selidiki apakah ada bilangan bulat
positip m dan n yang memenuhi S(m) + S(n) + S(m + n) = 2011
5. Perhatikan kubus ABCD.EFGH dengan panjang rusuk 2 satuan. Titik A, B, C, dan D terletak
pada bidang sisi bagian bawah. Titik I merupakan titik perpotongan garis diagonal pada bidang
sisi atas. Selanjutnya dibuat limas I.ABCD. Jika limas I.ABCD dipotong oleh bidang diagonal
yang menghubungkan titik-titik A, B, G, dan H, tentukan volume limas terpancung bagian bawah.
OLIMPIADE SAINS NASIONAL SMP
SELEKSI TINGKAT NASIONAL
TAHUN 2010

HARI PERTAMA
1. Sebuah pecahan disebut Toba-n pecahan itu mempunyai pembilanag 1 dan penyebut . Jika
adalah jumlah dari semua pecahan Toba-101, Toba-102, Toba-103, sampai dengan Toba-200,

tunjukkan bahwa .

2. Jika dan memenuhi sistem persamaan

{
Tentukan nilai ( )
3. Diketahui segitiga . Jika titik terletak di tengah-tengah , titik terletak di tengah-
tengah , dan titik adalah sebarang titik pada . Tentukan luas segitiga .

4. Diberikan aturan pergerakan suatu partikel pada bidang datar sebagai berikut.
( ) ( )
( ) ( )
Dengan dan adalah bilangan-bilangan bulat.
Berapa banyak lintasan berbeda dari ( ) ke ( ) dengan menggunakan aturan di atas?
5. Andra dan Dedi bermain “SUPER-AS”. Aturan permainan ini sebagai berikut. Pemain
bergantian mengambil kelereng dari suatu kaleng yang berisi 30 kelereng. Untuk setiap
pengambilan, pemain dapat mengambil paling sedikit 1 dan paling banyak 6 kelereng. Pemain
yang mengambil kelereng terakhir dinyatakan sebagai pemenang. Jika Andra memulai
permainan dengan mengambil 3 kelereng terlebih dahulu, tentukan berapa kelereng yang harus
diambil oleh Dedi dan bagaimana strategi pengambilan selanjutnya agar Dedi bisa menjadi
pemenang.
HARI KEDUA
1. Jika , tunjukkan bahwa

( )( )( )
2. Tentukan bentuk paling sederhana dari

3. Diketahui dan adalah dua jajargenjang. Titik terletak pada dan titik
terletak pada . Luas adalah 20 satuan. adalah titik pada sehingga tegak
lurus . Jika panjang adalah 5 satuan, tentukan panjang .

4. Setiap ruangan pada gambar berikut akan dicat sehingga setiap dua ruangan yang terhubung
langsung dengan pintu diberi warna berbeda. Jika disediakan 10 macam warna yang berbeda
dan 4 warna di antaranya tidak boleh digunakan berdekatan untuk dua ruangan yang terhubung
langsung dengan pintu, tentukan banyak cara berbeda mewarnai 4 ruangan tersebut.

5. Lantai suatu aula berbentuk persegi panjang dengan meter meter.


Seekor kucing berada diposisi . Melihat ada kucing, seekor tikus yang berada di tengah-
tengah lari dan mencoba menghindari kucing. Tikus berlari dari tempatnya menuju titik
dengan kecepatan meter/detik. Lintasannya berbentuk garis lurus. Melihat tikus melarikan
diri, dalam waktu bersamaan dari titik kucingpun mengejar dengan kecepatan 5 meter/detik.
Jika lintasan kucing juga berbentuk garis lurus dan tikus tertangkap sebelum sampai di ,
tentukan persamaan yang bisa digunakan untuk menentukan posisi dan waktu tikus tertangkap
kucing tersebut.
OLIMPIADE SAINS NASIONAL SMP
SELEKSI TINGKAT NASIONAL
TAHUN 2009

HARI PERTAMA
1. Sebuah persamaan kuadrat memiliki akar-akar bilangan asli a dan b. Persamaan kuadrat lainnya
memiliki akar-akar b dan c dengan a ¹ c . Jika a, b, dan c merupakan bilanganbilangan prima yang
kurang dari 15, ada berapa macam pasangan yang mungkin memenuhi syarat tersebut (dengan
syarat koefisien dari suku kuadratnya sama dengan 1)?
2. Di Indonesia, dahulu dikenal pecahan yang disebut “Pecahan Nusantara”. Pecahan Nusantara
adalah pecahan demikian sehingga a dan b adalah bilangan-bilangan asli dan a < b . Tentukan

jumlah semua pecahan nusantara mulai dari pecahan dengan b = 2 sampai dengan b = 1000.
3. Perhatikan gambar berikut. Huruf-huruf a, b, c, d, dan e di dalam kotak akan diganti dengan angka-
angka dari 1, 2, 3, 4, 5, 6, 7, 8, atau 9, dengan syarat a, b,c, d, dan e harus berlainan. Jika diketahui
ae = bd, ada berapa susunan yang mungkin terjadi?

4. Diketahui segitiga ABC dengan A sebagai puncak dan BC sebagai alas. Titik P terletak pada sisi
CA. Dari titik A ditarik garis sejajar PB dan memotong perpanjangan alas di titik D. Titik E
terletak pada alas sehingga CE : ED = 2 : 3. Jika F adalah tengah-tengah antara E dan C, dan luas
segitiga ABC sama dengan 35 cm2, berapakah luas segitiga PEF?
5. Setiap sisi suatu kubus dituliskan sebuah bilangan asli. Setiap titik sudutnya diberi nilai yang
merupakan hasil kali dari tiga bilangan pada tiga sisi yang berpotongan di titik sudut tersebut. Jika
jumlah semua bilangan pada titik-titik sudut tersebut sama dengan 1001, tentukan jumlah semua
bilangan yang dituliskan pada sisi-sisi kubus tersebut.
HARI KEDUA
1. Suatu nomor telepon dengan 7 angka disebut Nomor Cantik bilamana angka-angka yang muncul
pada tiga angka pertama (ketiganya harus berbeda) berulang pada pada tiga angka berikutnya atau
tiga angka terakhirnya. Contoh beberapa Nomor Cantik: 7133719, 7131735, 7130713, 1739317,
5433354. Jika angka-angkanya diambil dari 0, 1, 2, 3, 4, 5, 6, 7, 8 atau 9, tetapi angka pertama
tidak boleh 0, berapakah banyaknya Nomor Cantik yang bisa diperoleh.
2. Tentukan banyaknya bilangan asli n demikian sehingga n3 +100 terbagi habis oleh n +10 .
3. Suatu fungsi f didefinisikan seperti pada tabel berikut.

Berdasarkan definisi fungsi f di atas, selanjutnya didefinisikan suatu barisan bilangan dengan
rumus umum suku-sukunya sebagai berikut. dan ( ), untuk
4. Pada suatu segitiga ABC, titik D terletak pada sisi AB dan titik E terletak pada sisi AC. Tunjukkan
bahwa :

5. Pada turnamen catur, seorang pemain hanya bermain satu kali dengan pemain lainnya. Seorang
pemain memperoleh nilai 1 menang, 0 jika kalah, dan ½ jika imbang. Setelah kompetisi berakhir,
diketahui bahwa ½ total nilai yang diperoleh oleh setiap pemain didapatkan dari bermain dengan
10 pemain yang mendapatkan total poin terendah. Khusus untuk yang berada di peringkat sepuluh
terbawah, ½ total nilai yang diperolehnya didapatkan dari bermain dengan 9 pemain lainnya.
Berapakah banyaknya pemain dalam kompetisi tersebut?
OLIMPIADE SAINS NASIONAL SMP
SELEKSI TINGKAT NASIONAL
TAHUN 2008

HARI PERTAMA
1. Lingkaran M adalah lingkaran dalam dari ABC, sedangkan lingkaran N merupakan lingkaran
dalam dari ACD . Lingkaran M dan N bersinggungan di titik E. Jika panjang sisi AD = x cm , AB
= y cm, BC = z cm, tentukan panjang sisi DC (nyatakan dalam x, y dan z.)

2. Alamat rumah di Jalan Bahagia hendak diberi nomor dengan aturan sebagai berikut. Satu sisi
jalan dinomori dengan nomor bilangan genap berurutan mulai dari nomor 2. Sisi seberangnya
dinomori dengan nomor ganjil mulai dari nomor 3. Pada deretan rumah bernomor genap, terdapat
beberapa tanah kosong yang belum dibangun rumah. Rumah pertama yang bernomor 2 memiliki
tetangga di sebelahnya. Pada waktu pengurus RT memesan nomor-nomor rumah tersebut,
diketahui biaya pembuatan setiap digitnya adalah Rp. 12.000,-. Untuk itu, total biaya yang harus
dikeluarkan adalah Rp. 1.020.000. Diketahui pula bahwa biaya seluruh nomor rumah sisi genap
adalah Rp. 132.000,- lebih murah dibanding soso ganjil. Apabila tanah kosong nanti sudah
dibangun rumah, banyaknya rumah di sisi genap dan ganjil adalah sama. Tentukan banyaknya
rumah yang sekarang telah ada di Jalan Bahagia tersebut.

3. Diberikan suatu soal berikut: " Setiap unsur dalam himpunan A = 10, 11, 12 ,…, 2008 dikalikan
dengan setiap unsur dalam himpunan B = 21, 22, 23, … , 99. Hasil-hasil kali itu dijumlahkan
sehingga memberikan nilai X. Tentukan nilai X ". Seseorang menjawab soal tersebut dengan cara
mengalikan 2016991 dan 4740. Bagaimana kalian bisa menjelaskan bahwa cara orang itu masuk
akal?
4. Misalkan P adalah himpunan semua bilangan bulat positif antara 0 dan 2008 yang dapat
dinyatakan sebagai jumlah dari dua bilangan bulat positif berurutan atau lebih. (Contoh: 11 = 5 +
6,90 = 29 + 30 + 31, 100 = 18 + 19 + 20 + 21 + 22. Jadi 11, 90, 100 adalah beberapa anggota P)
Tentukan jumlah dari semua anggota P!
5. Bilangan empat angka akan dibentuk dari angka - angka pada 0, 1, 2, 3, 4, 5 dengan syarat angka-
angka pada bilangan tersebut tidak berulang, dan bilangan yang terbentuk merupakan kelipatan 3.
Berapakah peluang bilangan yang terbentuk mempunyai nilai kurang dari 3000?
HARI KEDUA
1. Misalkan himpunan
A = x, y  3x  5 y  15, x  y 2  25, x  0, x, y bilangan bulat. Tentukan semua pasangan (x,zx)  A

dengan syarat z merupakan bilangan bulat tidak nol.


2. Seorang pemilik toko menginginkan bisa menimbang berbagai macam berat benda (dalam
bilangan asli) hanya dengan 4 anak timbangan yang berbeda. (Sebagai contoh, jika dia memiliki
anak timbangan 1, 2, 5 dan 10. Dia bisa menimbang berat 1 kg, 2 kg, 3 kg (1 + 2), 4 kg (5 - 1), 5
kg, 6 kg, 7 kg, 8 kg, 9 kg(10 - 1), 10 kg, 11 kg, 12 kg, 13 kg(10 + 1 + 2), 14 kg(10 + 5 - 1), 15 kg,
16 kg, 17 kg dan 18 kg). Kalau dia ingin bisa menimbang semua berat dari 1 kg sampai dengan 40
kg, tentukan empat anak timbangan yang harus dimilikinya. Berikan penjelasan bahwa jawaban
kalian benar !
3. Perhatikan tabel berikut !

Tabel 4 x 4 ini merupakan gabungan empat bagian tabel yang lebih kecil berukuran 2 x 2. Tabel
ini akan diisi dengan empat bilangan bulat berbeda sedemikian sehingga:
 Jumlah mendatar bilangan - bilangan pada setiap barisnya sama dengan 10.
 Jumlah vertikal bilangan - bilangan pada setiap kolomnya sama dengan 10.
 Jumlah empat bilangan dalam setiap bagian 2 x 2 yang dibatasi garis tebal tersebut juga sama
dengan 10.
Tentukan berapa banyaknya susunan yang mungkin terbentuk.
4. Suatu barisan bilangan real mempunyai suku-suku didefinisikan sebagai berikut.
U n  a.r n 1 , jika n = 4m – 3 atau n = 4m – 2 jika n = 4m – 1 atau n = 4m dengan a> 0, r > 0, dan

m bilangan bulat positif. Buktikan bahwa jumlah semua suku ke-1 sampai dengan suku ke-2009

adalah.

a 1  r  r 2009  r 2010 
1 r 2

5. Kubus ABCD. EFGH dipotong menjadi empat bagian oleh dua bidang. Bidang pertama sejajar
dengan sisi ABCD dan melalui titik tengah rusuk BF. Bidang kedua melalui titik-titik tengah AB,
AD, GH, dan FG. Tentukan rasio volume dari bagian ruang yang terkecil dan bagian yang
terbesar.
OLIMPIADE SAINS NASIONAL SMP
SELEKSI TINGKAT NASIONAL
TAHUN 2007
HARI PERTAMA
1. Satu set kartu memuat 100 kartu masing-masing ditulisi bilangan dari 1 sampai dengan 100. Pada
setiap dua sisi kartu ditulis bilangan yang sama, sisi pertama berwarna merah dan sisi yang lain
berwarna hijau. Pertama Leny menyusun semua kartu dengan tulisan mearah menghadap ke atas.
Kemudia Leny melakukan tiga langkah berikut ini :
I. Membalik semua kartu yang nomornya habis dibagi 2
II. Membalik lagi semua kartu yang nomornya habis dibagi 3
III. Membalik lagi semua kartu yang nomornya habis dibagi 5, namun tidak membalik semua
kartu yang nomornya habis dibagi 5 dan 2.
Tentukan banyaknya kartu Leny sekarang yang bernomor yang berwarna merah dan menghadap
ke atas.
2. Hitunglah luas daerah dari tiga daerah setengah lingkaran yang beririrsan seperti tampak pada
gambar berikut.

1 Ax 2 5
3. Diketahui bahwa x   7 . Tentukan nilai A agar 4 
x x  x 1 6
4. Ada 13 kado yang berbeda yang dibagikan semua kepada Ami, Ima, Mai, dan Mia. Jika Ami
mendapat paling sedikit 4 kado, Ima dan Mai masing-masing mendapat paling sedikit 3 kado, dan
Mia mendapat paling sedikit 2 kado, ada berapa banyak susunan kado yang mungkin diperoleh?
5. Suatu bilangan asli disebut bilangan kuaprim jika memenuhi keempat syarat berikut.
I. Tidak memuat angka nol.
II. Angka-angka penyusun itu berbeda.
III. Satu angka pertama dan satu angka terakhir merupakan bilangan prima atau bilangan
kuadrat.
IV. Setiap pasang angka berurutan membentuk bilangan prima atau bilangan kuadrat.
Sebagai contoh, kita periksa bilangan 971643.
I. 971643 tidak memuat angka nol.
II. Angka-angka penyusun 971643 berbeda.
III. Satu angka pertama dan satu angka terakhir dari 971643, yaitu 9 dan 3 merupakan bilangan
prima atau bilangan kuadrat.
IV. Setiap pasang angka berurutan, yaitu 97, 71, 16, 64, dan 43 membentuk bilangan prima atau
bilangan kuadrat.
Jadi 971643 merupakan bilangan kuaprim
a. Carilah bilangan kuaprim 6-angka paling besar.
b. Carilah bilangan kuaprim 6-angka paling kecil.
c. Angka berapa yang tidak pernah termuat dalam sebarang bilangan kuaprim? Jelaskan.

HARI KEDUA
1. Empat bangun berbentuk laying seperti gambar berikut (a > b, a dan b bilangan asli kurang dari
10) ditata sedemikian rupa sehingga membentuk persegi dengan lubang berbentuk persegi pula di
tengah-tengahnya. Lubang berbentuk persegi di tengah-tengah tersebut memiliki keliling 16
satuan panjang. Berapakah keliling yang mungkin diperoleh dari persegi terluar yang terbentuk
jika diketahui pula bahwa a dan b adalah bilangan-bilangan yang relative prima.
2. Jika a = 3p, b = 3q, c = 3r, dan d = 3s dan p, q, r, dan s adalah bilangan asli. Berapakah nilai
terkecil dari p x q x r x s yang memenuhi a2 + b3 + c5 = d7 ?
3. Ucok bermaksud menyusun suatu kode kunci (password) yang terdiri atas 8 angka dan memenuhi
ketentuan berikut :
a. Angka yang dipakai adalah 1, 2, 3, 4, 5, 6, 7, 8, dan 9.
b. Angka pertama yang dipakai adalah minimal 1, angka kedua minimal 2, angka ketiga minimal
3, dan seterusnya.
c. Angka yang sama bisa digunakan beberapa kali.
a. Berapa banyak password berbeda yang mungkin disusun Ucok?
b. Berapa banyak password berbeda yang mungkin disusun Ucok, jika ketentuan (iii) diganti
dengan : tidak boleh ada angka yang digunakan lebih dari satu kali.
4. Untuk sebarang bilangan bulat a, b, dan c berlaku
a x (b + c) = (a x b) + (a x c)
a. Cari contoh yang menunjukkan bahwa
a + (b x c)  (a + b) x (a + c)
b. Kapan berlaku
a + (b x c) = (a + b) x (a + c)?
Jelaskan jawaban Anda.
5. Hasil survey terhadap N orang dengan pertanyaan apakah mereka memelihara anjing, burung,
atau kucing di rumah adalah sebagai berikut : 50 orang memelihara burung, 61 orang tidak
memelihara anjing, 13 orang tidak memelihara kucing, dan paling sedikit ada 74 orang yang
memelihara paling sedikit 2 jenis binatang di rumah. Berapa nilai maksimum dan minimum dari
nilai N yang mungkin?
OLIMPIADE SAINS NASIONAL SMP
SELEKSI TINGKAT NASIONAL
TAHUN 2006
HARI PERTAMA
1. Diketahui N = 9 + 99 + 999 + … + ⏟ .

Tentukan nilai N.
2. Segitiga ABC pada gambar berikut ini adalah sama kaki, dengan AB = AC = 90 cm dan BC = 108
cm. Titik P dan Q masing – masing terletak pada BC sedemikian sehingga BP : PQ : QC = 1 : 2 :
1. Titik S dan R berturut – turut terletak tepat ditengah AB dan AC. Dari kedua titik ini masing –
masing ditarik garis tegak lurus terhadap PR sehingga memotong di PR di titik M dan N. Tentukan
panjang MN.
3. Apabila delapan segitiga sama sisi yang sisinya 12 cm disusun seperti pada gambar berikut,
diperoleh suatu jaring – jarring octahedron. Tentukan volume dari octahedron tersebut.
4. Diketahui a2 + b2 = 1 dan x2 + y2 = 1.
lanjutkan proses aljabar berikut
(a2 + b2)( x2 + y2) –(ax + by)2 = ….
a. Hubungan apakah yang bisa disimpulkan antara ax + by dengan 1
b. Mengapa ?
5. Satu set soal terdiri dari 3 soal dengan pilihan jawaban benar (B) atau soal salah (S), serta 3 soal
pilihan ganda dengan jawaban A,B,C dan D. Seseorang menjawab semua soal secara acak.
Berapa peluang ia hanya benar 2 soal ?

HARI KEDUA
1. Dua bilangan bulat m dan n dikatakan elatif prima jika ada bilangan bulat a dan b sedemikian
sehingga am + bn = 1.
Tunjukkan bahwa untuk setiap bilangan bulat p, pasangan bilangan yang dibentuk oleh 21p + 4
dan 14p + 3 senantiasa relative prima.
2. Dua orang petani, Si A dan Si B bermaksud mengubah batas tanah mereka sehingga menjadi
seperti garis lurus, tidak berbelok – belok seperti pada gambar dibawah. Mereka tidak ingin luas
daerah asalnya berkurang. Coba tentukan garis batas yang seharusnya mereka sepakati, dan
jelaskan mengapa batas baru tersebut tidak mengurangi luas daerah asalnya masing – masing.
3. Diketahui system persamaan empat variable :
23x + 47y – 3z = 434
47x – 23y – 4w = 183
19z + 17w = 91
Dengan x, y, z, dan w adalah bilangan bulat positif.
Tentukan nilai dari
(13x – 14y)3 – (15z + 16w)3
4. Seseorang mengendarai kendaraan bermotor sehingga diperoleh grafik bahan bakar yang
digunakannya sebagai berikut :

Sisa bahan
bakar

7
5
3
1
Lama perjalanan dalam jam

0 2 6
Mula – mula kendaraannya berisi 3 liter bahan bakar. Setelah dua jam perjalanan bahan bakarnya
tersisa 1 liter.
a. Jika dalam 1 liter dia bisa menempuh jarak sejauh 32 km, berapakah jarak yang ditempuhnya
secara keseluruhan. Jelaskan mengapa anda menjawab seperti itu ?
b. Sesudah dua jam perjalanan, apakah terjadi percepatan atau perlambatan ? Jelaskan jawab
anda !
c. Tentukan berapa kecepatan rata – rata kendaraan tersebut !
5. Amir akan membuat lukisan dari lingkaran – lingkaran yang setiap lingkarannya diisi dengan
bilangan. Lukisan lingkaran tersebut disusun mengikuti pola berikut.

Dia membuat aturan bahwa empat lingkaran terbawah akan diisi dengan bilangan – bilangan
positif kurang dari 10 yang dapat diambil dari angka – angka pada tanggal kelahirannya, yakni 26
– 12 – 1961, tanpa berulang. Sementara itu, lingkaran – lingkaran diatasnya akan diisikan dengan
bilangan – bilangan yang merupakan hasil kali dua bilangan pada lingkaran – lingkaran
dibawahnya
a. Ada berapa carakah dia menempatkan bilangan – bilangan itu dari kiri ke kanan pada
lingkaran – lingkaran terbawah agar diperoleh nilai terbesar pada lingkaran yang paling atas?
Jelaskan.
b. Pada kesempatan yang lain, dia berencana memasukkan semua angka pada tanggal
kelahirannya tersebut sehingga jumlah lingkaran terbawah sekarang harus sebanyak 8
lingkaran. Dia tidak lagi memperhatikan berulang tidaknya bilangan – bilangan itu
1. Agar diperoleh nilai terkecil pada lingkaran yang paling atas, bagaimanakah bilangan –
bilangan itu disusun
2. Ada berapa susunan yang patut dipertimbangkan untuk menghasilkan nilai terkecil ?
OLIMPIADE SAINS NASIONAL SMP
SELEKSI TINGKAT NASIONAL
TAHUN 2005

HARI PERTAMA
1. A adalah suatu himpunan bilangan. Himpunan A memiliki sifat tertutup terhadap pengurangan,
artinya hasil pengurangan dua bilangan di A akan menghasilkan bilangan di A juga. Jika
diketahui dua anggota dari A adalah 4 dan 9, tunjukkan bahwa :
a. 0  A
b. –13  A
c. 74  A
d. selanjutnya, daftarlah semua anggota himpunan A.
2. Pasangan empat bilangan (2, 0, 4, 1) adalah salah satu selesaian/jawab dari x1 + x2 + x3 + x4 = 7.
Jika semesta pembicaraan pada persamaan ini adalah himpunan semua bilangan bulat tidak
negatif, tentukan banyak selesaian/jawab yang mungkin dari x1 + x2 + x3 + x4 = 7!
3. Adi adalah karyawan pada salah satu perusahaan tekstil yang bertugas menyimpan data. Suatu
ketika Adi diminta pimpinan perusahaan untuk menyiapkan data tentang kenaikan produksi
selama lima periode. Setelah dicari Adi hanya menemukan empat kenaikan data, yaitu 4%, 9%,
7%, dan 5%. Satu data lagi, yaitu data ke–5, tidak ditemukan. Selidiki data kenaikan produksi ke–
5, bila Adi hanya ingat bahwa rata–rata hitung dan median dari lima data tersebut adalah sama!
4. Tentukan semua pasangan bilangan bulat (x,y) yang memenuhi system persamaan berikut
x(y + 1) = y2 – 1
y(x + 1) = x2 – 1
5. Diketahui gambar berikut. ABCD adalah persegi, dan E adalah titik sembarang di luar persegi
ABCD. Selidiki apakah berlaku hubungan
AE2 + CE2 = BE2 + DE2 pada gambar di bawah!

D C

A B
HARI KEDUA
1 1
1. Di antara bilangan dan terdapat tak hingga banyak bilangan pecahan. Tentukan 999
5 4
1 1
bilangan pecahan di antara dan sehingga selisih antara bilangan pecahan berikutnya dengan
5 4
bilangan pecahan sebelumnya konstan!
(maksudnya : jika x1, x2, x3, x4, x5, …, xn) adalah bilangan pecahan yang dimaksud, maka x1 – x2
= x3 – x2 = … = xn – xn–1 = … = x999 – x998).
2. Pola pada gambar–gambar di bawah adalah :”Gambar berikutnya diperoleh dengan menambah
gambar segitiga sama sisi berwarna hitam yang ukuran sisinya setengah dari sisi masing–masing
segitiga warna putih yang tersisa pada gambar sebelumnya”. Misalkan pola tersebut berkelanjutan
(kontinu) sampai tak hingga.

1 2 3 4
a. Jika diketahui bahwa segitiga sama sisi pada Gb–1 adalah 1 satuan luas, tentukan luas
keseluruhan daerah yang dibentuk oleh segitiga–segitiga hitam pada Gb–5
b. Andaikata anda diminta untuk menemukan luas keseluruhan daerah yang dibentuk oleh
segitiga–segitiga hitam pada gambar ke–20, rumus yang bagaimanakah yang dapat anda
gunakan?
3. Untuk setiap pasangan bilangan asli a dan b, kita definisikan a*b = ab + a – b. Bilangan asli x
dikatakan penyusun bilangan asli n jika terdapat bilangan asli y yang memenuhi x*y = n. Sebagai
contoh, 2 adalah penyusun 6 karena terdapat bilangan asli 4 sehingga 2*4 = 2.4 + 2 – 4 = 8 + 2 –
4 = 6. Tentukan semua penyusun 2005
4. Tiga orang hendak makan siang di rumah makan. Untuk menentukan siapa yang membayar
mereka membuat suatu permainan. Masing–masing mengetos satu koin secara bersama–sama.
Jika hasilnya muka semua atau belakang semua, maka mereka mengetos lagi. Jika tidak demikian,
maka “orang ganjil” (yaitu orang yang koinnya muncul berbeda dari dua orang lainnya) yang
membayar. Tentukan banyaknya semua hasil yang mungkin jika permainan berakhir pada
pengetosan :
a. pertama
b. kedua
c. ketiga
d. kesepuluh
5. Diketahui bentuk x2 + 3y2 = n, dengan x dan y adalah bilangan bulat.
a. Jika n < 20, bilangan berapa sajakah n tersebut, dan diperoleh dari pasangan (x,y) apa saja?
b. Tunjukkan bahwa tidak mungkin menghasilkan x2 + 3y2 = 8!
OLIMPIADE SAINS NASIONAL SMP
SELEKSI TINGKAT NASIONAL
TAHUN 2004

HARI PERTAMA
1. Diketahui titik A(–1,2), B(0,2), C(3,0), dan D(3,–1) seperti terlihat pada gambar berikut.
Tentukan besar sudut AOD!

A B

C
O
D

2. Tentukan semua bilangan prima p > 2 sehingga p membagi 712 – 372 – 51!
3. Suatu bola jika dijatuhkan tegak lurus ke tanah dari suatu ketinggian maka ia akan memantul
kembali tegak lurus sepanjang sepertiga tinggi semula, turun kembali tegak lurus dan memantul
kembali sepertiga tingginya, dan seterusnya. Jika jarak yang ditempuh bola tersebut pada saat
menempuh tanah yang keempat sama dengan 106 m, dari ketinggian berapakah bola tersebut
dijatuhkan?
4. Balok ABCD.EFGH diperoleh dengan menempelkan dua kubus satuan ABCD.PQRS dan
PQRS.EFGH. Titik K adalah titik tengah rusuk AB, sedangkan titik L adalah titik tengah rusuk
SH. Berapakah panjang ruas garis KL?
5. Ada berapa banyakkah bilangan asli yang tidak lebih besar dari 2004 yang bersisa 1 ketika dibagi
2, bersisa 2 ketika dibagi 3, bersisa 3 ketika dibagi 4, dan bersisa 4 ketika dibagi 5?

HARI KEDUA
1. Sebuah dadu biasa 6–muka dilempar 3 kali. Berapakah besarnya kemungkinan jumlah mata dadu
pada ketiga lemparan adalah 12?
1 1
2. Diberikan dua bilangan real positif x dan y sehingga xy = 1. tentukan nilai minimum  !
x 4 4y 4
3. Diketahui rangkaian persegi yang disusun secara kesinambungan dan membentuk sudut–sudut
sebagaimana pada gambar berikut. Tentukan nilai sudut yang diberikan tanda x!

400 1250 350

4. Tentukan bilangan asli n terkecil sehingga jumlah ukuran sudut–sudut segi–n, n > 6 kurang dari n2
derajat!
5. Ada suatu kartu ajaib. Dengan menyebut di kartu yang mana suatu bilangan berada, tanpa melihat
kartu sama sekali, seseorang dengan tepat bisa menebak bilangan yang dimaksud. Kalau bilangan
tersebut pada kartu A dan B, maka bilangan yang dimaksud 1 + 2 (jumlah bilangan pada pojok
kiri atas) kartu A dan B. Kalau bilangan tersebut ada di A, B, dan C, bilangan yang dimaksud 1 +
2 + 4 atau sama dengan 7 (yang diperoleh dengan menambahkan bilangan–bilangan di pojok kiri
atas masing–maisng kartu A, B dan C).

A B C

1 3 2 3 4 5

5 7 6 7 6 7

A. Bagaimana hal ini bisa dijelaskan?


B. Misalkan kita akan membuat kartu yang memuat bilangan dari 1 sampai dengan 15
berdasarkan aturan di atas. Coba buatkan kartu–kartunya!
OLIMPIADE SAINS NASIONAL SMP
SELEKSI TINGKAT NASIONAL
TAHUN 2003

HARI PERTAMA
1. Pola ABBCCCDDDDABBCCCDDDDABBCCC….
Berulang sampai tak terhingga. Huruf apakah yang menempati urutan 2533?
2. Buktikan bahwa jika a > 2 dan b > 3, maka ab + 6 > 3a + 2b
3. Diberikan persegi panjang ABCD dengan ukuran 16 cm  25 cm, EFBG layang–layang, dan AE
= 5 cm. Tentukan panjang EF!
A B

D C
F

4. Perhatikan kumpulan pernyataan berurut berikut.


Diketahui bahwa x = 1
Karena x = 1 maka x2 = 1
Sehingga x2 = x
Akibatnya
x2 – 1 = x – 1
(x – 1)(x + 1) = (x – 1) . 1
dengan aturan pencoretan, diperoleh
x+1=1
1+1=1
2=1
Pernyataannya,
a. Kalau 2 = 1, maka setiap bilangan asli sama dengan 1. Tunjukkan!
b. Hasil 2 = 1 adalah sesuatu yang tidak mungkin. Tentu ada yang salah di dalam argumen di
atas. Dimanakah letak kesalahannya? Mengapa itu kamu anggap salah?
5. Untuk menghitung (1998)(1996)(1994)(1992) 16 ,

seseorang melakukannya dengan cara yang sederhana sebagai berikut : 20002 – 2  5  2000 + 52
– 5? Apakah cara yang dilakukan orang itu dapat dibenarkan? Mengapa?
6. Untuk menarik minat pelanggan, suatu restoran penjual makanan cepat saji memberikan kupon
berhadiah kepada setiap orang yang membeli makanan di restoran tersebut dengan nilai lebih dari
Rp 25.000,00. Di balik setiap kupon tersebut tertera salah satu dari bilangan–bilangan berikut : 9,
12, 42, 57, 69, 21, 15, 75, 24, dan 81. Pembeli yang berhasil mengumpulkan kupon dengan
jumlah bilangan di balik kupon tersebut sama dengan 100 akan diberi hadiah berupa TV 21”.
Kalau pemilik restoran tersebut meyediakan sebanyak 10 buah TV 21”, berapa banyak yang harus
diserahkan kepada pelanggannya?
7. Diketahui bentuk gambar di bawah ini.
Titik–titik pusat B, C, D, dan E diletakkan pada garis tengah lingkaran A dan garis tengah
lingkaran B sama dengan jari–jari lingkaran A. Lingkaran C, D dan E sama besar dan sepasang–
sepasang bersinggungan di luar sehingga jumlah panjang garis tengah ketiga lingkaran tersebut
sama dengan jari–jari lingkaran A.

A
D
B

C E

Bagaimanakah perbandingan lingkaran A dengan jumlah keliling lingkaran B, C, D, dan E?


8. Diketahui a + b + c = 0. Tunjukkan bahwa a3 + b3 + c3 = 3abc!

HARI KEDUA
1 1
1. Diketahui a1 = 2, a2 = 3. Untuk k > 2 didefinisikan bahwa ak  ak 2  ak 1 . Tentukan jumlah tak
2 3

hingga dari a1 + a2 + a3 + … !
2. Bilangan terkali adalah bilangan asli dalam bentuk dua digit diikuti oleh hasil kalinya. Sebagai
contoh 7  8 = 56, maka 7856 dan 8756 adalah bilangan terkali. 2  0 = 0, maka 200 adalah
bilangan terkali. Sebagai catatan, digit pertama bilangan terkali tidak boleh 0.
a. Berapakah selisih bilangan terkali terbesar dengan bilangan terkali terkecil?
b. Carilah semua bilangan terkali terdiri dari 3 digit yang masing–masing digitnya merupakan
bilangan kuadrat!
c. Diberikan kotak–kotak berikut yang harus diisi dengan bilangan terkali.

Tentukan isi dari kotak yang diarsir. Apakah isi ini merupakan satu–satunya?
d. Lengkapi semua kotak kosong di atas dengan bilangan terkali!

3. Perhatikan susunan tiga persegi di bawah


Buktikan bahwa BAX + CAX = 45

B C

A x

4. Buktikan bahwa (n – 1) n (n3 + 1) senantiasa habis dibagi oleh 6 untuk semua bilangan asli n!
KUNCI JAWABAN SOAL OSN
OLIMPIADE MATEMATIKA

OLIMPIADE SAINS NASIONAL (OSN) SMP 2003 – 2018


TINGKAT KABUPATEN/ KOTA,PROPINSI & NASIONAL

0 KDR FAIZAL AHMAD AL FAIZY | ERICK INSTITUTE INDONESIA


KUNCI SOAL OSK MATEMATIKA SMP TAHUN 2003 - 2018

OSK SMP TAHUN 2018

BAGIAN PERTAMA

1. D 13. B
2. C 14 C
3. C .15 A
4. C 16. B
5. D 17. C
6. D 18 . C
7. B 19 .B
8. B 20. C
9. B 21 C
10. C 22. C

11 D 23 B

12 A 24 B

25 D

OSK SMP TAHUN 2017 BAGIAN KEDUA


BAGIAN PERTAMA 1. k = 97
1. A 2.
2. B 3. Jadi dari persamaan (5) diperoleh:
3. B
4. B 4. Luas CDE = Luas ABC = 6
5. B 5.
6. A
7. B
8. D
9. D
10. B
OSK SMP TAHUN 2016
BAGIAN PERTAMA BAGIAN KEDUA
1. A 1. 4/9
2. C 2. 26
3. B 3.
4. B 4. 216
5. B 5.
6. D 6. 2017
7. A
7. 5100
8. D
8. 71
9. B
9. 420
10. B
10. 60
11. B
12. D
13. C
14. B
15. C

OSK SMP TAHUN 2015


BAGIAN PERTAMA BAGIAN KEDUA
1. D 1. – 1 atau -4
2. B 2. 12
2
3. C 3. √ cm
4. B 4. 55:153
5. B 5. 19
6. C 6. 10080
7. B 7. 1:2
8. D 8. 28 cara
9. A 9. A (-8,6), B (-8,10) dan C (-4,6)
10. C 10. 61608 cara
11. B
12. D
13. C
14. B
15. D
OSK SMP TAHUN 2014
BAGIAN PERTAMA BAGIAN KEDUA
1. A 1. 26
2. C 2. 35
3. A 3. 566 batang
4. B 4. 688
5. C 5.
6. B 6. 505
7. D 7. 254
8. D 8. 1/320
9. D 9. 4√
10. D 10. 2240 cara
11. B
12. C
13. C
14. C
15. D
16. B
17. C
18. B
19. C
20. B

OSK SMP TAHUN 2013


BAGIAN PERTAMA BAGIAN KEDUA
1. A 1. 17
2. B 2. 1
3. C 3.
4. E 4. -3
5. B 5. 2x atau 2y
6. B 6. 1
7. C 7. 2013
8. E 8. 3:4
9. E 9. 615
10. 6 (tidak ada dipilihan jawaban) 10. 2/3
11. E
12. E
13. B
14. D
15. D
16. C
17. A
18. A
19. B
20. A
OSK SMP TAHUN 2012
BAGIAN PERTAMA BAGIAN KEDUA
1. C 1. 4021; 4023; 4024; 4025; 4027
2. B 2. 1
3. A 3. √
4. B 4. x=1 atau x=3
5. C 5. 1013
6. E 6. 125 meter
7. E 7. 48
8. B 8. 1/2
9. B 9. 3/5√ cm
10. B 10. 27
11. D
12. C
13. B
14. E
15. A
16. A
17. A
18. D
19. E
20. C

OSK SMP TAHUN 2011


BAGIAN PERTAMA BAGIAN KEDUA
1. C 1. 2/5
2. E 2. 81
3. D 3.
4. B 4. 2.023.066
5. D 5. 29.701
6. A 6. (3,3) DAN (3,-3)
7. C 7. 144
8. B 8. 21
9. C 9. 0
10. A 10. 169
11. E
12. A
13. E
14. B
15. A
16. B
17. C
18. A
19. D
20. E
OSK SMP TAHUN 2010
BAGIAN PERTAMA BAGIAN KEDUA
1. D 1. 36O
2. E 2. 8
3. B 3.
4. D 4. 10
5. B 5. 1:2
6. D 6. 3
7. A 7. 4
8. A 8. 250
9. A 9. 1
10. C 10. 100 cara
11. D
12. A
13. C
14. E
15. C
16. C
17. B
18. A
19. E
20. D

OSK SMP TAHUN 2009


BAGIAN PERTAMA BAGIAN KEDUA
1. B 1. 4
2. C 2. 24
3. A 3.
4. A 4. 20
5. C 5. 1089
6. A 6. 8.01
7. B 7. 2008
8. D 8. 50
9. C 9. 242+168√
10. C atau D 10. 3
11. B
12. C
13. B
14. B
15. C
16. C
17. A
18. B
19. D
20. D
OSK SMP TAHUN 2008
BAGIAN PERTAMA BAGIAN KEDUA
1. C 1. 120.000
2. D 2. 17,5
3. B 3.
4. A 4. 0,2 jam atau 12 menit
5. B 5. m
6. C 6.
7. A 7. Tiga
8. C 8. 43210
9. D
9. √
10. C
10. 3
11. C
12. D
13. C
14. D
15. D
16. E
17. C
18. A
19. C
20. B

OSK SMP TAHUN 2007


BAGIAN PERTAMA BAGIAN KEDUA
1. B 1. 0 (tidak ada)
2. D 2. 65 km
3. D 3.
4. E 4. 57
5. E 5. 4
6. D 6.
7. B 7. 8
8. C 8. { }
9. C 9.
10. A 10. 3
11. D
12. A
13. D
14. D
15. D
16. B
17. C
18. E
19. C
20. C
OSK SMP TAHUN 2006
BAGIAN PERTAMA BAGIAN KEDUA
1. A 1. 5/4
2. C 2. 12.24
3. C 3.
4. C 4. x = 0, 2, 3, 5, 6, 8
5. B 5. 44
6. D 6. { }
7. A 7. √
8. E 8.
9. C 9.
10. B 10. √ √
11. D
12. C
13. A
14. B
15. B
16. C
17. B
18. C
19. D
20. B

OSK SMP TAHUN 2005


BAGIAN PERTAMA BAGIAN KEDUA
1. C 1. 180
2. E 2. 10 cm2
3. E 3.
4. B 4. -3,1
5. A 5. 3
6. C 6.
7. E 7.
8. C 8.
9. E 9.
10. C 10.
OSK SMP TAHUN 2004
BAGIAN PERTAMA BAGIAN KEDUA
1. C 1. 124O
2. C 2. Hari Minggu
3. C 3. dan 5
4. D 4.
5. E 5. 308 liter
6. E 6.
7. B 7.
8. A 8.
9. A 9.
10. E 10.

OSK SMP TAHUN 2003


BAGIAN PERTAMA BAGIAN KEDUA
1. B atau C 1. 41312432
2. B 2. 105
3. D 3.
4. B 4.
5. A 5. 10 segitiga
6. B 6. cm2
7. E 7. satuan luas
8. B 8. cm
9. C 9.
10. A 10. 12,5 cm
KUNCI SOAL OSP MATEMATIKA SMP TAHUN 2003 - 2016

OSP SMP TAHUN 2016


BAGIAN PERTAMA BAGIAN KEDUA
1. 4031 1.
2. a,b,c = 8,5,2 2. 140 cara
3. 2 cm 3.
4.
5. 18 cara
6. m=2
7. 6567
8.
9. 4 point
10. 18

OSP SMP TAHUN 2015


BAGIAN PERTAMA BAGIAN KEDUA
1. 60 1. 1220 cara
2. 100 ekor 2.
3. 3.
4. 322
5. 11
6. Ke-45
7.
8. 15252 cara
1. 1977
2. Lihat gambar

OSP SMP TAHUN 2014


BAGIAN PERTAMA BAGIAN KEDUA
1. (2,141), … dst ada 15 solusi 1.
2. ½ 2. 7
3. satuan luas 3.
4. Ada 4
5. 120 cara 4. 232800
6. 192 satuan luas 5. 1006,5
7. -11
8. 2a-b-1
9. 20 fungsi
10.
OSP SMP TAHUN 2013
BAGIAN PERTAMA BAGIAN KEDUA
1. 1. A= 20 buku,
B= 36 buku,
2. 20 C= 48 buku,
3. 21 pengambilan D= 40 buku
4.
2.
5.
6. 101 3.
7. 0,7 cm 4.
8. 296 dan 305 a=3 b=11 47 53
9. 351 c=23 37 41 d=13
29 61 e=7 f=17
10. 772
59 g=5 h=19 31

5. 5

OSP SMP TAHUN 2012


BAGIAN PERTAMA BAGIAN KEDUA
1. 1. x=0
2. 6 2.
3.
1. 13 cm
4. 2/35
2. 9933
5. 3. 128
6. 41325
7. 62
8. 18.184.525
9.
10.

OSP SMP TAHUN 2011


BAGIAN PERTAMA BAGIAN KEDUA
1. 219483 1. 21 tahun
2. 0 2.
3. 2 3. 3362
4. satuan luas 4. y=3x-1
5. 3:2
5. 120 bilangan
6. 18 anak
7.
8. 45
9. ¼
10. 4
OSP SMP TAHUN 2010
BAGIAN PERTAMA BAGIAN KEDUA
1. 3A + 36435270 1. 287
2. 55 2.
3. 18775 3.
4. 7
4. 90 cara
5.
6. 5,6 cm 5.
7. 36 cara
8. 1838 cara
9.
10. 48

OSP SMP TAHUN 2009


BAGIAN PERTAMA BAGIAN KEDUA
1. 22 1.
2. 2.019.045 2. 27 dan 37
3. 3. Tidak dapat ditentukan
4. 2x
5. 1250
6. 7,55
7.
8.
9.
10.
11. 77
12.
13.
14. 8
15. 55
16.
17. 11 cara
18. n=1
19. 21 meter
20. 2010
OSP SMP TAHUN 2008
BAGIAN PERTAMA BAGIAN KEDUA
1. 67678 1. 9
2. 24 2. 8 detik
3. 720 3. Tidak mungkin
4. 16 4. 48
5. 6 5. 670,224,10,4,2
6. 2
7. 52,5
8.
9. 1
10. 1.287
11. 36
12. 9801
13. 1:2
14. 182
15. 4
16. a=44 dan x=6, maka a+x=50
17. -1
18. Data terkecil = 25,
Data terbesar = 100,

19. 59690400 cara


20.

OSP SMP TAHUN 2007


BAGIAN PERTAMA BAGIAN KEDUA BAGIAN KEDUA
1. A 1. 60 1. a=20301 kubus dan
2. D 2. n=12 b= Rp. 61.807.500
3. B 3. 2. 6,23 detik
4. C
5. B
6. C 4.
7. C
5. – 2006
8. A
9. B 6.
10. D 7.
8. 1077
9.
10.
OSP SMP TAHUN 2006
BAGIAN PERTAMA BAGIAN KEDUA
1. 8 1.
2. 10 2.
3. 180.000 3.
4. 31/128 4. 630 batang korek api
5. 8,05 5. ½
6. 2 pasang
7. 5
8. Senin
9. 3
10. -2.013.021
11.
12. – 12 atau 28
13.
14.
15. 9
16. 26
17. 48 cara
18. 0
19.
20. 1

OSP SMP TAHUN 2005


BAGIAN PERTAMA BAGIAN KEDUA
1. 22 1. 400
2. 4008 2. 24 kombinasi
3. 240 dan 270 3.
4. 4. Terbukti
5. 5. Ada 2 pasang yaitu (4,140) dan
6. 102,105,108,111,114,117,120 (20,28)
7. (10,10,10), (9,10,11), (8,10,12),
(9,9,12), (8,11,11)
8. 63
9.
10. Benar. Prinsip
OSP SMP TAHUN 2004
BAGIAN PERTAMA BAGIAN KEDUA
1. 25 1. 31 kali
2. 3 2. 8/3 satuan luas
3. 18 3. {– 1,1}
4. 3 4. 0,163 kg
5. 4321312121 5. Terbukti
6. 3
7.
8. 12 satuan panjang
9. 15
10.

OSP SMP TAHUN 2003


BAGIAN PERTAMA BAGIAN KEDUA
1. 1.
2. 23 kali 2.
3. 12,5 satuan luas 3. 353700
4. 48 cm 4.
5.
6. 7 liter 5. 2
7.
8. 6 cara
9.
10. Gaji mereka sama
KUNCI SOAL OSN MATEMATIKA SMP TAHUN 2003 - 2016

OSN SMP TAHUN 2016


HARI PERTAMA
1. x=1
2. 2.402.230
3. Ya. Sama kaki
4. 99 kode
5. 349,5 gram

HARI KEDUA
1. dan
2. Bukan bilangan bulat
3. ( √ ) dm3
4. 120.960
5. Lihat tabel

OSN SMP TAHUN 2015


HARI PERTAMA
1. 619,737,131,179
2. 196,2016,2500
3. 60
4. Benar panjang busur AY = panjang busur AZ
5.
Jadwal Untuk
Kelompok Kelompok Kelompok Kelompok Kelompok
Percobaan
A B C D E
Minggu Ke-V
Jadwal I 3 1 4 5 2
Jadwal II 2 1 3 5 4

HARI KEDUA
1. 1475 dan 1534
2. terbukti
3.
4. QT= 4 satuan
5. 1977
6. h = 4 detik
OSN SMP TAHUN 2014
HARI PERTAMA
1. Hari Rabu, 26 Februari 2014
2. Minggu ke-23
3.
4. 7
5. dan

HARI KEDUA
1. Model B = 66 model, C=1 model, D= 1 model ; lebar halaman = 324 cm dan panjang halaman

2. 1280 cm2
3. 4:5
4. (0, ) dan (4, )
5. 112 cara

OSN SMP TAHUN 2013


HARI PERTAMA
1. √ atau √
2. Terbukti
3. Permutasi dari (a,b,c)=(2,3,8) dan (2,4,5) ada 12 solusi
4. Terbukti
5.

HARI KEDUA
1. tidak ada
2.

3. cm2
4. 15120 x 8! cara
5. cara

OSN SMP TAHUN 2012


HARI PERTAMA
1.
2. a. menyebut angka-angka 3,4,6,7. Jadi bilangan yang disembunyikan adalah 7
b.Gunakan cara penyajian bilangan
3.
4. √ meter
5.

HARI KEDUA
1. 1:99
2.
3. √ cm
4. 8 cm2 dengan ukuran 4 cm x 2 cm
5.
OSN SMP TAHUN 2011
HARI PERTAMA
1. atau 1,3,3,3,3,3,3,4,4
2. pasang
3.
4. 53130
5.

HARI KEDUA
1. 11 dengan n = 41
2. cara
3. = 19, terkecil =14
4. Tidak ada yang memenuhi
5. satuan volume

OSN SMP TAHUN 2010


HARI PERTAMA
1.
2.
3. luas segitiga ABC
4. 34 lintasan
5.

HARI KEDUA
1. terbukti
2. atau
3. satuan panjang
4. 2400
√ √
5. dengan

OSN SMP TAHUN 2009


HARI PERTAMA
1. pasangan
2.
3. susunan
4. 7 cm2
5.

HARI KEDUA
1. 77.760
2. bilangan
3.
4. Terbukti
5. atau 25 pemain
OSN SMP TAHUN 2008
HARI PERTAMA
1. cm
2.
3. adalah hasil kali 2016991 dan 4740 seperti yang dimaksudkan dalam soal
4. 2012981
5.
HARI KEDUA
1. (x,zx)=(2,2),(1,3),(3,3),(1,4),(2,4),(4,4)
2.
3. susunan
4. Terbukti
5.

OSN SMP TAHUN 2007


HARI PERTAMA
1. kartu

2. satuan luas
3.
4. 3.963.960
5.
b.136479
c.angka 8
HARI KEDUA
1. 24, 40 dan 56
2.
3. cara
b.256
4. a. a+(bxc) (a+b)x(a+c)
b. a+(bxc)=(a+b)x(a+c)
jika dan hanya jika a=0 atau a+b+c=1
5. Nilai maksimum N=tak terbatas dan minimum N=86

OSN SMP TAHUN 2006


HARI PERTAMA
1.
2. √
3. 576√ cm3
4. ax+by 1
5. 83 = 512
HARI KEDUA
1. terbukti
2. lihat gambar disamping
3.
4. 42 km/jam
5. a. ada 4 cara (1962,1692,2961,2691). Nilai terbesar diperoleh jika bilangan – bilangan yang
terbesar diletakkan ditengah sementara yang kecil bagian pinggir
b. bilangan – bilangan yang kecil ditengah-tengah dan terbesar di ujung. Ada 8 susunan yaitu :
62111269,92111266,96111226,66111229,62211169,92211166,66211129 dan 96211126
OSN SMP TAHUN 2005
HARI PERTAMA
1.

dst
A={ }
2.
3. 4%,5%,7%,9% dan X
jika x<5 maka mediannya=5%
jika 5<x<7 maka mediannya=X%
jika x>7 maka mediannya=7%
rata-ratanya=5%+ %
Jadi kemungkinan nilai X sbb:
a. 5=5+ , maka X=0
b. X=5+ , maka X=25/4
c. 7=5+ , maka X=10
4. (-1,1), (-1,0), (0,-1)
5.

HARI KEDUA
1.
2. satuan luas

1-( )
3. 5
4. a. 6
b.12
c.24
d.3072
5. a. ada 37 kemungkinan pasangan
b. terbukti
OSN SMP TAHUN 2004
HARI PERTAMA
0
1.
2. dan 71
3. meter

4.
5. 33 bilangan

HARI KEDUA
1.
2.
o
3.
4. 178
5. a. sudah jelas
b.
1 3 2 3 4 5 8 9
5 7 6 7 6 7 10 11
9 11 10 11 12 13 12 13
13 15 14 15 14 15 14 15

OSN SMP TAHUN 2003


HARI PERTAMA
1.
2.
3. √ cm
4. a. sudah ditunjukkan
b. kesalahannya terletak pada pencoretan
5.
6. Tidak ada TV yang perlu diserahkan
7.
8.

HARI KEDUA
1.
2.
b.100,111,144,199,400,414,900,919
c.isi kotak yang diarsir adalah 5 dan
merupakan satu-satunya
d.

3. o

4. Terbukti

Anda mungkin juga menyukai